2023年中考考前最后一课 您所在的位置:网站首页 壮美和崇高之美的异同点有哪些作文 2023年中考考前最后一课

2023年中考考前最后一课

2023-05-09 22:44| 来源: 网络整理| 查看: 265

这是一份2023年中考考前最后一课-语文,共320页。试卷主要包含了语言表达与应用29,文学文化常识与名著阅读58,名句名篇默写81,诗词曲鉴赏92,文言文阅读110,小说阅读130,散文阅读158,说明文阅读186等内容,欢迎下载使用。

目 录考前预测篇一、字、词、句1二、语言表达与应用29三、文学文化常识与名著阅读58四、名句名篇默写81五、诗词曲鉴赏92六、文言文阅读110七、小说阅读130八、散文阅读158九、说明文阅读186十、议论文阅读207十一、非连续性文本阅读229十二、写作255考前技巧篇一、字、词、句篇287二、语言表达与应用篇290三、文学文化常识与名著阅读篇291四、名句名篇默写篇292五、诗词曲鉴赏篇294六、文言文阅读篇295七、小说阅读篇298八、散文阅读篇300九、说明文阅读篇302十、议论文阅读篇304十一、非连续性文本阅读篇306十二、写作篇307考前心理篇中考获胜第一步:培养健康心理310送给考生的中考小贴士312终极考问:遇上不会做的题怎么办?313考场注意篇做好技术准备,决胜中考考场315避免失分,我有对策315考后心理篇7种方法舒缓考生考试后心理压力317考后提醒:中考结束适当娱乐可以放松不可放纵318【考前预测篇】一、字、词、句 题型一:字音1.下列各组词语中,加点字的读音全都正确的一组是( )A.娉婷(pīng) 筵席(yán) 顷刻(qīng) 强聒不舍(qiǎng)B.鲜妍(yán) 掺杂(cān) 挑剔(tī) 浮光掠影(luè)C.佝偻(lóu) 熏陶(tá) 诡谲(jué) 间不容发(fà)D.游弋(yè) 恣雎(suī) 囫囵(hú) 怒不可遏(è)【答案】C【解析】本题考查字音。A.顷刻(qīng)——qǐng;B.掺杂(cān)——chān;浮光掠影(luè)——lüè;D.游弋(yè)——yì;故选C。2.下列词语中,加点为字的读音有误的一项是( )A.纤绳(qiàn) 掺杂(chān) 恶劣(liè) 喷薄而出(bó)B.侥幸(jiǎ) 贬谪(zhé) 镰刀(lián) 抽丝剥茧(bō)C.恪守(kè) 诓骗(kuāng) 契合(qiè) 强(qiǎng)聒不舍D.发怔(zhèng) 惘然(wǎng) 眺(tià)望 前仆后继(pū)【答案】C【解析】本题考查字音。C.契合(qiè)——qì;故选C。3.下面各组词语中加点字的注音,完全正确的一项是( )A.辐射(fú) 粲然(càn) 繁弦急管(xuán)B.干涩(sè) 篝火(gōu) 获益匪浅(fēi)C.荆棘(jí) 烙饼(là) 更胜一筹(chóu)D.胚芽(pī) 陨石(yǔn) 恍然大悟(huǎng)【答案】C【解析】考查字音的识记。A.繁弦急管(xuán)——xián;B.获益匪浅(fēi)——fěi;D.胚芽(pī)——pēi;故选C。4.下面各组词语中加点字的注音,完全正确的一项是( )A.阔绰(chuò) 粗拙(zhuō) 诲人不倦(huī)B.伫立(zhù) 热忱(chéng) 忍俊不禁(jīn)C.濒临(bīn) 愚钝(yú) 春寒料峭(qià)D.默契(qì) 襁褓(qiǎng) 鲜为人知(xiān)【答案】C【解析】考查字音。A.诲人不倦(huī)——huì;B.热忱(chéng)——chén;D.鲜为人知(xiān)——xiǎn;故选C。5.下列句子中加点字的字音完全正确的一项是( )A.富有创造力的人总是孜(zī)孜不倦地汲(jí)取知识,使自己学识渊博。B.玄德见孔明身长八尺,面如冠玉,头戴纶(lún)巾,身披鹤氅(chǎng),飘飘然有神仙之概。C.要论中国人,必须不被搽(tú)在表面的自欺欺人的脂粉所诓(kuāng)骗,却看看他的筋骨和脊梁。D.我想起悠远的车铃,晴天里马儿戴着串铃在溜直的大道上跑着,狐仙姑深夜的谰(lán)语,原野上怪诞的狂风……这时我听到故乡在召(zhā)唤我,故乡有一种声音在召唤着我。【答案】A【解析】本题考查基础字词的字音。B.纶(lún)巾——guān;C.搽(tú)——chá;D.召(zhā)唤——zhà;故选A。6.下面加点字注音有误的一项是( )A.林荫(yīn) 侍弄(shì) 矫揉造作(jiǎ)B.毋宁(wú) 恫吓(xià) 怏怏不乐(yàng)C.恣睢(suī) 粗拙(zhuō) 悲天悯人(mǐn)D.稽首(qǐ) 畴躇(chú) 前仆后继(pū)【答案】B【解析】考查字音的识记。B.恫吓(xià)——hè;故选B。7.下面各组词语中加点字的注音,完全正确的一项是( )A.缅怀(miǎn) 雕镂(lòu) 销声匿迹(ruò)B.呢喃(nán) 带挈(qì) 伛偻提携(lǚ)C.崩塌(tān) 冗杂(rǒng) 阡陌交通(qiān)D.贮藏(zhù) 伶仃(líng) 怏怏不乐(yàng)【答案】D【解析】考查字音的识记。A.销声匿迹(ruò)——nì;B.带挈(qì)——qiè;C.崩塌(tān)——tā;故选D。8.下面各组词语中加点字的注音,完全正确的一项是( )A.瓦砾(shuò) 租赁(lìn) 悲天悯人(mǐn)B.山涧(jiàn) 踱步(duó) 风雨如晦(huì)C.斟酌(zhēn) 簇拥(cù) 自惭形秽(suì)D.倔强(juè) 体恤(xuè) 草长莺飞(zhǎng)【答案】B【解析】本题考查字音。A.瓦砾(shuò) ——lì;C.自惭形秽(suì)——huì;D.倔强(juè) ——jué,体恤(xuè) ——xù;故选B。9.下面各组词语中加点字的注音,完全正确的一项是( )A.窘迫(jiǒng) 胆怯(què) 如释重负(shì)B.恍惚(hū) 麂子(jǐ) 目眩神迷(xuàn)C.箱箧(qiè) 着想(zhá) 荡然无存(dànɡ)D.斑蝥(má) 腈纶(qíng) 骇人听闻(hài)【答案】B【解析】本题考查字音。A.胆怯(què)——qiè;C.着想(zhá)——zhuó;D.腈纶(qíng)——jīng;故选B。10.下列加点字的注音正确的一项是( )A.亘古(gèn) 干涸(hé) 追溯(sù) 潜心贯注(qiǎn)B.绮丽(qǐ) 藩篱(fán) 亵渎(xiè) 风雪载途(zài)C.剽悍(piā) 谀词(yú) 蓦然(mò) 龙吟凤哕(huì)D.诘责(jié) 胆怯(qiè) 嘈杂(cā) 大庭广众(tíng)【答案】C【解析】本题考查字音。A.潜心贯注(qiǎn)——qián;B.藩篱(fán)——fān;D.嘈杂(cā)——cá;故选C。11.下面各组词语中加点字的注音,完全正确的一项是( )A.锱铢(zì) 瞥见(piē) 黝黑(yǒu) 惟妙惟肖(xià)B.诓骗(kuāng) 演绎(yì) 稽首(qí) 如坐针毡(zhān)C.濒临(bīn) 睥睨(nì) 诡谲(jué) 殚精竭虑(dān)D.连翘(qiá) 襁褓(qiáng) 凫水(fú) 鸠占鹊巢(jiū)【答案】C【解析】本题考查字音。A.锱铢(zì)——zī;B. 稽首(qí)——qǐ;D. 襁褓(qiáng)——qiǎng;故选C。12.下面各组词语中加点字的注音,完全正确的一项是( )A.澄澈(chéng) 体恤(xù) 两肋插刀(lèi)B.浊流(zhuó) 侍弄(cì) 擎天柱(qíng)C.贮蓄(chǔ) 娴熟(xián) 呕心沥血(lì)D.麾下(há) 赦免(shè) 相濡以沫(rú)【答案】A【解析】考查字音的识记。B.侍弄(cì)——shì;C.贮蓄(chǔ)——zhù;D.麾下(há)——huī;故选A。13.下面各组词语中加点字的注音,完全正确的一项是( )A.妥当(dāng) 坍塌(tān) 乘奔御风(yù)B.兴味(xìng) 胸脯(pú) 间不容发(fà)C.别墅(shù) 怯懦(ruò) 信手拈来(diān)D.盘桓(yuán) 褴褛(lǚ) 触目伤怀(chù)【答案】B【解析】本题考查字音正确辨析能力。A.妥当(dāng)——dàng;C.怯懦(ruò)——nuò,信手拈来(diān)——niān;D.盘桓(yuán)——huán;故选B。14.下列各组词语中加点字注音,完全正确的一项是( )A.提防(tí) 称职(chèn) 粗糙(cǎ) 不省人事(xǐng)B.氛围(fēn) 污秽(huì) 炽热(zhì) 拈轻怕重(niān)C.匿笑(nì) 陨石(yǔn) 静谧(mì) 戛然而止(jiá)D.倔强(jué) 粗犷(guǎng) 蓦然(mù) 彬彬有礼(bīn)【答案】C【解析】本题考查字音辨析。A.提防(tí)——dī ,粗糙(cǎ)——cā;B.炽热(zhì)——chì;D.蓦然(mù)——mò;故选C。15.下列各组词语中,加点字的读音全都正确的一组是( )A.遒劲(jìng) 一绺(liǔ) 坊间(fāng) 大煞风景(shā)B.歼灭(jiān) 承载(zǎi) 逻辑(jí) 屏息敛声(bǐn)C.秀颀(qí) 幽悄(qiǎ) 轧扁(zhá) 强词夺理(qiáng)D.鬈发(quán) 倔强(jué) 羁绊(bàn) 张目结舌(jiē)【答案】A【解析】本题考查字音。B.承载(zǎi)——zài;屏息敛声(bǐn)——bǐng;C.轧扁(zhá)——yà;强词夺理(qiáng)——qiǎng;D.张目结舌(jiē)——jié;故选A。题型二:字形16.下列词语中,没有错别字的一项是( )A.煞白 要决 眼花缭乱 雕梁画栋B.鲁钝 隔膜 郑重共事 置知不理C.戳破 惊骇 走头无路 如雷贯耳D.体恤 避讳 味同嚼蜡 矫揉造作【答案】D【解析】本题考查字形辨析。A.要决——要诀;B.郑重共事——郑重其事,置知不理——置之不理;C.走头无路——走投无路;故选D。17.下列语句中书写正确的一项是( )A.在她头顶上打转的失重的铅笔,不知为什么,一闭上眼睛,总在我的眼前漂浮。B.到了秋天,果实成熟,植物的叶子渐渐变黄,在秋风中漱漱地落下来。C.要论中国人,必须不被搽在表面的自欺欺人的脂粉所诓骗,却看看他的筋骨和脊梁。D.他身材很高大,青白脸色,皱纹间时常夹些伤痕,一部乱篷篷花白的胡子。【答案】C【解析】考查字形的识记辨析。A.漂浮——飘浮;B.漱漱——簌簌;D.乱篷篷——乱蓬蓬;故选C。18.下列各组词语中,汉字书写全都正确的一组是( )A.燥热 萤光 因地制宜 不修边幅B.震撼 辐射 和颜悦色 周尔复始C.抉择 题跋 名副其实 人情世故D.彗星 捶击 振聋发聩 成群结对【答案】C【解析】本题考查字形辨析。A.萤光——荧光;B.周尔复始——周而复始;D.成群结对——成群结队;故选C。19.下列语句中书写正确的一项是( )A.对同志对人民不是满腔热诚,而是冷冷清清,漠不关心,麻木不仁。这种人其实不是共产党员,至少不能算一个纯粹的共产党员。B.有翅膀在水上的拍打声,有蹼的划动而发出的声音,还有观战者们激烈的辩论所发出的呼叫声。随后,一个深沉的声音算是最后发言。C.规划自己的职业生涯,使事业和人生呈现出缤纷和谐、相得益彰的局面,是第二间精神小屋坚固优雅的要决。D.《大学》本身就说,格物致知的目的,是使人能达到成意、正心、修身、治国的田地,从而追求儒家的最高理想——平天下。【答案】B【解析】本题考查汉字书写能力辨析。A.热诚——热忱;C.要决——要诀;D.成意——诚意;故选B。20.下列词语中没有错别字的一项是( )A.耸峙 菡萏 心有灵犀 屏息凝神B.粗犷 决别 语无伦次 颠沛流离C.猥琐 坍塌 截然不同 诃捐杂税D.震悚 竹篾 浑为一谈 怪诞不经【答案】A【解析】本题考查字形辨析。B.决别——诀别;C.诃捐杂税——苛捐杂税;D.浑为一谈——混为一谈;故选A。21.下列词语中没有错别字的一项是( )A.不省人事 走头无路 不足为据B.不言而喻 置之不理 自爆自弃C.迫不及待 根深缔固 行之有效D.如雷贯耳 锲而不舍 孜孜不倦【答案】D【解析】本题考查成语字形。A.走头无路——走投无路;B.自爆自弃——自暴自弃;C.根深缔固——根深蒂固;故选D。22.下列词语中没有错别字的一项( )A.翻来覆去 锋芒必露 振聋发聩 人情世故B.神采弈弈 妇孺皆知 重峦叠嶂 无缘无故C.哗众取宠 油光可签 混为一谈 眼花缭乱D.怪诞不经 诲人不倦 杳无消息 自惭形秽【答案】D【解析】本题考查常见易错字。A.锋芒必露——锋芒毕露;B.神采弈弈——神采奕奕;C.油光可签——油光可鉴;故选D。23.下列各组词语中,汉字书写全都正确的一组是( )A.蛾眉 熏淘 胃同嚼蜡 经世奇才B.喷薄 伶俐 不屑置辩 雕梁画栋C.聒燥 侦缉 根深蒂固 型销骨立D.统筹 惦纪 忸怩做态 如坐针沾【答案】B【解析】本题考查字形。A.熏淘——熏陶,胃同嚼蜡——味同嚼蜡;C.聒燥——聒噪,型销骨立——形销骨立;D.忸怩做态——忸怩作态, 如坐针沾——如坐针毡;故选B。24.下列词语中书写完全正确的一项是( )A.遨游 决择 亢奋 不屑置辩B.侥幸 视察 治裁 附庸风雅C.云霄 闲暇 旁骛 锲而不舍D.篡改 诘责 安详 锋芒必露【答案】C【解析】本题考查字形。A.决择——抉择;B.治裁——制裁;D.锋芒必露——锋芒毕露;故选C。25.下列词语中,没有错别字的一项是( )A.鲁钝 誊写 毛骨悚然 李代桃疆B.糟蹋 聒燥 如雷贯耳 樯倾楫摧C.恣睢 撺掇 纷至踏来 鸠占鹊巢D.秀颀 荼毒 与日俱增 坦荡如砥【答案】D【解析】本题考查字形。A.李代桃疆——李代桃僵;B.聒燥——聒噪;C.纷至踏来——纷至沓来;故选D。26.下列词语中没有错别字的一项是( )A.无可奈何 吹毛求疵 雕梁画栋B.哗众取宠 信手拈来 无精打彩C.锲而不舍 大厅广众 诲人不倦D.美不盛收 骇人听闻 翻来覆去【答案】A【解析】本题考查字形。B.无精打彩——无精打采;C.大厅广众——大庭广众;D.美不盛收——美不胜收;故选A。27.下列各组词语中,汉字书写全都正确的一组是( )A.嘹亮 恣雎 摇蓝 翻来覆去B.睿智 分歧 企盼 通霄达旦C.娉婷 旁骛 诸侯 金碧辉煌D.亵渎 祥和 锲合 雕粱画栋【答案】C【解析】本题考查字形。A.恣雎——恣睢,摇蓝——摇篮;B.通霄达旦——通宵达旦;D.锲合——契合,雕粱画栋——雕梁画栋;故选C。28.根据拼音写汉字。夜幕下看宛园,仿佛置身灯的海洋。水幕电影是在中心湖区的位置,播放节目有扇面秀、双龙戏珠以及京剧脸谱等内容,配合现场jīng xīn dònɡ pò( )的音乐,给游客们打造非常zhèn hàn( )的视觉效果。【答案】惊心动魄 震撼【解析】本题考查根据拼音写汉字。jīng xīn dònɡ pò——惊心动魄,意思是原指文辞优美,意境深远,使人感受极深,震动极大。后常形容使人惊骇紧张到极点。zhèn hàn——震撼,意思是指内心受到强烈的冲击或感动,精神或情绪剧烈起伏或波动。29.根据拼音写汉字。我生来就是高山而非溪流,我欲于群峰之巅俯视平庸的沟壑。我生来就是人杰而非草芥,我站在伟人之肩miǎ shì( )卑微的懦夫;我生来就是hóng hú( )而非燕雀,我欲于苍穹之上小视地上的蝼蚁。【答案】藐视 鸿鹄【解析】考查字形。藐视,miǎ shì,看不起;小瞧。鸿鹄,hóng hú,天鹅。30.根据拼音写出相应的词语。(1)pái huái _____了一会子,窗外雷声作了。(2)周围一片寂静,我趴在岩石上,神情huǎng hū _____,害怕和疲劳已经让我麻木。(3)趁这个机会,我pá gēn wèn dǐ _____,才从他嘴里知道了一些事。(4)研究动物行为的科学家实验的方法guài dàn bù jīng _____。【答案】徘徊 恍惚 刨根问底 怪诞不经【解析】(1)徘徊:在一个地方来回地走。注意其写法。(2)恍惚:神志不清、精神不集中;(记得、听得、看得)不真切、不清楚。注意“惚”的写法。(3)刨根问底:比喻追究底细,注意“刨”的写法。(4)怪诞不经:古怪荒唐,不合常理。注意“诞”的写法。题型三:字音和字形31.小语在初中语文教材里搜集井摘录“以物载情”的素材,以下是一些摘自她笔记本的词语,其中,字形和加点字的读音准确无误的一项是( )A.②⑤⑧⑫B.①⑥⑨⑪C.③④⑨⑩D.①④⑦⑪【答案】C【解析】本题考查字音字形。②帐蓬——帐篷;⑥拜谒(è)——yè;⑦亘(gèng)古——gèn;⑧妥贴——妥帖;⑪如座针毡——如坐针毡;⑫深恶(è)痛绝——wù;故选C。32.下面是某同学做的字音字形梳理记录,其中有错误的一项是( )A.注意纠正因方言发音而造成的误读,如:“惩(chēng)戒”应读成“惩(chéng)戒”,“畏罪潜(qiǎn)逃”应读成“畏罪潜(qián)逃”。B.注意纠正人名、地名的误读,如:“华(huá)罗庚”应读成“华(huà)罗庚”,“瀛(yín)洲”应读成“瀛(yíng)洲”。C.注意因音近或形似造成的误写,如:“啜泣”不能写成“辍泣”,“晕眩”不能写成“晕炫”。D.注意成语字形的误写,如:“美不胜收”应写成“美不盛收”,“眼急手快”应写成“眼疾手快”。【答案】D【解析】本题考查字音字形辨析能力。D.“美不盛收”应写成“美不胜收”。故选D。33.下列句子中加点字的注音和字形全都正确的一项是( )A.我是在春风的襁褓里长大的,记忆深处的那些桃花,总会在眼前倏(shù)然闪现。B.这是大自然的馈(kuì)赠,河面波光粼粼,两岸群山环绕,仿佛混然天成的山水画。C.老家就像一个跋(bá)山涉水时可以休息的驿站,总能让风尘仆仆的人充满幸福感。D.广大青年只有经受严格的淬(cuì)炼,才能在复杂严竣的斗争中经风雨、壮筋骨。【答案】C【解析】考查字音字形。A.倏(shù)——shū;B.混然天成——浑然天成;D.严竣——严峻;故选C。34.下列句子中加点字的注音和字形全都正确的一项是( )A.春分之后,枝头的蓓(péi)蕾已愈发饱满,似乎一阵暖风,就可以拉开盛大的帷幕。B.潺潺(chán)的流水声,应和着踏青姑娘的欢歌笑语,不决于耳。C.一草一木都在认认真真地赴岁月之约,心无旁骛地走在春意盎(àn)然的季节里。D.我怀抱着岁月的素笺(jiān),写下这大自然的馈赠,写下满心的欢喜与怀想。【答案】D【解析】本题考查字音字形。A.蓓(péi)蕾——bèi;B.不决于耳——不绝于耳;C.盎(àn)然——ànɡ;故选D。35.下列词语中加点字的字音和字形都正确的一组是( )A.元勋(xūn) 蓦然(mò) 咄咄逼人(duó) 美不盛收(shèng)B.瞥见(piē) 追溯(sù) 张目结舌(jié) 纷至沓来(tà)C.逞能(chéng) 桑梓(zǐ) 浮光略影(lüè) 信手拈来(niān)D.恬静(tián) 炽热(zhì) 见异思迁(yì) 油光可见(jiàn)【答案】B【解析】本题考查字音字形。A.咄咄逼人(duó)——duō,美不盛收——美不胜收;C.逞能(chéng)——chěng,浮光略影——浮光掠影,D.炽热(zhì)——chì,油光可见——油光可鉴;故选B。36.下列加点字,字形和注音全部正确的一项是( )A.摩擦(cā) 化妆(zhuāng) 安营扎寨(zhā) 春寒料峭(qià)B.胚胎(péi) 眩晕(xuàn) 浮想连翩(lián) 如履薄冰(lǚ)C.蜗行(wō) 通缉(jí) 重蹈覆辙(fù) 闲情逸志(zhì)D.考订(dīng) 颤抖(chàn) 正禁危坐(jīn) 不修边幅(fú)【答案】A【解析】本题考查字音字形。B.胚胎(péi)——pēi;浮想连翩——浮想连翩;C.通缉(jí)——jī;闲情逸志——闲情逸致;D.考订(dīng) ——dìng;正禁危坐——正襟危坐;故选A。37.选出下列词语中加点字音形完全正确的一项( )A.决别(jué) 龟裂(jūn) 拖沓(tà) 振聋发聩(kuì)B.篡改(cuàn) 燥热(zà) 镌刻(juān) 坦荡如砥(dǐ)C.晕厥(yūn) 秀颀(qí) 倜傥(tì) 家谕户晓(yù)D.倔强(jué) 遒劲(jìng) 绯闻(fěi) 深恶痛疾(wù)【答案】B【解析】考查字音字形的识记。A.决别——诀别;C.家谕户晓——家喻户晓;D.绯闻(fěi)——fēi,故选B。38.下列词语中加点字的字音、字形完全正确的一项是( )A.契和(hé) 教诲(huì) 束缚(fù) 风雪载途(zǎi)B.烦琐(suǒ) 帐篷(péng) 作揖(zuō) 因地治宜(zhì)C.劝戒(jiè) 疙瘩(gā) 流盼(liú) 间不容发(jiān)D.创伤(chuāng) 屏息(bǐng) 滞碍(zhì) 望眼欲穿(chuān)【答案】D【解析】本题考查字音字形。A.契和——契合, 风雪载途(zǎi)——zài ;B. 作揖(zuō)——zuò,因地治宜——因地制宜;C.劝戒——劝诫,疙瘩(gā)—— gē ;故选D。39.下列词语中、字形和加点字的注音完全正确的一项是( )A.妖娆(rá) 芭蕾(léi) 屏障 振耳欲聋B.折本(shé) 恣睢(suī) 坍塌 经世奇才C.字帖(tiē) 赧愧(nǎn) 烦燥 与日俱增D.栈桥(zhàn) 亘古(gèng) 端详 以身作责【答案】B【解析】考查字音字形。A.芭蕾(léi)——lěi,振耳欲聋——震耳欲聋;C.字帖(tiē)——tiè,烦燥——烦躁;D.亘古(gèng)——gè,以身作责——以身作则;故选B。40.下列词语中加点字的字音、字形完全正确的一项是( )A.练达(dá) 姑且(qiě) 睥睨(pì) 狂妄自大(wàng)B.诘难(jí) 寂寥(jì) 雷庭(tíng) 吹毛求疵(cī)C.藻饰(zǎ) 矿远(kuàng) 虐待(nuè) 浮光掠影(lüè)D.惆怅(chóu) 收揽(lǎn) 侦缉(jí) 信手沾来(niān)【答案】A【解析】考查字音字形。B.诘难(jí)——jié,雷庭——雷霆;C.矿远——旷远,虐待(nuè)——nüè;D.侦缉(jí)——jī,信手沾来——信手拈来;故选A。41.下列词语中加点字的字音、字形完全正确的一项是( )A.憎恶(zèng) 哀悼(dà) 愧怍(zuò) 大廷广众(tíng)B.晌午(shǎng) 妥贴(tiē) 殉职(xùn) 诲人不倦(huǐ)C.嘹亮(liá) 渲染(xuàn) 粗拙(zhuō) 悲天悯人(mǐn)D.托沓(tuō) 潜逃(qiǎn) 挟持(xié) 言外之意(yì)【答案】C【解析】本题考查字音、字形。A.憎恶(zèng)—— zēng,大廷广众——大庭广众;B.妥贴——妥帖,诲人不倦(huǐ)——huì;D.托沓——拖沓,潜逃(qiǎn)——qián;故选C。42.下列句子中,加点字词字形与注音不完全一致的一项是( ) A.各种花的香在空气里酝(yùn)酿,鸟儿将窠(kē)巢安在繁花嫩叶当中。B.他从唐诗下手,目不窥(kuī) 园,足不下楼,兀兀穷年,呖(lì)尽心血。C.要论中国人,必须不被搽(chá)在表面的自欺欺人的脂粉所诓(kuānɡ)骗。D.如果不怕刺,还可以摘到覆盆子,像小珊瑚珠攒(cuán)成的小球,又酸又甜,色味都比桑椹(shèn)要好得远。【答案】B【解析】本题考查字音字形。B.呖尽心血——沥尽心血。故选B。43.下列词语中,字形和加点字的注音完全正确的一项是( )A.悄然(qiǎ) 峻峭(jùn) 纤细(qiān) 司空见贯(guàn)B.顷刻(qǐng) 翠绿(lǜ) 醇香(chún) 竭尽全力(jié)C.抉择(jué) 炊烟(cuī) 潮讯(xùn) 天涯海角(yá)D.混淆(hǔn) 衔接(xián) 颁奖(bān) 和言悦色(yán)【答案】B【解析】本题考查字音字形。A.纤细(qiān)——xiān,司空见贯——司空见惯;C.炊烟(cuī)——chuī,潮讯——潮汛;D.混淆(hǔn)——hùn,和言悦色——和颜悦色;故选B。44.下列词语中字形和加点字的注音正确的一项是( )A.飞窜(cuàn) 瘦削(xiā) 统畴 暴风骤雨B.炽热(zhì) 威吓(hè) 赦免 无可耐何C.内疚(jiù) 绢本(juàn) 典籍 功成名遂D.倒戈(gē) 症结(zhèng) 忸怩 微不足到【答案】C【解析】本题考查汉字字音字形。A.瘦削(xiā)——xuē,统畴——统筹;B.炽热(zhì)——chì,无可耐何——无可奈何;D.症结(zhèng)——zhēng,微不足到——微不足道;故选C。45.下列各项中,汉字书写和加点字注音全都正确的一项是( )A.喷簿(bó) 狡黠(jié) 诡谲(jué) 如坐针毡(zhān)B.侍候(sì) 恐吓 (xià) 瘦削(xuē) 雕粱画栋(dòng)C.簇新(cù) 暄染(xuàn) 瞭望(liá) 吹毛求庛(cī)D.笔砚(yàn) 要诀(jué) 稽首(qǐ) 铢两悉称(zhū)【答案】D【解析】考查字音字形。A.喷簿——喷薄,狡黠(jié)——xiá;B.侍候(sì)——shì,恐吓 (xià)——hè,雕粱画栋——雕梁画栋;C.暄染——渲染,瞭望(liá)——lià,吹毛求庛——吹毛求疵;故选D。题型四:词语综合运用46.下列加点成语使用恰当的一项是( )A.央视热播剧《人世间》剧情抑扬顿挫,细节感人,深受观众喜爱。B.在逐梦太空的征途上,我国航天事业开启了举世瞩目的新篇章。C.奥运健儿在赛场上顽强拼搏,不断超越自我,赢得了观众振聋发聩的欢呼声。D.端午节当天的龙舟赛场,彩旗飘扬、人头攒动,远远望去,人声鼎沸。【答案】B【解析】本题考查成语运用。A.抑扬顿挫:指(声音)高低起伏和停顿转折,不能用来形容剧情,使用有误;B.举世瞩目:全世界的人都注视着,形容影响很大。在本句中用来形容航天事业的影响很大,符合语境;C.振聋发聩:声音很大,使耳聋的人也听得见。比喻用语言文字唤醒糊涂麻木的人,使他们清醒过来。本句用来形容观众欢呼声之大,对象误用,使用不正确;D.人声鼎沸:形容人声嘈杂喧闹,而彩旗飘扬,人头攒动都是视觉画面,使用有误;故选B。47.下列句子中加点的词语使用不恰当的一项是( )A.人的一生,有艰难困苦的逆境,也有峰回路转的风景。B.就写景来说,无论何种景物,要描写得惟妙惟肖,都要费许多笔墨。C.面对妈妈精心烹制的一桌美味菜肴,求职失利的他吃着却味同嚼蜡,毫无食欲。D.季羡林讲课,语调平稳,语言朴实,没有一丝的煽情,却叫人不能不正襟危坐,洗耳恭听。【答案】C【解析】本题考查成语的运用。A.峰回路转:形容山峰、道路迂回曲折。有时也比喻经过挫折后出现转机。此处形容“风景”,使用正确;B.惟妙惟肖:形容描写或模仿得非常好,非常逼真。此处形容描写非常好,非常逼真,使用正确;C.味同嚼蜡:味道像嚼蜡一样。形容说话或文章枯燥乏味。此处形容吃东西,使用不当;D.正襟危坐:整好衣襟,端端正正地坐着。形容严肃、恭敬或拘谨的样子。此处形容人们听季羡林讲课严肃、恭敬或拘谨的样子,使用正确;故选C。48.下列各句中,加点词语使用错误的一项是( )A.举目回望,那一个个奔忙而坚定的身影,那一张张疲惫却坚毅的面孔,顷刻间汇聚成一往无前的钢铁洪流,是无数血肉之躯筑起抵御疫情的新的长城。B.这堂课让同学们仿佛置身于祖国广茂大地,汲取生命的养料,感悟奋斗的真谛,真切感受祖国的命运与自己休戚相关。C.在“传统文化进校园”主题班会上,他的一番高谈阔论获得了大家的肯定和赞许。D.许多院士表示,医生不仅要有妙手回春的本领,更要有医者仁心的大爱。【答案】C【解析】本题考查词语正确运用的能力。A.顷刻:指片刻,表示行动或事情在极短的时间内完成。符合语境;B.汲取:意思是吸取、吸收。符合语境;C.高谈阔论:多指不着边际地大发议论。与“获得了大家的肯定和赞许”语境不符;D.妙手回春:比喻将濒于死亡的人救活。形容医术高明,能把生命垂危的病人医治好。符合语境;故选C。49.某同学给大家介绍学习心得,其中加点词使用最恰当的一项是( )七年级上册《寓言四则》里讲了这样一个故事:一只蚊子专门挑狮子鼻子等没有毛的地方攻击,狮子气得不卑不亢,却无计可施,蚊子得意洋洋,高唱着凯歌飞起来,没想到却被一张蜘蛛网给粘住了!它无力还击,只好一蹴而就了,这恰好印证了一句话:鹬蚌相争,渔翁得利。A.不卑不亢B.得意洋洋C.一蹴而就D.鹬蚌相争,渔翁得利【答案】B【解析】本题考查对成语的理解与运用。A.“不卑不亢”为褒义,形容待人态度得体,分寸恰当,用在此处不恰当,可以改为“暴跳如雷”;B.“洋洋得意”形容得意时神气十足的姿态,使用正确;C.“一蹴而就”形容事情轻而易举,一下子就能完成,可以改为“束手就擒”;D. “鹬蚌相争,渔翁得利”形容两方争执会造成两败俱伤,让第三者获利,可以改为“螳螂捕蝉,黄雀在后”;故选B。50.下面句子中加点词语运用不合适的一项是( )A.面对美国咄咄逼人的战略打压,中国唯有斗争才能捍卫自身利益!B.一草一木都在认认真真地赴岁月之约,心无旁骛地走在万物生长的春天。C.石头垒砌的房子,依山而建,鳞次栉比。沿河而上的石阶,斑斑驳驳,曲曲折折。D.12位非遗传人讲述了灯彩、核雕、苏绣等的前世今生,还有络绎不绝的哲思匠心。【答案】D【解析】考查词语运用。A.咄咄逼人:气势汹汹,使人惊惧。正确;B.心无旁骛:心思没有另外的追求,形容心思集中,专心致志。正确;C.鳞次栉比:像鱼鳞和梳子齿那样有次序地排列着,多用来形容房屋或船只等排列得很密很整齐。正确;D.络绎不绝:形容行人、车马、船只等来往频繁,连续不断。不能形容“哲思匠心”,用错对象;故选D。51.下列句子中加点的词语使用不恰当的一项是( )A.我们深切缅怀袁隆平的同时,更应该铭记他的人生箴言,永不停止追求。B.文化需要交流借鉴,只有在不同文化的碰撞下才能赋予自身文化新的意义。C.从二十四节气创意倒计时,到“折柳”送别来宾,冬奥会开、闭幕式的设计巧妙绝伦。D.振兴乡村,把绿水青山变成金山银山,让乡亲们过上富裕文明的生活,这是乡村干部们孜孜不倦的梦想。【答案】D【解析】本题考查词语使用。A.箴言:规谏劝诫的话。使用正确;B.赋予:给予,交给,寄托(重大任务、使命等)。本句中用来形容不同文化的碰撞给予了文化自身新的意义,符合语境,使用正确;C.巧妙绝伦:指精巧奇妙到了极点,在同类事物中没有能与之相比的。用来形容“设计”,符合语境和用法,使用正确;D.孜孜不倦:指工作或学习勤奋不知疲倦。这里用来修饰“梦想”,属于使用对象错误,使用不正确;故选D。52.下面语句中加点的成语使用有误的一项是( )A.苏州园林的匠师们借鉴了各地园林的标本,自出心裁,修建成功的园林各具特色。B.对于他人的置之不理,我们说再多也无济于事,不如坚定自己的内心,无畏前行。C.在残破的棚屋里,居里夫人呕心沥血,坚持数年后,终于见到了镭的美丽颜色。D.几千年来“无后不孝”的传统观念,在文化落后的山区,至今仍根深蒂固。【答案】A【解析】此题考查成语运用。A.自出心裁:出于自己心中的设计或筹划。与句中“借鉴了各地园林的标本”相矛盾;B.置之不理:放在一边,不予理睬。形容对某人某事十分冷淡。与语意相符;C.呕心沥血:形容费尽心血和精力。与语意相符;D.根深蒂固:比喻根基牢固,不可动摇。与语意相符。故选A。53.依次填入下面一段文字横线处的词语,最恰当的一项是( )二十世纪八十年代,现代工笔画复兴,这体现了中国古典艺术传统的不断 ,也体现了新时代中国画艺术的一种革新、新生和希望。中国工笔画要走向现代,不能够把传统精神与现代语言对立起来,应该对传统精神进行有效的把握和 ,使之与现代题材相结合,创造出有东方 的现代作品,使工笔画获得新的发展。A.延续 继承 意蕴B.延续 承担 意境C.继续 承担 意蕴D.继续 继承 意境【答案】A【解析】本题考查词语的运用。延续:强调照原样继续,侧重行为状态的延伸。继续:强调连下去,不间断。空一,根据“这体现了中国古典艺术传统的不断 ,也体现了新时代中国画艺术的一种革新、新生和希望”语境,此处是强调照原样继续,应使用“延续”。排除CD;继承:泛指把前人的作风、文化、知识等接受过来,也指按照法律或遵照遗嘱接受死者的财产、职务、头衔、地位等。承担:担负,担当。空二,根据“应该对传统精神进行有效的把握和 ”语境前文“不能够把……对立起来”可知,此处是把前人的作风、文化、知识等接受过来,应该用“继承”。排除B;意境:作品或景象的神韵给人带来的回味。意蕴:作品里面渗透出来的理性内涵,即事物的内容或含义。空三,根据“创造出有东方 的现代作品,使工笔画获得新的发展”语境,可知,此处是有内容或含义的意思,应使用“意蕴”更准确。仍排除B。故选A。54.依次填入下面一段文字横线处的词语,最恰当的一项是( )人们在工作时会产生一种被支配感,时间是不自由的,心理上是紧张 的。如果再加班的话,剩余的放松时间更是少之又少。而人们在内心往往会 工作与放松的平衡,睡觉在很多人眼中被视为占据自由时间的 。报复性睡前拖延症就产生了。A.急迫 追求 行动B.紧急 追寻 行动C.急迫 追求 行为D.紧急 追寻 行为【答案】C【解析】本题考查词义辨析。第一空:急迫:马上需要应付或办理,不容许迟延,形容心情;紧急:形容形势紧迫,时间短促,没有缓冲的余地。与“心理”搭配的应该是“急迫”。第二空:追求:尽力寻找、探索。侧重点是表述的主体对某一结果产生的内心期望以及努力;追寻:用积极的行动来争取达到某种目的,是对动作的表述。与“平衡”搭配,用“追求”更恰当。第三空:行动:为达到某种目的进行的活动;行为:有意识表现出来的行动。“睡觉”应该是“行为”。故选C。55.依次填入下面一段文字横线处的词语,最恰当的一项是( )汉字无论是字形的演变,还是其中的文化意蕴,都 了中华民族几千年沉淀下来的精神财富。认识和辨析汉字,熟悉它们的 ,是我们了解历史、了解文化的重要 ,对于继承优秀的文化基因和弘扬优秀的民族文化意义重大。A.凝聚 内涵 途径B.凝结 内容 途径C.凝聚 内容 路径D.凝结 内涵 路径【答案】A【解析】考查正确运用词语的能力。第一空,凝聚:指气体由稀变浓或变成液体;积聚;聚合。凝结:气体变成液体或液体变成固体。“凝聚”可以指物质方面也可指精神方面,“凝结”多指物质形态的聚集。因为精神财富是无形的,所以第一处用“凝聚”;第二空,内涵:指一个概念所反映的事物的本质属性的总和;内在的涵养。内容:指事物所包含的实质性事物。内涵是一种抽象的但绝对存在的感觉,是某个人对一个人或某件事的一种认知感觉。“内涵”不是表面上的东西,而是隐藏在事物深处,需要探索、挖掘才可以看到,填入这个词语符合“认识和辨析汉字”的结果,应使用“内涵”;第三空,“途径”比喻为达到目的采取的方式方法,“路径”多用于指现实道路,结合“了解历史、了解文化”的内容可知,此处指的文化精神层面,并非现实中,应选择“途径”;故选A。56.下面句子中加点词语运用不合适的一项是( )A.面对美国咄咄逼人的战略打压,中国唯有斗争才能捍卫自身利益!B.只有在创新基础上的守正,才不会固步自封,才能与时俱进、推陈出新。C.12位非遗传人讲述了灯彩、核雕、苏绣等的前世今生,还有美轮美奂的哲思匠心。D.石头垒砌的房子,依山而建,鳞次栉比。沿河而上的石阶,斑斑驳驳,曲曲折折。【答案】C【解析】本题考查成语的辨析和使用。A.咄咄逼人:形容气势汹汹,盛气凌人,使人难堪;也指形势发展迅速,给人压力。修饰“战略打压”,符合语境;B.固步自封:比喻守着老一套,不求进步。与“与时俱进、推陈出新”相对,符合语境;C.美轮美奂:多用于形容建筑物雄伟壮观,富丽堂皇。此处用于形容“哲思匠心”,不符合语境;D.鳞次栉比:像鱼鳞和梳子的齿一样,一个挨着一个地排列着,多用来形容房屋等密集。此处用于修饰“房子”,符合语境;故选C。57.依次填入下面一段文字横线处的词语,最恰当的一项是( )每一次抵达,都意味着新的出发。奋进新______,大力弘扬北京冬奥精神,崇扬______卓越、______的精神境界,攻坚克难、砥砺前行,不弃微末、不舍寸功,积小胜为大胜,我们就一定能继续创造令人刮目相看的新的奇迹,不断赢得优势、赢得主动、赢得未来。A.旅程 追逐 精益求精B.旅程 追求 精雕细刻C.征程 追求 精益求精D.征程 追逐 精雕细刻【答案】C【解析】本题考查词语运用。旅程:旅行的路程。征程:远去的路程。空一:根据“奋进”“新”,可知应使用“征程”;追求:用积极的行动来争取达到某种目的或特指向异性求爱。追逐:指迅速积极地追寻逃跑的东西。空二,根据涉及对象“卓越”,可知应使用“追求”;精益求精:(学术、技术、作品、产品等)好了还求更好。精雕细刻:用刀在器物上精心细致地雕刻。比喻创作文学、艺术作品时十分认真、非常细致地加工刻画。也比喻做事认真细致。空三,根据所修饰的词语“精神境界”,可知应使用“精益求精”;故选C。58.下面一段话选自小明同学在百日誓师大会上的发言稿,其中加点词使用不恰当的一项是( )六月的辉煌属于谁?我想说,必定属于朝气蓬勃、青春年少的我们。因为每一位父母都强聒不舍,每一位老师都恪尽职守,我们每一位初三学子都在发愤图强。心中有信仰,脚下有力量!今日我们击缶起誓,明日我们凯歌而还!A.朝气蓬勃B.强聒不舍C.恪尽职守D.发愤图强【答案】B【解析】考查成语运用。A.朝气蓬勃:形容充满了生气和活力的样子。合乎语境,正确;B.强聒不舍:形容别人不愿意听,还絮絮叨叨说个不停。为贬义词,用在此处形容父母,情感色彩不符;C.恪尽职守:谨慎认真地做好本职工作。使用正确;D.发愤图强:下定决心,努力谋求强盛,使用正确;故选B。59.下列句子中加点词语运用有误的一项是( )A.11月20日,2022年卡塔尔世界杯开幕。从场馆、裁判、周边商品、赞助商到“最萌使者”大熊猫,“中国元素”如满天繁星,在这届世界杯大放异彩。B.面对美国的技术封锁,中国科学家白手起家。多少人殚精竭虑,青丝变白发,建成中国第一艘航母“辽宁舰”。C.2022年新冠疫情呈现多点爆发态势。各地方舱医院,医护人员和病患守望相助,共享天伦之乐,成为一道别样的风景。D.美国众议长佩洛西窜访台湾,一边公然制造“一中一台”假象,一边声称维护中国的统一,不过是玩了一出自欺欺人的把戏。【答案】C【解析】考查成语运用。A.大放异彩:闪耀着无比灿烂的光辉,散发出奇异的光彩,比喻有着优异的表现或是突出的成就,句中比喻“中国元素”像“满天繁星”大放异彩,十分贴切;B.白手起家:形容原来没有基础或条件很差而创立一番事业。句中形容中国航母发展历程,正确;C.天伦之乐:泛指家庭骨肉团聚的欢乐。句中形容医护人员和病患之间的关系,用错对象;D.自欺欺人:用自己都难以置信的话或手法来欺骗别人。符合佩洛西打着民主幌子,干涉中国内政之实的语境,正确;故选C。60.下列加点成语使用恰当的一项是( )A.学校开展“垃圾分类进校园”活动,他首当其冲,报名参加了学校“垃圾分类小分队”。B.“整本书阅读”受到广泛关注,这项教学活动在我市部分中小学也开展得绘声绘色。C.袁隆平常下到田间,前仆后继进行高产杂交水稻研究,是一位真正的耕耘者。D.在中国,很多农村居民可能不知道一些现代节日,但是对“二十四节气”却耳熟能详。【答案】D【解析】本题考查成语的正确使用能力。A.使用有误,首当其冲:比喻最先受到攻击或遭到灾难。该成语不合语境;B.使用有误,绘声绘色:形容叙述、描写生动逼真。本句用来形容活动开展得好,用错对象;C.使用有误,前仆后继:指前面的人倒下了,后面的人继续跟上去。英勇斗争,不怕牺牲。与句子语境不符,且用于袁隆平一人,使用不恰当;D.耳熟能详:听的次数多了,熟悉得能详尽地说出来。使用正确;故选D。61.下列句子中加点的词语使用错误的一项是( )A.在飞逝的时光里,我们看到的、感悟到的中国,是一个坚韧不拔、欣欣向荣的中国。B.在我们的日常生活、学习和工作中,必须要有坚持不懈、锲而不舍的精神。C.在植树节活动中,同学们妙手回春,为龙江大地多种树、多添绿,建设美丽家乡。D.雷锋的名字家喻户晓,雷锋的事迹深入人心。【答案】C【解析】本题考查成语运用。A.欣欣向荣:原形容草木生长得茂盛。现多比喻事业蓬勃发展,繁荣兴旺。句中用来形容中国的发展态势,符合语境;B.锲而不舍:不断地镂刻。比喻坚持不懈。句中用来形容我们在学习生活中应该有的精神状态,符合语境;C.妙手回春:形容医术高明,能把生命垂危的病人医治好。句中用来描写同学们的植树活动,使用对象不对;D.家喻户晓:家家户户都知道。形容尽人皆知。句中用来形容雷锋的名声很大;符合语境;故选C。62.下列句子中加点的词语使用不恰当的一项是( )A.孔繁森不仅告诫别人,也时时警醒自己,他在笔记本上写下箴言:“勤奋、廉洁、坦诚、开拓”。B.“严谨和自律”一直是方明恪守的准则,他极少出错,被业内人士奉为学习的楷模。C.不怨天尤人,不妄自菲薄,我们才能眼里有光,脚下有路,才会有进取的无限动力。D.小李做事十分严谨细致,平时大家不以为然的一些小细节,他总能注意到。【答案】D【解析】本题考查词义辨析。A.箴言:规谏告诫的话。“勤奋、廉洁、坦诚、开拓”就是规谏告诫的话,符合语境;B.恪守:谨慎而恭顺地遵守、严格遵守。“恪守准则”,使用恰当;C.妄自菲薄:过分地看轻自己。这里形容对自己的要求,符合语境;D.不以为然:指不认为是对的,表示不同意。不能形容不在意小细节,使用有误;此处应为“不以为意”;故选D。63.下列句子中加点成语使用恰当的一项是( )A.北宋王希孟的青绿山水画《千里江山图》气势磅礴、朴质典雅,色彩美轮美奂。B.拔河比赛激烈无比,选手拼尽全力,同学们的加油呐喊声如雷贯耳。C.小荔发言完毕,小夏说:“刚才小荔同学抛砖引玉,下面我也谈谈我的看法。”D.我们做事应认真分析可能出现的问题,既不要杞人忧天,也不要麻痹大意。【答案】D【解析】本题考查成语运用。A.美轮美奂:形容房屋高大华丽。选项说的是画,对象错误;B.如雷贯耳:形容响亮得像雷声传进耳朵里,形容人的名声大。选项说的是同学们的加油呐喊声,对象有误;C.抛砖引玉:比喻用自己不成熟的意见或作品引出别人更好的意见或好作品,是谦辞。本句敬辞和谦辞使用有误;D.杞人忧天:比喻缺乏根据和不必要的忧虑。使用正确;故选D。64.下列句子中加点的词语使用不恰当的一项是( )A.衔泥筑巢时,雌燕和雄燕常常形影不离,呢喃着只有彼此才能听得懂的情话。B.广州岭南印象园将岭南特色风光浓缩于此,园中秀丽的景色撩逗着游人的心。C.《为歌而赞》节目中,嘉宾多次引用诗句进行点评,画龙点睛,引发歌手和观众的共鸣。D.经过与李教授的一番长谈,我逐渐顿开茅塞,之前疑惑不解的地方全都明白了。【答案】D【解析】本题考查词语辨析。A.呢喃:意思是形容燕子的叫声 ;形容小声说话,轻声细语。使用正确;B.撩逗:意思是挑逗、挑动。使用正确;C.画龙点睛:多比喻写文章或讲话时,在关键处用几句话点明实质,使内容更加生动有力。使用正确;D.顿开茅塞:比喻忽然开窍,醒悟或明白了一个道理。此处与“逐渐”前后矛盾;故选D。65.下列各句中加点的成语使用恰当的一项是( )A.电视连续剧《狂飙》的演员把角色演绎得真实可信,栩栩如生,深受观众好评。B.在运动会筹备期间,他首当其冲,第一个报名参赛。C.演讲比赛中,选手们夸夸其谈,精彩的表现让观众赞叹。D.延安,曾经是中共中央的所在地,是“延安精神”的发源地,也是无数人魂牵梦萦的地方。【答案】D【解析】本题考查成语的理解与运用。A.栩栩如生:通常比喻画作、雕塑中的艺术形象生动逼真,像活的一样。不能用来形容演员,适用对象错误;B.首当其冲:比喻最先受到攻击或遇到灾难。与“在运动会筹备期间”“第一个报名参赛”的语境不符,使用不正确;C.夸夸其谈:指浮夸空泛地大发议论或写文章浮夸,不切合实际。句中用来描写演讲的选手的精彩表现,感情色彩不对;D.魂牵梦萦:形容思念情切。使用正确;故选D。66.下列句子中,加点词语使用恰当的一项是( )A.为了将课本剧《变色龙》中的角色演得入木三分,大家都全身心地投入到排练中。B.有些人凭空想了许多念头,络绎不绝地说了许多空话,可是从来没认真做过一件事。C.经过一段时间的重新装修,教学楼里里外外都改头换面,给了师生们一个惊喜。D.文言文至现代汉语再到翻译的过程,有一点点的误差,表达的含义就会大相径庭。【答案】D【解析】本题考查成语的理解与运用。A.入木三分:形容书法笔力刚劲有力,比喻对文章或事物见解深刻、透彻。不能用来形容表演的角色贴近作品,用错对象;B.络绎不绝:形容行人车马来来往往,接连不断。不能用来形容说了很多话,用错对象;C.改头换面:指人的容貌发生了变化,现在多比喻只改变外表和形式,内容实质不变。不能用来形容教学楼整修后的样子,使用有误;D.大相径庭:比喻相差很远,大不相同。符合语境。故选D。67.下列句子中,加点的成语使用无误的一项是( )A.队伍分散开去,摩肩接踵,前后长达一英里左右。B.但是大手笔只选择两三件事轻描淡写一下,境遇便完全呈露眼前,惟妙惟肖。C.写文章能文不加点,一气呵成的,一定是文字修养极高的人。D.只要你能身临其境地为父母想一想,你就会理解父母的心情,就不会对他们求全责备了。【答案】C【解析】本题考查成语运用。A.有误,摩肩接踵:意思是肩碰肩,脚碰脚,形容来往的人很多,很拥挤。本句使用与前面的“队伍分散开去”语境不符;B.有误,惟妙惟肖:形容描写或模仿得非常好,非常逼真。本句是表达境遇呈现像真实场景,不能用“惟妙惟肖”,可用“栩栩如生”;C.正确,文不加点:文章一气写成,不用涂改。形容文思敏捷,下笔成章。符合语境,指文学修养高的人会写文章。使用正确;D.有误,身临其境:亲自到了那个环境。语境是说要替父母的处境考虑,应该用“设身处地”。使用有误;故选C。68.下列句子中加点成语使用恰当的一项是( )A.人类面对城市交通拥堵现象,目前当务之急是采取节能减排措施。B.很多人喜欢将一些不刊之论发布到自媒体上,造成了极坏的社会影响。C.鲁迅先生是五四时期的先锋人物,其文章振聋发聩,促进了中华民族的觉醒。D.“此爱跨山海”的援藏教师陈洪顺,为教育事业悲天悯人,得到了全国人民的赞许。【答案】C【解析】本题考查词语使用。A.当务之急:指当前急切要办的事,与“目前”连用,语义重复,使用错误;B.不刊之论:比喻不能改动或不可磨灭的言论,用来形容文章或言辞的精准得当,无懈可击。不能形容不好的言论,望文生义,使用错误;C.振聋发聩:比喻用语言文字唤醒糊涂麻木的人,使他们清醒过来。鲁迅用文字唤醒中华民族的觉醒,符合语境,使用正确;D.悲天悯人:对社会的腐败和人民的疾苦感到悲愤和不平。不能形容用为教育事业尽心尽力援藏教师陈洪顺,的错对象,使用错误;故选C。69.下列句子中加点词语使用有误的一项是( )A.英雄烈士不容亵渎。以任何方式侵害英烈名誉的人,都应受到严惩。B.在志愿者的帮助下,这名失学儿童终于重返校园,与在校师生共享天伦之乐。C.他们两拨人的争吵越来越激烈,幸好老师及时赶到,从中斡旋,才得以缓和。D.如今,源远流长的传统文化正在被赋予新的生命力,焕发出勃勃生机。【答案】B【解析】考查词语运用。A.不容亵渎:不容许轻慢、不尊敬。正确;B.天伦之乐:泛指家庭的乐趣。不能用在师生之间,用错对象;C.斡旋:调解;扭转僵局。正确;D.赋予:指给予,交给,寄托(重大任务、使命等)。正确;故选B。70.下列各句中,加点成语使用不恰当的一项是( )A.这篇小说把非洲风景描写得形象逼真,读完使人有身临其境的感觉B.在这次学术研讨会上,王老师高谈阔论,积极发表自己的见解。C.这篇通讯展现了志愿军可歌可泣的英雄事迹,只有打动人心的力滑。D.春天,青岛中山公园的楼花开放了,灿若云霞,美不胜收【答案】B【解析】本题考查成语的使用。A.身临其境:亲自到了那个环境。此处形容小说把非洲风景描写得形象逼真,让人有亲自到了那个环境的感觉,使用正确;B.高谈阔论:漫无边际地大发议论(多含贬义)。此处形容王老师高在学术研讨会上发言,使用不当;C.可歌可泣:值得歌颂,使人感动得流泪,指悲壮的事迹使人非常感动。此处形容志愿军的英雄事迹使人非常感动,使用正确;D.美不胜收:美好的东西很多,一时看不过来。此处形容青岛中山公园的春景美好一时看不过来,使用正确;故选B。题型五:病句辨析与修改71.下面是小文同学摘录的部分句子,其中有语病的一项是( )A.诵读古代优秀诗歌,可以让我们的心灵得到滋润和净化,情感变得丰富。B.在经典文化的浸润下,使学校逐渐营造了善思善行、求实求是的好风气。C.十年来,我国重大创新成果竞相涌现,科技创新的体系化能力不断增强。D.各国只有风雨同舟、团结合作,才能书写构建人类命运共同体的新篇章。【答案】B【解析】考查病句的辨析。B.成分残缺缺主语。应删去“在……下”或“使”;故选B。72.下列句子没有语病的一项是( )A.线上教学是否有效,关键是要看教师突破传统教学的局限,学生改进学习方法。B.南极洲恐龙化石的发现,有力地证明地壳在进行缓慢但又不可抗拒的运动。C.袁隆平一生致力于杂交水稻技术的应用、推广与研究,为世界粮食供给做出了杰出贡献。D.每到暑假,学校都会反复向学生发出不要到陌生水域游泳,更不要在没有大人陪同的情况下到河、湾、塘、坝去游玩。【答案】B【解析】本题考查病句的辨析。A.两面对一面。应在“教师”和“学生”后加“能否”;C.语序不当。应将“应用、推广与研究”改成“研究、应用与推广”;D.成分残缺。应在“游泳”后加上“的倡议”;故选B。73.下面是张华同学在午间给同学们播报的一段关于“双减”的新闻,其中表述有语病的一句是( )①“双减”背景之下,学校和教师要深刻领悟国家政策。②首先是思想上“更新”,从实施素质教育的大局出发,切实减轻学生过重负担,以学生全面健康的学习生活为最终教育目的。③学校教育要努力使学生通过不断学习扩大知识面,看到纷繁美丽的大千世界,体味到成长中的快乐,④只有把学习融入到学生成长的过程之中,就能为学生的终身学习打下良好的基础。A.①B.②C.③D.④【答案】D【解析】考查病句修改。D.关联词使用不当,应为“只有……才”。故选D。74.下列句子没有语病的一项是( )A.中学生要提升文学素养,养成爱读书,尤其是读经典名著,让书香浸润心灵。B.一个人是否拥有健康的体魄,关键在于持之以恒、科学健康地参加体育锻炼C.《史记》被称为“百科全书式的通史”,主要是因为它是研究古代典章制度、人文历史、自然科学、经济文化的重要史料的原因。D.在青少年心中深植劳动的种子,有助于让他们懂得“伟大出自平凡,平凡造就伟大”的价值理念。【答案】D【解析】本题考查病句辨析。A.成分残缺,“养成……”缺宾语,在“名著”后面加上“的习惯”;B.两面对一面,“是否……关键在于持之以恒”两面对一面,可在“持之以恒”前加上“能否”;C.句式杂糅,“主要是因为……的原因”句式杂糅,可删掉“的原因”;故选D。75.下面选项中没有语病的一项是( )A.大数据的价值生成有其内在规律,只有深刻掌握并认识这些规律,才能提高科学运用大数据的能力。B.国庆假期,我省共接待游客大约2000多万人次左右,共实现旅游收入150多亿元。C.近年来,世界移动通信大会已成为中国制造商展示其最新成果的一个重要平台。D.在日益激烈的竞争中,能否具有良好的心态,审慎的态度,是一个人成功的关键。【答案】C【解析】本题考查病句的辨析与修改。A.语序不当,可将“掌握”与“认识”互换位置;B.表意重复,可将“大约”“多”“左右”删去其中的任意两个;D.两面对一面,可将“能否”删去;故选C。76.下列句子中没有语病的一项是( )A.北京冬奥会与冬残奥会的巨大成功,完美地向世界解答了新时代中国可信、可爱、可敬的形象。B.保护好、传承好、利用好民间艺术,对延续历史文脉、建设社会主义文化强国具有重要意义。C.我国废旧手机资源回收利用率仅4%左右,每年新增闲置手机进入正规回收领域的不足30%左右。D.广大青年是否崇德向善,不仅关乎人生道路走得正、走得远,更关乎整个社会风清气正、朝气蓬勃。【答案】B【解析】本题考查病句的辨析与修改。A.搭配不当。“解答了……形象”搭配有误,应改为“展示了新时代中国可信、可爱、可敬的形象”;C.表述矛盾。将“不足30%左右”改为“不足30%”;D.两面对一面。将“是否崇德向善”改为“崇德向善”。故选B。77.下面一段文字中有语病的一项是( )①每一本好书,都是黑暗中的一道亮光。②这一道道亮光,将给我们这一叶叶扁舟暗空下的引航,直至寻找到风平浪静且又万家灯火的港湾。③我们应有这样的古风:沐浴双手,然后捧卷。④在一番庄严肃穆的感觉之中,使你必将得到书的神谕。A.第①句B.第②句C.第③句D.第④句【答案】D【解析】本题考查语病辨识。第④句“在一番庄严肃穆的感觉之中,使你必将得到书的神谕”有语病,缺少主语,应删去“使”。故选D。78.下面的句子,没有语病的一项是( )A.元代,伴随着济州河的开凿和漕运的发展,特别是大运河,济宁建造了城池,形成了城市。B.初中生要学会制订自己的阅读计划,广泛阅读各种类型的读物,课外阅读总量不少于260余万字。C.推动构建人类命运共同体的不懈努力,见证着新时代中国“为人类谋进步、为世界谋大同”的担当与作为。D.航天英雄们在圆满地完成飞行任务后凯旋而归。【答案】C【解析】本题考查病句辨析。A.成分残缺,在“大运河”之后,加上“全线通航以后”;B.不合逻辑,“不少于”和“余”互相矛盾,应删掉“不少于”或“余”;D.语义重复,“凯旋”意为:打了胜仗归来;“归”也是回来的意思,“凯旋”和“归”语义重复,可删掉“而归”;故选C。79.下列语句中没有语病的一项是( )A.ATP250网球男单决赛中,吴易昺首进巡回赛决赛就夺冠,创造了中国的历史记录。B.截至2月12日,土耳其南部强震在土耳其和叙利亚造成超过2.8万人左右遇难。C.随着青海湖生态环境逐渐提高,一万余只赤麻鸭由候鸟逐渐变成当地的留鸟。D.通过广博的认知和开阔的视野,能让一个人看到事物本质,思考问题也会更深入。【答案】A【解析】考查病句的辨析。B.语意矛盾,去掉“超过”或“左右”;C.搭配不当,将“提高”改为“改善”;D.缺少主语,删去“通过”;故选A。80.下面一段文字中有语病的一项是( )①古典诗词在诞生伊始,就与音乐结下不解之缘。②《诗经》300余篇,几乎均可和乐歌唱。③这种“和诗以歌”的形式,从源头诗词所蕴含的韵律美、声调美、节奏美等。④与其说《经典咏流传》是对传统文化的创新发展,毋宁说,它是在追寻并努力重现千百年前“歌诗一家”的文化。A.第①句B.第②句C.第③句D.第④句【答案】C【解析】本题考查病句的辨析与修改。③处成分残缺。缺少谓语,应在“从源头”后加上“奠定了”;故选C。81.下面一段文字中有语病的一项是( )在今天,①拥有一部电脑就好比拥有整个世界。于是,有的人开始厌倦纸质图书。②对纸质图书的命运,不管怎样担忧,我都始终抱有希望,因为只要你想借助阅读享受快乐,这种方式就永远不会消亡。③而且,随着整个社会浮躁心态的改变,使传统阅读方式将会受到大众的钟爱。④目前,古典著作图书受到读者追捧,就是最好的印证。A.第①句B.第②句C.第③句D.第④句【答案】C【解析】本题考查病句辨析及修改。C.缺少主语,可删去“使”。故选C。82.下列句子有语病的一项是( )A.每年中考,考生在考场上专心答卷,而场外的老师和家长无时无刻都在焦急地等待。B.慈善晚会成功募得近千万善款,用于品学兼优但家境贫寒的学生的资助。C.实现教育现代化,需要建设一支品德高尚、业务精通的大批高素质教师队伍。D.传统中国对如何读书有过很多讨论,其中,朱熹的《朱子读书法》流传甚广,成为许多人读书指南。【答案】D【解析】考查句子语病辨析修改。A.用词不当,“无时无刻”没有时刻。常与否定词连用,指时时刻刻、随时的意思。将“都”改为“不”;B.偷换主语,可在“用于”前添加“这项善款”;C.搭配不当,应将“大批”删去;故选D。83.下列语句中没有语病的一项是( )A.科学社会主义在二十一世纪的中国焕发出新的蓬勃生机,中国式现代化为人类实现现代化提供了新的保障。B.中国共产党第二十次全国代表大会报告指出:推进马克思主义中国化时代化是一个追求真理、揭示真理、笃行真理的过程。C.体育应该是基础教育不可分割的重要组成部分,体育教育必须坚决贯彻落实,体育强国不能不只成为一个口号。D.2022年11月3日,“梦天”在轨完成与“天和”“问天”的组装,形成“T”字基本构型。建成国家太空实验室的历史时刻,如今已近在眼前。【答案】B【解析】考查病句辨析。A.搭配不当,应将“保障”改为“选择”;C.否定错误,“不能不”改为“不能”;D.“如今已”与“近在眼前”矛盾。可删掉“,如今已”;故选B。84.下列句子中没有语病的一项是( )A.党的十八大以来,中国接入互联网的学校已接近100%左右。B.昼上超然楼,碧波荡漾的大明湖与风光旖旎的鹊华山尽收眼底。C.为了避免校园安全事故不再发生,我市中小学都加大了校园安全工作力度。D.驻青高校筹划、开展的“我为峰会添光彩”,得到了广大师生的热烈响应。【答案】B【解析】本题考查病句辨析。A.重复累赘,“接近”“左右”去掉其一;C.否定不当,去掉“不”;D.成分残缺,在“我为峰会添光彩”后添加“活动”;故选B。85.下列句子没有语病的一项是( )A.中国不仅是“一带一路”建设的倡议者,更是负责任的参与者、有担当的行动者。B.中华民族之所以成为伟大的民族,靠的是悠久的历史文明造就的。C.“世界园艺博览会”5月在中国北京举行,每天参观的人数超过10万左右。D.一个班级能否形成良好的班级文化,对学生的健康成长起着至关重要的作用。【答案】A【解析】本题考查病句辨析。B.句式杂糅,去掉“靠的”或“造就的”;C.前后矛盾,去掉“超过”或“左右”;D.两面对一面,去掉“能否”;故选A。86.下列说法正确的一项是( )A.“对于XBB1.5毒株,我们切忌不要恐慌,也不要轻视,保持良好心态等健康生活方式。”这句话没有语病。B.“再高的山也挡不住,再深的谷也堵不住,再危险的道也敢去涉足。”这句是假设关系的复句。C.古代文人好以居士命名,如白居易号香山居士,欧阳修号易安居士,李白号青莲居士。D.我国古代许多词语具有特定的代称,如“布衣”指平民,“汗青”指史册,“社稷”指国家。【答案】D【解析】考查语法知识、病句修改、文学和文化常识。A.本句有语病,“切忌”与“不要”语义重复,去掉“切忌”;B.有误。这是一个并列复句;C.有误。欧阳修号六一居士,李清照号易安居士;故选D。87.下面选项中没有语病的一项是( )A.大数据的价值生成有其内在规律,只有深刻掌握并认识这些规律,才能提高科学运用大数据的能力。B.国庆假期,我省共接待游客2000多万人次左右,共实现旅游收入150多亿元。C.近年来,世界移动通信大会已成为中国制造商展示其最新成果的一个重要平台。D.在日益激烈的竞争中,能否具有良好的心态,审慎的态度,是一个人成功的关键。【答案】C【解析】本题考查病句辨析及修改。A.语序不当,将“认识”与“掌握”互换位置;B.语义矛盾,删去“多”或“左右”;D.两面对一面,可在“一个人”的后面加上“能否”;故选C。88.下列句子没有语病的一项是( )A.“二十四节气”具有深远的文化意义,它体现了中国人尊重自然的理念。B.这部电影开创了中国科幻电影新纪元,填补了中国“硬科幻”电影。C.只有意识到这一点,你就能深刻地了解我们战士的胸怀是多么宽广。D.近年,经三路老商埠成为网红打卡地,是因为它承载着济南人记忆的原因。【答案】A【解析】考查病句的辨析。B.成分残缺,在句尾加上“的空白”;C.关联词语搭配不当,应把“就”改为“才”;D.句式杂糅,删去“原因”,可将“是因为它承载着济南人记忆的原因”改为“这是因为它承载着济南人的记忆”;故选A。89.下列句子表达正确,没有语病的一项是( )A.通过观看央视热播节目《典籍里的中国》,让我深深感受到中华优秀文化典籍的魅力。B.学校组织消防逃生演练活动,目的是为了提高学生的消防安全意识和自救能力。C.春天到来,植物园里的游客络绎不绝,聆听着大自然的鸟语花香,心情无比舒畅。D.国家能源局消息,到2月份,全国太阳能发电装机容量达4.1亿千瓦,同比增长30.8%。【答案】D【解析】本题考查辨析并修改病句的能力。A. 缺少主语,去掉“让”;B. 句式杂糅,去掉“目的”或“为了”;C. 搭配不当,“花香”不能“聆听”,可将“聆听”改为“享受”;故选D。90.下列句子没有语病的一项是( )A.事实证明,网络谣言的泛滥和滋生,会使许多人成为受害者。B.我国渤海大型油气田原油探明地质储量超过一亿吨,对于能否保障国家能源安全有着重要意义。C.全球疫情蔓延,中国不仅做好自身防控,还研制疫苗为世界防疫做出了重要贡献。D.为了提高大家阅读的兴趣,我校文学社开展了一系列的名著阅读和主题诗歌朗诵。【答案】C【解析】本题考查病句辨析。A.语序不当,将“泛滥”和“滋生”调换位置;B.一面对两面,去掉“能否”;D.成分残缺,在句末加“活动”;故选C。91.下面病句修改不正确的一项是( )A.随着新能源汽车商业保险专属条款的正式实践,不少新能源汽车的保费出现了大幅提升的现象。(将“实践”改为“实施”)B.语文学习不仅要发挥听说读写的兴趣,还要养成良好的读书习惯。(将“读书”与“良好的”互换位置。)C.计算机汉字字体应用场景日益丰富,在较好满足社会多样化用字需求的同时,也出现了部分字体使用场合不当、质量水平较低。(在句末加上“的问题”)D.政府部门要加大法治理论研究和宣传力度,引导广大群众遇事找法、自觉守法、解决问题用法。(将“遇事找法”和“自觉守法”互换位置)【答案】B【解析】本题考查语病辨识与修改。B.修改不正确。“语文学习不仅要发挥听说读写的兴趣,还要养成良好的读书习惯”语病是“发挥”与“兴趣”动宾搭配不当,应将“不仅要发挥听说读写的兴趣”改为“不仅要提高听说读写的能力”。故选B。92.对病句的修改不正确的一项是( )A.一个热爱读书的人,即使不能拥有舒适的书房,也会一直始终保持纯洁的读书情感。去掉“一直”。B.市卫生监督部门加大了对市场上牛肉的抽样检测,防止不合格肉制品重现白姓餐桌。去掉“不”。C.中国新一代隐形战斗机试飞成功的消息一经披露,广泛引起了全世界的关注。“广泛”移至“关注”前。D.消费者通过网络平台购买商品,其合法权益受到损害的,可以向销售者要求偿还。“偿还”改为“赔偿”。【答案】B【解析】本题考查病句修改。B.成分残缺,应该在“检测”之后加上“力度”,而不是去掉“不”,选项修改不正确。故选B。93.下列对病句的修改不正确的一项是( )A.深中通道全长约24公里左右,是集“桥、岛、隧、水下互通”于一体的大湾区超级工程,预计于2024年建成通车。(把“约”或“左右”去掉)B.粤剧《将军令》大量采用蔡李佛的武打技巧并运用南北武戏的精髓,使高水准的武戏成为该剧的一大亮点。(把“运用”改为“吸收”)C.饮食结构和运动量不足会引起严重的健康问题,因此学校应加强学生的健康管理,确保学生“吃动平衡”。(在“饮食结构”前加上“由于”)D.课本剧演出之前,我们要分配角色,选择剧本,合作排练。(将“分配角色”和“选择剧本”互换位置)【答案】C【解析】本题考查病句修改。C.主谓搭配不当,应在“饮食结构”后加上“不合理”;故选C。94.下面病句修改不正确的一项是( )A.注重平时的身体锻炼,决定体育测试能否考出好成绩。(删除“否”)B.加强校园治安管理,防止校园欺凌事件不再发生。(删除“不”)C.这次会议上,针对如何提高学习效率的问题,大家交换了广泛的意见。(“广泛的”移到“交换”前)D.为了优化育人环境,提升办学水平,学校加快了校园环境改造的速度和规模。(“加快”改成“扩大”)【答案】D【解析】本题考查病句的辨析与修改。D.病因是“搭配不当”。“加快”不能与“规模”搭配,故可将“和规模”删去或者将最后一句改为“学校加快了校园环境改造的速度,扩大了校园环境改造的规模。”故选D。95.下列对病句的修改不正确的一项是( )A.从某种意义上来说,做好奥运遗产的继承和转化,也是衡量一届奥运会成功与否的重要标志之一。(在“做好”前加上“能否”)B.各高校纷纷推出别具“心意”的文创产品,不仅能增进同学们对学校的了解,也能激发众多莘莘学子求知的热情。(把“热情”改为“兴趣”)C.汉字是意美、形美、音美“三美”兼具的文字,也是世界上少数几种兼具实用价值和审美价值、可以构成书法艺术。(在句末加上“的字体之一”)D.湛江雷剧与雷州半岛人民的生活习俗有着千丝万缕的联系,为研究、了解雷州半岛政治、经济、文化的发展提供了宝贵的资料。(“研究”与“了解”互换位置)【答案】B【解析】本题考查病句辨析及修改。B.成分赘余,“众多”“莘莘”任删其一。故选B。96.对病句的修改不正确的一项是( )A.2023年2月,中国新能源汽车产销约93.3万辆左右。去掉“约”或者“左右”。B.在日常生活中,班级干部应该充分发挥先锋模范传统。“传统”改为“作用”C.在复习过程中,我们要努力改正并随时发现自己的弱点。将“努力”与“随时”调换位置。D.经过大家共同努力,使我们出色地完成了任务。删掉“使”。【答案】C【解析】本题考查病句的辨析与修改。C.修改不正确。“努力改正并随时发现”语序不当,可将“努力改正”与“随时发现”调换位置。故选C。97.对下列病句修改不正确的一项是( )A.实现中华民族伟大复兴就是最伟大中华民族近代以来的梦想。将“最伟大”移至“以来”后面。B.看了电视剧《狂飙》,引发了观众对剧中人物和剧情的热议。删去“看了”。C.这次会议制定每一位发言者的发言时间最多不能超过30分钟。将“制定”改为“规定”。D.推进素质教育是保证青少年健康的条件之一。在“健康”的后面加“成长”。【答案】C【解析】本题考查修改病句。C.语义重复;删去“最多”或“不能超过”;故选C。98.对病句的修改不正确的一项是( )A.伴随阵阵大风,使北方多个城市笼罩在黄沙中。(将“使”删去)B.2023年度载人航天飞行任务标识召集活动落下帷幕。(将“召集”改为“征集”)C.具备顽强的毅力,是一个人在事业上能否取得成功的关键。(删去“能否”)D.近日,首家全国“中国移动One NET 台测试认证中心”正式落地西安。(将“落地”改为“安家”)【答案】D【解析】考查病句修改。D.“首家全国”语序不当,应为:全国首家。故选D。99.下列对病句的修改不正确的一项是( )A.2022年北京冬奥会上,经过中国小将谷爱凌奋力拼搏,终于获得自由式滑雪女子大跳台金牌。(删去“经过”)B.近段时间以来,全国各地开展广泛“喜迎二十大、永远跟党走、奋进新征程”等主题教育活动。(将“开展”和“广泛”调换位置)C.实施新修订的《义务教育课程方案和课程标准》,对增强义务教育高质量发展、全而建设社会主义现代化强国具有重要意义。(将“增强”改为“加强”)D.为激发同学们的阅读兴趣,我校语文组开展了一系列名著阅读和主题诗歌朗诵。(在句末加“的活动”)【答案】C【解析】本题考查病句的辨析及修改。常见病句类型有搭配不当,用词不当、成分残缺、语序不当、句式杂糅、前后矛盾不合事理、重复多余等。C有误,把“增强……发展”搭配不当,因此“增强”改为“推动”;故选C。100.选出对下列病句修改有误的一项是( )A.由于这所大学忽视理论的应用,使该大学输出的有用的科学技术人才逐年减少。(修改:删掉“由于”或“使”)B.一个人工作能力的高低,不在于他掌握了多少知识,关键是看他做出了突出的成绩。(修改:“看他”后面加“是否”)C.董卿在主持《朗读者》节目时,旁征博引,妙语连珠,令广大观众目不暇接。(修改:“旁征博引”和“妙语连珠”互换)D.许多超市将个头大的西瓜切成小块出售,一块西瓜不超过5块钱左右,非常划算。(修改:“不超过”和“左右”去掉一个)【答案】C【解析】考查病句修改。C.“目不暇接”不合语境,应改为“赞不绝口”。故选C。二、语言表达与应用 题型一:句子衔接与排序1.填入下面一段文字横线处的语句,最恰当的一句是( )近期,改编自同名小说的《人世间》《风起陇西》等影视剧取得了很好的口碑。此类作品在改编时注重彰显时代气质,在视听呈现、情节叙述和价值表达上更为精进,能够更好地满足当下观众的观剧品味和诉求,全方位提升了影视创作的水准,__________。A.凸显出国产剧精品化的创作趋势B.国产剧精细化创作趋势日渐凸显C.国产剧凸显出精品化的创作趋势D.凸显出国产剧创作的精细化趋势【答案】A【解析】本题考查句子内容和句式的连续性和一致性。横线前的句子“全方位提升了影视创作的水准”中“全方位”包含着下一句内容,下一句的开头词语要与“提升了”相一致,所以下一句的句首要用“凸显出”;“此类作品在改编时注重彰显时代气质,在视听呈现、情节叙述和价值表达上更为精进”是对国产剧的精品化创作趋势提出的要求;同时“提出”和“凸显”都是动词,“凸显出国产剧精品化的创作趋势”与“提升了影视创作的水准”句式相同;故选A。2.小红准备分享作文立意小技巧,横线处的语句,链接最恰当的组是( )作文立意要明确,表达的思想要有深度。同时,还要求新,避免思维定式,人云亦云。____________,____________,____________,____________。①深入发掘题目的内涵②甚至从常见观点的对立角度反向立意③都可以使立意新颖④变换不同的角度思考问题A.①④②③B.①③④②C.④①③②D.②①④③【答案】A【解析】本题考查语言衔接的能力。由前文“表达的思想要有深度”可知,这个立意需要深刻,那么就要往深处去思考,所以应该放在第一处的是“①”,正好和前文这句话形成照应;由前文“还要求新,避免思维定式”可知,立意应该需要从不同的角度切入,与之照应的应该是“④”;由前文“避免思维定式,人云亦云”可知立意也不能观点陈旧,随大流,最好和平常观点不一样,所以与之照应的是“②”;最后,第③“都可以使立意新颖”可以作为总结放在最后。故选A。3.依次填入下面文段横线处的语句,衔接最恰当的一项是( )读书并不在多,最重要的是选得精,读得彻底。与其读十部无关轻重的书,不如以读十部书的时间和精力去读一部真正值得读的书;与其十部书都只能泛览一遍,不如取一部书精读十遍。“______”,这两句诗值得每个读书人悬为座右铭。买书原为自己受用,多读不能算是荣誉,少读也不能算是羞耻。少读如果彻底,必能养成深思熟虑的习惯,涵泳优游,以至于变化气质;多读而不求甚解,______。①读书破万卷,下笔如有神②好书不厌百回读,熟读深思子自知③则如驰骋十里洋场,虽珍奇满目,徒惹得心花意乱,空手而归④虽珍奇满目,徒惹得心花意乱,则如驰骋十里洋场,空手而归A.①③B.①④C.②③D.②④【答案】C【解析】这段文字主要表明读书要选得精,读得彻底。根据第一空前的句子“与其读十部无关轻重的书,不如以读十部书的时间和精力去读一部真正值得读的书;与其十部书都只能泛览一遍,不如取一部书精读十遍”可知,该句强调的是一部书应多读、精读。根据第一空后面的“这两句诗”可知,所填的句子是两句诗。据此可知应填句②。根据第二空前的“多读而不求甚解”可知,该空要填的句子应是阐述什么是“多读而不求甚解”。句③运用比喻的手法闹释了什么是多读而不求甚解。应填句③。故选C。4.下面语段横线处依次填入的语句,衔接最恰当的一项是( )亲爱的朋友们,当你坐上早晨第一列电车驰向工厂的时候,当你扛上犁耙走向田野的时候,当你喝完一杯豆浆提着书包走向学校的时候,当你坐到办公桌前开始这一天工作的时候,当你往孩子口里塞苹果的时候,当你和爱人一起散步的时候……______朋友!你是这么爱我们的祖国,爱我们的伟大领袖毛主席,你一定会深深地爱我们的战士——他们确实是我们最可爱的人!(节选自魏巍《谁是最可爱的人》)①请你意识到这是一种幸福吧②你也许很惊讶地说:“这是很平常的呀!”③因为只有你意识到这一点,你才能更深刻地了解我们的战士在朝鲜奋不顾身的原因④可是,从朝鲜归来的人,会知道你正生活在幸福中⑤朋友,你是否意识到你是在幸福之中呢A.①③⑤④②B.⑤①②④③C.①③⑤②④D.⑤②④①③【答案】D【解析】考查句子衔接的能力。阅读文段内容可知,文段中给出的六句排比句都是我们日常生活中常见的情景,所以我们可能不会把这样的场景和幸福联系在一起,因此第一句应该是⑤句,对于这样普通的情景提出问题,接着②句承接⑤句,对这样的观点表示惊讶,④句一个“可是”表示转折,转入朝鲜归来的人对这些情景的感触,①句承接④句呼吁我们将会在也这样平常的事情看作是幸福的事情,最后③句表明原因。因此正确的排序为⑤②④①③;故选D。5.依次填入下面文段横线处的语句,符合语境的一项是( )毕业慢悠悠地到来,又急匆匆地溜走。在期盼未来生活的同时,也要好好跟自己的这一段青春告别:再走一走______,摸一摸______,抱一抱______,和大家好好说再见,然后奔赴你的下一段旅程。①陪伴你三年的黑板桌椅②朝夕相处的老师和同学③校园里每天必经的小路A.①②③B.②①③C.③①②D.③②①【答案】C【解析】此题考查语句的衔接。根据每个空前的动词,填出对应的事物即可。第一空“走”对应“小路”,选③;第二空“摸”对应“桌椅”,选①;第三空“抱”对应“老师和同学”,选②。故选C。6.填写在横线上的句子,衔接最恰当的一项是( )在古代,待客宴饮,并不是等仆从将酒肴摆满就完事了,主人还要作引导,作陪伴,主客必须共餐。 。少者吃饭时还得小口小口地吃,而且要快些咽下去,随时要准备回复长者的问话,谨防发生喷饭的事。①长者表示不必如此,少者才返还入座而饮。②侍食年长位尊的人,少者还得记住要先吃几口饭,谓之“尝饭”。③尤其是老幼尊卑共席,那礼节就多了。④虽先尝食,却又不得自己先吃饱完事,必得等尊长者吃饱后才能放下碗筷。⑤陪伴长者饮酒时,酌酒时须起立,离开座席面向长者拜而受之。A.②④⑤①③B.②⑤①③④C.③②④⑤①D.③⑤①②④【答案】D【解析】本题考查句子衔接,要求学生根据句意及句子间的相互关系对句子进行排序,考查分析理解的综合能力,从题干和所给句子可知,接下去所写内容为老幼尊卑共席时的礼节,故③句放在最前面,统领下文,⑤句和①句均写饮酒时礼仪且句意上有承接关系,故顺序为⑤①,②④句讲“尝饭”,根据句意④句承接②句故顺序为②④,根据②句中“还得”及题干末句句意可知本题顺序为③⑤①②④,故选D。7.在下边横线处依次填入语句最恰当的一项是( )面对地图,就是面对五色焕然的土地。而实实在在的土地,其颜色要比地图上的色彩丰富得多,复杂得多。在喜马拉雅山上,土地是银色的,________;在西部广阔的沙漠里,土地是苍黄的,________;在东北广袤的原野中,土地是黑色的,________;在南方的丛山丘陵中,土地是绿色的,________;在坦荡无眼的平原里,土地是金色的,现代神话正在天天演绎;在革命老区,土地是红色的,培育了伟大的民族精神。①留下无穷的关于生命的思索②把丰收和喜悦送给人间③呈现白皑皑的景观④孕育着无限生机和希望A.③①②④B.①②④③C.③①④②D.①③④②【答案】A【解析】本题考查句子排序。结合“土地是银色的”可知,银色就是白色,喜马拉雅山上的白色是雪的颜色,故选③;结合“在西部广阔的沙漠里,土地是苍黄的”可知,沙漠几乎看不见生命,是荒凉的,引发人的思索,故选①;结合“土地是黑色的”可知,黑土地是肥沃的土地,带来丰收,故选②;结合“在南方的丛山丘陵中,土地是绿色的”可知,绿色象征希望和生机,故选④;故选A。8.下列语段空白处填入四个句子,最恰当的一项是( )在春日踏青,青芜如毯,_________;夏日听雨,雨声淅沥,_________;秋日看花,花叶相辉,_________;冬日观雪,雪意阑珊,_________。好诗如四季,岁岁不相同。①诗是千缕缤纷下那一丝虚幻的朦胧 ②诗是玉田琼屋上那一份惊艳的洁白③诗是绿茵遍野时那一抹久违的清新 ④诗是红叶清风里那一派无尽的潇洒A.③④②①B.③①④②C.④③①②D.④②③①【答案】B【解析】本题考查语句的衔接。按照对应法,第一、二、三、四空分别描述春草、夏雨、秋叶、冬雪四个季节的四种景物,结合①句中的“千缕缤纷”,②句中的“惊艳的洁白”,③句中的“绿茵遍野”,④句中的“红叶清风”这几个关键词,即可对应。故依次是③①④②。故选B。9.结合语境,填入下面横线处最恰当的一项是( )合作,没有重量,却________,为了诺言,你甘愿劳苦奔波;合作,没有体积,却_________,彼此守护着一片真诚的热土;合作,没有色彩,__________,吹醒桃红柳绿的绚丽世界。A.载着沉甸甸的承诺 占据着两个人的心 却让你快乐如风B.载着沉甸甸的承诺 却让你快乐如风 占据着两个人的心C.占据着两个人的心 载着沉甸甸的承诺 却让你快乐如风D.却让你快乐如风 载着沉甸甸的承诺 占据着两个人的心【答案】A【解析】第一空:“重量”与“沉甸甸”相对。故填:载着沉甸甸的承诺。第二空:“体积”与“占据”相对。故填:占据着两个人的心。第三空:“吹醒”与“风”相对。故填:却让你快乐如风。故选A。10.依次填入下面文字空白处的语句,衔接恰当的一项是( )赛场上, ;赛事中, ;赛场外, 。共燃冰雪梦,激情向未来,这是中国人民与世界人民携手同行、共创未来的美好期许,是少年奥运健儿初心使命深层次的你现,向世界展现中国青年的气质。①超3亿人携手奥运五环,让原本有些“高冷”的冰雪运动大众化、全民化②以苏翊鸣为代表的运动员赛出了青春的力量③众多00后奥运志愿者,在场馆管理赛事服务颁奖礼仪等领城施展才能,履行自己的“冰雪之约”A.③②①B.②③①C.③①②D.②①③【答案】B【解析】本题考查语句衔接。第②句中的“运动员”对应“赛场上”,第③句的“志愿者”对应“赛事中”,第①句的“超3亿人”对应“赛场外”;故分别填入②③①;故选B。11.下列句子排序正确的一项是( )①树树海棠,枝枝红梅,在雨中绽开或红或粉的笑颜,细雨滋润后的锦官城,穿越千年光阴而来,将春的信息写在枝头、湖畔,似乎在向踏春者招手。②亭南,鹭雁飞掠,鸳鸯戏水,一壶清甜的暖茶入胃,即便不懂诗的人们,也会一不留神滑入那些瑰丽多姿的唐诗意境。③春日的雨,笼盖住湖边的桃红柳绿,用温情的方式催促它们成长,让这些花花草草在雨中更加青翠欲滴。④邀上三五好友,在湖畔寻一茶亭坐下,梨花枝头白,树下盖碗绿,真乃人生一大惬意之事。⑤在成都赏雨,真有诗和远方的感悟。A.②⑤③①④B.③②①④⑤C.③①④②⑤D.②④③⑤①【答案】C【解析】考查句子排序的能力。阅读语段内容可知,该语段主要描写的是春雨的场景,所以③句应为首句,接着①句对于雨后不同景物的描写承接③句中“让这些花花草草在雨中更加清脆欲滴”的内容,然后④句“邀上三五好友”的内容承接①句中“似乎在向踏春者招手”的内容,②句“亭南,鹭雁飞掠,鸳鸯戏水,一壶清甜的暖茶入胃”描写的是亭中喝茶的场景,承接④句“邀上三五好友,在湖畔寻一茶亭坐下”的内容,最后⑤句做总结,所以正确的排序为③①④②⑤;故选C。12.下面是南沙同学“读书的乐趣”的演讲提纲,排列顺序正确的一项是( )①为什么读书?读书就是在乐趣之中求真善美。②读书是我们选择的一种生活方式,让我们在阅读中沉淀,在书香中成长。③读书可以不断启蒙自己,更新自己,是之谓“求真”。④读书的乐趣发自内心,读书可以陶冶性情,是之谓“求美”。⑤读书,对我来说是一种享受。我所说的读书,是读怎样的书。⑥读书是为了安身立命,是为了不断的精神追求,是之谓“求善”。A.①⑤④③⑥②B.⑤①③⑥④②C.②⑤④⑥③①D.①②⑥④③⑤【答案】B【解析】考查对内容的排序。这是“读书的乐趣”的演讲提纲,首先⑤中的“我所说的读书,是读怎样的书”引出演讲的话题,可放在第一句;①承接提出“为什么读书”的问题并从“真善美”三个方面概括;然后③⑥④是分别从“真善美”三个方面依次谈读书的好处;最后②是总结。由此可知,正确安排顺序是:⑤①③⑥④②;故选B。13.下列语句排序正确的一项是( )①如果说中国传统文化阻碍了中国古代科技的发展,那么中国古代科学技术的辉煌成就从何得来?②科学是理论化、系统化的知识体系,是人类对自然、社会和自身的本质和规律性的认识活动和实践活动。③事实上,中国传统文化早在两千多年前的春秋战国时期,已经有了文化和科技教育融合的影子。④科学还是一种文化,科学文化理所当然属于先进文化。⑤儒学的开门祖师孔子,他对学生进行礼、乐、书、数、御、射“六艺”教育。其中数即数学,乐和声学有关,御和力学有关,射和机械有关。A.①②④⑤③B.②①④⑤③C.①④②③⑤D.②④①③⑤【答案】D【解析】本题考查语言的衔接、排序。解答时通读全句,理解文字的主要内容。本段文字主要讲述科学文化,所以由②“科学是理论化、系统化的知识体系”可知,②为首句;由④“科学还是一种文化”可知,④在②之后;由①“如果说中国传统文化阻碍了中国古代科技的发展”提到了“中国传统文化”对“中国古代科技”的阻碍影响,这是④“科学还是一种文化,科学文化理所当然属于先进文化”的转折,所以,①在④之后;③“事实上,中国传统文化早在两千多年前的春秋战国时期,已经有了文化和科技教育融合的影子。”可知这在阐述中国古代科学技术的由来,所以,③在①之后;⑤“儒学的开门祖师孔子,他对学生进行礼、乐、书、数、御、射“六艺”教育。其中数即数学,乐和声学有关,御和力学有关,射和机械有关。”这是中国古代科学技术的由来的具体阐述,所以,⑤在③之后;由此可知,顺序是②④①③⑤;故选D。14.2016年,“二十四节气”列入联合国教科文组织《人类非物质文化遗产代表作名录》;2022年2月4日,立春,第二十四届北京冬奥会开幕式上,二十四节气倒计时惊艳世人。考场里的你知道吗?几天前,2022年6月21日,是二十四节气中的夏至。下列介绍“夏至”的语句排序正确的一项是( )①夏至后,气温逐渐升高,过两三个星期,炎炎盛夏才真正来临。②在二十四节气中,夏至是第十个节气,在每年6月21日或22日。③这是一年中北半球白天最长、夜晚最短的一天。此后,白昼渐短,黑夜渐长。④“夏至”的“至”,是“极,到达极点”的意思,是指这一天太阳几乎直射北回归线。⑤吃过夏至面,虽然意味着夏季开始了,但夏至还不是一年中最热的日子。⑥所以民间会有“吃过夏至面,一天短一线”的说法。⑦二十四节气是中国传统文化的重要组成部分,被称为“时间里的中国智慧”。A.④⑦②⑤③⑥①B.⑦②④③⑥⑤①C.⑦②④⑥⑤③①D.④⑦③②⑤⑥①【答案】B【解析】本题考查句子排序。⑦“二十四节气是中国传统文化的重要组成部分,被称为‘时间里的中国智慧’”,总说二十四节气的意义,是首句;②“在二十四节气中,夏至是第十个节气,在每年6月21日或22日”,说明夏至的时间,是次句;④“‘夏至’的‘至’,是‘极,到达极点”的意思,是指这一天太阳几乎直射北回归线”,解释“夏至”的命名缘由,是第三句;③“这是一年中北半球白天最长、夜晚最短的一天。此后,白昼渐短,黑夜渐长”紧随第④句,是“太阳直射北回归线”的结果,是第四句;⑥“所以民间会有‘吃过夏至面,一天短一线’的说法”,是对第③句的解释说明,是第五句;⑤“吃过夏至面……”紧承第⑥句,是第六句;①“夏至后,气温逐渐升高,过两三个星期,炎炎盛夏才真正来临”承接第⑥句句中“夏至还不是一年中最热的日子”进行说明,是最末句;因此正确排序为:⑦②④③⑥⑤①;故选B。15.将下面句子组成语意连贯的一段话,顺序排列最恰当的一项是( )①从战国到魏晋长达八百年的岁月里,中国人主要用竹简写字、刻字、著书立说。②中国先民不仅编竹为筏,还将竹片作为文字的载体。③孔子因勤于读书,把牛皮绳多次翻断,于是有了“韦编三绝”的佳话。④儒家经典很多写在竹简上。⑤人们先砍竹削片,晒干后钻孔,再用牛皮绳串起来编结成书,就是所谓的“韦编”。⑥由于竹简的利用,中国文字记载的经典得以保存传世,可以说竹子为中国文化的发展及历史文献的传播立下了汗马功劳。A.⑥④①②③⑤B.②⑥④⑤③①C.⑥②①④⑤③D.②⑤③①④⑥【答案】D【解析】考查排序。按时间为序。根据②句中的“中国先民”,可知应为首句。⑤句中的“人们先砍竹削片”紧承②句中的“将竹片作为文字的载体”,应为第二句。③句中的“把牛皮绳多次翻断”紧承⑤句中的“用牛皮绳串起来编结成书”,应为第三句。根据①句中的“从战国到魏晋”,可知应为第四句。④句中的“竹简”紧承①句中的“中国人主要用竹简写字、刻字、著书立说”,应为第五句。⑥为末句,总结了竹简的意义。故排序为:②⑤③①④⑥。故选D。16.将下列句子组成语意连贯的一段话,排序最恰当的一项是( )①同样一句饱含真理的话语,不同的人讲出来,会因人格不同,产生大不相同的结果。②文天祥抗元被俘,正气浩然,留下了“人生自古谁无死,留取丹心照汗青”的名句。③由于缺少人格作支撑,再漂亮的话语也最终成了污人耳目、遭人嘲讽的沉渣泡沫。④这句话的力量,“至大至刚”,流布宇内,穿越千古,至今读来,仍令人振奋。⑤而那些“两面人”,台上巧舌如簧、信誓旦旦,台下却任性贪婪、劣迹斑斑。A.④③②⑤①B.②⑤④③①C.①②④⑤③D.①⑤③②④【答案】C【解析】①句是对语段的综述,表明不同的人讲同样包含真理的话语,会产生不相同的结果。其他四句对①句进行了论证,所以①句应该在最前面。②句是从正面进行举例,④指出文天祥这句话给人以振奋的力量,所以④句应该在②句后,对①句进行正面论证。⑤句的“而”表示转折,即从反面进行举例,所以⑤句应该在④句后。③句指出“两面人”话语最终成了污人耳目、遭人嘲讽的原因,与⑤句一起从反面对①句进行论证,所以③句应该在⑤句后。正确的顺序应为①②④⑤③。故选C。17.将下面句子组成语意连贯的一段话,排序最恰当的一项是( )①你会发现这些“梗”既勾勒出社会发展的大事小情,②是表达赞美敬意的刷屏词“yyds”,还是袒露内心触动的惯用语“破防了?”③也蕴藏着人们对日常生活的直观体察,还折射出社会语言文化的最新变化。④你使用最为频繁的流行“梗”是哪一个?⑤不可否认的是,以“梗”对话已成为互联网时代一道独特的“景观”。A.④②⑤①③B.⑤④②①③C.④②①③⑤D.⑤①③④②【答案】A【解析】本题考查排序。纵观整个句群,是对互联网时代以“梗”对话的看法。④句首先询问你使用最为频繁的流行“梗”,引出要讨论的话题,应排首位;②句揣摩的语气回答④句的疑问,应排第二位;⑤句说明“梗”对话现象已经成为“景观”,不可忽视(或否认);①句承⑤句指出“梗”已分布生活生活的大小事情,是比较普遍的现象;③句点明这一现象的实质,应排最后。因此,排序为④②⑤①③。故选A。18.将下列句子组成语段,顺序排列合理的是( )①人文景观,其布局重点从泰城西南祭地的社首山、蒿里山至告天的玉皇顶,形成“地府”、“人间”、“天堂” 三重空间。②泰山风景名胜以泰山主峰为中心,呈放射状分布,由自然景观与人文景观融合而成。③岱庙是山下泰城中轴线上的主体建筑,前连通天街,后接盘道,形成山城一体。由此步步登高,渐入佳境,而由“人间”进入“天庭仙界”。④泰山山体高大,形象雄伟。尤其是南坡,山势陡峻,主峰突兀,山峦叠起,气势非凡,蕴藏着奇、险、秀、幽、奥、旷等自然景观特点。A.①④③②B.④①③②C.②④①③D.②①④③【答案】C【解析】本题考查排序。纵观四个句子,讲的是泰山风景名胜的总体特点。②句“由自然景观与人文景观融合而成”是对泰山的总体评价,属于总说句,所以放在第一位;④句谈“自然景观”,根据与中心句对应的原则,应排第二位;①句谈“人文景观”的整体布局,应排第三位;③句谈人文景观的具体分布(或游览线路应排在最后;因此语序为②④①③。故选C。19.将下列句子组成语意连贯的一段话,排序最恰当的一项是( )①鲁迅先生在众多领域都颇有建树。②他一生研习不辍,留下600多万著译文字、3万多页手稿,古今中外版画4000多幅。③他以无限的热忱、不懈的努力创造出惊人的成绩,令人感佩。④在文学创作、文化历史研究、古籍校勘整理等方面,他都有开拓性贡献。⑤他酷爱美术,喜爱钻研汉画像和碑帖,喜爱书籍装帧设计,还是篆刻高手。A.⑤①②③④B.⑤④①③②C.①②⑤③④D.①④⑤②③【答案】D【解析】①句总领全文,点明写作对象,为第一句;④句“在文学创作、文化历史研究、古籍校勘整理等方面”紧承“在众多领域”,为第二句;⑤具体写鲁迅先生的成就,为第三句;②总括写鲁迅先生的成就,为第四句;③总写鲁迅先生的精神与成绩令人敬佩,为末句。故选D。20.下列句子组成语段顺序排列正确的一项是( )①向日葵式的仿生建筑也能够随时追踪太阳的方向进行旋转,太阳落山后,控制程序会让房屋自动恢复初始位置。②众所周知,向日葵从发芽到花盘盛开这一段时间,其叶子和花盘会一直追随太阳的位置以获得最充足的阳光。③加上其表面安装了大量的太阳能光电板,它每天生产的电能远远大于旋转所消耗的。④其旋转的动力全都来自自身的“光合作用”,即由屋顶的太阳能光电板和小型的太阳能电动机提供动力,十分节能。⑤于是住户便将多余的电能存入社区电网,冬天或者阴天时再拿出取用,剩余的还能卖钱。A.④②①③⑤B.②①④③⑤C.④②⑤①③D.②③①④⑤【答案】B【解析】考查句子的排序能力。解答此类题目时,先找到总起句和总结句。很显然,语段主要讲述了“向日葵式的仿生建筑”是如何借鉴向日葵向阳原理的,因此应将②句作为首句,先讲述向日葵向阳,然后引出①句“向日葵式的仿生建筑也能够随时追踪太阳的方向进行旋转”,接着④句讲述向日葵式的仿生建筑旋转的动力的来源,其次③句讲述其节能的原因,最后⑤句做总结,这样的仿生建筑不仅可以节约能源,还可以让住户赚钱,因此正确的排序应该是②①④③⑤;故选B。题型二:语言表达与运用21.结合上下文,在画线处补写句子,使语意连贯。花草是有记忆的,________________。虽说花草的生命很短,从春到秋,但它们把春秋一世的记忆藏进了种子,一代又一代传下去,花草就这样记住了时间。树也是有记忆的,它能记住更长时间的变化。树的生命记忆不仅有春有秋,还有一年又一年的岁月。春天发芽,秋天结果,然后脱下满树的黄叶,这记忆能让我们看到。可是,_________________,植物学家看到这一圈圈的树纹,就知道,哪年风调雨顺,哪年遇涝逢早。【答案】 它们记得季节的变化 我们没看到的记忆是树干里的年轮【解析】本题考查语句衔接。第一空:根据“花草是有记忆的”,联系后文“但它们把春秋一世的记忆藏进了种子,一代又一代传下去,花草就这样记住了时间”可知,所填写的语句应该体现“花草”记得的内容——季节、时间;可补写为:它们记得季节与时间的变化。第二空:“树也是有记忆的,它能记住更长时间的变化。树的生命记忆不仅有春有秋,还有一年又一年的岁月。春天发芽,秋天结果,然后脱下满树的黄叶,这记忆能让我们看到”,写的是我们能看到的树的记忆;“可是”一词表转折,联系后文“植物学家看到这一圈圈的树纹,就知道,哪年风调雨顺,哪年遇涝逢早”可知,后文应是我们没看到的树的记忆——年轮;可拟写为:我们没看到的树的记忆是年轮。22.下图是“生活垃圾分类处理流程图”,请你简要介绍图片内容,不超过80字。【答案】示例:生活垃圾中的餐厨垃圾,可资源化处理,比如供热发电、堆肥回用:大件垃圾和可回收垃圾可回收后再利用:其他垃圾需运往转运站集中处理。【解析】本题考查图文转换。按照流程图由总到分有序说明即可,注意字数的限制。示例:生活垃圾分为四类:餐厨垃圾、大件垃圾、可回收垃圾和其他垃圾。对于餐厨垃圾,可资源化处理,比如供热发电、堆肥回用等;对于大件垃圾和可回收垃圾,可回收后再利用;对于其他垃圾可直接运往转运站集中处理。23.下图为4月23日世界读书日的宣传图片,请对该图片的画面及寓意进行说明。要求:①信息完整、准确、有效;②语言简明,说明顺序合理;③不超过120字。【答案】示例:4月23日世界读书日宣传图片分为三个部分,主体部分为四本书,其中下面三本书是关上的,最上面一本书是摊开的;摊开的书本上有一个追寻梦想的女孩;书的右侧是一架梯子。宣传图片寓意着阅读是人类实现梦想的阶梯。【解析】考查图文转换。看图可知,图中主体部分是撂在一起四本书。下面的三本是合起来的,最上面的一本是打开的。打开的书本上有一个女孩子,双手上举,目视远方,远方有她的梦想。四本书的右侧是一架梯子。由此可分析,这幅宣传画的寓意是阅读如同梯子,可以帮助人们看到并实现自己的梦想。24.奋进新时代,开启新征程。为了更好地传承“志存高远,奋发进取”的精神,希望中学特举行校徽征集活动。假如你参与了此活动,请你给评委说明校徽主体部分的创意。(不少于80字)写作提示:①校徽外形为同心圆;②从备选元素中选择3~4个设计元素;③主要从色彩、图形、寓意方面说明创意。外形:备选元素:我选择的元素:___________________创意说明:_______________________【答案】 书、火焰、奔跑的人、跑道。 校徽的主体图案由两部分构成。上半部分由奔跑的人、脚下的跑道、头顶的红色火焰组成,寓意奋发进取﹑积极向上;下半部分是一本翻开的书,寓意勤奋好学。校徽由红、蓝、绿三色组合而成,简洁大方,是生命,是青春,是希望。(我选择的元素:浪花、帆船、祥云。创意说明:浪花起伏的蓝色海面上,一艘帆船正乘风破浪,驶向远方,海面上长空万里,白云朵朵,整个画面给人以广阔、拼搏、积极向上之感。校徽的主体图案寓意学校蓬勃发展,莘莘学子充满朝气,积极进取,放飞梦想,勇往直前!)【解析】本题考查图标、徽标。徽标即徽章、标志,它不是一般的图标,是具有指代意义、具有标识性质的图形。此题首先需要观察校徽,透过现象看本质,分析校徽反映的是什么主题,再观察题目中的元素,将有关联的元素组合在一起,写出它们的寓意,结合题文中“志存高远,奋发进取”的主题精神,答案不唯一,言之有理即可。示例:我选择的元素是跑道、奔跑的人、火焰。奔跑的人指的是学生在一直奋进努力,一直在奔跑前进着;跑道是指人生中的不同道路,有学习的道路、职业的道路,都有不同的跑道和人生追求;火焰是奔跑的人燃起的火焰。整个元素组合起来是指学生在各自的人生跑道上奋力奔跑,燃起属于自己的火焰与希望。25.班级开展“信息改变生活”主题活动。你作为主持人,想借用下面的漫画写一段开场白,以激起同学们对活动主题的思考和讨论。写作提示:①结合漫画内容和寓意;②100字左右。【答案】同学们,请看漫画:一位戴着眼镜的人在成堆“学习资料”中搜寻,累得满头大汗,书本散落一地,却一无所获,不禁叹道:“资料找时方恨多啊!”信息时代,对海量信息的筛选、辨析、汲取等,常令人苦不堪言。如何避免 信息焦虑,高效享受信息红利,需要我们积极思考、热烈讨论。【解析】本题考查开场白和图文转换。结合题干,借用图中的漫画写一段开场白。开场白一般要由称谓、问候语+开展活动的原因、目的或意义+导入语构成,拟写开场白时,要注意情景场合,语言要得体,表达要富有文采,能够营造氛围,引入活动。本题是开展“信息改变生活”主题活动,称呼是“尊敬的老师和亲爱的同学们”,问候语是“大家好”,内容要围绕漫画内容,激发学生对“信息改变生活”的活动主题的思考。漫画内容要理清漫画内的人物关系,并按照一定顺序介绍漫画的主要内容,介绍人物时要注意神态、动作、语言等细节部分,注意漫画有无标题、注解等细节部分,再行分析漫画的寓意。观察漫画,会发现有一个人站在梯子上查找资料,资料浩如烟海,这个人累的满头大汗,说道“资料找时方恨多啊”,图画寓意当今社会是一个知识信息量爆炸的时代,各种资料良莠不齐,筛选辨析让人很累。据此理解即可拟出开场白。示例:尊敬的老师,亲爱的同学们,大家好!我们先来看一副漫画,图中的一位戴眼镜的男子趴着梯子在一大堆“学习资料”中查找自己需要的信息,可是累的满头大汗,而且口中还在感叹“资料找时放恨多啊”我们现在进入到信息时代,各种信息浩如烟海,可是内容也良莠不齐,怎么样才能高效利用信息呢?下面就让我们进入“信息改变生活”的主题活动,希望大家积极思考,踊跃发言!26.在确定本届亚运会主题口号为“心心相融,@未来”后,亚组委向社会征集亚运志愿者口号。以下三则入选30强的口号中,作为志愿者的你会投选哪则?请从内容主题、语言表达等方面简述理由。【甲】同e舞台,同e精彩。 【乙】心在,爱在,我在 【丙】IT世界,爱连你我。【答案】示例:我选【乙】。语言上很简洁,三个“在”,押韵顺口;“心在”,内容上与主题口号也有呼应;“我在”志愿者的主题也很突出。【解析】本题考查语言表达。此题可任选一则口号,然后从内容主题、语言表达等方面简述理由,即能够说明选择的志愿者口号与本届亚运会主题口号“心心相融,@未来”怎样契合,并从语言表达方面说明选择理由。示例一:我选【甲】。语言上,采用复沓的修辞,形成了回环美,强调了“同e”;内容上,用表示互 联网的“e”代替“一”,形式活泼,和主题“@未来”相呼应;“同e舞台”和“同e精彩”,也展现了志愿者们将和运动员们在亚运会上的舞台上有同样精彩的表现。示例二:我选【乙】。语言简单,连续用三个“……在”的句式,运用排比的手法,增强气势,押韵顺口;扣合“心心相融”的主题,“我在”体现大家争当志愿者,为亚运会服务。示例三:我选【丙】。语言上,言简意赅,朗朗上口。“IT”和主题“@未来”相呼应,有时代科技感;“世界相连”,把“世界”与“你我”对应,体现了体育运动的意义,也体现了志愿者的主题。27.阅读下面的图文内容,按要求做题。你的同学文文希望制作的风筝能参加“筝迎新年”作品展。下面三种图案中,你推荐了最合适的一种给他,并告知他理由。【答案】示例:选择C。理由:风筝图案由鱼和莲花组成。“鱼”谐音“余”,寓意“吉庆有余”;“莲”谐音“连”。整个图案寓意“连年有余”,符合“筝迎新年”作品展的要求。【解析】本题考查图文转换能力。解答此类题目时,首先要通读题干,抓住主要信息,明确答案要求;然后找出图中所含的信息:标题、项目、各种数据等,并对其进行比较;最后依据题干要求,把数字转化为语言,规范作答。解答时,围绕“筝迎新年”的主题,结合题干所给风筝图片,分析其与主题的关联,再进行选择叙述即可。A图图案为一只猫头鹰,与“新年”主题无关。B图图案由祥云组成蝴蝶形状,中间有“寿”,寓意“长寿”,与“新年”主题无关。C图图案为一只金鱼,金鱼两侧还有莲花图案,“鱼”和“余”同音,表示着年年有余,代表了人们对新的一年美好的祝愿。“荷”与“和”“合”谐音,“莲”与“联”“连”谐音,中华传统文化中,经常以荷花即莲花作为和平、和谐、合作、合力、团结、联合等的象征;以荷花的高洁象征和平事业、和谐世界的高洁。因此,某种意义上说,赏荷也是对中华“和”文化的一种弘扬。春节,大量的荷花图、鲤鱼荷花图的运用,是取其谐音,象征“和和美美”“连(莲)年有余(鱼)”。据此可知,C图最符合“筝迎新年”作品展的要求。28.初2003班班主任和班委会商议,决定由作为班长的你去联系学校团委赵老师,当面邀请他于4月24日下午第一、二节课到本班教室参加“文明礼仪伴我行”演讲比赛初赛。作为班长,你会对赵老师怎么说?【答案】示例:赵老师,您好!我是初2003班的班长,我们班准备于4月24日下午第一、二节课在本班教室进行“文明礼仪伴我行”演讲比赛初赛,想请您参加,您看行吗?(期待您的光临!)【解析】本题考查语言表达的能力。解答时,开头应有称呼,然后介绍自己,再向赵老师发出邀请,要交代清楚时间、地点和事由。还要注意语句连贯,语言得体。示例:赵老师,您好!我是初2003班的班长,我们班于4月24日下午第一、二节课在本班教室进行“文明礼仪伴我行”演讲比赛初赛,想请你给予指导,请问,您有时间吗?29.老师要求同学们各用一种颜色表达对杭城之美的理解。请你按照示例,仿写句子,表达你的理解,关联词自选。示例:我心目中的杭城之美是绿色的。因为绿色既是杭城诗意浓郁的江南之美的体现,也是杭州善城文化与自然景观和谐辉映的象征。【答案】示例:我心目中的杭城之美是红色的。因为红色既是杭城鲜花烂漫的江南之美的体现,又是杭州红色文化与自然景观相映成趣的象征。【解析】本题考查句子仿写。注意“用一种颜色”,表达对“杭城之美”的理解,参照句子格式进行仿写。根据例句,此题答题格式为:我心中的杭城之美是X色的。因为X色既是……的体现,也是……的象征。示例一:我心目中的杭城之美是黄色的。因为黄色既是杭城丹桂飘香的江南之美的体现,也是杭州诗意文化与自然景观交相呼应的象征。示例二:我心目中的杭城之美是金色的。因为金色既是杭城阳光灿烂的江南之美的体现,也是杭城美好品质与自然美景交相辉映的象征。30.在世界节水日到来之际,为了取得更好的宣传效果,同学们在宣传手册上选用了一幅漫画(见下图)。请你向人们进行讲解。写作提示:①说明漫画的构图要素和寓意;②80字左右。【答案】漫画以三个水龙头水流的大小,从左至右代表昨天、今天和明天人们浪费水资源的状况。暗喻我们肆意地浪费水资源,水资源将会越来越少,水龙头里的水就会越来越少,直至枯竭。【解析】本题考查图文转换。解答此题,首先要从漫画的构成元素上分析漫画所表现的主题。漫画中的水龙头放水量从大到小,对应的是下方“昨天”“今天”“明天”的字样,说明如果我们肆意浪费水资源,水资源将会越来越少,直至枯竭,借以提醒人们要有节水意识。据此总结作答即可。31.“中国水稻之父”袁隆平曾说过“人就像种子,要做一粒好种子”,引发了大家的思考。有同学提出“能否成为一粒好种子是由外部环境决定的”,有的同学认为“能否成为一粒好种子是由种子自身决定的”,为此班内组织了一场辩论赛,请你选择一方观点并阐述理由。(150字左右)【答案】示例一:我认为“能否成为一粒好种子是由外部环境决定的”。一粒健康的种子,如果没有合适的温度、水分、光照等适宜的外部环境,也不能正常萌发、成长,从而失去生命力。如同一个健康的人,只有处在天时、地利、人和都具备的环境中,才能获得成长所需的支持与力量,从而实现目标,到达成功的彼岸。示例二:我认为“能否成为一粒好种子是由种子自身决定的”。一粒种子如果本身就存在先天缺陷,缺少萌发所需的物质,即使有合适的温度、水分、光照等适宜的外部环境,也无法萌发。如同一个人如果自身有残疾或先天疾病,即便创造了良好的运动条件,很多运动项目也无法参加,达到这方面的要求。【解析】本题考查辩论。辩论赛的特点是双方各执相反观点,解答此题首先明确自己所代表的观点,选择“能否成为一粒好种子是由外部环境决定的”和“能否成为一粒好种子是由种子自身决定的”其中一个观点,接着阐述支持该观点的理由,语言流畅,言之有理即可。示例:我认为“能否成为一粒好种子是由种子自身决定的”。自身先天的条件会限制其发展的情况,如果种子本身就是坏的,再好的外部条件也长不出芽。比如一个先天在绘画方面没有天赋甚至色盲的人,外部条件的培养也很难使其在绘画方面有很高的成就。32.【专题讲座】为了进一步激发同学们的兴趣,学校拟在4月7日下午四点半在小礼堂举办主题为“中国传统文化里的色彩之美”的主题讲座,准备邀请书画院的王教授来我校主讲,请你以学校学生会的名义拟一份邀请函。【答案】示例:邀请函尊敬的王教授:您好!我校拟在4月7日下午四点半在小礼堂举办主题为“中国传统文化里的色彩之美”的讲座,在此诚恳邀请您为我们进行主讲,希望您能莅临参加! XX学校学生会4月3日【解析】考查语言表达。首行居中写“邀请函”三个字。第二行顶格写称呼“尊敬的王教授”。第三行空两个格写正文。交代活动举行的时间“4月7日下午四点半”,地点“小礼堂”,内容“主题为‘中国传统文化里的色彩之美’的讲座”,邀请的目的“请对方担任主讲”。最后表达希望对方莅临的意愿。右对齐写落款,单位在上,日期在下。单位是学校学生会,日期要早于活动举行的时间。33.为弘扬传统文化,学校要开设象棋社团,请你结合下面所给材料写一段话,动员同学们积极加入。(限80字以内)[材料一]在中国的棋林之中,影响最为深远者当属围棋和象棋。俗语有云:“棋局小世界,世界大棋局。”作为典型的中国文化载体,二者以棋盘和棋子折射出中国传统社会的缩影。相通之处在于棋盘都是阡陌纵横,呈格状分布;棋子以颜色不同构成两方以对垒拼杀。但再加考察,则大有区别。[材料二]围棋和象棋反映出中国文化在不同角度、不同阶段逐渐形成的两种不同的思维方式和价值观念。围棋的存在表明中华文化体系中存在着民主的精神、平等的理念。考虑到中国古代的爱国情操向以忠君行动来体现,象棋所表现出来的便可视为先人为保江山社稷而不惜一切代价的群体理念和视死如归、杀身成仁的牺牲精神。这两种精神和理念相辅相成,共同支撑着中华民族的精神大厦。【答案】示例:象棋是中国文化载体之一,是一种较为高雅的技艺。象棋表现出来的是为保江山社稷而不惜一切代价的牺牲精神。请同学们积极加入象棋社团,弘扬传统文化。【解析】本题考查语言表达能力。答案不唯一,写出象棋在中国传统文化的意义及学习象棋的好处即可,注意字数的限制。示例:象棋作为中华文化的典型代表,象棋表现出来的精神支撑着中华民族的精神大厦,还可以在象棋对局中培养自己良好的心态和抗挫折意识与能力。请同学们积极加入象棋社团。34.近期,一款“爱国”主题T恤为不少时尚青年所追捧。下面是该T恤上的徽标图案,请说说其设计巧妙之处。【答案】图中设计了中国的版图(五角星是国旗的象征,华表是中国的象征);英文字母“I”是“我”的意思,“心形”是爱的表示,“yu”是“你”的意思;整幅图表达了我爱我的祖国、我和我的祖国心连心、祖国在我心中等真挚的情感。【解析】本题考查图文转换。抓住构图要素的特点,按照一定的顺序进行说明即可。示例:图形由两部分组成,左面主体部分,是一个“心形,“心”中左侧是含有华表的“T”字,右侧是含有五星的中国的版图地图;右面的文字是“yu”。寓意为:我爱我的祖国(或我和我的祖国心连心、祖国在我心中等)。35.在学校举行的“非遗进校园·文化在身边”主题活动中,你将作为宣传组的志愿者对下面的宣传画进行解说。请将你要说的话写下来。写作提示:①任选一幅宣传画进行解说;②介绍画面内容和主题;③80字左右。【答案】示例一:大家好,请看第一幅画。画的下方是眉开眼笑的一家三口,正借助电子产品观看节目,中间有“非物质文化遗产”字样,上方是造型不一的四个戏剧人物。宣传画名为《“活”起来》,意在说明借助数字化媒介宣传非遗文化能更好地传承传统文化,使其焕发新的生机。示例二:大家好,我讲解的是第二幅画。画面中间是写有“人民的非遗·人民共享”的飘带,飘带上方是“山西非遗购物节”字样,画中还有酒、面、醋、葡萄等山西非遗物产元素。这幅宣传画《用起来》传达了以“山西非遗购物节”为契机,让人们享受非遗物产,传承非遗文化的愿望。【解析】本题考查图文转换。任选一幅画进行解说即可。如果选图一,首先描写画面,画面分为三个部分,画面下方是一家三口,父亲和孩子手捧着“数字化媒介”,也就是电子产品,一家三口正开心地观看节目;画面上方是四个戏剧人物的Q版形象,是一家三口关注的内容;而画面的正中间是“非物质文化遗产”的字样,表示图画下方一家三口观看的内容,同时也是画面上方四个形象代表的内容;接着再看宣传画的名字为《“活”起来》,结合画面内容可知,就是让“非物质文化遗产”“活”起来,借助的工具就是“数字化媒介”、电子产品,据此分析画面主旨,就是借助数字化媒介宣传非遗文化能更好地传承传统文化,使其焕发新的生机。如果选图二,首先描写画面,画面上方是“山西非遗购物节”字样,画面下方是酒、面、醋、葡萄等山西非遗产品,也就是此次购物节上会展销的商品;画面中间是写有“人民的非遗·人民共享”的飘带,代表着此次购物街的主题;接着看宣传画的名字为《用起来》,意思是指让大家购买山西非遗产品,使用山西非遗产品,以达到宣传作用。据此分析画面主旨,宣传“山西非遗购物节”,并让人们享受非遗物产,传承非遗文化的愿望。题型三:综合性学习36.为庆祝中国共产主义青年团成立100周年,学校举行主题演讲活动,你准备参加。(1)你的演讲主题是:___________________。 (2)围绕主题给演讲稿写开头(至少运用一种修辞手法,60字以内)。(3)为调动现场氛围,你准备在演讲时带领大家唱一首歌。下列两首歌曲你会选择哪一首?请说明理由。【甲】 我们是五月的花海,用青春拥抱时代。我们是初升的太阳,用生命点燃未来。“五四”的火炬,唤起了民族的觉醒,壮丽的事业,激励着我们继往开来……(节选自歌曲《光荣啊,中国共青团》)【乙】 我们迎着初升的太阳,走在崭新的道路上。我们是优秀的中华儿女,谱写时代的新篇章。我们迎着风雨向前方,万众一心挽起臂膀……(节选自歌曲《走向复兴》)【答案】(1)示例:在青春的赛道上奋力奔跑(贴近活动目的即可)(2)示例:同学们,大家好!沐浴明媚阳光,怀揣远大理想,吹响前进号角,我们是新时代的奋斗者,在青春的赛道上奋力奔跑。(3)示例1:选甲,理由:这段歌词表现了作为一名青少年乐于为国家、为社会奉献一切的精神。并且,这是歌唱中国共青团的,与“庆祝中国共产主义青年团成立100周年”的活动主题更接近。示例2:选乙,这段歌词唱出了作为中华儿女谱写新时代篇章的豪迈,符合本次演讲活动的主题“庆祝中国共产主义青年团成立100周年”。【解析】(1)考查演讲主题的把握。解答时,围绕“为庆祝中国共产主义青年团成立100周年”的主题,结合中学生实际,贴近活动目的进行拟写即可。示例:青春心向党,奋进新征程。(2)考查拟写演讲稿的能力。演讲稿也叫演讲词,它是在较为隆重的仪式上和某些公众场合发表的讲话文稿。演讲稿是进行演讲的依据,是对演讲内容和形式的规范和提示,它体现着演讲的目的和手段。演讲稿由开头、主体、结尾三部分构成。开头要先声夺人,富有吸引力:开门见山,亮出主旨;叙述事实,交代背景;提出问题,发人深思;引用警句,引出下文。本题作答时,要紧紧围绕上题所拟写的主题来进行叙述,并注意至少运用一种修辞手法和字数要求;示例一:同学们,大家好!青春像一轮太阳,释放出似火的热情。青年有理想担当,国家和民族就有希望。青春心向党,我们要在新征程上奋进!(3)考查语言表达能力。解答时,围绕“庆祝中国共产主义青年团成立100周年”的主题,结合对两首歌曲内容的理解进行选择即可。开放性试题,答案不唯一,言之有理即可。示例一:选甲,《光荣啊,中国共青团》是中国共青团的团歌。歌词内容表现出浑身充满朝气的青年一代,用青春拥抱时代,用生命点燃未来的博大胸怀。与“庆祝中国共产主义青年团成立100周年”的活动主题更接近。示例二:选乙,《走向复兴》表达了中国人民实现民族复兴的伟大理想和心愿,给人以勇往直前的力量,表达了我们把祖国变得更加富强的决心和信心。符合本次演讲活动的主题“庆祝中国共产主义青年团成立100周年”。37.五月的花海,时代的芬芳。建团百年,我们以青春之名,赴强国之约。华新中学举办“逐梦好青年,接续向未来”综合性学习活动,请你参与。(1)【秀·青春风貌】校团委邀请你拟写一条宣传语。请你结合语段中的划线句完成宣传语的上句。新时代中国青年刚健自信、担当有为,坚定拥护党的领导,奋力走在时代前列,展现出前所未有的昂扬风貌:追求远大理想,与国家同呼吸,与人民共命运,时刻彰显着鲜明的爱国主义精神;传承伟大使命,先天下之忧而忧,后天下之乐而乐,勇做开拓、进取的时代先锋。上句:____________________下句:传承使命勇做时代先锋(2)【访·青春榜样】为了让同学们感受到青年榜样的力量,校团委拟开展采访青年奋进者的活动。作为活动策划者,请简要阐述你的采访思路。我的采访思路:为了让采访高效进行,我们第一步要确定采访对象,第二步要拟定____________________,第三步要约定采访时间,第四步再进行____________________。(3)【燃·青春接力】跨越百年,三代青年相聚一堂。面对主持人“青春应该做什么”这个提问,请你补全当代青年的回答。李大钊:我生活在水深火热的旧中国。我的青春是用热血和生命,创造一个崭新的中国。雷锋:我生活在欣欣向荣的新社会。我的青春是用拼搏和汗水,建设一个美丽的中国。当代青年:我生活在繁荣富强的新时代。我的青春是____________________,____________________。主持人:跨越百年,传承一代代不变的梦想,我们用最好的成绩证明——中国青年,是这个世界上最炫的那一片风景!(4)【研·青春成才】综合下列两则材料,探究青春成才的主要因素有哪些。材料一:“只要祖国需要,我当然愿意”,这是青年彭士禄在组织征求他意见,是否愿意改行学原子能核动力专业时的坚定回答;“家是玉麦,国是中国”,这是西藏玉麦乡牧民卓嘎、央宗姐妹从小就树立并付诸行动的坚定信念;“到艰苦的地方去,到祖国需要我的地方去”,这是云南丽江华坪女子高级中学毕业生山启燕毕业后的坚定选择……材料二:青年成才因素调查图我的结论:___________________________________________________【答案】(1)【秀·青春风貌】示例:追求理想彰显爱国精神(2)【访·青春榜样】示例:采访提纲(方案、内容、问题) 实地(现场)采访(3)【燃·青春接力】示例一:用创新和梦想,成就一个强大的中国。 示例二:用努力和奋斗,成就一个伟大的中国。(4)【研·青春成才】示例一:高度的社会责任感和坚定的理想信念(上进心)是青春成才的主要因素。示例二:家国情怀和坚定的理想信念(上进心)是青春成才的主要因素。【解析】(1)本题考查宣传标语的拟写。根据例句以及“追求远大理想,与国家同呼吸,与人民共命运,时刻彰显着鲜明的爱国主义精神”概括可得:追求理想彰显爱国精神。(2)本题考查对采访思路的设计。根据对采访过程的掌握可知,我们第一步要确定采访对象,第二步要拟定采访提纲(方案、内容、问题),第三步要约定采访时间,第四步再进行实地(现场)采访。(3)本题考查句子仿写。结合例句可知,采用“用……和……,成就……”的形式,如:用信心和智慧,成就一个强盛的中国。答案是多元的。(4)考查对文本材料的探究。结合【研·青春成才】材料一中列举的青年彭士禄、西藏玉麦乡牧民卓嘎、央宗姐妹和山启燕事例,可以看出成才要有爱国情怀,要有坚定的信念;从材料二统计表中的数据可知,高度的社会责任感和坚定的理想信念(上进心)是青春成才的主要因素。据此分析,可得出探究的结论是:家国情怀和坚定的理想信念(上进心)是青春成才的主要因素。意对即可。38.阅读材料,完成下列各题。贵州省第十三届人民代表大会第四次会议在贵阳召开。省长李炳军代表人民政府向大会作政府报告。报告指出,生态文明建设取得重大成果。牢固树立绿水青山是金山银山理念,强力推进国家生态文明试验区建设。(1)为了贯彻落实贵州省第十三届人民代表大会第四次会议精神,某市正准备就“倡导低碳生活”开展宣传活动,请你为这次活动写一条宣传标语。(2)下表是某学校对学生进行“低碳生活方式”了解程度的调查表。请根据调查结果,用简洁的语言概括图表中的信息。(3)某班召开一次以“低碳生活,我在行动”为主题的班会,大家推荐你做主持人,请写出你的开场白。(80字左右)【答案】(1)建设生态文明 倡导低碳生活 树立低碳理念,创建绿色家园(2)大多数学生不太了解低碳生活方式。(3)大家好!低碳生活是指在我们的日常生活和学习中,最大限度降低地球能量的耗费,减低二氧化碳的排放量。低碳生活,它是一种生活态度。低碳生活,它代表着一种健康、一种自然的生活方式。现在让我们携起手来,行动起来,从身边小事做起。为人类美好的明天共同努力。【解析】(1)本题考查拟写宣传标语。本题设计的宣传标语应围绕“倡导低碳生活”这一主题展开,体现其内容或意义,可采用对偶等修辞手法进行拟写;如:追求节能时尚,拥抱低碳生活。(2)本题考查图文转换。本题中的柱状图统计了某学校对学生进行“低碳生活方式”了解程度;其中非常了解的占7.9%,比较了解的占24.2%,非常不太了解的占59.2%,不了解的占8.7%,据此可知大多数学生不太了解低碳生活方式。(3)本题考查拟写开场白。活动主题为“低碳生活,我在行动”,在拟写开场白时,首先要有礼貌用语,接下来围绕主题表达“低碳生活”的意义或与自身的关系,最后宣布班会开始。示例:同学们,大家好!你是否了解“低碳生活”呢?“低碳”是一种生活习惯,旨在提倡健康生活,“低碳生活”不再只是一种理想,更是一种值得期待的新的生活方式。让我们走进“低碳生活”,为我们的美好生活献计献策,共建我们最美好的家园吧!39.你们班级开展“身边的文化遗产”综合性学习,请你积极参与下面的活动。[研·淮水安澜]①同学们对文化遗产“水上长城——洪泽湖大堤”很感兴趣,在查阅了洪泽湖大堤的资料后,又向不同学科老师请教。请仿照示例,向老师提一个问题。 (1)示例:问地理老师 提问:老师您好! 请问洪泽湖大堤修建在现在位置的依据是什么?问____老师 提问:___________________________________________。(2)[赞·地标美食]②下列淮安特色美食入选“江苏百道乡土地标菜”,请从中任选一道,写一则广告语。(不超过20字)备选:洪泽杂鱼锅贴 金湖蒜泥龙虾 盱眙十三香龙虾 淮安钦工肉圆 涟水鸡糕 淮阴码头汤羊肉选:__________ 广告语:______________________。(3)[汇·名城风华]③在“身边的文化遗产”综合性学习汇报会上,大家用对联表达对淮安文化的热爱。有同学拟了上联,请从下列选项中选出最恰当的下联。上联:万里漕运通南北 下联 选( )备选:A.赞古楚人杰地灵B.千年淮味传古今C.淮剧一曲唱四海D.名著《西游》天下闻【答案】(1)示例:历史 老师您好!洪泽湖大堤修建的历史意义有哪些?(2)示例:洪泽杂鱼锅贴 广告语:活鱼锅贴,美食一绝。(3)B【解析】(1)本题考查语言表达能力。作答时,要结合洪泽湖大堤的相关资料内容提问。示例:语文老师您好!请问洪泽湖大堤的修建有哪些文化意义呢?(2)本题考查拟写广告语。作答时,任选一道淮安特色美食,写出吸引人的广告语。示例一:金湖蒜泥龙虾。广告语:龙虾驾到,滋味美妙。示例二:盱眙十三香龙虾。广告语:盱眙十三香,唇齿留香。示例三:淮安钦工肉圆广告语:古老工艺匠心传承,铁棒锤出鲜香年味 示例四:涟水鸡糕广告语:淮安涟水鸡糕,让乾隆帝都赞不绝口。示例五:淮阴码头汤羊肉淮阴码头汤羊肉,涮出来的好味道。(3)本题考查对联。对联写作上要做到词性相同、结构一致;平仄对应、音韵协调;内容相关、意境完美。A.偏正短语“万里漕运”与动宾短语“赞古楚”不相对;B.偏正短语“万里漕运”与“千年淮味”相对,动宾短语“通南北”与“穿古今”相对,恰当;C.下联末字“海”是仄声字,不符合“仄起平收”的原则;D.动宾短语“通南北”与主谓短语“天下闻”不相对;故选B。40.中考结束后,班级要制作一个视频,回顾多彩的初中生活,珍藏美好的青春记忆。(1)本视频的主题是“深情寄母校 青春向未来”,你想为片头选择下面的哪种字体,请说出你的理由。我选择___________(字体),因为_______________________________________________。(2)视频分为“校园撷景”“教师风采”“同学画像”“班级史话”四个部分。请你任选一个部分,写一段话放在它的前面。(不少于30字)(3)视频的最后,同学们要朗读“志存高远,放飞青春梦想;脚踏实地,书写人生华章”这句话,请你从朗读感情、朗读技巧等任一角度对同学们做出提示。【答案】(1)示例:隶书 方劲古拙,蚕头雁尾,给人以古朴大方之感。(2)示例:校园撷景:春天使大地焕然一新,春天给学校满园春色,春天给我们欢乐和希望,催促我们奋发向上。让我们带着感恩之心走进我们的母校吧。(3)示例:朗读感情,要情感充沛,要读出自己的喜悦和期待之情。【解析】(1)本题考查字体识别及鉴赏。任选一种字体进行鉴赏,如隶书的“蚕头雁尾”、“一波三折”,楷书形体方正,笔画平直,行书的行云流水等特点,言之有理即可。示例一:楷书。字迹工整,笔法清秀,易于表达感情。示例二:行书。字体行云流水、舒展有型,赏心悦目。(2)本题考查语言表达能力。任选“校园撷景”“教师风采”“同学画像”“班级史话”一个部分,语言得体,言之有理即可。示例一:教师风采:大家还记得在讲台上激扬文字、指点江山的诸位老师吗?让我们走进老师,感受他们的风采吧。示例二:同学画像:曾记得操场上奔跑的身影吗?曾记得教师内琅琅的读书声吗?那是我们的同学,我们一生的朋友。示例三:班级史话:这是个团结奋进、朝气蓬勃、温暖和谐的集体,35张笑脸,35个不同的个性支撑起了这个班级大厦。在这个大家庭里,我们感受到了不一样的快乐和温暖。(3)本题考查朗读运用能力。作答时,从朗读感情、朗读技巧等任一角度进行分析,如朗读情感的充沛,朗读技巧中的重读等。示例:朗读技巧,要重读“青春梦想”“人生华章”,这样才能体现出自己的激扬青春的情感。41.同学们发现语文课要求阅读的许多名著书目都与英雄有关,但是同学们对于“英雄”的认识各不相同,大家打算举办一次“何为英雄”的主题班会,交流分享对英雄的认识。(1)为了筹备主题班会,班委会对班级42位同学进行了问卷调查,下图是根据同学们对“你对哪一类英雄了解最多”的回答结果所作的数据统计:根据上图,能够得出的结论是( )。A.该班同学了解的中国古代英雄比中国近代革命英雄多。B.该班同学了解的作品中虚构的英雄比中国当代英雄少。C.该班同学了解最多的英雄类型是中国古代英雄。D.该班同学了解最少的英雄类型是中国当代英雄。【答案】D【解析】根据图表内容分析即可。A.有误。该班同学了解的中国古代英雄比中国近代革命英雄少;B.有误。该班同学了解的作品中虚构的英雄比中国当代英雄多;C.有误。该班同学了解最多的英雄类型是中国近代英雄;故选D。(2)同学们为主题班会设计了一张海报,请根据海报内容和班会主题把宣传语补充完整。【答案】示例:梦想追逐者;平凡岗位上的努力前行者【解析】本题考查拟写宣传语。根据班会主题是“何为英雄”和海报内的人物都是一线的劳动者,有吹号的军人,有拿书的教师,有拿锄头的农民,有提包奔走的知识分子等,他们在奋力拼搏,努力前行。据此拟写宣传语即可。示例:追逐梦想的英雄。(3)班级同学小A的妈妈是一名医生,曾经主动申请前往一线抗疫,大家决定要在班会课上通过视频连线的方式采访她,听听她对于“何为英雄”的看法。以下是同学们草拟的采访问题,请你筛选出其中不合适的两项( )。A.你在医生这个行业干了几年?B.您为什么主动报名前往一线支援抗疫?C.您觉得英雄的评价标准是什么?D.您最欣赏的英雄是谁?E.您是如何教育自己的孩子的?【答案】AE【解析】A.“你在医生这个行业干了几年?”与“何为英雄”的主题不相符;B.她主动参加抗疫,这是“英雄”的表现;C.“您觉得英雄的评价标准是什么?”与“何为英雄”的主题相符;D.“您最欣赏的英雄是谁?”与“何为英雄”的主题相符;E.“您是如何教育自己的孩子的?”谈论的是如何教育孩子,与“何为英雄”的主题不相符;故选AE。(4)班会课上,班长请你结合名著阅读篇目,简要谈谈自己对“何为英雄”的看法,请完成一篇100字以内的演讲稿。可结合名著:《红星照耀中国》/《水浒传》/《钢铁是怎样炼成的》(任选其一)【答案】参考样例:①大家好!我认为英雄要有着崇高信念并坚定执着地为之奋斗。正如《钢铁是怎样炼成的》一书中的保尔柯察金,他在身有残疾的时候依然不改革命信念,始终坚守在自己的工作岗位上,为了理想而奉献生命。这样的人是我心中的英雄。②大家好!我认为英雄是能够在国家民族处于危难之际挺身而出的人。正如《红星照耀中国》中的革命领袖和普通战士,他们都在为了民族解放和独立而奋斗,不顾个人的安危。他们都是英雄。③大家好!我认为英雄是嫉恶如仇、有胆有识的人。正如《水浒传》当中的鲁智深,他得知镇关西欺凌弱女子金翠莲,便主动站出来帮助翠莲父女二人,不仅给了他们盘缠,还拳打镇关西,让正义得以伸张,这样的人是当之无愧的英雄。【解析】本题考查语言表达能力。首先要有称呼,因为在班会课上,面对的是全体同学,可以用“大家好!”作为称呼语。然后用简洁的语言并结合所给名著篇目,讲述自己关于英雄的认识即可。示例:大家好!英雄者,意志坚定,互相扶持,不畏艰难险阻,救黎民于水火,解百姓于倒悬!《红星照耀中国》中红军将士们面对险恶环境和层层围堵,意志坚定,互相鼓励扶持,完成了两万五千里长征的创举,可谓真正的英雄。42.湖湘中学准备组织一次“走近文化遗产,弘扬传统文化”的研学活动,请你参加并完成下列任务:(1)下面是中国文化遗产标志,请你根据所提供的基本信息,说说徽标的寓意。(50字以内)【答案】示例:中国文化遗产标志代表着政府和人民对文化遗产的保护,表达的是追求光明、团结奋进、和谐包容的精神寓意。【解析】本题考查表达应用能力,依据徽标内容理解寓意。中国文化遗产标志中的成都金沙“四鸟绕日”是中华各民族对太阳神崇拜的体现,表达了对光明的追求,是蓬勃向上、团结奋进、和谐包容精神的象征;徽标采用圆形标志,体现了中华民族强烈的凝聚力和向心力,表现了中华民族自强不息、昂扬向上的精神。设立中华文化遗产标志代表了我国政府和人民对文化遗产的重视和保护。总结作答时要精炼,注意字数要求。(2)学校拟邀请岳麓书院的王教授于5月28日下午三点在学校图书馆为同学们作“走近岳麓书院,感受千年文化”的专题讲座。请你以湖湘中学德育处的名义,写一份邀请函。邀请函尊敬的王教授:_________________________________湖湘中学德育处2021年5月21日【答案】示例:您好,我是湖湘中学的学生xxx,谨代表我校,诚挚地邀请您于5月28日下午三点在我校图书馆为同学们作“走近岳麓书院,感受千年文化”的专题讲座。期待您的到来!【解析】本题考查表达能力,补写邀请函的正文。正文是告知被邀请方举办礼仪活动的缘由、目的、事项及要求,写明礼仪活动的日程安排、时间、地点,并对被邀请方发出得体、诚挚的邀请,结尾一般要写常用的邀请惯用语,如“敬请光临”“期待您的到来”等。本题中要把活动安排的时间:5月28日下午三点;地点:学校图书馆,邀请目的:为同学们作“走近岳麓书院,感受千年文化”的专题讲座等内容表述清楚。(3)请运用对联知识,把下列拆散的对联重新组合。传承 吟诗词曲赋 颂扬 国学经典 山河桑田 诵经史子集上联:_________________________ 下联:_________________________【答案】上联:诵经史子集传承国学经典 下联:吟诗词曲赋颂扬山河桑田【解析】本题考查表达应用能力,重新组合对联。首先,依据对联知识先将拆散的对联两两配对。根据词性相同、字数相同的原则可知,“传承”对“颂扬”,“吟诗词曲赋”对“诵经史子集”,“国学经典”对“山河桑田”;其次,根据词义分别组成句子。传承指对前人的经验进行传授和继承并发扬发展的过程,故与“国学经典”搭配;颂扬意思是称颂褒扬,可与“山河桑田”搭配;国学经典包括经、史、子、集、蒙,因此“诵经史子集”与“传承国学经典”组成一句话,“吟诗词曲赋”与“颂扬山河桑田”搭配成一句话;最后,确定上下联。上联最后一个字必须是“仄”声字,下联最后一个字必须是“平”声字,也就是仄起平收,“典”是仄声,“田”是平声。43.岁月如歌,三年青春相聚的一团热火,就要散作满天繁星。回首逝去的日子,我们洒下的汗水,收获的欢乐都将成为亲切的怀恋。九年级(一)班开展“岁月如歌”主题活动,请你参与活动,完成以下任务。(1)【班史拟名】为了见证同学们的成长,纪念这段美好的初中生活,班级将制作一本班史。请你为班史拟写一个有意义的名字。示例:《花样年华——我们的初中生活》【答案】示例:我们;青葱岁月……(能体现纪念的、青春等的皆可,注意简洁)【解析】本题考查拟名的能力。班史名称围绕初中生活、离别在即拟写,简短,能表达怀念、热爱、赞美等情感即可。示例:《亲切的怀念——我们的初中岁月》。(2)【栏目解说】班史计划分“班级留影”“班级大事记”“师生风采”三个板块来编写,请你作为栏目负责人,参考示例,向参与编辑的同学们说明“师生风采”的栏目内容。示例:班级留形——该栏目拟选取若干张初中生活的照片,含荣誉照片、生活照片等,配上文字解说,记录我们共同经历的岁月。班级大事记——该栏目按时间顺序,记录三年来班级发生的重大事件。提纲挈领,不展开叙述。师生风采——【答案】师生风采:该栏目以师生为原型,以文字的形式叙述每一位师生的典型事迹,或截取某一生活片段或代表性语言,彰显其特点。(扣住“风采”一词,能具体说明本栏目的内容,言之成理皆可)【解析】本题考查栏目内容说明。“师生风采”栏目,显然是介绍全班老师和同学们的特点及事迹的栏目,仿照“班级留影”“班级大事记”的示例,向参与编辑的同学说明主要内容即可。示例:师生风采——该栏目收录老师和同学们的自我介绍,以个人小传的形式,请每位老师和同学写出自我简介,主要包括个人特点和三年来印象深刻的事迹,每人一个页面,并配个人生活照一张。(3)【邀请拟写】 “桃李芬芳,教泽绵长”。班级决定于6月10日19点在本班教室组织一场以“岁月如歌,师恩难忘”为主题的文艺晚会。请你代表全班同学邀请各科任课教师参加这次文艺晚会。邀请函尊敬的老师:您好!九年级(一)班全体同学2022年6月9日【答案】示例:为了记录我们的成长足迹,感恩老师对我们的教导,我班于6月10日19点在本班教室组织一场以“岁月如歌,师恩难忘”为主题的文艺晚会,我们真诚地邀请您参加,望您百忙之中拨冗莅临!【解析】本题考查邀请函的拟写。本题只要求拟写邀请函的正文部分,根据所给的情境组织文字进行表述。本题设置的情境:邀请各科任老师参加主题为“岁月如歌,师恩难忘”为主题的文艺晚会。重点要说明邀请的事由(请老师参加班里组织的毕业文艺晚会)、具体的时间(6月10日19点)、地点(本班教室),要注意语句连贯,语言得体,最后表达热切期望老师参加的意思。44.“陋室常余书卷在,清新自有墨香来。”青春不息,学习不止,无论年长年少,都应捧起书本,沐浴书香。为此,初一22班的同学将在本班教室举行以“少年正是读书时”为主题的班会活动。请你积极参与活动的组织,完成下面的任务。(1)下列句子中,能体现读书重要性的句子是( )。①立身以立学为先,立学以读书为本。②纸上得来终觉浅,绝知此事要躬行。③读书勤乃有,不勤腹空虚。④读书破万卷,下笔如有神。⑤书籍是人类进步的阶梯。⑥天行健,君子以自强不息。A.①④⑤B.②④⑤C.②⑤⑥D.①③⑤【答案】A【解析】本题考查名言警句的理解。①句意:修养品行从学习开始,学习以读书为根本,体现了读书的重要性;②句意:从书本上得到的知识终归是浅薄的,未能理解知识的真谛,要真正理解书中的深刻道理,必须亲身去躬行实践,体现的是实践的重要性;③句意:读书学习勤奋肚子里才有(学问),(读书学习)不勤奋肚子里就是空的(没有学问),强调了读书的方法是“勤”;④句意:形容博览群书,把书读透,这样落实到笔下,运用起来就会得心应手,体现了读书的重要性;⑤句意:把书籍比喻成人类进步的阶梯,也就是说人类的进步离不开书籍,书籍为人类提供了很多的知识、经验,体现了读书的重要性;⑥句意:宇宙不停运转,人应效法天地,永远不断地前进,体现的是“自强不息”,与读书无关;①④⑤体现了读书的重要性,故选A。(2)下面是本次活动的宣传标语,请你仿照前一句,将后面的句子补充完整好读书,读书让生活变得充实;读好书,____________________________。【答案】好书能净化心灵。【解析】本题考查宣传标语的补充。这里要紧扣“少年正是读书时”的主题进行拟写,仿照“读书让生活变得充实”的句式、内容特点,写出读好书的积极作用即可。如:好书让思维变得缜密,好书让知识变得渊博,好书让思想变得澄澈等。(3)活动中,同学们都积极交流自己的阅读经验,甲同学说:“我会努力记住书中的关键人物和情节。”乙同学说:“我会拿出一个笔记本做摘录。”你认为谁的读书方法更好,请说明理由。【答案】示例:我认为乙同学的读书方法好,俗话说好记性不如一根烂笔头,再说在摘抄中可以放慢阅读速度,从而增加了品味的时间。【解析】本题考查观点表达。首先明确自己的观点,然后阐述理由,这是开放性试题,言之有理即可。如:我认为甲同学的读书方法好。“我会努力记住书中的关键人物和情节”这一方法的好处是,有利于快速把握作品的内容,能对信息进行快速筛选整合以及对人物形象的快速把握。又如:我认为乙同学的读书方法好。“我会拿出一个笔记本做摘录”的好处是,这样做可以加深印象,记录下阅读瞬间的思想火花,并将我们的思考引向深刻,使读书的效果加倍,同时也有利于词句、思想、素材的积累。45.一百年前,我们党在一艘游船上诞生。从此,这艘游船成为永载史册的“红船”、学校准备举办一场弘扬红色精神,“爱国爱党”为主题的综合实践活动,请你积极参加并完成以下任务。(1)请为本次活动写一则宣传标语。(不超过10个字)【答案】红色精神薪火相传【解析】本题考查拟写宣传标语的能力。宣传标语要简洁、明确,宜使用一定的修辞手法。此题围绕“弘扬红色精神,‘爱国爱党’”的主题进行拟写,符合标语的一般要求即可。如:红色精神薪火相传;闪闪红星照耀中国;红色历史永记心中。(2)初三(1)班收集整理了红色旅游发展现状资料,如下图所示,请根据统计图中的数据整体变化情况组织语言表述信息。2015-2019年中国红色旅游行业市场规模(单位:亿元)【答案】2015-2019年中国红色旅游行业市场规模呈上升趋势。【解析】本题考查图文转换能力。此图显示的是2015-2019年中国红色旅游行业市场规模,根据图中表示市场规模的柱体从2015年—2019年逐年升高可见,这段时间中国红色旅游行业市场规模呈逐年上升、稳步提高的趋势。可据此概括作答,不需要列出具体数字。(3)本次活动中有一场关于“弘扬红色精神是继承传统更重要还是融入流行更重要”的辩论赛。请根据流程将表格内容补充完整。【答案】①开始时正方的任何一位队员先起立发言。在他结束发言后,反方任何一位队员立即发言双方依次轮流发言直到时间用完为止。②梳理全场脉络,找出交锋点,然后指出对方在此犯下的错误,提出正确的理解,再从更高的角度阐述己方观点【解析】本题考查设计活动方案的能力。第①空,属于攻辩之后的自由辩论阶段,可由正反双方任一队员发言和辩驳;步骤可先由正方任一队员发言,发言结束后反方任一队员进行辩驳或阐述己方观点;然后再由正方任一队员发言,依次类推,双方可相互反驳对方观点,轮流自由辩论,直到本阶段“自由辩论”结束;第②空,是双方“总结陈词”阶段,双方各派一名队员,各自总结己方观点,指出对方错误;在总结中,要根据对方的反驳,对己方观点进行完善,从而更好地反驳对方,堵住漏洞,全面准确地阐述己方的观点。根据以上分析,简要概括两个阶段的正反双方的任务即可。三、文学文化常识与名著阅读 题型一:文学文化常识1.下列文学常识表述错误的一项是( )A.鲁迅作品的主题有的轻松,如《朝花夕拾》中的《从百草园到三味书屋》《阿长与山海经》叙写的是童趣;有的沉重,如《呐喊》中的《故乡》《孔乙己》反映的则是社会的病态。B.明代小说家吴敬梓的《儒林外史》没有贯穿全书的中心人物和主要情节,而是由众多故事连缀而成,表现的是普通士人日常生活中的生存状态与精神世界。C.《我爱这土地》的作者是艾青,他的代表作有《大堰河——我的保姆》《光的赞歌》《向太阳》。D.《海燕》是短篇小说“幻想曲”《春天的旋律》的结尾部分。作者高尔基,苏联作家,代表作有自传体小说《童年》《我的大学》《在人间》。【答案】B【解析】本题考查文学常识。B.清康熙四十年(1701年),吴敬梓出生在安徽全椒一个“科第仕宦多显者”的官僚家庭,所以他是清代的,选项“明代小说家吴敬梓”有误。故选B。2.下面是小青同学积累的文学、文化知识,其中不正确的一项是( )A.王安石,北宋政治家、文学家、思想家,“唐宋八大家”之一。B.《诗经》是我国最早的一部诗歌总集,分为风、雅、颂三个部分。C.《资治通鉴》是北宋司马光主持编纂的一部编年体通史。D.《论语》,儒家经典著作,与《大学》《尚书》《孟子》合称为“四书”。【答案】D【解析】本题考查对文学常识的识记。D.有误。《尚书》不属于“四书”,应该改为《中庸》。3.下列对文学文化常识的表述,不正确的一项是( ) A.对联是我国传统文化的瑰宝,也是活的文化遗产,有春联、寿联、名胜古迹联等。B.中国传统文化中,许多事物以“阴阳”划分,如“山南水北”为阳,“山北水南”为阴。C.《诗经》是我国最早的一部诗歌总集,收录从西周到春秋时期的诗歌305篇。D.古代的年龄有特定的称谓,如小孩子七八岁时叫“始龁”,男子十八岁时叫“弱冠”。【答案】D【解析】本题考查文学文化常识。D.“男子十八岁时叫‘弱冠’”有误,男子二十岁时叫“弱冠”;故选D。4.下列对文学文化常识的表述,不正确的一项是( )A.风骚,本指《诗经》里的《国风》和《楚辞》中的《离骚》,后来泛指文章辞藻。B.欧阳修,字永叔,自号醉翁,晚年又号六一居士,北宋文学家,“唐宋八大家”之一。C.辛弃疾,字幼安,号稼轩,北宋著名豪放派词人。《南乡子•登京口北固亭有怀》是他的作品。D.《简•爱》采用第一人称的写法,小说中的主人公敢和骄横残暴的表哥发生冲突,毅然离开惩罚凌辱她的寄宿学校,爱上与她地位悬殊的罗切斯特,都是因为她坚信人在精神上是平等的。【答案】C【解析】本题考查文学文化常识识记。C.有误。辛弃疾是南宋词人;故选C。5.下列关于文学、文化常识的表述,正确的一项是( )A.除夕之夜,按习俗应全家团聚吃汤圆、观花灯,寓意着“圆满”,也寓意着“团聚”。B.《资治通鉴》是北宋司马光主持编撰的一部纪传体通史,记载了从战国到五代共1362年间的史事。C.“达于汉阴”中的“汉阴”是指“汉水南面”;在我国古代文学作品中,常常用“桑梓”“桃李”“婵娟”“丝竹”来分别指代家乡、学生、月亮和音乐。D.“冠”是古代贵族所戴帽子的总称。古时男子十八岁举行加冠(束发戴帽)仪式,表示已经成人。后人常用“冠”或“加冠”表示年已十八。【答案】C【解析】本题考查识记文学、文化常识。A.“除夕之夜,按习俗应全家团聚吃汤圆、观花灯”错误,吃汤圆、观花灯是元宵节的习俗并非是“除夕”的习俗;B.《资治通鉴》是编年体,而非“纪传体”;D.古时男子二十岁举行加冠仪式,表示已经成人。后人常用“冠”或“加冠”表示年已二十。选项中“十八”的表述错误;故选C。6.欣赏下面的书法作品和对联,下列和它们有关的说法正确的是( )A.作品一的内容是三国政治家、军事家曹操的四言诗《观沧海》,该诗直抒胸臆,意境开阔,气势雄浑,苍凉悲壮,真实地再现了沧海之景。B.作品一的落款“己亥”与“元丰六年”“庆历四年”的纪年方式相同。C.作品二中“亭台依旧羡他烟水全收”是上联,“风月无边到此胸怀何似”是下联。D.作品二中的对联构思精巧,对仗工整,动静结合,韵味无穷。【答案】D【解析】考查文学和文化常识。A.曹操是东汉末年人,不是“直抒胸臆”,应是借景抒情,诗中还有想象;B.后两者是按照皇帝的朝代纪年,“己亥”是天干地支纪年法;C.上下联有误,应是仄起平收,上下联互换;故选D。7.下列表述有误的一项是( )A.《孟子》《庄子》是儒家经典著作,《礼记》《左传》是道家经典著作。B.古代常用官职或出生之地来称呼一个人,如“杜工部”指杜甫,“柳河东”指柳宗元。C.杨绛,翻译家、作家,代表作有《干校六记》等。叶圣陶,原名叶绍钧,代表作有童话集《稻草人》等。D.部编版初中语文教材选入多位诺贝尔文学奖获得者的文章,如印度泰戈尔的《金色花》,英国罗素的《我为什么而活着》。【答案】A【解析】考查文学常识。A.《孟子》《礼记》《左传》是儒家经典著作,《庄子》是道家经典著作。故选A。8.下列有关课文内容和文学文化常识表述完全正确的一项是( )A.《诗经》是我国最早的一部诗歌总集,也是我国诗歌现实主义传统的源头。《诗经》中的诗当初都是配乐的歌词,按所配乐曲的性质分成风、雅、颂三类。“风”是正统的宫廷乐歌,用于宴会的典礼;“雅”是各地方的民歌民谣;“颂”是祭祀乐歌,用于宫廷宗庙祭祀。B.“世先生同在桑梓”中的“桑梓”指家乡;“居庙堂之高则忧其民”中的“庙堂”指朝廷;“黄发垂髫”分别指老人与小孩;“瀚海阑干百丈冰”中的“瀚海”指沙漠;“复立楚国之社稷”中的“社稷”指国家。C.怡宝学成归来,受邀回母校做讲座。来到母校,他说:“贵校师生特别热情,老师在门口恭候我光临,我非常感动。”这样的情境中,怡宝的话是表达得体的。D.小说中,“变”字贯穿了全文。法国作家莫泊桑的《我的叔叔于勒》一文中,围绕着金钱,菲利普夫妇对于勒的态度一直在变;俄国作家屠格涅夫的《变色龙》一文中,随着“狗的主人是谁”的猜测不断改变,奥楚蔑洛夫的态度和裁断也一直在变。【答案】B【解析】本题考查文学常识、文化常识。A.“‘风’是正统的宫廷乐歌,用于宴会的典礼;‘雅’是各地方的民歌民谣”表述不正确。“风”是各地方的民歌民谣;“雅”是正统的宫廷乐歌,用于宴会的典礼;C.“怡宝的话是表达得体的”表述不正确。怡宝的话中的“恭候光临”表达不得体,“恭候光临”是指自己恭候别人的到来,谦辞,不能用于别人;D.“俄国作家屠格涅夫的《变色龙》一文中”表述不正确。《变色龙》的作者是俄国作家契诃夫;故选B。9.下列关于文学和文化常识表述不正确的一项是( )A.《曹刿论战》节选自《春秋左氏传》。主人公曹刿是春秋时期鲁国人;而《左传》一书旧传为春秋时期左丘明所作,近人则认为是战国时人所编写。B.《山坡羊·潼关怀古》中的“山坡羊”是曲牌名。作者明代张养浩,字希孟,他所作的散曲以豪放著称。C.《史记》是我国第一部纪传体通史,作者西汉司马迁。我们所学过的《陈涉世家》就节选自该书。D.唐宋时期,诗歌和散文创作都达到了很高的水平。其中散文创作名家有韩愈、柳宗元、欧阳修、曾巩、“三苏”、王安石,此八人被称为唐宋散文八大家。【答案】B【解析】考查文学和文化常识的掌握。B.有误,张养浩是元代散曲家,不是明代;故选B。10.下列对文学文化常识的表述,不正确的一项是( )A.古代有许多称谓有特定的含义,如“汗青”指史册,“青鸟”指信使,“黄发”指小孩。B.古代春社日,祭社神(土地神),祈求丰收。C.“冀之南,汉之阴,无陇断焉”中的“阴”指的是汉水的南部。D.我国古代常用“迁”指调动官职,“左迁”“迁谪”都表示降职贬官。【答案】A【解析】本题考查文学文化常识。A.“黄发”指的是老人,“垂髫”指的是小孩;故选A。11.下列关于文学、文化常识的表述,错误的一项是( )A.老舍,原名舒庆春,字舍予,北京人,满族,作家。主要作品有小说《骆驼祥子》《四世同堂》,话剧《茶馆》《龙须沟》等。B.普希金,俄国诗人。代表诗作有《自由颂》《致恰达耶夫》《致大海》等。C.消息是迅速、简要地报道曾经发生的事件的一种新闻体裁,极为常见,运用广泛。D.读书人的自谦辞有小生、晚生、晚学等。【答案】C【解析】考查文学、文化常识的识记。C.有误,消息是迅速、简要地报道新近发生的事件的一种新闻体裁,消息的最大特点是时效性强和真实客观;故选C。12.下列关于名著内容的表述,错误的一项是( )A.长妈妈、寿镜吾、范爱农、孔乙己,都是鲁迅《朝花夕拾》中的人物。B.美国记者埃德加·斯诺的《红星照耀中国》描述了中国工农红军长征的经过,向全世界全面报道了这一举世无双的军事壮举。C.《钢铁是怎样炼成的》中的主人公保尔·柯察金的身上凝聚着那个时代最美好的精神品质——为理想而献身的精神、钢铁般的意志和顽强奋斗的高贵品质。D.在《儒林外史》中科举制成为作者揭露和讽刺的主要对象。【答案】A【解析】考查名著内容的识记辨析。A.有误,长妈妈、寿镜吾、范爱农是散文集《朝花夕拾》中的人物,孔乙己是鲁迅所著的短篇小说《孔乙己》中的人物;故选A。13.下面有关文学作品的表述,错误的一项是( )A.《儒林外史》是由清代讽刺小说家吴敬梓创作的章回体长篇小说。主要描写了封建社会后期读书人的生活和精神状态,文笔淋漓酣畅,是中国古代讽刺文学的典范。B.《格列佛游记》的作者是法国18世纪前期最优秀的讽刺作家和政论家乔纳森·斯威夫特。童话色彩是小说的表面特征,热情的讴歌与赞颂是作品的灵魂所在。C.墨子、孟子、老子均是先秦时期著名的思想家。他们分别是墨家、儒家、道家的代表人物。D.契诃夫的小说《变色龙》和安徒生的童话《皇帝的新装》都运用了对比、夸张等艺术手法,表现了很强的讽刺意味。【答案】B【解析】本题考查文学常识。B.有误,《格列佛游记》的作者乔纳森·斯威夫特是英国人,童话色彩是小说的表面特征,尖锐而深刻的讽刺是作品的灵魂所在;故选B。14.下列关于文学、文化常识的表述,完全正确的一项是( )A.在人际交往中,我们要尽量做到用语得体。比如:初次见面用“久违”,好久不见用“久仰”,请人帮忙用“拜托”,麻烦别人用“劳驾”。B.诗句“遥知兄弟登高处,遍插茱萸少一人”和“遥怜故园菊,应傍战场开”都与中国传统节日——重阳节有关。重阳节的时间为农历九月初九。登高赏秋与感恩敬老是当今重阳节的两大重要主题。C.《天净沙•秋思》是元代作家马致远所写的最为有名的一首词,被誉为“秋思之祖”,其中“天净沙”是词牌名,“秋思”是题目。D.古代立春后第五个戊日为春社日,人们在这一天祭社神,祈求丰收。社神就是谷神。【答案】B【解析】本题考查文学、文化常识。A.初次见面用“久仰”,好久不见用“久违”;C.《天净沙•秋思》是元代戏曲作家马致远所写的最为有名的一首小令,被誉为“秋思之祖”,其中“天净思”是曲牌名,“秋思”是题目;D.社神是土地神,并非“谷神”;故选B。15.下列关于文学文化常识的表述,正确的一项是( )A.维克多·雨果,英国作家,代表作品有小说《巴黎圣母院》《悲惨世界》《九三年》,《就英法联军远征中国致巴特勒上尉的信》选自《雨果文集》。B.苏轼,字子瞻,号东坡居士,北宋文学家,我们学过他的《水调歌头·明月几时有》《定风波·莫听穿林打叶声》等。C.“横眉冷对千夫指,俯首甘为孺子牛”是鲁迅一生的写照,《阿Q正传》是以“鲁迅”为笔名发表的第一篇白话文小说。《琐记》选自他的散文集《朝花夕拾》。D.古人称谓丰富:所谓“名”实则“本名”;“字”则叫“表字”,是除本名外另取一个与本名有所关联的名字;“谥号”则多为自己所取,表示自己的志趣爱好。【答案】B【解析】本题考查文学文化常识。A.有误。雨果是法国作家;C.有误。《狂人日记》是以“鲁迅”为笔名发表的第一篇白话文小说;D.有误。“谥号”是古人死后,其他人依其生前行迹而为之所立的称号;故选B。题型二:名著阅读16.下列有关名著的表述中正确的一项是( )A.《西游记》是最成功的带有神话色彩的文学巨著,其中孙悟空勇闯龙宫、勾销生死簿、搅乱蟠桃会、打碎琉璃盏、大闹凌霄殿,其反抗精神呼之欲出。B.《儒林外史》中周进的姐夫等人可怜周进,凑钱替他捐了个监生,得以直接考举人,考中;后来又考中进士,任广东学道。周进遇范进考秀才,因仰慕他而录取。范进后又考中举人,张乡绅来结交,赠予银子及房子。C.鲁迅在《朝花夕拾》的《五猖会》中记述了作者儿时盼望观看迎神赛会时的急切、兴奋的心情,并借此对“正人君子”予以辛辣的讽刺。D.《钢铁是怎样炼成的》“筑路”这一章节中,险恶的环境描写给读者留下了深刻的印象,这些环境描写表现了保尔和战友们在极其艰难困苦的条件下显现出来的英雄本色。【答案】D【解析】本题考查名著内容。A.“打碎琉璃盏”有误。打碎琉璃盏的是沙僧。沙僧本是天界的卷帘大将,只因在蟠桃会上打碎玉帝的琉璃盏,被玉帝打了800锤丢到流沙河中;B.“因仰慕他而录取”“周进的姐夫等人可怜周进,凑钱替他捐了个监生”表述有误。几个商人同情周进,帮助他捐了个监生。不久,周进凭着监生的资格竟考中了举人。过了几年,他又中了进士,升为御史,被指派为广东学道。在广州,周进发现了范进。为了照顾这个54岁的老童生,他把范进的卷子反复看了三遍,终于发现那是一字一珠的天地间最好的文章,于是将范进取为秀才;C.“借此对‘正人君子’予以辛辣的讽刺”有误。《五猖会》记述的是作者儿时盼望观看迎种赛会的急切,兴奋的情绪,而这种少年的高兴心情却因被父亲强迫背诵《鉴略》而冲淡,从而揭示了旧式教育对于儿童天性的压制,揭露了封建教育制度对儿童的摧残。文中没有表现出对“正人君子”予以辛辣嘲讽;故选D。17.下列对相关名著的解说,不正确的一项是( ) A.《昆虫记》每个章节中都详细、生动地描绘一种或几种昆虫的生活,它们有的恪守自然规则,不敢稍越雷池;有的为了生存繁衍,不懈努力。这些描写充满了法布尔对自然的热爱,对生命的敬畏之情。B.艾青的诗歌中主要意象是“土地”和“光明”。他的长诗《向太阳》《火把》,借助太阳、索求火把,表达了驱逐黑暗、坚持斗争、争取胜利的美好愿望,诗人也因此被称为“太阳”和“火把”的歌手。C.《西游记》中有许多脍炙人口的故事都与“三”有关,如三打白骨精、三借芭蕉扇、斗法降三怪、三探无底洞、三打祝家庄等。正所谓“九九数完魔灭尽,三三行满道归根”。D.《水浒传》塑造了众多梁山好汉的形象,其中我们都很喜欢的武松崇尚忠义、勇而有谋、有仇必复、有恩必报的个性。他醉打蒋门神、大闹飞云浦、血溅鸳鸯楼等故事极为精彩。【答案】C【解析】考查名著常识。C.“三打祝家庄”是《水浒传》中的情节;故选C。18.下列对相关名著的解说,不正确的一项是( )A.《红星照耀中国》中,苏维埃政府对农民最有重要意义的四项举措是重新分配土地、取消高利贷、取消苛捐杂税、消灭特权阶级。因此,苏维埃政府得到了农民的广泛支持。B.尼摩船长是法国作家凡尔纳的小说《海底两万里》的主人公,他对民族压迫和殖民主义极端痛恨,向往民主与自由。C.《水浒传》是元末明初小说家施耐庵写的一部以北宋末年宋江起义为题材的长篇白话小说。D.老舍的《骆驼祥子》富有浓郁的老北京风情,小说刻画了祥子自尊好强、吃苦耐劳、始终不向命运低头的平民形象。【答案】D【解析】本题考查名著内容识记和理解。D.解说不正确。根据《骆驼祥子》中祥子最后“由人变成鬼”可知祥子屈服于命运,而非“始终不向命运低头”。故选D。19.下列表述正确的一项( )A.《水浒传》中的武松疾恶如仇,侠肝义胆,有仇必复,有恩必报。从为兄报仇开始,他一步步走向反抗的道路,是下层英雄好汉中富有血性和传奇色彩的人物。B.《西游记》中唐僧师徒取经成功后,观音菩萨封唐僧为“金蝉子”,封孙悟空为“金身罗汉”,封猪八戒为“天蓬元帅”,封沙僧为“斗战胜佛”。C.《钢铁是怎样炼成的》以主人公奥斯特洛夫斯基的成长经历为线索,热情讴歌了苏联青年人在血与火的洗礼中大无畏的革命英雄主义和乐观精神。D.《红岩》中,叛徒甫志高假冒共产党员潜入刘公馆,前来了解刘思扬在狱中的表现,并要他详细汇报狱中地下党的情况。刘思扬识破了他的伪装,亲手将他击毙。【答案】A【解析】本题考查名著内容的识记。B.《西游记》中唐僧师徒四人取经成功后,如来封唐僧为“旃檀功德佛”,封孙悟空为“斗战胜佛”,封猪八戒为“净坛使者”,封沙僧为“金身罗汉”;C.《钢铁是怎样炼成的》作者是奥斯特洛夫斯基,小说主人公是保尔·柯察金;D.特务郑克昌冒充共产党员,而非“甫志高”;李敬原送来情报揭穿伪装,而非“刘思扬识破了他的伪装”;故选A。20.下列关于名著的表述,不正确的一项( )A.《骆驼祥子》中虎妞如愿嫁给祥子,与刘四爷反目,可祥子觉得日子并不如意。B.《水浒传》中武松斗杀西门庆后被发配,途中在十字坡酒店险些被孙二娘所害。C.《朝花夕拾》中在迎神会上的无常,名为鬼实为人,借鬼讽人,伸张正义。D.《海底两万里》想象丰富,全书用第三人称,语言平实,将科学与文学巧妙地结合,体现了作者的科学精神和非凡的想象力。【答案】D【解析】此题考查名著文学常识。D.有误,《海底两万里》是用第一人称写的,语言生动形象;故选D。21.下列关于名著的说法中不正确的两项是( )( )A.《骆驼祥子》中二强子酗酒成性,喝了酒在家发脾气,结果将自己的妻子打死了,还逼着女儿小福子卖身养活一家人,可见酒能泯灭人性。B.宋江被奸臣蔡京、高俅等用“皇帝赐酒”的手段毒杀。宋江因为害怕花荣造反,破坏了自己的“忠义之名”,欺骗他喝了毒酒而死。可见酒伤英雄心。C.猪八戒酒后戏嫦娥而被罚下人间,杨志在景阳冈上被酒麻翻丢了生辰纲,祥子醉后被虎妞诱惑导致被骗婚,可见喝酒误事。D.陆谦请林冲喝酒,是想骗他离家便于高衙内调戏林娘子;西门庆请何九叔喝酒,是要拉他下火坑;武松请邻居喝酒,是让他们见证斩杀潘金莲,可见喝酒是个“局”。E.《儒林外史》里娄家两公子、牛布衣等把酒论文,虞博士等人乘酒兴和诗,可见樽酒论文是古代文人的雅好。【答案】BC【解析】本题考查名著相关知识。B.“……宋江因为害怕花荣造反……欺骗他喝了毒酒而死。可见酒伤英雄心”表述错误。宋江害怕造反,破坏了自己的“忠义之名”的人是李逵,不是花荣;C.“杨志在景阳冈上被酒麻翻丢了生辰纲”表述错误。杨志在黄泥冈上被酒麻翻丢了生辰纲,不是景阳冈;故选BC。22.下列有关文学名著的表述,有误的一项是( )A.《智取生辰纲》中杨志受梁中书的派遣押送生辰纲前往东京。五月天气酷热,杨志却让军士顶着烈日赶路,众人怨声载道。B.《阿长与〈山海经〉》中写了长妈妈睡相不好、在床上摆“大”字、懂得许多规矩和麻烦的礼节、讲长毛的故事、谋害“我”的隐鼠等情节。C.《格列佛游记》中慧骃国用比赛绳技的方法来选拔官员,候选人必须冒着跌断脖子的危险来表演绳技。D.《西游记》中孙悟空的出身和大闹天宫等故事,为表现他的神通广大和后来追随唐僧去西天取经做了铺垫。【答案】C【解析】本题考查名著常识。C.《格列佛游记》中小人国用比赛绳技的方法来选拔官员,候选人必须冒着跌断脖子的危险来表演绳技。故选C。23.下列对相关名著内容的理解,不正确的一项是 ( )A.《水浒传》中武松十分敬重宋江,但当听说宋江抢走了刘太公的女儿时,砍倒了杏黄旗,大闹忠义堂。B.《朝花夕拾》中的长妈妈是鲁迅幼年的保姆,在《狗•猫•鼠》和《阿长与<山海经>》中都提到她害死隐鼠之事。C.法布尔所著的《昆虫记》,真实地记录了昆虫的生活,表达了对生命的关爱和对自然万物的赞美之情。D.《我爱这土地》这首诗的感情基调悲怆、深沉、坚定,多用修饰语来呈现诗人的情感,如通过“悲愤”“激怒”等词表现沉重,用“永远汹涌”“无止息地吹刮”等短语来表现坚定。【答案】A【解析】本题考查文学常识。A.有误,《水浒传》中李逵十分敬重宋江,但当听说宋江抢走了刘太公的女儿时,砍倒了杏黄旗,大闹忠义堂。故选A。24.文学名著中有很多经典的人物形象。下列名著片段中的“他”是( )平生只好结识江湖上好汉:但有人来投奔他的,若高若低,无有不纳,便留在庄上馆谷,终日追陪,并无厌倦;若要起身,尽力资助,端的是挥霍,视金似土。人问他求钱物,亦不推托。且好做方便,每每排难解纷,只是周全人性命。如常散施棺材药饵,济人贫苦,周人之急,扶人之困。以此山东、河北闻名,都称他做及时雨,却把他比的做天上下的及时雨一般,能救万物。A.柴进B.晁盖C.宋江D.王伦【答案】C【解析】考查名著人物。根据选段中的“以此山东、河北闻名,都称他做及时雨”可知,此人是宋江,绰号及时雨,梁山聚义后被众人推举为首领,一心谋求招安,最后被朝廷赐下毒酒害死。故选C。25.下列关于名著的表述有误的一项是( )A.《钢铁是怎样炼成的》在艺术上取得了很高的成就。它写人物以叙事和描写为主,同时穿插内心独白、格言警句、书信和日记等,使人物形象有血有肉。B.《红岩》讲述了解放战争时期,重庆地区的地下党人英勇斗争的故事,刻画了一批意志坚定、形象高大的共产党人形象,如江姐、许云峰、成岗、华子龙等。C.《西游记》是神魔鬼怪小说的翘楚,是中国古典文学中最富有想象力的作品之一,也是我国古代第一部长篇白话小说。D.《骆驼祥子》是现代作家老舍的代表作。小说描写了一个普通人力车夫的一生,他放弃了自己的理想,从一个诚实可爱的青年变成了麻木、潦倒、狡猾、自暴自弃的行尸走肉。【答案】C【解析】本题考查名著常识。C.有误。《西游记》是中国古代第一部浪漫主义章回体长篇神魔小说;我国古代第一部章回体长篇白话小说是《水浒传》;故选C。26.下面有关名著内容的表述,不正确的一项是( )A.《傅雷家书》中有父亲的谆谆教诲、有孩子与父母的真诚交流,字里行间洋溢着真情。B.《海底两万里》中,尼摩船长用潜水艇上的电击退了章鱼的攻击,挽救了船员的生命。C.《钢铁是怎样炼成的》中,保尔在朱赫来的影响下,逐步走上革命道路,最终成长为无产阶级战士。D.《西游记》中的孙悟空有超凡的能力,他曾经大闹天宫、三打白骨精、三调芭蕉扇。【答案】B【解析】本题考查名著内容识记。B.表述不正确。根据《海底两万里》中“在巴布亚新几内亚他们的船搁浅了,遇到当地土著人的攻击,尼摩船长用他的闪电挡住土著人进入‘鹦鹉螺’号”可知,尼摩船长用潜水艇上的电击退了“土著人”的攻击,挽救了船员的生命,而非“章鱼”。故选B。27.下列关于名著与文学常识表述正确的一项是( )A.《红岩》着力于塑造英雄群像,如精警老练的许云峰,在越狱战斗的最后时刻,屹立在高耸巨大的岩石上,吸引住敌人的火力,以掩护其他同志突围。B.他对四家邻舍道:“小人因与哥哥报仇雪恨,犯罪正当其理,虽死而不怨。却才甚是惊吓了高邻……”“他”是《水浒传》中的武松。C.《钢铁是怎样炼成的》作品中,谢廖沙做事认真负责,很有主见,沉着冷静,善交朋友,对保尔的思想成长起到决定性的作用。D.《白雪歌送武判官归京》是一首歌行体古诗,作者岑参,是唐代诗人,其边塞诗尤多佳作,诗风沉郁顿挫。【答案】B【解析】本题考查识记名著相关知识及文学常识。A.《红岩》中舍己为人,在越狱战斗中,为掩护其他同志突围,他吸引住敌人的火力,壮烈牺牲的是“齐晓轩”而不是“许云峰”。表述错误;C.《钢铁是怎样炼成的》中,做事认真负责,很有主见,沉着冷静,善交朋友,对保尔的思想成长起到决定性的作用的人是“朱赫来”而不是“谢廖沙”。表述错误;D.盛唐边塞诗人岑参的边塞诗雄健奔放,表现了一种阳刚豪健的风骨美和奇特壮丽的异域美,并不是“诗风沉郁顿挫”。表述错误;故选B。28.选出下列对名著有关内容表述有误的一项( )A.《红岩》中许云峰在白公馆地下室挖了一个地道,留给同志们越狱。B.《琐记》中鲁迅最先去的学校是雷电学校,后来又去的矿路学堂。C.《鲁滨逊漂流记》中,鲁滨逊因为海难而流落荒岛,在岛上生活了二十八年,救了个野人。D.《骆驼样子》中,“祥子”把卖骆驼所得到的三十元钱放在了虎妞那里存着,后来虎妞把钱又还给祥子。【答案】D【解析】考查名著内容的理解识记。D.有误,祥子把卖骆驼后剩下的30块钱交给刘四爷保管,希望攒满后再买车;一天晚上,虎妞把他存在刘四爷那里的30元钱还给他,要他腊月二十七给刘四爷拜寿,讨老头子喜欢,再设法让刘四爷招他为女婿;故选D。29.下列关于文学作品的表述,不正确的一项是( )A.《格列佛游记》中飞岛国的人很注重科学研究,他们设计从黄瓜里提取出阳光,用实验法把粪便还原为食物,用猪耕地,利用蜘蛛结网,用风箱打气治病——他们是一群空想、不尊重科学规律的“万能学者”。B.蟠桃会上,孙悟空喝光宴会用的仙酒,吃尽太上老君的金丹。太上老君大怒,随即和哪吒太子带10万天兵去花果山捉拿悟空,被悟空打败。C.《昆虫记》是一部既有趣又有益的书。在作者笔下,杨柳天牛像个吝啬鬼,身穿一件似乎“缺了布料”的短身燕尾礼服:被毒蜘蛛咬伤的小麻雀,也会愉快地进食,如果我们喂食动作慢了,他甚至会像婴儿般哭闹。D.《童年》的整体基调虽然严肃、低沉,但小说以一个小孩的眼光来描述,给一幕幕悲剧场景蒙上了一层天真烂漫的色彩,读来令人悲哀但又不过于沉重,使人在黑暗中看到光明,在邪恶中看到善良,在冷酷无情中看到人性的光芒,在悲剧的氛围中感受到人们战胜悲剧命运的巨大力量。【答案】B【解析】本题考查名著内容的识记。B.和哪吒太子带10万天兵去花果山捉拿悟空的是李靖,不是太上老君。此项“太上老君大怒,随即和哪吒太子带10万天兵去花果山捉拿悟空”说法有误;故选B。30.下列对相关名著的解说,正确的一项是( )A.《昆虫记》:法布尔往往在一个章节中介绍两种不同类的昆虫以对比分析。B.《水浒传》:以揭露上层贵族醉心名利之嘴脸为主题的讽刺小说。C.《简·爱》:艾米莉·勃朗特以浪漫主义笔调谱写的一部爱情喜剧。D.《艾青诗选》:以自由体诗抒发了诗人艾青对祖国的热爱和对光明的歌颂。【答案】D【解析】本题考查名著阅读。A.《昆虫记》往往一个章节只介绍一种昆虫;B.《水浒传》主题是人民反抗上层残暴统治,且这不是讽刺小说;C.《简·爱》作者是夏洛蒂•勃朗特,且这不是一部爱情喜剧;故选D。31.阅读材料,请你完成下面任务。学校举行“青春向党,喜迎二十大”主题演讲。下面是洋洋演讲稿的结尾部分。请你从备选名著中提炼素材,帮他完善讲稿。面对军统特务的威逼利诱,许云峰不为所动,拒绝与反动派合作,因为他有坚定的信念;(1)______,因为他有奉献的精神;(2)_____,因为他有顽强的意志……我们要继承这些宝贵的精神财富!备选名著:《红星照耀中国》 《钢铁是怎样炼成的》【答案】面对艰难恶劣的生活环境,周恩来没有退缩,扛起革命的大任 面对身体残疾的磨难坎坷,保尔没有屈服,拿起笔书写红色篇章【解析】本题考查句子的衔接和名著识记能力。从内容上考虑,要从所给的两部名著中选择两个人物,突出他们的奉献精神和顽强意志;从结构上看,要和划线句保持一致。句式:面对+偏正短语,人名+四字短语(是选择),写具体的行动。《红星照耀中国》是美国著名记者埃德加·斯诺,在1936年穿越重重封锁深入根据地,切实了解中国共产党人的生活经历和革命精神后所作,又名《西行漫记》,是一部文笔优美的纪实性很强的报道性作品,属于报告文学。全书12篇,对中国共产党和中国革命做出了客观评价,并向全世界做了公正报道。主要内容包括采访了众多共产党领袖和红军将领,如毛泽东、周恩来、彭德怀、林伯渠、邓发、徐海东等,其中周恩来是一个纯粹的知识分子、书生出身的“造反者”。第一次见面用英语跟斯诺打招呼。还为斯诺规划采访行程。把胳膊爱护地搭在“红小鬼”的肩上,走过乡间田埂。他是一个平易近人,温和文雅,头脑冷静,善于分析推理,讲究实际经验,细心热情,善于计划,生活朴素,充满活力的人。毛泽东曾到湖南省立图书馆看书自修半年,是一个精明而又博学多才的知识分子;他为红军做出了长征的决定,同时对世界政治惊人地熟悉,提出的世界政局时事问题有时连斯诺都无法回答,是个天才的军事家和政治战略家;他的财物只是一条毛毯、几件个人的用品,伙食也同每个人一样,将自己的上衣脱给战士穿,长征中和普通战士一样步行,体现了他的生活简朴、廉洁奉公,能够吃苦耐劳,精力过人,身体像铁打的;他虽被南京政府悬赏,却能泰然地和民众一起看电影,说明他是一个受苏区人民拥护,却被南京政府恨之入骨的人;毛泽东同斯诺谈论共产党的政策及抗日战争,代表了中国人民大众的迫切要求。朱德出身富农家庭,却坚持寻找并加入中国共产党,寻找新的革命道路,后在柏林参加共产党,因参加革命活动被捕。朱德指挥全军打过几百次小仗,几十次大仗,经历了敌人的五次大围剿,在长期作战中形成自己带兵的特殊战术。他沉默谦虚、爱护部下、天性温和,言谈直截了当,是个很活泼的人,有大智大勇、爱国爱民,为革命甘愿放弃个人财富和地位。《钢铁是怎样炼成的》是苏联作家尼古拉·奥斯特洛夫斯基所著的一部长篇小说。讲述了保尔柯察金从一个不懂事的少年到成为一个忠于革命的布尔什维克战士,再到双目失明却坚强不屈创作小说,成为无产阶级革命钢铁战士的故事。保尔当过童工,从小就在社会最底层饱受折磨和侮辱,后来在朱赫来的影响下逐步走上革命道路。随后经历了一系列人生挑战,但无论是战场上的搏杀、感情上的波折,还是工地上的磨难,都没能使他倒下,反而使他更加勇敢、坚强。在伤病无情地夺走他的健康,使他不得不躺在病床上的时候,他仍然不向命运屈服,拿起笔以顽强的毅力进行文学创作,重新回到战斗的行列。保尔的身上凝聚着那个时代最美好的精神品质——为理想而献身的精神、钢铁般的意志和顽强奋斗的高贵品质。示例1:面对穷凶险恶的残酷敌人,朱德没有后退,带领革命战士奋勇战斗。示例2:面对身体病痛的磨练坎坷,保尔没有放弃,拿起笔来进行文学创作。32.读书贵有所得,请你从以下几个人物中选择一位,结合原著内容讲述他具有代表性的一件事(50字左右),并谈谈带给你的启示。A.保尔 B.孙悟空 C.尼摩船长 D.藤野先生【答案】示例:保尔忍受着肉体和精神上的巨大痛苦,先是用硬纸板做成框子写,后来是自己口述,请人代录。在母亲和妻子的帮助下,他用生命写成的小说《暴风雨所诞生的》终于出版。启示:面对挫折要有钢铁般的意志和毅力,要有不畏艰难困苦一往无前的气概。在取经路上,孙悟空迎难而上,在降服红孩儿一回中,唐僧被红孩儿捉去,孙悟空智斗失败后决定硬闯,结果被烟熏瞎了眼睛,但他依然不忘救师父,就算是死,也不畏惧。启示:遇到挫折艰险不轻言放弃,意志坚定,敢于拼搏。当到达印度半岛南端的锡兰岛,阿龙纳斯接受尼摩船长的建议,步行到海底采珠场。忽然,有条巨鲨向采珠人扑来。尼摩船长手拿短刀,挺身跟鲨鱼搏斗。最后还送了采珠人价值不菲的珍珠。启示:我们要有同情心,在别人危急时要施以援助之手,面对挫折挑战要勇敢。藤野先生检查并从头到末地修改“我”抄的讲义,细致地指出“我”绘的解剖图中的错误。藤野先生治学严谨,一丝不苟。启示:对待工作和学习不能马虎,凡是都要认真对待。【解析】本题考查对名著人物的理解。结合合原著内容讲述他具有代表性的一件事并谈及启示,语言流畅通顺即可。示例一:《钢铁是怎样炼成的》中的保尔以“解放全人类”为理想,在全身瘫痪、双目失明的情况下,仍未放弃“战斗”。他选择文学创作实现理想,先用硬纸板做成框子来写作,后来自己口述,请人代录。在母亲和妻子的帮助下,用生命写成了《暴风雨所诞生的》。保尔克服肉体和精神上的痛苦,坚持不懈地创作,体现了保尔理想信念的坚定。是保尔让我明白人生路上所遇到的磨砺,会让我们变得更加坚强。我们应坚定理想信念,不断奋斗。示例二:孙悟空是《西游记》中的人物,当他得知弼马温这一官职只是一个养马的卑贱官职后,大闹天宫,从中我感受到了孙悟空勇于追求自由,敢于反抗的精神。这启示我们要勇于追求自由,面对不公正的事情要敢于反抗。示例三:尼摩船长是法国著名小说家儒勒•凡尔纳的小说《海底两万里》中的人物,“鹦鹉螺号”潜水艇的船长。尼摩船长自信、勇敢,他毫不畏惧地带领船员们克服了“鹦鹉螺号”搁浅、被土著人围攻、与鲨鱼搏斗、冰山封路、章鱼袭击等重重困难,启示我们要不怕困难。示例四:藤野先生是鲁迅先生的恩师,他收到“我”讲义第二天便还给了“我”,并且还从头到尾用红笔进行添改。从中可以看出藤野先生是一个认真负责,细心细致之人。我们在今后的学习中也要做一个认真负责,细心细致之人。33.中国书信文化,从远古走来,绵延至今。见字如面,书信有一种神奇的力量。为吸引更多的同学去阅读书信类作品,请从下列作品中任选一部,写一则简短的推荐语。(1)《傅雷家书》(傅雷) (2)《给青年的十二封信》(朱光潜)【答案】示例一:我推荐《傅雷家书》。两地书,父子情。傅雷通过书信的形式关心在外求学的儿子的生活、事业,与儿子谈做人、文学、艺术等话题,指导、激励儿子做德艺兼备、人格卓越的艺术家。阅读这本书,我们能获得思想的启迪、艺术的熏陶、做人的道理,还可以学习如何和父母沟通相处等。示例二:我推荐朱光潜的《给青年的十二封信》。作者与青年人谈人生修养、谈文学、谈艺术、谈学习生活等,亲切平等的对话方式、优美的散文笔调、生动的比喻说理,把深刻的人生道理讲得有理有趣,给我们指明人生的方向,解决生活中的难题。(抓住每本书的主旨、特色以及有益启示进行推荐,从而答到吸引阅读的目的即可)【解析】本题考查推荐语。推荐语重在“推介”,目的在于让人喜欢。一般写法:首先告诉大家文章的主要内容,然后介绍这篇文章值得推荐的地方,即文章的主要特点;可从主题理解、语言风格、文章写作方法等方面入手。示例一:我推荐《傅雷家书》。《傅雷家书》是文艺评论家傅雷写给儿子的书信集,十二年通信数百封,贯穿着傅聪出国学习、演奏成名到结婚生子的成长经历,映照着傅雷的翻译工作、朋友交往以及傅雷一家的命运起伏。将父亲对孩子的爱都藏在字里行间,他的爱润物无声,他的爱坚实到风雨不能剥蚀,令读者为之动容。示例二:我推荐朱光潜的《给青年的十二封信》。从短短的十二封信中,可以看出朱先生的确博闻强识,善于思辨。在告诫青年的信中,他又是务实的,真诚的。朱光潜先生在书里谈生活,谈理想、谈学习、谈人生,在多角度、多层次剖析社会现象的基础上点明了青年人的发展方向。,给我们以启迪。34.为积极响应国家“精准扶贫”政策,班级将捐献一批图书给山区的学生,请你从下列书中任选一本,参照已给出的示例,在扉页上书写赠语。①《艾青诗选》 ②《水浒传》示例:《红星照耀中国》是一部纪实作品,记录了长征时期作者在我国西北革命根据地实地采访的所见所闻。我希望收到这本书的你,永远铭记在曾经的中国大地上,有一群红军战士抛头颅洒热血,他们艰苦奋斗、不畏牺牲,只为了让中国人民过上美好的生活。【答案】示例一:《艾青诗选》是一部诗集,“土地”、“太阳”是其中常见的诗歌意象,寄托着诗人艾青对祖国真挚浓烈的热爱,以画人境,诗韵悠然。我希望收到这本书的你,能沉浸在艾青的“诗歌国度”,读诗,品画,入情。示例二:《水浒传》是一部古典小说,描写了以宋江为首的梁山好汉们反抗欺压,揭竿而起,聚义梁山,直至接受招安,最后失败的故事。四海之内皆兄弟,我希望收到这本书的你,也能和梁山好汉们一样,结交志同道合、胸怀忠义的人生挚友。【解析】本题考查语言表达。在书的扉页上写赠语,实际上就是给书写推荐语。示例:《艾青诗选》是一部真正的名家名作,诗坛泰斗的精品。解放前,艾青以画家的身份,进入诗坛,他追求光明,热爱土地,他脚踏大地,手持火把,心向太阳。一首朴实无华的《大堰河我的保姆》感动了无数代人。解放后备受打击,但矢志不移,作为归来的诗人,他焕发创作的第二个春天,《鱼化石》富含哲理,发人深思。 《水浒传》是一部是一部有审美价值和社会意义的中国优秀长篇小说。人物个性鲜明有血有肉,栩栩如生,跃然纸上,最大特点是作者善于把人物置身于真实的历史环境中,扣紧人物的身份、经历和遭遇来刻画他们的性格。35.学校将在世界读书日举行“好书我推荐”活动,你将带着《昆虫记》参赛。请你用简洁的语言为这部书写一段精彩的推荐语。120字左右。【答案】示例:《昆虫记》是法国杰出昆虫学家、文学家法布尔的传世佳作。书中作者对昆虫最直观的研究记录,影响了无数科学家、文学家及普通大众,其文学及科学非凡的成就受到举世推崇。全文用大量篇幅介绍了昆虫的生活习性,行文优美, 生动活泼,充满了盎然的情趣和诗意,被公众认为跨越领域、超越年龄的不朽传世经典!【解析】本题考查语言表达。推荐语,重在“推介”,目的在于让人喜欢。一般写法:首先告诉大家文章的主要内容,然后介绍这篇文章值得推荐的地方,即文章的主要特点。可以集中阐述其中的一个方面,也可以阐述其几个主要方面。一般可从主题理解、语言风格、文章写作方法等方面入手。本题的开放性较大,可根据自身对《昆虫记》知识的日常阅读积累,阐释自己的观点。符合经典阅读的主要思想,合情合理即可。示例:《昆虫记》是法国杰出的昆虫学家、文学家法布尔耗费毕生心血写成的一部昆虫学的传世佳作。全书详细、深刻地展示了蜘蛛、蜜蜂、螳螂、蝎子、蝉、甲虫、蟋蟀等多昆虫的生活情景。在对昆虫的本能、习性、劳动、婚恋、繁衍、死亡的描述中,无不渗透着人文关怀,并以生物性反观人性。全书充满了对生命的关爱之情和对自然万物的赞美。阅读下面文字,完成下面小题。自此严监生的病,一日重似一日,毫无起色。诸亲六春都来问候。五个侄子穿梭地过来陪郎中弄药。到中秋以后,医家都不下药了。把管庄的家人都从乡里叫了上来。病重得一连三天不能说话。晚间挤了一屋的人。桌上点着一盏灯。严监生喉咙里痰响得一进一出,一声接一声的,总不得断气。还把手从被单里拿出来,伸着两个指头。大侄子走上前来问道:“二叔,英不是还有两个亲人不曾见面?”他就把头摇了两三摇。二侄子走上前来问道,“二叔,莫不是还有两笔银子在那里,不曾吩咐明白?”他把两眼睁得溜圆,把头又狠狠摇了几摇,越发指得紧了。奶妈抱着儿子插口道:“老爷想是因两位舅爷不在眼前,故此惦念。”他听了这话,两眼闭着摇头,那手只是指着不动。赵氏慌忙揩揩眼泪,走近上前道:“老爷,别人都说的不相干,只有我晓得你的意思!"……赵氏分开众人,走上前道:“老爷,只有我能知道你的心事。你是为灯盏里点的是两茎灯草,不放心,恐费了油。我如今挑掉一茎就是了”说罢,忙走去挑掉一茎。众人看严监生,点一点头,把手垂下,登时就没了气……36.下列说法正确的选项是( )A.范进是《儒林外史》的中心人物,从而把众多的故事连缀起来,表现出了普通士人日常生活中的生存状态与精神世界。B.《儒林外史》塑造了很多淡泊名利、恪守道德的贤者,寄托了作者对理想社会的追求。C.《儒林外史》的讽刺艺术突出表现在对精彩情节的刻画上,范进中举就是其中一道亮丽的风景线。D.《儒林外史》中的人物常常在登场数回之后,旋即退场,从此不再出现。37.在赵氏之前有好几个人猜测严监生为什么“伸着两个指头”的原因,作者为什么这样写,而没有一下子就让赵氏来说?38.严监生为多燃了一根灯草而迟迟不肯咽气,这极具讽刺效果。请根据你的阅读积累。再列举出一部文学作品中采用这种写作手法的相关情节。【答案】36.D 37.前面的人所猜测的内容,为赵氏的话做铺垫,可以起到更好地衬托的艺术效果,侧面写出了严监生的吝啬。38.示例:周进在贡院内哭得口吐鲜血,只因为自己一辈子没考中。当听到人们原意出银子资助他捐一个监生的时候,竟然跪下磕头感谢众人说是“再生父母”。【解析】36.本题考查内容理解。A.“范进是中心人物”有误。清代小说家吴敬梓的长篇讽刺小说《儒林外史》没有贯穿全书的中心人物和主要情节,是由众多故事连缀而成,表现的是普通士人日常生活中的生存状态与精神世界。《范进中举》就是其中一个故事;B.“塑造了很多”有误。在《儒林外史》中,作者将科举制度作为主要揭露和讽刺的对象,并通过书中少数淡泊名利、恪守道德的贤者,寄托了自己对理想社会的追求;C.“表现在对精彩情节的刻画上”有误。这篇小说的讽刺性得益于精心巧妙的情节设置。小说不长,但却一波三折,极富戏剧性。其线索可用三个字来描述:盼一赶一避。这种戏剧性突转的巧妙之处在于,阅读期待视野与结果的完全相反,从而达到一种绝妙的讽刺效果;故选D。37.考查写作手法。结合“大侄子走上前来问道:‘二叔,英不是还有两个亲人不曾见面?’他就把头摇了两三摇。二侄子走上前来问道,‘二叔,莫不是还有两笔银子在那里,不曾吩咐明白?’他把两眼睁得溜圆,把头又狠狠摇了几摇,越发指得紧了。奶妈抱着儿子插口道:‘老爷想是因两位舅爷不在眼前,故此惦念’”等内容可知,作者描写赵氏之前有好几个人猜测严监生为什么“伸着两个指头”的原因,意在为后文赵氏的出场做铺垫;联系“赵氏分开众人,走上前道:‘老爷,只有我能知道你的心事。你是为灯盏里点的是两茎灯草,不放心,恐费了油。我如今挑掉一茎就是了’说罢,忙走去挑掉一茎。众人看严监生,点一点头,把手垂下,登时就没了气”可知,以众人的猜测引出赵氏正确的作答,具有更好的衬托作用,同时更能突出严监生的吝啬,增强了文章的讽刺效果。38.本题考查阅读拓展。答案不唯一。示例:《格列佛游记》是乔纳森·斯威夫特的一部杰出的游记体讽刺小说,出版两个多世纪以来,被翻译成几十种语言,在世界各国广为流传。作品运用讽刺影射的手法,反映了18世纪前半期,英国的社会矛盾,揭露了当时统治集团的腐败和罪恶,并抨击了侵略战争和殖民主义。阅读《简·爱》节选部分,完成下面小题。①我一路抵抗。以前我从未如此大胆,贝熙和阿伯特小姐原本便已听了许多关于我的坏话,这时更加认定我是一个坏孩子。其实我只是不知所措,或者像法国人说的那样,无法自控。我深知刚才一瞬间的负隅顽抗,会让我遭受各种难以想象的酷刑。但绝望透顶的我就像造反的奴隶,下定决心抗拒到底。②“抓紧她的胳膊,阿伯特小姐。她就像一只发疯的猫。”③“不要脸,不要脸,”女主人的丫鬟说,“你太过分了,爱小姐,你怎么可以动手打一个年轻的绅士,他可是你的恩主的儿子!是你的少爷。”④“什么少爷!他怎么可能是我的少爷?难道我是佣人吗?”⑤“不是,你比佣人还不如,因为你光吃饭不干活。去那边坐下,好好反省你刚才邪恶的行为。”⑥这时她们已经按照瑞德太太的吩咐,把我带进那个房间,扔到一张圆凳上。我本能地像弹簧一样跳起来,她们立刻用双手摁住我。⑦“如果你不老老实实坐好,我就把你绑起来,”贝熙说,“阿伯特小姐,你的袜带借我用一下,我的太细,她一挣扎就会断。”⑧阿伯特小姐转过身去,准备从粗壮的小腿上卸下那根必不可少的绳索。想到等下被绑起来多么丢人,我不禁有点慌张。⑨“别脱了,”我哭着说,“我不会乱动的。”⑩我双手抓住座位,坐得端端正正。⑪“最好是。”贝熙说。她看到我确实没有反抗,便不再紧紧摁着我,而是和阿伯特小姐抱着手,黑着脸站在旁边,盯着我的眼神充满狐疑,似乎并不相信我已恢复了理智……39.请用简洁的语言概括选段内容。40.请根据选段的内容,简要概括简爱的个性特点。(两点即可)【答案】39.答案示例1:简爱被认为是坏孩子(因为打恩主的儿子)而被阿伯特等人抓进房间反省。答案示例2:简爱因为打了恩主的儿子而受到惩罚(被要求反省)。 40.有反抗精神有自尊心、勇敢。【解析】39.本题考查名著情节的概括。阅读材料,联系第①段“贝熙和阿伯特小姐原本便已听了许多关于我的坏话,这时更加认定我是一个坏孩子”及第⑥段“这时她们已经按照瑞德太太的吩咐,把我带进那个房间,扔到一张圆凳上”可知,选文内容为简·爱被关进红房子的故事情节。简·爱父母双亡,从小寄住在里德太太家中,简·爱的表哥约翰·里德自私、霸道,一直在欺侮简·爱。一天,约翰·里德找到在早餐室的窗座上看书的简·爱,责骂她,并拿书扔简·爱,使她的头部受伤,简·爱发了疯似的和他对打起来,也因此惹怒了里德太太,被关到了红房子里。可概括故事情节为:简·爱被认为是坏孩子(因为打恩主的儿子)而被阿伯特等人抓进房间反省。40.本题考查名著中人物形象的分析。选文中有大量关于简·爱的语言描写和心理描写。由“我一路抵抗”“我深知刚才一瞬间的负隅顽抗,会让我遭受各种难以想象的酷刑。但绝望透顶的我就像造反的奴隶,下定决心抗拒到底”“我本能地像弹簧一样跳起来,她们立刻用双手摁住我”等描写可以看出可以看出简·爱具有反抗精神,且十分勇敢;由“什么少爷!他怎么可能是我的少爷?难道我是佣人吗?”可以看出简·爱有自己的自尊心,追求平等。按要求完成各题。[文段一]“真的,我得走!”她有点恼火了,她反驳说,“你以为我会留下来,成为你觉得无足轻重的人吗?……你以为,因为我穷、低微、不美、矮小,我就没有灵魂没有心吗?你想错了!一我的灵魂跟你的一样,我的心也跟你的完全一样!要是上帝赐予我一点美和一点财富,我就要让你感到难以离开我,就像我现在难以离开你一样。我现在跟你说话,并不是通过习俗、惯例,甚至不是通过凡人的肉体——而是我的精神在同你的精神说话;就像两个都经过了坟墓,我们站在上帝脚跟前,是平等的——因为我们是平等的!”[文段二]沈琼枝在宋家过了几天,不见消息,想道:“彼人一定是安排了我父亲,再来和我歪缠。不如走离了他家,再作道理。”将他那房里所有动用的金银器皿、珍珠首饰,打了一个包袱,穿了七条裙子,扮做小老妈的模样,买通了那丫鬟,五更时分,从后门走了,清晨出了门上船。那船是有家眷的。沈琼枝上了船,自心里想道:“我若回常州父母家去,恐惹故乡人家耻笑。”细想:“南京是个好地方,有多少名人在那里。我又会做两句诗,何不到南京去卖诗过日子?或者遇着些缘法出来也不可知。”立定主意,到仪征换了江船,一直往南京来。吴敬梓《儒林外史》41.聚焦典型:文段(一)中的“我”是____________ (填人名),她是世界文学画廊中的典型形象,代表着____________的性格。42.专题探究:文段(一)中的“我”和文段(二)中的沈琼枝有一个共同的行动“走”,请你分析两人在“走”这件事情上原因的异同。43.延伸阅读:近年来,“她力量”逐渐成为社会形容女性群体的关键词,结合小说和生活谈谈你对“她力量”的理解。【答案】41.简•爱 自尊自主、叛逆反抗、追求平等(意思对即可)42.沈琼枝反抗盐商宋为富的婚姻欺骗行为,不愿做妾,逃离宋家到南京。简•爱积极进取,她敢于反抗、敢于追求爱情,追求平等。她们都聪明伶俐、敢于反抗、自尊自爱、独立自强。43.“她力量”指的是女性身上展现出的意气风发、斗志昂扬的力量,具有“她力量”的女性不屈从于压迫,敢于反抗,高度自尊自立,以自身的智慧化解生活中的种种不如意等。例如奥运冠军谷爱凌,刻苦学习艰苦训练,让她以自信的面貌赢得了世界的喝彩。【解析】41.本题考查识记名著内容和分析形象。结合“因为我穷、低微、不美、矮小,我就没有灵魂没有心吗?你想错了!一我的灵魂跟你的一样,我的心也跟你的完全一样!”可知,人物是《简•爱》中的简•爱。性格:“你以为我会留下来,成为你觉得无足轻重的人吗?”写简•爱敢于反抗;“你以为,因为我穷、低微、不美、矮小,我就没有灵魂没有心吗?你想错了!一我的灵魂跟你的一样,我的心也跟你的完全一样!”写简•爱坚强,独立自主,追求尊严人格;“就像两个都经过了坟墓,我们站在上帝脚跟前,是平等的——因为我们是平等的!”写她追求自由平等。42.本题考查比较阅读。文段(一)写罗切斯特试探简·爱,告诉简·爱他要结婚了,新娘就是英格拉姆小姐,并推荐简·爱去爱尔兰做家庭教师。简·爱伤心极了,之后对罗切斯特说的这段话。根据“你以为我会留下来,成为你觉得无足轻重的人吗?……你以为,因为我穷、低微、不美、矮小,我就没有灵魂没有心吗?你想错了!——我的灵魂跟你的一样,我的心也跟你的完全一样”可知,简·爱对自己的命运、价值、地位的思考和努力把握,对自己的思想和人格有着理性的认识,对自己的幸福和情感有着坚定的追求。表现了简·爱的自尊、自主、自立,具有反抗精神和独立精神,追求精神上的自由。文段(二)出自《儒林外史》,沈琼枝是一个很鲜明的女性人物形象,常州才女,善诗书、重名节。因不从盐商宋为富,父女遭迫害,只身流落金陵,以卖诗顾绣为生。后参殿试,中同进士。“彼人一定是安排了我父亲,再来和我歪缠。不如走离了他家,再作道理”写她敢于反抗封建婚姻制度,敢于同封建势力做坚决斗争,“我若回常州父母家去,恐惹故乡人家耻笑”写她聪明智慧;“南京是个好地方,有多少名人在那里。我又会做两句诗,何不到南京去卖诗过日子?或者遇着些缘法出来也不可知”写她追求独立自主的生活。可见,二人都聪明伶俐、敢于反抗、自尊自爱、独立自强。43.本题考查理解名著内容和联系实际的能力。“她力量”是指女性群体在社会各个领域产生的不可忽视的力量,“她力量”包括女性本身的力量、女性和男性融合的力量、女性对男性激励的力量。简·爱自尊、自主、自立,具有反抗精神和独立精神,追求精神上的自由。沈琼枝敢于反抗封建婚姻制度,敢于同封建势力做坚决斗争,不慕财富,不惧官府,有文才、有胆识、有志气,力图掌握自己的命运,努力维护妇女的独立人格。二人都具有“她力量”。实际生活中,很多女性具有“她力量”,如北京2022年冬残奥会残奥高山滑雪女子滑降(坐姿)季军刘思彤,不因自身的残疾而自卑胆怯,敢于追求梦想。通过高山滑雪运动,在不断挑战自己的过程中接受了自己身体上的不完美,以为国争光的荣耀,为自己的生命增添意义和价值。没羽箭飞石打英雄①徐宁飞马直取张清,两马相交,双枪并举。斗不到五合,张清便走,徐宁去赶。张清把左手虚提长枪,右手便向锦袋中摸出石子,扭回身,觑得徐宁面门较近,只一石子,可怜悍勇徐宁,石子眉心早中,翻身落马。龚旺、丁得孙便来捉人。宋江阵上人多,早有吕方、郭盛,两骑马,两枝戟,救回本阵。宋江等大惊,尽皆失色。再问:“那个头领接着厮杀?”宋江言未尽,马后一将飞出。看时,却是锦毛虎燕顺。宋江却待阻当,那骑马已自去了。燕顺接住张清,斗无数合,遮拦不住,拨回马便走。张清望后赶来,手取石子,看燕顺后心一掷,打在镗甲护镜上,铮然有声,伏鞍而走。宋江阵上一人大叫:“匹夫何足惧哉!”拍马提槊飞出阵去。宋江看时,乃是百胜将韩滔,不打话便战张清。两马方交,喊声大举。韩滔要在宋江面前显能,抖擞精神,大战张清。不到十合,张清便走。韩滔疑他飞石打来,不去追赶。张清回头不见赶来,翻身勒马便转。韩滔却待挺槊来迎,被张清暗藏石子,手起,望韩滔鼻凹里打中。只见鲜血迸流,逃回本阵。彭玘见了大怒,“量这等小辈,何足惧哉!”不等宋公明将令,手舞三尖两刃刀,飞马直取张清。两个未曾交马,被张清暗藏石子在手,手起,正中彭玘面额,丢了三尖两刃刀,奔马回阵。②宋江见输了数将,心内惊惶,便要将军马收转。只见卢俊义背后一人大叫:“今日将威折了,来日怎地厮杀!且看石子打得我么!”宋江看时,乃是丑郡马宣赞,拍马舞刀,直奔张清。张清便道:“一个来,一个走!两个来,两个逃!你知我飞石手段么?”宣赞道:“你打得别人,怎近得我!”说言未了,张清手起一石子,正中宣赞嘴边,翻身落马。龚旺、丁得孙却待来捉,怎当宋江阵上人多,众将救了回阵。③宋江见了,怒气在心,掣剑在手,割袍为誓:“我若不拿得此人,誓不回军!”呼延灼见宋江说誓,便道:“兄长此言,要我们弟兄何用!”就拍踢雪乌骓,直临阵前,大骂张清:“小儿得宠,一力一勇!认得大将呼延灼么?”张清便道:“辱国败将之人,也遭我毒手!”言未绝,一石子飞来。呼延灼见石子飞来,急把鞭来隔时,却中在手腕上,早着一下,便使不动钢鞭,回归本阵。④宋江道:“马军头领,都被损伤。步军头领,谁敢捉这张清?”只见部下刘唐手拈朴刀,挺身出阵。张清见了大笑,骂道:“你那败将,马军尚且输了,何况步卒!”刘唐大怒,径奔张清。张清不战,跑马归阵。刘唐赶去,人马相迎。刘唐手疾,一朴刀砍去,却砍着张清战马。那马后蹄直踢起来,刘唐面门上扫着马尾,双眼生花,早被张清只一石子,打倒在地。急待挣扎,阵中走出军来,横拖倒拽,拿入阵中去了。宋江大叫:“那个去救刘唐?”只见青面兽杨志便舞刀直取张清。张清虚把枪来迎。杨志一刀刺去,张清镫里藏身,杨志却砍了个空。张清手拿石子,喝声道:“着!”石子从肋窝里飞将过去。张清又一石子,铮的打在盔上,吓得杨志胆丧心寒,伏鞍归阵。[A]⑤宋江看了,辗转寻思:“若是今番输了锐气,怎生回梁山泊!谁与我出得这口气?”朱仝听得,目视雷横说道:“捉了刘唐去,却值甚的!一个不济事,我两个同去夹攻。”朱仝居左,雷横居右,两条朴刀,杀出阵前。张清笑道:“一个不济,又添一个!由你十个,更待如何!”全无惧色。在马上藏两个石子在手。雷横先到,张清手起,势如招宝七郎,石子来时,面门上怎生躲避,急待抬头看时,额上早中一石子,扑然倒地。朱仝急来快救,脖项上又一石子打着。关胜在阵上看见中伤,大挺神威,轮起青龙刀,纵开赤兔马,来救朱仝、雷横。刚抢得两个奔走还阵,张清又一石子打来。关胜急把刀一隔,正打着刀口,迸出火光。关胜无心恋战,勒马便回。[B]⑥双枪将董平见了,心中暗忖:“吾今新降宋江,若不显我些武艺,上山去必无光彩。”手提双枪,飞马出阵。张清看见,大骂董平:“我和你邻近州府,唇齿之邦,共同灭贼,正当其理。你今缘何反背朝廷?岂不自羞!”董平大怒,直取张清。两马相交,军器并举。两条枪阵上交加,四双臂环中撩乱。约斗五七合,张清拨马便走。董平道:“别人中你石子,怎近得我!”张清带住枪杆,去锦袋中摸出一个石子,手起处真如流星掣电,石子来吓得鬼哭神惊。董平眼明手快,拨过了石子。张清见打不着,再取第二个石子,又打将去,董平又闪过了。两个石子打不着,张清却早心慌。那马尾相衔,张清走到阵门左侧,董平望后心刺一枪来。张清一闪,镫里藏身,董平却搠了空,那条枪却搠将过来。董平的马和张清的马两厮并着。张清便撇了枪,双手把董平和枪连臂膊只一拖,却拖不动。两个搅做一团。(选自《水浒传》,作者:施耐庵)44.读书要梳理:选文第④段画横线的宋江话语,在行文结构上有什么妙处?45.读书要鉴赏:选文[A][B]两处加点句写得同中有异,异中有同,试作分析。46.读书要探究:张清飞石连伤数将,然而两次都未能打中董平,这是为什么?47.读书要勾连:书中另一好汉叫“张青”,试从其绰号、事迹方面简要介绍。【答案】44.承上启下(过渡) 45.两句都在于凸显张清飞石了得,打得水浒英雄败下阵来。A 句写飞石打在盔上,侧重声音,B 句写飞石打在刀口,侧重火光,写得有声有色,异彩纷呈。46.因为董平新降宋江,想要在梁山好汉面前显些武艺,以免上山后脸上无光,所以打斗时他格外卖力而小心。47.张青,绰号“菜园子”,与孙二娘结为夫妻,曾在十字坡开设酒店,以蒙汗药伤害过往行人,卖人肉包子,后随二龙山众头领加入梁山泊。【解析】44.本题考查句段作用。第④段中宋江的话“马军头领,都被损伤”承接上文徐宁、燕顺、韩滔、彭玘、宣赞、呼延灼被张清的石子打伤;“步军头领,谁敢捉这张清”引出下文刘唐等与张清征战的情节。故画线的宋江话语在文中起了承上启下的过渡作用。45.本题考查句子赏析。[A]句“铮”是拟声词,从听觉的角度写出张清飞石打在“盔上”时的声音,“吓得杨志胆丧心寒,伏鞍归阵”以杨志的表现侧面写出张清打出石子的威力;[B]句“迸出火光”,从视觉的角度写出张清飞石打在刀口时火光飞溅的情况,“关胜无心恋战,勒马便回”以关胜的表现侧面写出张清打出石子的威力。根据以上分析,写出[A][B]句描写的异同点。46.本题考查内容理解。由选文第⑥段“董平见了,心中暗忖:‘吾今新降宋江,若不显我些武艺,上山去必无光彩。’”“董平眼明手快,拨过了石子”可知,董平认为新降宋江,想要在梁山好汉面前显些武艺,以免上山后脸上无光,所以打斗时他格外卖力而小心。47.本题考查名著阅读积累。张青,本在孟州道光明寺种菜,因此唤做“菜园子”,却因为小事杀了光明寺里的僧人,逃出后在大树坡做劫匪,结识了孙二娘,二人结为夫妻,便在十字坡开设酒店,用蒙汗药杀死过往行人,做人肉包子的生意。后来跟随二龙山众头领加入梁山泊。梁山大聚义时一百零八将,坐第一百零二把交椅,司职“打探声息”、“邀接来宾头领”并负责管理“西山酒店”,星号地刑星。在征讨方腊接近尾声的歙州之战时,恰逢摩云金翅欧鹏阵亡,军心大乱之际,张青在乱军之中战死,死后追封:义节郎。48.学校开展“阅读名著,分享经典”活动,阅读名著要善于运用对比勾连的方法,请你完成以下任务。语段一:①这沙僧倒身下拜,拜罢抬头正欲告诉前事,忽见孙行者站在旁边,等不得说话,就掣降妖杖望行者劈脸便打。这行者更不回手,彻身躲过。沙僧口里乱骂道:“我把你个犯十恶造反的泼猴!你又来影瞒菩萨哩!”菩萨喝道:“悟净不要动手,有甚事先与我说。”沙僧收了宝杖,再拜台下,气冲冲的对菩萨道:“这猴一路行凶,不可数计。……他就帅众拿我,是我特来告请菩萨。不知他会使筋斗云,预先到此处,又不知他将甚巧语花言,影瞒菩萨也。”菩萨道:“悟净,不要赖人,悟空到此今已四日,我更不曾放他回去,他那里有另请唐僧、自去取经之意?”沙僧道:“见如今水帘洞有一个孙行者,怎敢欺诳(欺骗迷惑)?”②菩萨道:“既如此,你休发急,教悟空与你同去花果山看看。是真难灭,是假易除,到那里自见分晓。”这大圣闻言,即与沙僧辞了菩萨。(节选自《西游记》)语段二:李逵已听得了,便道:“叵耐这厮!我倒与了他一个银子,又饶了性命,他倒又要害我。这个正是情理难容!”一转踅到后门边。这 A 却待出门,被李逵劈揪住。那妇人慌忙自望前门走了。李逵捉住 A ,按翻在地,身边掣出腰刀,早割下头来。拿着刀,却奔前门寻那妇人时,正不知走那里去了。再入屋内来,去房中搜看,只见有两个竹笼,盛些旧衣裳,底下搜得些碎银两并几件钗环,李逵都拿了。又去 A 身边搜了那锭小银子,都打缚在包裹里。却去锅里看时,三升米饭早熟了,只没菜蔬下饭。李逵盛饭来,吃了一回,看着自笑道:“好痴汉!放着好肉在面前,却不会吃!”吃得饱了,把 A 的尸首拖放屋下,放了把火,提了朴刀,自投山路里去了。那草屋被风一扇,都烧没了。有诗为证:劫掠资财害善良,谁知天道降灾殃。家园荡尽身遭戮,到此翻为没下场。(节选自《水浒传》)(1)语段一中“水帘洞有一个孙行者”是__________(填名),沙僧在菩萨面前见到行者劈脸便打,是因为沙僧误认为是行者打晕唐僧,抢走行李和通关文牒。语段二中A是________(填人名),李逵说“与了他一个银子,又饶了性命”,是因为动了恻隐之心。(2)上面两语段出自“真假美猴王”“真假李逵”,结合“假者”最后的结局以及语段二划线的诗句,谈谈这两个故事给你的启示。(3)古典小说中有许多关于“闹”的情节,请结合自己的阅读体验,完成表格。(4)今年是农历兔年,小樊从两部名著中找到有关于“兔”的情节,同学们对此各抒己见,请把他们的对话补充完整。小襄:我觉得《西游记》中关于兔子的描绘很精彩。玉兔①______(简述情节:在哪儿干什么),想诱取唐僧元阳,后被悟空识破,太阴星君把玉兔收走。小阳:《西游记》的描绘是不错,但我觉得《水浒传》中关于兔子的描写颇有特色。李逵回乡接老娘,途中遇一白兔。他动了童心,一路追赶,那兔子跑得无影无踪。②___(简要赏析),对比鲜明、描写精妙。【答案】(1)六耳弥猴 李鬼(2)示例1:无论是假悟空,还是假李逵,二者虽心狠手辣,但最终都被打死,罪有应得。示例2:真的假不了,假的真不了,人要努力做好自己,切不可邯郸学步、东施效颦。(3)孙悟空大闹天宫 嫉恶如仇 重情重义(4)在天竺国假扮公主; 李逵五大三粗,形如黑塔,兔儿小巧玲珑,状如白团;李逵笨拙,兔儿灵敏(能指出一项对比即可)。【解析】(1)本题考查名著故事情节和人物识记。《西游记》“真假美猴王”故事情节:唐僧因悟空又打死拦路强盗,再次把他撵走。六耳狝猴精趁机变作悟空模样,抢走行李关文,又把小妖变作唐僧、八戒、沙僧模样,欲上西天骗取真经。沙僧从观音处找来悟空,真假猴王大战,观音、玉帝、唐僧、阎王等无法分辨。最后如来佛识破六耳猕猴,悟空一棍将他打死,师徒团聚。《水浒传》“真假黑旋风”故事情节:李逵上梁山后,思母心切,为了让母亲享福,特地回家去接老母亲上梁山。途中路过一片险恶的松林。在树林里,李逵遇到了一个冒充自己的面黑如炭的大汉李鬼在此拦路抢劫,李逵识破了他的阴谋,就要动手杀掉他,对方却谎称自己因老母饥饿才出此下策,李逵心软,放了他。这时饥饿的李逵也想找地方吃饭,恰巧遇到一户人家,没想到却是那冒充李逵的恶徒的家,他们想在饭中放药把李逵毒死,恰好被李逵听见,李逵将其杀死,并烧掉了贼窝。(1)根据语段一“这沙僧倒身下拜,拜罢抬头正欲告诉前事,忽见孙行者站在旁边,等不得说话,就掣降妖杖望行者劈脸便打……沙僧道:‘见如今水帘洞有一个孙行者,怎敢欺诳(欺骗迷惑)?’”可知,选文出自《西游记》“真假美猴王”,回顾原著内容可知,“水帘洞有一个孙行者”是六耳弥猴;(2)根据语段二“李逵已听得了,……有诗为证:劫掠资财害善良,谁知天道降灾殃。家园荡尽身遭戮,到此翻为没下场”可知,选文出自《水浒传》“真假黑旋风”,回顾原著内容可知,A是李鬼。(2)本题考查启示。根据“真假美猴王”和“真假李逵”故事结局,“假者”都被杀死,结合语段二划线诗句“劫掠资财害善良,谁知天道降灾殃。家园荡尽身遭戮,到此翻为没下场”可谈:看事情要全面,不能只看表面要通过表面看本质;要不怕困难勇于面对挫折,敢于和困难及邪恶势力作斗争;假的真不了,邪恶势力一定会打败;等等。示例3:做人要正直,心怀鬼胎、恩将仇报,终是害人害己。(3)本题考查名著情节识记和人物形象理解。孙悟空大闹天宫:孙悟空去东海龙宫抢了金箍棒,又去地府强销生死簿。龙王、阎君去天庭告状,玉帝把孙悟空召入天庭,授他做弼马温。悟空嫌官小,打回花果山,自称“齐天大圣”。玉帝派天兵天将捉拿孙悟空,没有成功,便让孙悟空管理蟠桃园。孙悟空偷吃蟠桃,毁了王母的蟠桃宴,又偷吃了太上老君的金丹后逃离天宫。玉帝再派李天王率天兵捉拿;观音菩萨举荐二郎真君助战;太上老君在旁使暗器帮助,最后悟空被擒。悟空被刀砍斧剁、火烧雷击,甚至置丹炉锻炼四十九日,依然无事,还在天宫大打出手。玉帝降旨请来如来佛祖,才把孙悟空压在五行山下。鲁智深大闹野猪林:林冲的上司太尉高俅义子高衙内霸占林冲的妻子不成,便设下毒计要害林冲。于是,林冲被判了重刑,刺配沧州,又用重金收买了押送的差役,让他们在半路上结果林冲的性命。走了不几天,来到一个险恶的地方,叫野猪林。正准备下手,只听“当啷”一声,薛霸的棍子没有打着林冲,却被打飞了。从树后跳出一个胖大和尚,抡起禅杖就打,吓得两个差役慌忙跪地求饶。林冲睁眼一看,原来是鲁智深前来相救。鲁智深听说林冲被发配沧州,一直在暗地里跟着,果然在野猪林救了林冲。并一路护送着林冲到了沧州。(1)根据题干《西游记》、“‘闹’的原因”是“嫌弼马温官小,又怒王母娘娘未请他参加蟠桃会”“人物形象”是“桀骜不驯敢于反抗”可知,“闹”的情节是孙悟空大闹天宫;(2)根据题干《水浒传》、“‘闹’的情节”是“鲁智深大闹野猪林”、“‘闹’的原因”是“林冲被公差押解至野猪林时,公差要杀害林冲”,结合原著鲁智深与林冲是结义兄弟和两位公差要助纣为虐杀林冲,可知“人物形象”是嫉恶如仇、重情重义。(4)本题考查名著情节识记与赏析,《西游记》天竺国故事情节:玉兔精原是广寒宫捣药的玉兔,在广寒宫时被素娥仙子打了一拳。因而怀恨。素娥下界投生天竺国皇室,玉兔为报私仇也私自下界,在毛颖山中兴妖作怪,手使一条名叫捣药杵的短棍,善于变化。她摄去素娥转世的天竺国公主,扮成假公主。知道唐僧取经要路经天竺国,想招圣僧为丈夫,采取元阳真气,以便得道成太乙上仙。多亏孙悟空火眼金睛,识破妖精的真相,两人一场大战,不分胜败。孙悟空扔起金箍棒,万千变化,打得妖怪化作清风,逃到南天门上,孙悟空紧追不舍,又打回地上,妖怪难敌悟空的如意金箍棒,遁入毛颖山。孙悟空追到山上,找到妖精,妖精无奈出洞迎战,正当招架不住时,太阴星君和嫦娥仙子赶到,救了妖怪的性命,带回天宫。悟空救出了真公主,那国王自然对唐僧师徒千恩万谢,一番盛情款待后,唐僧师徒继续西去。《水浒传》李逵回乡老娘故事情节。李逵上梁山后,思母心切,为了让母亲享福,特地回家去接老母亲上梁山。路遇李鬼扮成“李逵”剪径,李鬼谎称家中有九十老母,便被李逵放走。后来李逵路过李鬼家,发现李鬼不仅所言不实,且企图害他,便将他杀了。回到家中后,李逵背着双目失明的老母亲奔梁山而去,途中路过沂岭时,李逵放下母亲去接水。回来后,发现老母被老虎所吃。李逵气愤之下杀了一窝四只老虎。(1)根据题干“玉兔 (简述情节:在哪儿干什么),想诱取唐僧元阳,后被悟空识破,太阴星君把玉兔收走”的提示,回顾原著相关情节可知:玉兔在天竺国假扮公主;(2)《水浒传》中描写李逵回乡接老娘,天色渐渐微明,露草之中,突然赶出一只兔子。李逵动了童心,一路追赶,那兔儿跑得无影无踪。李逵笑道:“那畜生倒引了我一程路。”此寥寥-笔,可谓施耐奄神来之笔。李逵五大三粗,形如黑塔,兔儿小巧玲珑,状如白团;李逵笨拙,兔儿灵敏;对比鲜明、描写精妙。四、名句名篇默写 1.根据所给信息默写相应内容。①山舞银蛇,__________________________,欲与天公试比高。(毛泽东《沁园春·雪》)②天接云涛连晓雾,__________________________。(李清照《渔家傲》)③__________________________,山入潼关不解平。(谭嗣同《潼关》)④最爱湖东行不足,__________________________。(白居易《钱塘湖春行》)⑤安得广厦千万间,__________________________!(杜甫《茅屋为秋风所破歌》)⑥《回延安》中,贺敬之重回延安,激动万分:“__________________________,紧紧儿贴在心窝上。”⑦《岳阳楼记》中描写鸟儿时飞时停、鱼儿潜游水中的语句是:“________________________,________________________。”⑧文天祥《过零丁洋》中表现舍生取义、视死如归的坚定信念和昂扬斗志的千古名句是:“__________________________?__________________________。”【答案】原驰蜡象 星河欲转千帆舞 河流大野犹嫌束 绿杨阴里白沙堤 大庇天下寒士俱欢颜 手抓黄土我不放 沙鸥翔集 锦鳞游泳 人生自古谁无死 留取丹心照汗青【解析】本题考查默写古诗文。默写题作答时,一是要透彻理解诗文的内容;二是要认真审题,找出符合题意的诗文句子;三是答题内容要准确,做到不添字、不漏字、不写错字。“蜡、舞、嫌、阴、汗”等字词容易写错。2.用课文原句填空。风景这边独好,“一切都像刚睡醒的样子,(1)________________________”(朱自清《春》)。恋春光,“草树知春不久归,(2)________________________”(韩愈《晚春》);抒豪情,“(3)________________________,便引诗情到碧霄”(刘禹锡《秋词》)。时空不是距离,友谊自在心间,“(4)________________________,________________________”(王勃《送杜少府之任蜀州》);师者无时不有,学习无处不在,“(5)________________________,其不善者而改之”(《论语·述而》)。万里奔赴,扛起的是家国重任,“万里赴戎机,(6)________________________”(《木兰诗》);倾城出猎,抒发的是报国之志,“会挽雕弓如满月,(7)________________________,________________________”(苏轼《江城子·密州出猎》)。【答案】欣欣然张开了眼 百般红紫斗芳菲 晴空一鹤排云上 海内存知己 天涯若比邻 择其善者而从之 关山度若飞 西北望 射天狼【解析】本题考查名句名篇默写。默写题作答时,一要透彻理解诗文内容;二要认真审题,找出符合题意的诗文句子;三是答题内容要准确,做到不添字、不漏字、不写错字。本题中注意“芳菲、晴、涯、邻、择、度”等字词的正确书写。3.默写与运用(1)山不在高,________________________。(刘禹锡《陋室铭》)(2)而今识尽愁滋味,________________________。 (辛弃疾《丑奴儿·书博山道中壁》)(3)____________________________,在乎山水之间也。(欧阳修《醉翁亭记》)(4)傍晚时分,小华漫步山间,看到鸟儿结伴归巢,不由地用陶渊明《饮酒(其五)》中的诗句来形容所见之景:“__________________________,_____________________________。”【答案】有仙则名 欲说还休 醉翁之意不在酒 山气日夕佳 飞鸟相与还【解析】诗词默写要求:一、正确理解诗文句子含义;二、把握题干要求,选择合适的诗文句子填入;三、不添字,不漏字,不错字。本题注意:休、意、夕。4.默写。(1)海内存知己,________________________。(王勃《送杜少府之任蜀州》)(2)________________________,月有阴晴圆缺。(苏轼《水调歌头》)(3)落红不是无情物,________________________。(龚自珍《己亥杂诗》)(4)________________________,自将磨洗认前朝。(杜牧《赤壁》)(5)身不得,男儿列,心却比,男儿烈。 (( )《满江红》)【答案】天涯若比邻 人有悲欢离合 化作春泥更护花 折戟沉沙铁未销 秋瑾【解析】本题中“邻”“化作”“戟”“销”“瑾”等是易错字词,作答时要注意。5.默写。(1)推此志也,____________________________ 。(司马迁《屈原列传》)(2)________________________,凌万顷之茫然。(苏轼《__________________________》(3)杜甫《登高》中“________________________,__________________________”一联由眼前之景联想到沦落他乡、年老多病的处境。【答案】虽与日月争光可也 纵一苇之所如 赤壁赋/前赤壁赋 万里悲秋常作客 百年多病独登台【解析】默写题作答时,一是要透彻理解诗文的内容;二是要认真审题,找出符合题意的诗文句子;三是答题内容要准确,做到不多字,不少字,不写错字。本题中注意“虽、纵、苇、如、常”等字要正确书写。6.用课文原句填空。(1)若夫日出而林霏开,____________________________。(《醉翁亭记》)(2)学而不思则罔,________________________。(《十二章》)(3)_____________________________,寒光照铁衣。(《木兰诗》)(4)_____________________________,要留清白在人间。(《石灰吟》)(5《破阵子·为陈同甫赋壮词以寄之》中,从视觉和听觉两方面,再现紧张激烈的战斗场面的句子是:__________________________,__________________________。(6)“只有站得高,才能看得远”,王之涣写到“欲穷千里目,更上一层楼”,杜甫的《望岳》中“________________________,________________________”与此有异曲同工之妙。【答案】云归而岩穴暝 思而不学则殆 朔气传金柝 粉身碎骨浑不怕 马作的卢飞快 弓如霹雳弦惊 会当凌绝顶 一览众山小【解析】课文原句填空作答时,一是要深刻理解诗文内容;二是要认真审题找出符合题意的诗文的语句;三是作答内容要准确,做到不加字、不少字、不写错字。本题中注意“暝、殆、柝、霹雳、凌”等字词的书写。7.诗、文名句填空。①大漠孤烟直,________________________。(王维《使至塞上》)②忽如一夜春风来,_____________________________ (岑参(白雪歌送武判官归京》)③________________________, 蜡炬成灰泪始干。(李商隐《无题》)④_____________________________,不亦乐乎? (《论语》)⑤龚自珍《己亥杂诗》中表现诗人虽脱离官场,依然关心着国家的命运,不忘报国之志的诗句是:__________________________,___________________________。【答案】长河落日圆 千树万树梨花开 春蚕到死丝方尽 有朋自远方来 落红不是无情物 化作春泥更护花【解析】本题考查名句名篇默写。默写题作答时,一是要透彻理解诗文的内容;二是要认真审题,找出符合题意的诗文句子,三是答题内容要准确,做到不添字、不漏字、不写错字。本题中的“树、梨、蚕、泥”等字容易写错。8.古诗文默写填空。(1)蒹葭苍苍,白露为霜。所谓伊人,________________________。(《诗经·蒹葭》)(2)________________________,________________________。谈笑有鸿儒,往来无白丁。(刘禹锡《陋室铭》)(3)在《破阵子·为陈同甫赋壮词以寄之》中,辛弃疾用“________________________,________________________”表达了率师北伐、收复失地和建功立业的愿望。(4)龚自珍《己亥杂诗》(其五)中的“________________________,________________________”可以用来赞美为实现民族复兴而无私奉献的中华儿女。【答案】在水一方 苔痕上阶绿 草色入帘青 了却君王天下事 赢得生前身后名 落红不是无情物 化作春泥更护花【解析】默写题作答时,一是要透彻理解诗文的内容;二是要认真审题,找出符合题意的诗文句子;三是答题内容要准确,做到不添字、不漏字、不写错字。本题中的“痕、帘、赢”等字容易写错。9.默写。⑴稻花香里说丰年,________________________。(《西江月·夜行黄沙道中》)⑵____________________________,家书抵万金。(《春望》)⑶僵卧孤村不自哀, _____________________________。(《十一月四日风雨大作》)⑷___________________________,不可久居,乃记之而去。(《小石潭记》)⑸至若春和景明,波澜不惊,上下天光,_________________________。 (《岳阳楼记》)⑹___________________________,远小人,此先汉所以兴隆也。(《出师表》)【答案】听取蛙声一片 烽火连三月 尚思为国戍轮台 以其境过清 一碧万顷 亲贤臣【解析】默写题作答时,一是要透彻理解诗文的内容;二是要认真审题,找出符合题意的诗文句子;三是答题内容要准确,做到不添字、不漏字、不写错字。本题中的“蛙、烽、戍、顷”等字词容易写错。10.诗文默写。(1)_____________________________,干戈寥落四周星。(文天祥《过零丁洋》)(2)为什么我的眼里常含泪水?________________________ ……(艾青《我爱这土地》)(3)_____________________________,到乡翻似烂柯人。(刘禹锡《酬乐天扬州初逢席上见赠》)(4)杜甫《茅屋为秋风所破歌》一诗中表现诗人舍己为人的献身精神、博大的胸襟和至死不悔的决心的诗句是:_____________________________,_____________________________!(5)一个时代的性格是青年代表的性格,一个时代的精神是青年代表的精神。梦想越伟大,任务越艰巨,越需要青年迎难而上,担负起时代的重任。请抛弃那“爱上层楼,________________________”(辛弃疾《丑奴儿·书博山道中壁》)故作深沉的无病呻吟吧。遭遇挫折,当有“__________________________,____________________________”(李白《行路难》其一)的壮志豪情;实现追求,当有“_____________________________,____________________________”(李商隐《无题》)的坚贞与无私。无论身处何种境地,只有坚定理想信念,勇担时代重任,青春才能在担当与奋进中大放异彩。【答案】辛苦遭逢起一经 因为我对这土地爱得深沉 怀旧空吟闻笛赋 何时眼前突兀见此屋 吾庐独破受冻死亦足 为赋新词强说愁 长风破浪会有时 直挂云帆济沧海 春蚕到死丝方尽 蜡炬成灰泪始干【解析】默写题作答时,一要透彻理解诗文内容;二要认真审题,找出符合题意的诗文句子;三是答题内容要准确,做到不添字、不漏字、不写错字。本题中注意“逢、吟、赋、见、庐、沧、蜡炬”等字的正确书写。11.依据课文填空。(1)________________________,切问而近思,仁在其中矣。(《论语》)(2)非独贤者有是心也,人皆有之,________________________。(《孟子》)(3)沙鸥翔集,________________________。(《岳阳楼记》)(4)________________________,水善利万物而不争,处众人之所恶,故几于道。(《老子》)【答案】博学而笃志 贤者能勿丧耳 锦鳞游泳 上善若水【解析】本题考查学生对古诗文名句的识记能力。解答此类题目,我们需要在平时的学习中,做好积累,根据提示语句写出相应的句子,尤其要注意不能出现错别字。本题中的“笃、鳞、游、善”等字词容易写错。12.文言诗文默写填空。(1)问君何能尔?________________________。(陶渊明《饮酒》)(2)海内存知己,________________________。(王勃《送杜少府之任蜀州》(3)________________________,直挂云帆济沧海。(李白《行路难》)(4)出淤泥而不染,________________________。(周敦颐《爱莲说》)(5)念天地之悠悠,________________________!(陈子昂《登幽州台歌》)(6)人生自古谁无死?________________________。(文天祥《过零丁洋》)(7)________________________,佳木秀而繁阴。(欧阳修《醉翁亭记》)(8)但愿人长久,________________________。(苏轼《水调歌头》)(9)苟全性命于乱世,________________________。(诸葛亮《出师表》)(10)马作的卢飞快,________________________。(辛弃疾《破阵子·为陈同甫赋壮词以寄之》)(11)白居易的《钱塘湖春行》一诗中,直抒胸臆,表明诗人余兴未阑,完全陶醉在美好的湖光山色之中的两句诗是:________________________,________________________。【答案】心远地自偏 天涯若比邻 长风破浪会有时 濯清涟而不妖 独怆然而涕下 留取丹心照汗青 野芳发而幽香 千里共婵娟 不求闻达于诸侯 弓如霹雳弦惊 最爱湖东行不足 绿杨阴里白沙堤【解析】诗词默写要求:一、不能添字,不能少字;二、字的笔画要准确。注意:偏、濯、涟、怆、涕、汗青、霹雳、阴。13.古诗文填空。(1)将军百战死,________________________。(《木兰诗》)(2)________________________,可以为师矣。(《<论语>十二章》)(3)感时花溅泪,________________________。(杜甫《春望》)(4)________________________,浅草才能没马蹄。(白居易《钱塘湖春行》)(5)了却君王天下事,________________________。(辛弃疾《破阵子•为陈同甫赋壮词以寄之》)(6)________________________,在乎山水之间也。(欧阳修《醉翁亭记》)(7)龚自珍《己亥杂诗》中表现虽“老骥伏枥”,仍怀奉献精神的诗句是:________________________,________________________。(8)请你写出描写边塞风光的连续两句古诗词:________________________,________________________。【答案】壮士十年归 温故而知新 恨别鸟惊心 乱花渐欲迷人眼 赢得生前身后名 醉翁之意不在酒 落红不是无情物 化作春泥更护花 大漠孤烟直 长河落日圆(塞下秋来风景异 衡阳雁去无留意)【解析】课文原句填空作答时,一是要深刻理解诗文内容;二是要认真审题找出符合题意的诗文的语句;三是作答内容要准确,做到不加字、不少字、不写错字。本题中注意“渐、赢、生前身后、酒、落、作”等字词的书写。描写边塞风光的连续两句古诗词:秦时明月汉时关,万里长征人未还。角声满天秋色里,塞上胭脂凝夜紫。14.古诗文默写。(1)高峰入云,_______________。(陶弘景《答谢中书书》)(2)深林人不知,_____________。(王维《竹里馆》)(3)____________________,漏断人初静。(苏轼《卜算子·黄州定慧院寓居作》)(4)____________________,落花时节又逢君。(杜甫《江南逢李龟年》)(5)《过零丁洋》中概写诗人被捕前的经历,突出自己艰难遭遇的句子是:________________,_________________。(6)“_____________________,____________________”,只要我们坚定信心,不惧“卡脖子”,就一定能实现科技的自立自强。(请从《行路难·其一》中选用合适的句子作答)【答案】清流见底 明月来相照 缺月挂疏桐 正是江南好风景 辛苦遭逢起一经 干戈寥落四周星 长风破浪会有时 直挂云帆济沧海【解析】默写题作答时,一是要透彻理解诗文的内容;二是要认真审题,找出符合题意的诗文句子;三是答题内容要准确,做到不添字、不漏字、不写错字。本题中的“疏、干戈、寥、济、沧”等字词容易写错。15.用古诗文原句填空。①____________________,夜吟应觉月光寒。(李商隐《无题》)②万籁此都寂,____________________。(常建《题破山寺后禅院》)③____________________,不可知其源。(柳宗元《小石潭记》)④吾视其辙乱,____________________,故逐之。(《曹刿论战》)⑤《红楼梦》中有诗云:“质本洁来还洁去,强于污淖陷渠沟。”《爱莲说》一文表意与之相近的文句是:“____________________,____________________。”【答案】晓镜但愁云鬓改 但余钟磬音 其岸势犬牙差互 望其旗靡 出淤泥而不染 濯清涟而不妖【解析】本题考查名句名篇默写。默写题作答时,一是要透彻理解诗文的内容;二是要认真审题,找出符合题意的诗文句子;三是答题内容要准确,做到不添字、不漏字、不写错字。本题中的“鬓”“钟磬”“差互”“靡”“淤”“濯”“涟”容易写错,需引起注意。16.根据提示填空。(1)_____________,夜泊秦淮近酒家。(杜牧《泊秦淮》)(2)_____________,一览众山小。(杜甫《望岳》)(3)窈窕淑女,_____________。(《关雎》)(4)锦帽貂裘,_____________。(苏轼《江城子·密州出猎》)(5)古今中外,凡成就一番事业的人,他们在成功与挫折面前始终保持着“____________,___________”的心态。(范仲淹《岳阳楼记》)(6)“无私奉献精神”是时代永恒的主旋律,请你写出连续两句与“奉献”有关的诗句:“______________,____________。”(课内外均可)【答案】烟笼寒水月笼沙 会当凌绝顶 君子好逑 千骑卷平冈 不以物喜 不以己悲 落红不是无情物 化作春泥更护花(春蚕到死丝方尽,蜡炬成灰泪始干。(符合题意即可))【解析】默写题作答时,一是要透彻理解诗文的内容;二是要认真审题,找出符合题意的诗文句子;三是答题内容要准确,做到不添字、不漏字、不写错字。注意重点字“笼、凌、逑、骑、冈、己”的书写。⑦⑧空,要选择与“奉献”有关的诗句,如:采得百花成蜜后,为谁辛苦为谁甜?横眉冷对千夫指,俯首甘为孺子牛。鞠躬尽瘁,死而后已;等等。17.古诗文默写。(1)__________,不知饴阿谁。(《十五从军征》)(2)纷纷暮雪下辕门,___________。(岑参《白雪歌送武判官归京》 )(3)俗子胸襟谁识我,___________。(秋瑾《满江红》)(4)__________,病树前头万木春。(刘禹锡《酬乐天扬州初逢席上见赠》)(5)北宋文学家欧阳修的《醉翁亭记》一文以色彩鲜明的语言,描绘春夏之景的句子是:___________,___________。(6)春秋时期,鲁与齐交战,起初鲁庄公不待齐军疲惫就要出战,被曹刿阻止了,曹刿采取“敌疲我打”的方针,战胜了齐军,成为了中国战争史上以弱胜强的有名战例。在《曹刿论战》中反映曹刿提出的“敌疲我打”方针的句子是“____________,______________,_____________。______________,故克之。”【答案】羹饭一时熟 风掣红旗冻不翻 英雄末路当磨折 沉舟侧畔千帆过 野芳发而幽香 佳木秀而繁阴 一鼓作气 再而衰 三而竭 彼竭我盈【解析】默写题作答时,一要透彻理解诗文内容;二要认真审题,找出符合题意的诗文句子;三是答题内容要准确,做到不添字、不漏字、不写错字。本题中注意“羹、掣、末、畔、阴、作、衰、竭、盈”等字的正确书写。18.请你完成下面古诗文名句填空。【答案】月是故乡明 仍怜故乡水 凫雁满回塘 却话巴山夜雨时 凭君传语报平安 采菊东篱下 夜阑卧听风吹雨 铁马冰河入梦来 例:安得广厦千万间 大庇天下寒士俱欢颜【解析】默写题作答时,一是要透彻理解诗文的内容;二是要认真审题,找出符合题意的诗文句子;三是答题内容要准确,做到不多字,不少字,不写错字。本题中注意“怜、凫、雁、凭、篱、阑”等字要书写正确;第(8)小题只要是表达家国情怀的诗句即可,示例“人生自古谁无死,留取丹心照汗青”。19.根据原文默写。(1)学而不思则罔,____________。(《论语·为政》)(2)___________________,留取丹心照汗青。(文天祥《过零丁洋》)(3)感时花溅泪,___________________。(杜甫《春望》)(4)溪云初起日沉阁,__________________。(许浑《咸阳城东楼》)(5)_______________,只有香如故。(陆游《卜算子·咏梅》)(6)《白雪歌送武判官归京》中诗人以春花喻冬雪的千古名句是:_______________,_______________。(7)《行路难》中表现李白浪漫主义创作风格,道出诗人坚信抱负必能实现的诗句是:__________,__________。【答案】思而不学则殆 人生自古谁无死 恨别鸟惊心 山雨欲来风满楼 零落成泥碾作尘 忽如一夜春风来 千树万树梨花开 长风破浪会有时 直挂云帆济沧海【解析】默写题作答时,一是要透彻理解诗文的内容;二是要认真审题,找出符合题意的诗文句子;三是答题内容要准确,做到不添字、不漏字、不写错字。本题中的“殆、满、零、碾、作、忽、梨、沧”等字词容易写错。20.请将下面古诗文语句补充完整。(1)大漠孤烟直,__________________。(王维《使至塞上》)(2)会当凌绝顶,__________________。(杜甫《望岳》)(3)__________________,随君直到夜郎西。(李白《闻王昌龄左迁龙标遥有此寄》)(4)人生自古谁无死?__________________。(文天祥《过零丁洋》)(5)子曰:“__________________,可以为师矣。”(《论语·为政》)(6)春蚕到死丝方尽,__________________。(李商隐《无题》)(7)“诗言志”是中国自古的文化传统。王安石在《登飞来峰》一诗中,借景抒怀,用“__________________,__________________”表达自己满怀信心、锐意改革的政治抱负。【答案】长河落日圆 一览众山小 我寄愁心与明月 留取丹心照汗青 温故而知新 蜡炬成灰泪始干 不畏浮云遮望眼 自缘身在最高层【解析】诗词默写要求:一、不能添字,不能少字;二、字的笔画要准确。注意:圆、丹、汗、蜡炬、缘。21.默写。⑴________________________,江春入旧年。(王湾《次北固山下》)⑵箫鼓追随春社近,_______________________。(陆游《游山西村》)⑶我报路长嗟日暮,______________________。(李清照《渔家傲》)⑷料峭春风吹酒醒,微冷,____________________。(苏轼《定风波》)⑸关关雎鸠,在河之洲。窈窕淑女,______________________。(《诗经·周南》)⑹山舞银蛇, ____________________。(毛泽东《沁园春∙雪》)⑺______________________ ,无案牍之劳形。(刘禹锡《陋室铭》)⑻选贤与能,__________________。(《大道之行也》)⑼_________________________,万钟于我何加焉!(孟子《鱼我所欲也》)⑽______________________,处江湖之远则忧其君。(范仲淹《岳阳楼记》)⑾冬雪不能阻隔春天翩然而至的脚步,同样疫情也一样不能阻止滚滚前进的历史车轮。正如刘禹锡《酬乐天扬州初逢席上见赠》中所写:“___________,___________。”【答案】海日生残夜 衣冠简朴古风存 学诗谩有惊人句 山头斜照却相迎 君子好逑 原驰蜡象 无丝竹之乱耳 讲信修睦 万钟则不辩礼义而受之 居庙堂之高则忧其民 沉舟侧畔千帆过 病树前头万木春【解析】默写题作答时,一是要透彻理解诗文的内容;二是要认真审题,找出符合题意的诗文句子;三是答题内容要准确,做到不添字,不漏字,不写错别字。本题中注意“生”“冠”“谩”“逑”“驰”“睦”“辩”“畔”等字的书写。22.填空。(1)为人谋而不忠乎?与朋友交而不信乎?___________________?(《〈论语〉十二章》)(2)落红不是无情物,___________________ 。(龚自珍《己亥杂诗(其五)》)(3)念天地之悠悠,___________________。(陈子昂《登幽州台歌》)(4)____________,寒光照铁衣。(《木兰诗》)(5)水何澹澹,___________________。(曹操《观沧海》)(6)潮平两岸阔,___________________ 。(王湾《次北固山下》)(7)《陋室铭》中写交往之雅的句子是“ ___________________,___________________ 。”(8)《过零丁洋》中表现文天祥舍生取义、视死如归的坚定信念和昂扬斗志的诗句是“ ___________________?___________________。”【答案】传不习乎 化作春泥更护花 独怆然而涕下 朔气传金柝 山岛竦峙 风正一帆悬 谈笑有鸿儒 往来无白丁 人生自古谁无死 留取丹心照汗青【解析】默写题作答时,一要透彻理解诗文内容;二要认真审题,找出符合题意的诗文句子;三是答题内容要准确,做到不添字、不漏字、不写错字。本题中注意“怆、涕、朔、柝、竦峙、鸿、汗”等字的正确书写。23.在下列横线上填写出相应的句子。(1)_____;若出其里。(曹操《观沧海》)(2)莫笑农家腊酒浑,_____。(陆游《游山西村》)(3)为人谋而不忠乎?_____?传不习乎?(《论语》十二章》)(4)古诗词中,关心百姓疾苦的作品随处可见。《十五从军征》中,通过“_____,_____”反映出乱世中百姓生活的艰辛。(5)《小石潭记》中“_____,_____,往来翕忽”写出了鱼儿游乐的情景。【答案】星汉灿烂 丰年留客足鸡豚 与朋友交而不信乎 舂谷持作饭 采葵持作羹 佁然不动 俶尔远逝【解析】默写题作答时,一是要透彻理解诗文内容;二要认真审题,找出符合题意的诗文句子;三,答题内容要准备,做到不添字、不漏字、不写错别字。本题中的“汉、豚、舂、葵、羹、然、俶、逝”等字词容易写错。24.请根据原文或提示,将下面古诗文语句补充完整。(1)_____________,衡阳雁去无留意。(范仲淹《渔家傲·秋思》)(2)山重水复疑无路,_____________。(陆游《游山西村》)(3)_______________,长河落日圆。(王维《使至塞上》)(4)乱花渐欲迷人眼,_____________。白居易《钱塘湖春行》)(5)海日生残夜,_______________。(王湾《次北固山下》)(6)_______________,何妨吟啸且徐行。(苏轼《定风波》)(7)文天祥的《过零丁洋》一诗中以磅礴的气势收束全篇,写出宁死不屈壮烈誓词的诗句是:“_________________?________________。”【答案】塞下秋来风景异 柳暗花明又一村 大漠孤烟直 浅草才能没马蹄 江春入旧年 莫听穿林打叶声 人生自古谁无死 留取丹心照汗青【解析】课文原句填空作答时,一是要深刻理解诗文内容;二是要认真审题找出符合题意的诗文的语句;三是作答内容要准确,做到不加字、不少字、不写错字。本题中注意“柳、漠、蹄、汗”等字词的书写。25.用原文补写出名篇名句中的空缺部分。(1)《次北固山下》中蕴含哲理的诗句是________,__________。(2)金樽清酒斗十千,____________。(李白《行路难》)(3)千古兴亡多少事?悠悠。____________。(辛弃疾《南乡子·登京口北固亭有怀》)(4)谈笑有鸿儒,_________。(刘禹锡《陋室铭》)(5)《〈论语〉十二章》阐明“学”与“思”关系的句子是:_______,______。(6)民谚云:“卑贱者最聪明,高贵者最愚蠢。”后一句在《曹刿论战》中也有类似的表达。曹刿对其乡人曰:“_____,______。”【答案】海日生残夜 江春入旧年 玉盘珍羞直万钱 不尽长江滚滚流 往来无白丁 学而不思则罔 思而不学则殆 肉食者鄙 未能远谋【解析】默写古诗文,要根据上句写出下句,根据下句默写上句,理解性默写要根据提示默写正确语句,如本题(6)需要默写出《曹刿论战》中与“卑贱者最聪明,高贵者最愚蠢”类似的表达,应该是“肉食者鄙,未能远谋”。本题注意“生”“直”“罔”“殆”的正确书写。26.经典诗文默写。(1) 海日生残夜,________________________。(王湾《次北固山下》)(2)________________________,坐断东南战未休。(辛弃疾《南乡子 登京口北固亭有怀》)(3) 我报路长嗟日暮,________________________。(李清照《渔家傲》)(4)________________________,虎啸猿啼。(范仲淹《岳阳楼记》)(5) 金樽清酒斗十千,____________________。(李白《行路难(其一)》)(6)________________________?英雄末路当磨折。(秋瑾《满江红》)(7) 录毕,走送之,________________________。(宋濂《送东阳马生序》)(8)______________________,肩膀上的红旗手中的书。(贺敬之《回延安》)(9)从这点出发,________________________。(毛泽东《纪念白求恩》)(10)读书使人充实,________________________,作文使人准确。(培根《谈读书》)【答案】江春入旧年 年少万兜鍪 学诗谩有惊人句 薄暮冥冥 玉盘珍羞直万钱 俗子胸襟谁识我 不敢稍逾约 东山的糜子西山的谷 就可以变为大有利于人民的人 讨论使人机智【解析】默写题作答时,一要透彻理解诗文内容;二要认真审题,找出符合题意的诗文句子;三是答题内容要准确,做到不添字、不漏字、不写错字。本题中注意“兜鍪、谩、暮、冥、羞、直、襟、逾、糜”等字的正确书写。27.古诗文默写。(1)__________________,君子好逑。(《诗经》)(2)沉舟侧畔千帆过,_____________________。(刘禹锡《酬乐天扬州初逢席上见赠》)(3)______________________,五十弦翻塞外声。(辛弃疾《破阵子·为陈同甫赋壮词以寄之》)(4)伤心秦汉经行处,____________。(张养浩《山坡羊·潼关怀古》)(5)《行路难》中,作者运用比喻的修辞手法生动地写出了前路充满艰难险阻的句子是:_________________ ,__________________ 。(6)苏轼《水调歌头》中,表达对亲人美好祝愿的千古名句是:___________,___________。(7)杜甫《望岳》中“ _______________, _________________。”两句诗由远观而近看,表现了诗人对大好河山无限眷恋和热爱之情。【答案】窈窕淑女 病树前头万木春 八百里分麾下炙 宫阙万间都做了土 欲渡黄河冰塞川 将登太行雪满山 但愿人长久 千里共婵娟 荡胸生曾云 决眦入归鸟【解析】默写题作答时,一是要透彻理解诗文的内容;二是要认真审题,找出符合题意的诗文句子;三是答题内容要准确,做到不添字、不漏字、不写错字。本题中的“窈窕、麾、炙、阙、婵娟、曾、决眦”等字词容易写错。五、诗词曲鉴赏 阅读下面的诗歌,完成下面小题。渔家傲·秋思范仲淹塞下秋来风景异,衡阳雁去无留意。四面边声连角起,千嶂里,长烟落日孤城闭。浊酒一杯家万里,燕然未勒归无计。羌管悠悠霜满地,人不寐,将军白发征夫泪。1.下列对这首词的理解和分析,不正确的一项是( )A.“塞下秋来风景异”,点明地域和季节,以“异”字统领边地风光的特点。B.“孤城闭”三个字真实地反映出当时的军事态势,并为下片的议论作铺垫。C.“将军白发征夫泪”一句,是说无论将军还是士兵们都白了鬓发,泪满衣襟。D.这首词的意境悲凉壮阔,形象鲜明生动,语言质朴凝练,读来真切感人。2.有些同学在默写时会把“衡阳雁去无留意”中的“雁”写成“燕”,请向他们解释为什么这里用“雁”字。【答案】1.B 2.①北雁南飞是塞外秋天特有的景象,用“雁”更符合地域和时令特点;②大雁是边塞诗中的常见意象,常寄寓着思乡之情,用“雁”更能传达出作者的情感。【解析】1.此题考查内容理解。B.有误,“孤城闭”三个字真实地反映出当时的军事态势,并为下片的抒情作铺垫,此项“并为下片的议论作铺垫”分析有误。故选B。2.此题考查词语理解。“雁”这个词在传统文化中被赋予一定的内涵,具有点缀秋冬景象,寄托对故乡、亲人的思念的象征意义,而“燕子”是春天的使者。根据“塞下秋来风景异”分析,此处的“雁”与“秋天”有关,更符合塞外地域和时令特点;“浊酒一杯家万里”与思乡有关,所以这里用“雁”字更能传达出作者的情感。阅读下面的诗歌,完成下面小题。江山秋色图①胡行简连山缥缈树槎牙②,江上西风雁影斜。万迭③秋光无限好,画围偏在野人④家。【注】①此诗为画作《江山秋色图》上的题诗。②槎(chá)牙:形容树木错杂不齐貌。③万迭:万重。④野人:山野之人。3.下列对这首诗的理解和赏析,不正确的一项是( ) A.本诗是一首题画七绝,读起来音韵和谐优美。B.前两句通过山、树、江、风、雁等意象,营造了秋的意境。C.前两句描绘了一幅生机勃勃的江山秋色图,与题目照应。D.本诗写景从视觉感受出发,写出了多种景物的形态。4.结合本诗三、四句,用自己的话说说诗人喜爱画作《江山秋色图》的原因。【答案】3.C 4.万重秋光风景无限美好,画中还有山野之人的活动,使画面增加了生趣。画作展示了万重秋光的美好,一重山色就是一重秋光,一重秋光就是一重美好,同时,画作还展现了山野之人的活动情景,既有自然风光,又有人文情趣,富有浓郁的生活气息,表达了作者对自然风光、山野生活的喜爱之情。【解析】3.本题考查诗歌内容的理解分析。C.“描绘了一幅生机勃勃的江山秋色图”有误,前两句中描写山的“缥缈”,树的“槎牙”,以及“西风”“雁影斜”等景象都展现了秋天松弛闲静、宁静悠远的特点,并非生机勃勃;故选C。4.本题考查诗句赏析。三、四句“万迭秋光无限好,画围偏在野人家”大意是:一重重的秋光风景无限美好,画中还有山野之人的活动,使画面增加了生趣。由此可知,联系前两句中的“连山缥缈”可知,画作展示了万重秋光的美好,一重山色就是一重秋光,一重秋光就是一重美好。除了自然风光以外,画作中还有很多山野之人的活动情景,景与人相融相合,使画面充满了浓郁的生活气息。阅读下面的诗歌,完成下面小题。夜雨寄北李商隐君问归期未有期,巴山夜雨涨秋池。何当共剪西窗烛,却话巴山夜雨时。秋思张籍洛阳城里见秋风,欲作家书意万重。复恐匆匆说不尽,行人临发又开封。5.这两首诗的体裁都是_____________。6.下列对两首诗的理解,不正确的一项是( )A.《夜雨寄北》中,“期”“巴山夜雨”两次出现,韵律回环往复,营造了时空交错的意境,达到了内容与形式的完美结合。B.《秋思》首句的“见”字平淡却富有意蕴。无形的秋风使城内花木凋落,游子满目凄凉,对家乡、亲人的思念油然而生。C.《夜雨寄北》中的“寄”和《秋思》中的“书”都表明他们身在异乡,两首诗都是在书信中表达自己对亲人的思念之情。D.《夜雨寄北》前两句由一问一答,写到眼前景色,寓情于景;而《秋思》前两句则紧承“见秋风”叙事,寓情于事。7.甲乙两诗的后两句在表现手法上各有特色,请分析其不同之处。【答案】5.七言绝句 6.B 7.【示例】《夜雨寄北》中“何当共剪西窗烛,却话巴山夜雨时”虚实结合,用想象未来团聚时的欢乐反衬出今夜离别的相思之苦;而今夜的苦又成了未来秉烛夜话的材料,增添了重聚时的乐。在时间和空间的回环对照中,将深挚的思念之情表达得曲折、细腻又含蓄隽永。《秋思》中“临发开封”是一个富于诗情的细节描写,把“复恐说不尽”的心态表现得栩栩如生,非常细腻地表达了作者对家乡亲人的深切思念。寄深情于浅淡,寓曲折于平缓,乍看起来,寥寥数语,细细吟诵,却意味无穷。【解析】5.本题考查诗歌的体裁。诗歌题材主要有古代诗歌和新诗两类,古代诗歌又分为近体诗和古体诗,近体诗又分为格律诗、绝句、词、曲,新诗包括自由诗、散文诗和民歌。古体诗是依照古代的诗体来写的,近体诗是同古体诗相对而言的。古体诗没有一定的格律,是古代的“自由诗”,一般分为四言诗、五言古体(五古)、七言古体(七古)、杂言体。近体诗有严格的格律,句数、字数、押韵、平仄等都有一定的规则,是古代的格律诗,一般包括律诗(五律、七律)、绝句(五绝、七绝)、排律(五言排律、七言排律)。律诗和绝句的区别主要在于句数上,绝句共有四句,律诗共有八句,八句以上的律诗称为排律。绝句可对仗,也可不对仗;但是律诗的颔联和颈联,则必须对仗。无论律诗还是绝句,都有平仄的要求。律诗的一、二两句为首联;三、四两句为颔联;五、六两句为颈联;七、八两句为尾联。这两首诗每首诗共四句,每句七言,总共二十八字。故是七言绝句。6.本题考查理解和赏析诗歌的能力。B项有误。“无形的秋风使城内花木凋落,游子满目凄凉”表述有误,结合诗句的内容没有对花木凋落、游子满目凄凉的描写。故选B。7.本题考查分析诗歌的表现手法的能力。诗歌常见的表现手法有:对比、夸张、借代、反衬、借古讽今、托物言志、虚实结合等等。作答此题时,首先要判断诗句使用的表现手法,然后结合诗句具体分析,最后分析作者借此传达的情感。《夜雨寄北》后两句运用了虚实相生的表现手法。“何当”是个表示愿望的词,是从“君问归期未有期”的现实中生发出来的,“共剪西窗烛”是由当前苦况所激发的对于未来欢乐的憧憬,是虚写。“却话巴山夜雨时”既是想象未来的答话,又是当前的实景。归期未定,作者心境之郁闷、孤寂,是不难想见的。而作者却跨越这一切去写未来,盼望在重聚的欢乐中追话今夜的一切。于是,未来的乐自然反衬出今夜的苦;而今夜的苦又成了未来剪烛夜话的材料,增添了重聚时的乐。两句诗,虚实相生,情景交融,构成完美的意境,余味无穷。《秋思》前后两句突出心理和动作描写。“复恐”二字,细致入微地刻画出似有千言万语而唯恐言之不尽地复杂微妙的心理;“临发又开封”的动作,既照应了“意万重”,又紧承“复恐”和“匆匆”,让人在看似平常的描写中体会到作者浓浓的思想之情。与其说是为了添写几句未说尽的内容,不如说是为了验证一下自己的疑惑和担心。作者寓情于事,将对家人的思念熔铸在自己的动作中。阅读下面的诗歌,完成下面小题。饮酒(其五)陶渊明结庐在人境,而无车马喧。问君何能尔?心远地自偏。采菊东篱下,悠然见南山。山气日夕佳,飞鸟相与还。此中有真意,欲辨已忘言。8.下列对这首诗的理解与分析,不正确的一项是( )A.本诗题目为《饮酒》,却未写饮酒之事,而是借“饮酒”的话题直抒胸臆,抒发对世事人生的感慨,字里行间透露出闲雅之趣。B.本诗按意境可划分为两层:前四句为一层,写诗人摆脱世俗烦恼后的感受;后六句为一层,写南山的美好晚景和诗人从中获得的乐趣。C.诗人借“山气”“飞鸟”等意象勾勒出黄昏将至时云入山峰、鸟入山林的静态之景,描绘出一幅悠远恬淡、自然和谐的画卷。D.“此中有真意”一句中的“真意”表现为“自然之趣”,既包含诗人对田园生活的热爱,也包含诗人归隐自洁、超脱世俗的人生追求。9.学完本诗后,小文有这样的疑问:“山气、飞鸟皆为‘望’中所得,诗人为何说悠然‘见’南山呢?”对此,你怎么看?请结合全诗阐述你的观点。【答案】8.C 9.①“见”字写出了一种无意得之、悠然忘我的心境,与全诗营造的闲远恬淡的氛围是吻合的。②“望”相较于“见”,更有目的性,更像是有意为之,破坏了诗歌自然而然、浑然天成的意境。【解析】8.本题考查对诗歌的理解和分析。C.有误,“山气日夕佳,飞鸟相与还。”诗人借“山气”“飞鸟”等意象勾勒出黄昏将至时云入山峰、鸟入山林的动态之景,“静态之景”表述有误;故选C。9.本题考查炼字。解答此题时,先要进行判断,然后分析这个字词的表达效果,再分析换掉后有怎样的变化,得出结论。本题中的“悠然见南山”的“见”字在从事家事活动的时候,不经意而看到了南山的景色,更能表现诗人的那种恬淡、闲适的心境;“见”字说明南山瞬间入眼,“望”字说明诗人有意远看,在寻找,不能体现“悠然”心情。阅读下面的诗歌,完成下面小题。【甲】行路难(其一)李白 金樽清酒斗十千,玉盘珍羞直万钱。停杯投箸不能食,拔剑四顾心茫然。欲渡黄河冰塞川,将登太行雪满山。闲来垂钓碧溪上,忽复乘舟梦日边。行路难!行路难!多歧路,今安在?长风破浪会有时,直挂云帆济沧海。【乙】渡荆门送别李白渡远荆门外,来从楚国游。山随平野尽,江入大荒流。月下飞天镜,云生结海楼。仍怜故乡水,万里送行舟。10.下列对诗歌的赏析不正确的一项是( )A.甲诗歌以叙事开篇,第一、二句营造了欢乐的宴饮气氛;第三、四句急转直下,“停杯投箸”“拔剑四顾”写出了诗人内心的抑郁和苦闷。B.第五、六句用“冰塞川”“雪满山”比喻仕途的艰难,形象化的语言中蕴含无限的失意;第七、八句诗人巧用吕尚和伊尹的典故,进一步抒发了政治失意的悲愤之情。C.“山随平野尽,江入大荒流”,描写了山峦渐渐消失,江水奔向广阔原野的景象,逼真如画,有如一幅长江出峡渡荆门长轴山水图,成为脍炙人口的佳句。D.“仍怜故乡水,万里送行舟”,这一句运用了拟人的修辞手法,将故乡水拟人化,借写故乡水有情,不远万里,依恋不舍送我远别故乡。11.两首诗分別抒写了诗人怎样的情感?(甲)____________________________________________________(乙)____________________________________________________【答案】10.B 11.【甲】本诗表现了诗人在政治道路上遭遇艰难时展现出的乐观自信的人生态度。(或:表达了诗人面对人生坎坷,仍盼望施展自己远大抱负的志向。) 【乙】表达了诗人离开故乡时依依不舍,思念故乡的感情。【解析】10.此题考查的是对诗歌的赏析。B.“抒发了政治失意的悲愤之情”理解有误。第七、八句“闲来垂钓碧溪上,忽复乘舟梦日边”诗人巧用吕尚和伊尹的典故,表现了作者渴望被朝廷重用的思想感情;故选B。11.此题考查的是对诗人感情的把握。甲诗“停杯投箸不能食,拔剑四顾心茫然”“欲渡黄河冰塞川,将登太行雪满山”“行路难!行路难!多歧路,今安在?长风破浪会有时,直挂云帆济沧海”等诗句,通过层层迭迭的感情起伏变化,既充分显示了黑暗污浊的政治现实对诗人的宏大理想抱负的阻遏,反映了由此而引起的诗人内心的强烈苦闷、愤郁和不平,同时又突出表现了诗人的倔强、自信和他对理想的执着追求,展示了诗人力图从苦闷中挣脱出来的强大精神力量。《渡荆门送别》是唐代大诗人李白青年时期在出蜀漫游的途中写下的一首诗。此诗由写远游点题始,继写沿途见闻和观感,后以思念作结。全诗意境高远,风格雄健,形象奇伟,想象瑰丽,以其卓越的绘景取胜,景象雄浑壮阔,其中“渡远荆门外,来从楚国游”抒发了诗人自己的少年意气,“仍怜故乡水,万里送行舟”两句诗,诗人在远游时内心仍旧牵系故乡的山水,表现了作者年少远游、倜傥不群的个性及浓浓的思乡之情。阅读下面的诗歌,完成下面小题。望洞庭湖赠张丞相孟浩然八月湖水平,涵虚混太清。气蒸云梦泽,波撼岳阳城。欲济无舟楫,端居耻圣明。坐观垂钓者,徒有羡鱼情。12.下列对这首诗的理解和分析,不正确的一项是( )A.首联起笔不凡,立足整体描写,表现洞庭湖水天相连、开阔浩渺的景象。B.颈联以欲渡无舟为喻,含蓄委婉地抒发从政心愿,期待得到丞相举荐。C.尾联的“徒”字,充满惋惜之意,表达自己空有才华无人赏识的忧愤。D.全诗把写景和抒情有机地结合起来,触景生情,情在景中,构思新颖。13.本诗中“波撼岳阳城”与“白水绕东城”(李白《送友人》)同是写水的佳句,其中水的特点有何不同?请简要分析。⁠【答案】12.C 13.示例:“波撼岳阳城”展现洞庭湖水波涛汹涌的景象,极具气势;“白水绕东城”描画白水缓缓流转之态,明丽秀美。【解析】12.考查诗句理解。C.“表达自己空有才华却无人赏识的忧愤”理解有误。尾联说自己坐在湖边观看那些垂竿钓鱼的人,却白白地产生羡慕之情。表现了作者希望得到张九龄的引荐,毫无忧愤之意;故选C。13.考查比较阅读。“波撼岳阳城”意思是“波涛奔腾,涌向东北岸,好像要摇动岳阳城似的”,“撼”表现了洞庭湖波浪的宏大声势。“白水绕东城”意思是“波光粼粼的流水围绕着城的东边”,“绕”字描画白水的动态美,宛转流畅,清新美丽。阅读下面的诗歌,完成下面小题。潼 关谭嗣同终古高云簇此城,秋风吹散马蹄声。河流大野犹嫌束,山入潼关不解平。14.下列对这首诗的理解和分析不正确的一项是( )A.首句从视觉角度,以一种远景式的遥望,展现潼关一带苍茫雄浑的景象。B.第二句从听觉角度渲染出寂寥的氛围,为全诗奠定了感伤的基调。C.第三、四句写黄河浩浩荡荡以及潼关境域内山峦起伏,将写景与言情巧妙地结合。D.诗人把自己的精神投射给高山、大河,这两个意象是其心态与胸怀的外化。15.诗中的“犹嫌束”“不解平”运用了哪种修辞手法?表达了诗人怎样的情感?请简要分析。【答案】14.B 15.示例一:运用了拟人的修辞手法,赋予河、山以人的情感,写出河的奔腾壮阔和山的巍峨险峻,表达了诗人渴望冲决罗网、勇往直前、追求个性解放的少年意气。示例二:运用了对偶的修辞手法,增强气势,让诗句富有音韵美,突出了河的奔腾壮阔和山的巍峨险峻,表达了诗人渴望冲决罗网、勇往直前、追求个性解放的少年意气。【解析】14.本题考查题内容理解。B.“为全诗奠定了感伤的基调”表述错误。第二句“秋风吹散马蹄声”意思是:秋风阵阵总是吹散哒哒的马蹄声。以轻捷、有力的笔调,将“秋风”“马蹄声”引入诗中,不但以听觉形象补充了前一句所造成的视觉形象,进一步渲染出潼关一带独具的氛围,而且打破了原先画面的静态,给全诗增添了一种动感。秋风中那矫健的马蹄声却更能催动豪情。他在壮阔的天地间策马驰骋,感到欣喜,感到痛快,感到精神上的极大的自由。可见并没有所谓“感伤”的基调。故选B。15.本题考查重点诗句赏析。“河流大野犹嫌束,山入潼关不解平”意思是:奔流的黄河流入辽阔的草原还嫌太束缚,秦岭山脉进去潼关以后就再也不知道何为平坦。本句将黄河人格化,运用了拟人的修辞手法,将写景与言情巧妙地结合起来,融进了诗人要求冲破约束的奔放情怀,是自我性格含蓄而又生动的描绘。写河、山雄伟的气势,表现其奔腾壮阔,让我们感受到诗人渴望冲决罗网,追求个性解放的少年意气与豪情,从而看出作者远大的抱负。阅读下面的诗歌,完成下面小题。木兰诗(节选) 《乐府诗集》东市买骏马,西市买鞍鞯,南市买辔头,北市买长鞭。旦辞爷娘去,暮宿黄河边,不闻爷娘唤女声,但闻黄河流水鸣溅溅。旦辞黄河去,暮至黑山头,不闻爷娘唤女声,但闻燕山胡骑鸣啾啾。万里赴戎机,关山度若飞。朔气传金柝,寒光照铁衣。将军百战死,壮士十年归。16.下列对这首诗的理解和分析,不正确的一项是( )A.这是一首叙事诗,叙述了木兰女扮男装,代父从军的故事,充满了传奇色彩。B.诗歌以排比、对偶、互文等修辞手法展开描写和叙述,具有强烈的艺术感染力。C.“朔气传金柝,寒光照铁衣”通过“朔气”“寒光”两个词表现边塞天气寒冷。D.“将军百战死,壮士十年归”中“百战”是虚指,“十年”是实指,虚实相应。17.全诗写人叙事,有时“泼墨如水”,有时“惜墨如金”,请结合节选部分的诗句分析这一手法。【答案】16.D 17.“泼墨如水”即详写,“东市买骏马……北市买长鞭”具体描写木兰出征前的准备(或:“旦辞爷娘去……但闻燕山胡骑鸣啾啾”具体描写木兰奔赴战场的场面); “惜墨如金”即略写,“万里赴戎机……壮士十年归”概写木兰的征战生活。【解析】16.本题考查内容分析。D.“将军百战死,壮士十年归”是互文,意思为:将士们经过无数次出生入死的战斗,有些牺牲了,有的十年之后得胜而归。据此分析,“百战”和“十年”都是虚指,该选项说法有误;故选D。17.本题考查写法。题目中的有时“泼墨如水”指的是详写,“惜墨如金”指的是略写。结合诗歌第一段“东市买骏马,西市买鞍鞯,南市买辔头,北市买长鞭。旦辞爷娘去,暮宿黄河边,不闻爷娘唤女声,但闻黄河流水鸣溅溅。旦辞黄河去,暮至黑山头,不闻爷娘唤女声,但闻燕山胡骑鸣啾啾”可知,该诗详写了木兰从军前的准备,再结合第二段“万里赴戎机,关山度若飞。朔气传金柝,寒光照铁衣。将军百战死,壮士十年归”可知打仗的过程是略写的。这样写体现作者并不是为了表现木兰时怎样取胜的,而是重在表现木兰的深明大义、性格纯真善良、不慕名利、品质高贵!阅读下面的诗歌,完成下面小题。江城子·密州出猎苏轼老夫聊发少年狂,左牵黄,右擎苍,锦帽貂裘,千骑卷平冈。为报倾城随太守,亲射虎,看孙郎。酒酣胸胆尚开张。鬓微霜,又何妨!持节云中,何日遣冯唐?会挽雕弓如满月,西北望,射天狼。18.下列对这首词的理解和分析,不正确的一项是( )A.本词主要写词人“少年狂”的表现,上片侧重表现“心狂”,下片侧重表现“形狂”。B.“千骑卷平冈”,千骑奔驰,腾空越野,好一幅壮观的出猎场面!“卷”极言行走之快。C.“鬓微霜,又何妨”从正面衬托词人的“少年狂”,表现出词人壮志未已的英雄本色。D.这首词的题材、艺术形象、语言风格都是粗犷的,充分体现了苏轼豪放派的诗词风格。19.“持节云中,何日遣冯唐”一句用了什么典故?请分析其作用。【答案】18.A19.本句运用冯唐去云中赦免魏尚的典故。词人用“冯唐持节”(或:词人以魏尚自许),表达了希望能得到朝廷重用、杀敌报国的雄心壮志。【解析】18.本题考查对词的理解与分析。A.结合上片中的“左牵黄,右擎苍,锦帽貂裘,千骑卷平冈”可以看出,上片写的是“出猎”这一特殊场合下表现出来的词人举止神态之“狂”,是“形狂”;结合下片中的“持节云中,何日遣冯唐?会挽雕弓如满月,西北望,射天狼”可以看出,下片由实而虚,进一步写词人“少年狂”的胸怀,抒发由打猎激发起来的壮志豪情,是“心狂”;故选A。19.本题考查内容理解与分析。“持节云中,何日遣冯唐”的意思是皇帝什么时候才派遣冯唐去云中郡,把边事委托给太守魏尚呢?冯唐是汉文帝时人。他曾向汉文帝陈说云中太守魏尚征战有功,不应当为一点小差错(报功时文书上所载杀敌的数字与实际不符)就治魏尚的罪,而应免罪赏功。汉文帝采纳了冯唐的意见,并派他到云中郡去赦免魏尚的罪,仍然让魏尚担任云中郡太守,还把冯唐升任为车骑都尉。事见《史记·冯唐列传》。苏轼此时任密州太守,故以魏尚自许,希望能得到朝廷的信任,像汉文帝派冯唐持节赦免魏尚一样,对自己委以重任,赴边疆抗敌。阅读下面的诗歌,完成下面小题。山寺夜起【清】江湜①月升岩石巅②,下照一溪烟。烟色如云白,流来野寺前,开门惜夜景,矫首③看霜天。谁见无家客,山中独不眠。【注】①江湜(1818-1866):清代诗人。字持正,又字弢叔,别署龙湫院行者,长洲(今江苏苏州)人,诸生。三与乡试,皆不第。在京师得亲戚资助,捐得浙江候补县丞。咸丰十年,奔走避兵,后忧愤而死。②巅:顶处,最高的地方。③矫首:抬头。矫,举。20.下面对本诗的理解不正确的一项是( )A.首联描写月亮从岩石之巅升起,月光普照大地,营造了清新明朗的氛围,奠定了轻松愉悦的情感基调。B.颔联紧承首联,写出了水汽自然飘动的动态之美。洁白的水汽缥缈轻盈、流落无定,不禁让作者联想到自己漂泊无依的生活。C.颈联中的“霜天”二字,既是写霜重气寒的眼前之景,又暗示了诗人心境,可谓传神之笔。D.尾联用反问加强直接抒情效果,与前面的景物描写遥相映衬,突出愁绝伤绝的自伤之境。21.结合全诗分析诗人“不眠”的原因。【答案】20.A 21.作者不眠原因有二:沉醉山寺夜景;自己为“无家客”,因思家而难以入眠。【解析】20.考查诗歌内容理解辨析。A.有误,首联描写月亮从岩石之巅升起,月光普照大地,为全诗营造了思乡的氛围,奠定了凄凉哀伤的感情基调;故选A。21.考查词句理解赏析。“开门惜夜景,矫首看霜天。谁见无家客,山中独不眠”意为:打开门,欣赏这夜景,抬头看那深寒的夜空。有谁见过我这等没有家的流浪者,在这深山中孤独的难以成眠。诗人无法入睡,只好自己打开寺门,站在寺外观赏美景。眼前美景,让作者感慨万千,不禁怜惜起来。据此可知,诗人“不眠”的原因既是对美好山寺夜景的沉醉,又暗含深沉的身世漂泊之感。阅读下面的诗歌,完成下面小题。破阵子•为陈同甫赋壮词以寄之醉里挑灯看剑,梦回吹角连营。八百里分麾下炙,五十弦翻塞外声,沙场秋点兵。马作的卢飞快,弓如霹雳弦惊。了却君王天下事,赢得生前身后名。可怜白发生!22.下列对这首词的理解和分析,不正确的一项是( )A.首句用“挑灯”“看剑”两个连贯动作,展示出主人公在夜深人静、万籁俱寂之时,思潮汹涌,重温军营生活。B.“八百里分麾下炙,五十弦翻塞外声”两句对仗工整,表现了雄壮的军容和士兵高昂的战斗情绪。C.这首词是辛弃疾写给志同道合的朋友陈同甫的,“壮词”即豪放之词。D.本词从思想感情上看,结尾直抒胸臆,抒发杀敌报国的豪情;从语言风格上看,展现了一种豪放壮美。23.任选角度赏析“马作的卢飞快,弓如霹雳弦惊”。【答案】22.D 23.示例一:运用比喻的修辞,把战马比作的卢马,弓弦声比作霹雳,形象生动地描写出惊险激烈的战斗场面,表现作者渴望重回战场的爱国激情和雄心壮志。示例二:运用典故,引用“的卢”的典故,写出战马像的卢一样,跑得飞快,写出惊险激烈的战斗场面,表现作者渴望重回战场的爱国激情和雄心壮志。示例三:从视觉和听觉两方面描写了激烈的战斗场面,塑造了一个奋勇向前的英雄形象,表现作者渴望重回战场的爱国激情和雄心壮志。【解析】22.本题考查对诗歌内容的理解。D.“可怜白发生”表明作者已年近半百,两鬓染霜,理想难以实现,充满了壮志未酬的抑郁、愤懑之情。“抒发杀敌报国的豪情”理解错误;故选D。23.本题考查赏析诗句。赏析诗句要从以下几方面入手:一、品味富于表现力的关键词;二、剖析诗词的思想内容;三、探究用典化句的妙用;四、体会修辞手法的表达效果;五、揭示其蕴含的哲理。此句可从运用修辞或是运用典故的角度赏析。“马作的卢飞快,弓如霹雳弦惊”意思是战马像的卢马那样跑得飞快,射箭时弓弦的响声像响雷一样,震耳离弦。采用比喻的修辞,将战马比作的卢马,是所见;将弓弦声比作霹雳,是所闻;马快弦急说明战斗的激烈和顺利,运用比喻的修辞,生动形象地描写出惊险激烈的战斗场面,侧面烘托出作者的英勇,表达了作者强烈的爱国激情和雄心壮志。“马作的卢飞快”是运用“的卢”典故,写出战马像的卢一样,跑得飞快。据此分析任选角度赏析即可。阅读下面的诗歌,完成下面小题。菩萨蛮•书江西造口壁辛弃疾郁孤台①下清江水,中间多少行人②泪!西北望长安,可怜③无数山④。青山遮不住,毕竟东流去⑤。江晚正愁余,山深闻鹧鸪。注释:①郁孤台:古台名。②行人:指因金兵南侵而逃难的人。③可怜:可惜。④无数山:这里指投降派(也可理解为北方沦陷国土)。⑤毕竟东流去:暗指力主抗金的潮流不可阻挡。24.下列关于本词的理解和分析,不正确的一项是( )A.开头两句起笔不凡,词人由眼前的“清江水”联想到“行人泪”,将无限哀痛、满腹幽怨巧妙地传达出来。B.三、四句写词人遥望汴京,眼前无数峰峦让他触目伤怀,自感年华已逝,功名难就,因而心生恨意。C.五、六句中“遮不住”三字将青山周匝围堵之感一笔推倒,“毕竟”二字使表达的情感更为深沉复杂。D.这首词以眼前景道心上事,寓悲愤之情于宏阔之景,丰厚蕴藉,沉郁顿挫,颇有“老杜”之风。25.清代陈廷焯《白雨斋词话》赞此词“结尾句号呼痛哭,音节之悲,至今犹隐隐在耳”,请结合全词,简述结尾二句所蕴含的思想感情。【答案】24.B25.追怀当年国事艰危的沉痛;对⾦兵⼊侵下百姓们流离失所的同情;对失去的国⼟的深情怀念;重山阻隔,故园难回的悲愤;未能恢复中原壮志难酬的抑郁和苦闷;对当权者⼀味妥协不思光复的愤懑。【解析】24.考查诗词赏析。B.在“西北望长安,可怜无数山”中,长安指汴京。本句是诗人因记起朋友被追而向汴京望去,然而却有无数的青山挡住了诗人。联系注释中的“无数山:这里指投降派(也可理解为北方沦陷国土)”可知,这两句诗表达了作者因为国土沦丧,朝廷中投降派占据主流而产生的悲愤之情。本项“自感年华已逝,功名难就,因而心生恨意”。故选B。25.考查词句赏析。联系“郁孤台下清江水,中间多少行人泪”,句中的“行人”指的是因金兵南侵而逃难的人。在建炎年间北宋南奔之际,自中原至江淮而江南,不知有多少行人流下无数伤心泪。表现了作者重临旧地,追思国事的沉痛,“行人泪”表现了作者对流离失所的百姓的同情之意。联系“西北望长安,可怜无数山”,根据注释“无数山:这里指投降派(也可理解为北方沦陷国土)”可知,表现了作者对朝廷中主张投降,不思光复的投降派的愤懑与批判,也有对沦陷国土的怀念。作者欲望长安,久去思归,却被无数山所阻隔,表现了作者重山阻隔,故园难回的悲愤。联系“青山遮不住,毕竟东流去”,中原仍未收复,作者举头眺望,视线却被青山遮断;但浩浩荡荡的江水冲破重重阻碍,奔腾向前。这既是眼前实景,又暗喻自己百折不回的意志,表达了作者争取最后胜利的信心。但一想到南归后的遭遇,又愁上心头,诗人的情绪又变得低沉,满是中原难复,壮志难酬的抑郁和苦闷。阅读下面的诗歌,完成下面小题。临江仙·送钱穆父①[宋]苏轼一别都门三改火②,天涯踏尽红尘。依然一笑作春温③。无波真古井④,有节是秋筠⑤。惆怅孤帆连夜发,送行淡月微云。尊前不用翠眉颦。人生如逆旅,我亦是行人。[注释]①钱穆父:名勰,又称钱四。被贬越州(今浙江绍兴)、瀛洲(今河北河间),赴任途经杭州,苏轼作此词以送。②改火:古代钻木取火,四季换用不同木材,称为“改火”,这里指年度的更替。③春温:指春天的温暖。④古井:比喻内心恬静,情感不为外界事物所动。⑤筠:竹。26.下列对诗歌的理解和赏析,不正确的一项是( )A.这是一首赠别词。词人将对老友的眷眷惜别之情写得缠绵感伤、哀怨愁苦。B.词的上片写与友人久别重逢。先从时间着笔,再就空间落墨,最后表达对友人赞颂之意。C.“惆怅孤帆连夜发,送行淡月微云”一句,描绘出一种凄清幽冷的氛围,渲染了作者与友人分别时抑郁无欢的心情。D.“尊前不用翠眉颦”一句,运用借代修辞,言说宴席中以歌舞相伴的歌女用不着为离愁别恨而哀怨。27.请赏析诗句“人生如逆旅,我亦是行人”。【答案】26.A27.这两句意思是:人生就是座旅店,我也是匆匆过客。运用比喻,形象写出不必为暂时离别伤情,其实人生如寄。作者以对友人的慰勉和开释胸怀总收全词,既动之以情,又揭示出得失两忘、万物齐一的豁达的人生态度。【解析】26.本题考查内容理解。A.“临江仙·送钱穆父”有误。《临江仙·送钱穆父》上片写与友人久别重聚,赞赏友人面对坎坷奔波时的古井心境和秋竹风节;下片切入正题,写月夜与友人分别,抒发了对世事人生的超旷之思;故选A。27.本题考查诗句赏析。“人生如逆旅,我亦是行人”意思是:人生在世就好像住旅馆,我也包括在旅行者里边。将人生的旅程比作住旅馆,运用了比喻的修辞手法;人生是短暂的,不必因离别而愁绪满怀,这是诗人对友人的劝慰,希望友人能够积极面对,表现其旷达的胸怀,诗人中寄寓着作者的情思,表现了诗人得失两忘、万物齐一的人生态度。阅读下面的诗歌,完成下面小题。少年游·玉壶冰莹兽炉灰欧阳修 (宋代)玉壶冰莹兽炉灰。人起绣帘开。春丛①一夜,六花②开尽,不待剪刀③催。洛阳城阙中天起,高下遍楼台。絮乱风轻,拂鞍沾袖,归路似章街④。注:①春丛:春季丛生的草木。②六花:雪花,因其结晶为六瓣,所以叫六花。③剪刀:喻春风。④章街:章台街,汉代都城长安街名,街旁多植柳树。28.首句“玉壶冰莹兽炉灰”有什么作用?29.赏析“絮乱风轻,拂鞍沾袖”的妙处。【答案】28.首句,写玉壶上结了一层晶莹透亮的冰,通过室内景表现天气之寒冷,为下文咏雪作铺垫。 29.运用比喻手法,将雪花比作柳絮,飘扬着的雪花似迎风飞舞的柳絮,有的拂过行人的马鞍,有的飘落在他的衣袖上。将雪花轻盈的形态以及似乎不舍归人离去的情态写得生动形象,颇有情趣。【解析】28.本题考查诗句的赏析和作用。首句“玉壶冰莹兽炉灰”意思是室内的玉壶上结了一层晶莹透亮的冰,取暖香炉内的香也烧成了灰烬。此句从闺中人的视角咏雪,通过室内之“玉壶冰莹”侧面表现天气之寒冷,同时,也为下文咏雪作铺垫。29.本题考查诗句的赏析。“絮乱风轻,拂鞍沾袖”一句意思是:飘扬着的雪花似迎风飞舞的柳絮,有的拂过行人的马鞍,有的飘落在他的衣袖上。此两句运用了比喻的修辞手法,将雪花比作飘飞的柳絮,生动形象地写出了雪花的轻盈。同时,“拂鞍沾袖”生动形象地赋予雪花人的情感,雪花拂过马鞍,沾着衣袖,似乎不愿归人离去,把雪花写得颇有意趣。30.阅读下面的诗歌,完成下面小题。【甲】渔家傲·秋思范仲淹塞下秋来风景异,衡阳雁去无留意。四面边声连角起,千嶂里,长烟落日孤城闭。浊酒一杯家万里,燕然未勒归无计。羌管悠悠霜满地,人不寐,将军白发征夫泪。【乙】从军行杨炯烽火照西京①,心中自不平。牙璋②辞凤阙③,铁骑绕龙城。雪暗凋旗画,风多杂鼓声。宁为百夫长,胜作一书生。【注】①西京:长安。②牙璋:古代发兵所用之兵符,分为两块,相合处呈牙状,朝廷和主帅各执其半。指代奉命出征的将帅。③凤阙:阙名。汉建章宫的圆阙上有金凤,故以凤阙指皇宫。(1)【乙】诗颈联从视听两个角度侧面展现了两军交锋的战斗场面,请用形象的语言加以描绘。(2)说说两首诗词流露的情感有何相同和不同?【答案】(1)漫天大雪,纷纷飘下,军旗上的彩画,都黯淡模糊,看不清了;北风呼啸,战鼓雷鸣,各种声响混杂在一起,回荡在边地沙场之上。(2)相同点:甲乙两诗都表达了作者建功立业,报效国家的愿望。不同点:甲词还抒发了将士久陷疆场,思念家乡,遥无归期的感慨;乙诗表达了做一名军队里的低级军官比做整天舞文弄墨高谈阔论者强的看法。(或表达了作诗人投笔从戎,保家卫国的爱国情怀。)(意思相近即可)【解析】(1)本题考查描绘诗中画面的能力。描绘诗中展现的画面时,要抓住诗中的主要景物,用自己的语言再现画面。描述时要忠于原诗,要用自己的联想和想象加以再创造,语言力求优美。颈联“雪暗凋旗画,风多杂鼓声”,前句从人的视觉出发:大雪弥漫,遮天蔽日,使军旗上的彩画都显得黯然失色;后句从人的听觉出发:狂风呼啸,与雄壮的进军鼓声交织在一起。两句诗,有声有色,各臻其妙。诗人别具机抒,以象征军队的“旗”和“鼓”,描绘了出征将士冒雪同敌人搏斗的坚强无畏精神和在战鼓声激励下奋勇杀敌的悲壮激烈场面。(2)本题考查诗歌感情分析。解答的关键在于通过句意,理解作者情感,此外,还可以通过结合诗句的上下文来帮助自己理解作者的情感。《渔家傲》的“浊酒一杯家万里,燕然未勒归无计”意思是“饮一杯浊酒,不由得想起万里之外的家乡,未能像窦宪那样战胜敌人,刻石燕然,不能早作归计”,句子透露了作者的思乡之情,同时表达了能早日战胜敌人,报效国家的伟大抱负;《从军行》的“宁为百夫长,胜作一书生”意思是“我宁作百夫长冲锋陷阵,也不耐守笔砚做个书生”,诗句表达了作者投笔从戎,想“冲锋陷阵”,在战场上战胜敌人,建功立业,报效国家的伟大志向。因而,两首诗的共同点是:都表达了作者渴望在战场上战胜敌人,建功立业,报效国家的伟大志向。不同点是:《渔家傲》还透露了将士长期征战沙场,思念家乡的情感;乙诗表达了做一名军队里的低级军官比做整天舞文弄墨高谈阔论者强的看法。31.阅读下面的诗歌,完成各题。咸阳城东楼许浑一上高城万里愁,蒹葭杨柳似汀洲。溪云初起日沉阁,山雨欲来风满楼。鸟下绿芜秦苑夕,蝉鸣黄叶汉宫秋。行人莫问当年事,故国东来渭水流。(1)下列对《咸阳城东楼》的赏析,有误的一项是( )A.颔联用云、日、风、雨层层推进,颈联以绿芜、黄叶营造渲染了萧条凄凉的意境。B.全诗情景交融,景中寓情,诗人赋予抽象情感以形体,表达了对历史和现实的深刻思考。C.“莫问”二字是劝诫之词,令人思索,表达了诗人相思的忧愁和感古伤今的悲凉。D.“山雨欲来风满楼”是全诗的警句,只用寥寥七个字便形象地写出了山城雨即将来临的情景。(2)晚唐著名诗人许浑从小便立下建功立业的远大抱负,然而报国无门,他郁郁寡欢、万念俱灰。“一上高城万里愁,蒹葭杨柳似汀洲”,一个“愁”字奠定了全诗的感情基调,请从表现手法角度赏析这两句。【答案】(1)C(2)通过描写、抒情的方式,表现出诗人思乡之愁、忧国之愁、怀古之愁。【解析】(1)本题考查诗歌内容理解。C.根据“行人莫问当年事,故国东来渭水流”可知,意思是:来往的过客不要问从前的事,只有渭水一如既往地向东流。“莫问”二字,并非劝诫之词,而是令人思索之语,它让读者从悲凉颓败的自然景物中探求历史的教训;表述有误;故选C。(2)本题考查诗句赏析。根据首联“一上高城万里愁,蒹葭杨柳似汀洲”可知,意思是:诗人一登上咸阳高高的城楼,向南望去,远处烟笼蒹葭,雾罩杨柳,很像长江中的汀洲;描绘了登楼所见之景;这是描写的表现手法;诗人游宦长安,远离家乡,一旦登临,思乡之情涌上心头。蒹葭杨柳,居然略类江南。万里之愁,正以乡思为始。运用抒情的方式,表达浓烈的思乡之愁;“一上”表明触发诗人情感时间之短瞬,“万里”则极言愁思空间之迢遥广大,一个“愁”字,奠定了全诗的基调。笔触低沉,景致凄迷,触景生情,苍凉伤感的情怀落笔即出,意远而势雄。32.阅读下面两首词,完成小题。卜算子·黄州定慧院寓居作(北宋)苏轼缺月挂疏桐,漏断人初静。谁见幽人独往来,缥缈孤鸿影。惊起却回头,有恨无人省。拣尽寒被不肯栖,寂寞沙洲冷。西江月(北宋)苏轼顷在黄州,春夜行薪水中,过酒家饮。酒醉,乘月至一溪桥上,解鞍曲脑,醉卧少休。及觉已晓,乱山攒拥,流水锵然,疑非尘世也。书此数语桥柱上。照野弥弥浅浪,横空隐隐层霄。障泥①未解玉骗骄,我欲醉眠芳草。可惜一溪风月,莫教踏碎琼瑶②。解鞍款枕绿杨桥,杜宇一声春晓。[注]①障泥:马韩,垂于马两侧以挡泥土。②琼瑶:美玉。这里形容月亮在水中的倒影。阅读上面两首词,回答问题。(1)第一首词的上阕通过描绘 _____、_____等景物,渲染了凄清孤寂的氛围。(2)第二首词中“照野弥弥浅浪,横空隐隐层霄”画面宁静美好,请用生动的语言加以描绘。(3)两首词都是苏轼谪居黄州时的作品,请分析其思想情感的不同之处。【答案】(1)缺月、疏桐 孤鸿(2)春夜,词人在蕲水边骑马而行,经过酒家饮酒,醉后乘着月色归去,经过一座溪桥。明月当空,春水涨满、溪流汩汩。广阔的天空还有淡淡的云层,隐隐约约在若有若无之间。(3)《卜算子•黄州定慧院寓居作》通过写鸿的孤独缥缈、惊起回头、怀抱幽恨和选求宿处,表达了作者贬谪黄州时期的孤寂心情和高洁自许、不愿随波逐流的人生追求。《西江月》苏轼被贬黄州期间,欣赏美丽的月色,被月色陶醉,他能以山水为乐。表现出一个物我两忘、超然物外的境界,抒发了作者乐观、豁达、以顺处逆的襟怀。【解析】(1)本题考试诗歌内容理解。上阕写的正是深夜院中所见的景色。“缺月挂疏桐,漏断人初静”,通过描绘缺月、疏桐,营造了一个夜深人静、月挂疏桐的孤寂氛围,为“缥缈孤鸿影”中的“孤鸿”的出场作铺垫。通过描绘缺月、疏桐、孤鸿的描绘,渲染了凄清孤寂的氛围。(2)本题考查描绘画面内容。此句意思:月光下小溪春水涨满、水波涌动,隐隐约约的看见天空中云气弥漫。“照野弥弥浅浪,横空隐隐层霄”是作者的归途所见。“照野”, 突出月色之佳。用“弥弥”形容“浅浪”, 把春水涨满溪流汩汩的景象表现出来了。广阔的天空还有淡淡的云层。“横空”,写出了天宇之广,说云层隐隐约约在若有若无之间。在此基础上加上自己的想象和联想,运用优美的语言进行描述即可。(3)本题考查情感主旨。《卜算子·黄州定慧院寓居作》下片“惊起却回头,有恨无人省”写突然惊起又回过头来,心有怨恨却无人知情;诗人没有知音,表现诗人的孤苦;“拣尽寒枝不肯栖,寂寞沙洲冷”,写孤鸿遭遇不幸,心怀幽恨,惊恐不已,在寒枝间飞来飞去,拣尽寒枝不肯栖息,只好落宿于寂寞荒冷的沙洲,度过这样寒冷的夜晚。词人以象征手法,通过鸿的孤独缥缈,惊起回头,怀抱幽恨和选求宿处,表达了作者贬谪黄州时期的孤寂处境和高洁自许、不愿随波逐流的心境。《西江月》上片写词人路上的见闻和醉态,下片写月下云雾朦胧之景,望水中之月,斜卧绿杨桥上入梦乡,言语间表现了词人对美好景物的怜惜之情,以空山明月般澄澈、空灵的心境,描绘了一个富有诗情画意的月夜人间仙境图,表现出一个物我两忘超然物外的境界,抒发了作者乐观、豁达、以顺处逆的襟怀。33.阅读下面的诗歌,完成后面的小题。浣溪沙①王安石百亩中庭半是苔,门前白道水萦回。爱闲能有几人来?小院回廊春寂寂,山桃溪杏两三栽。为谁零落为谁开?【注释】①这首诗大约写于宋神宗年间,王安石变法二度受挫,辞去相位后退居金陵。(1)请概括诗人归隐地的生活环境。(2)这首词以“为谁零落为谁开”问句结尾,有什么好处?【答案】(1)宽敞的庭院。洁净的小路,盘旋的溪水,优美、清闲,呈现出一种淡泊宁静的村野生活情景。(2)示例:用“为谁零落为谁开”十分贴切,这里词人以桃花自喻,表达自己也像它一样孤独,没有人问起,落寞之情溢于言表,反映环境的零落荒寂,令人唏嘘。这首词以问句结束,启迪读者思考词外之意,深化了词的内涵。【解析】(1)考查诗歌内容的理解。诗词大意:很大的庭院,里有一半长满了青苔,门前的白沙道曲曲折折,和清清的溪流缠绕在一起。平日里没有几个人过来,我喜爱这宁静的生活。小院和回廊在春天里都寂静无声,山坡上溪水边三三两两地栽着几棵桃树和杏树。在这个寂寞的地方。请问你们是为谁凋谢又为谁开放的呢?上片中“百亩中庭半是苔,门前白道水萦回”作者写到寓居的环境有宽敞的庭院,洁净的小路,盘旋的溪水;下片中“小院回廊春寂寂,山桃溪杏两三栽”,写到院子里曲折的回廊非常安静,山上的桃花开了,溪边的杏树,三三两两地种在一起,优美、清闲,呈现出一种淡泊宁静的村野生活情景。(2)考查诗歌内容的理解。“为谁零落为谁开”,山里的桃花已经开放了,不知道它们是为谁开放,为谁凋零。再美又有什么用,哪里会有人来这里欣赏呢。这小园风情景物,寓意颇深。王安石变法二度受挫,辞去相位后退居金陵。一场惊天动地的“变法”,其中酸苦,用“为谁零落为谁开”之问也十分贴切。这里词人以桃花自喻,表达自己也像它一样孤独,没有人问起,落寞之情溢于言表,反映环境的零落荒寂,令人唏嘘。同时,这首词以问句结束,启迪读者思考词外之意,深化了词的内涵,扩大了词的表现空间。34.阅读下面的宋词,完成问题。雁门太守行李贺黑云压城城欲摧,甲光向日金鳞开。角声满天秋色里,塞上燕脂凝夜紫。半卷红旗临易水,霜重鼓寒声不起。报君黄金台上意,提携玉龙为君死。(1)下面对本诗理解、分析不正确的一项是( )A.“雁门太守行”是古乐府曲名,“行”是古诗体裁,本诗借用它作诗题写当时战事。B.全诗以色彩斑斓的词语,浓墨重彩地描绘战争场景,构成了奇特的意境,歌颂守边战士浴血奋战、视死如归的英雄气概。C.诗的前四句写日落前的情景。“角声满天秋色里,塞上燕脂凝夜紫”,是说塞上的泥土在晚霞映衬下凝成胭脂色,写出了边塞风光的秀美。D.“半卷红旗临易水”使人联想起“风萧萧兮易水寒,壮士一去兮不复还!”喻示将士们无所畏惧,勇往直前。(2)下面选项中,对本诗理解有误的一项是( )A.作者描写战争的惨烈场景,并没有进行具体的战争场面的刻画,而是从颜色入手。B.首联渲染战前敌军压境,我军英勇应战的危急、紧张气氛。C.颈联中“声不起”写出战士们在沉重的鼓声中斗志难振。D.尾联运用典故,表达守城将士们誓死报效君王的决心。【答案】(1)C (2)C【解析】(1)本题考查诗歌理解。C.“塞上燕脂凝夜紫”这里指暮色中塞上泥土有如胭脂凝成。凝夜紫,在暮色中呈现出暗紫色。凝,凝聚。“燕脂”“夜紫”暗指战场血迹,所以并非“写出了边塞风光的秀美”,反而是反映战争的残酷,所以选项错误。故选C。(2)本题考查诗歌内容的理解。C.“霜重鼓寒声不起”描写苦战的场面:驰援部队一迫近敌军的营垒,便击鼓助威,投入战斗。无奈夜寒霜重,连战鼓也擂不响。面对重重困难,将士们毫不气馁。所以选项中“声不起”写出战士们在沉重的鼓声中斗志难振的说法是错误的。故选C。35.阅读下面一首词,完成问题。破阵子•为陈同甫赋壮词以寄之宋•辛弃疾醉里挑灯看剑,梦回吹角连营。八百里分麾下炙,五十弦翻塞外声。沙场秋点兵。马作的卢飞快,弓如霹雳弦惊。了却君王天下事,赢得生前身后名。可怜白发生!(1)辛弃疾是南宋著名的爱国将领,他笔下的战争场面气势磅礴。请任选一个角度,赏析“马作的卢飞快,弓如霹雳弦惊”。(2)有人认为“了却君王天下事,赢得生前身后名”流露了词人追名逐利的心理,你同意这种看法吗?说说你的理解。【答案】(1)运用比喻的修辞手法,写出了紧张激烈的战斗场面。(2)不同意。整首词抒发了作者想要杀敌报国,建功立业却已年老体迈的壮志未酬的思想感情。【解析】(1)本题考查赏析句子。“马作的卢飞快,弓如霹雳弦惊”的意思是:战马像的卢马一样跑得飞快,弓箭像惊雷一样,震耳离弦。此处采用比喻的修辞手法,将弓箭发出的响声比喻成惊雷,形象生动地写出了战场上的惊险与激烈。(2)本题考查学生对作者思想情感的理解能力。该词是作者失意闲居信州时所作,无前人沙场征战之苦,而有沙场征战的热烈,“了却君王天下事,赢得生前身后名”表现了作者杀敌报国,恢复祖国山河,建功立业的远大抱负;尾句“可怜白发生”,写作者已到暮年,抒发壮志不酬的悲愤心情。六、文言文阅读 一、阅读下面的文言文,完成下面小题。鱼我所欲也鱼,我所欲也;熊掌,亦我所欲也。二者不可得兼,舍鱼而取熊掌者也。生,亦我所欲也;义,亦我所欲也。二者不可得兼,舍生而取义者也。……万钟则不辩礼义而受之,万钟于我何加焉!为宫室之美、妻妾之奉、所识穷乏者得我与?乡为身死而不受,今为宫室之美为之;乡为身死而不受,今为妻妾之奉为之;乡为身死而不受,今为所识穷乏者得我而为之:是亦不可以已乎?此之谓失其本心。1.解释下面加点的词。(1)所识穷乏者得我与(2)是亦不可以已乎【答案】(1)同“德”,感恩(感激) (2)停止【解析】本题考查重点词语在文中的含义。词语解释时要注意词语在具体语言环境中的用法,如一词多义、古今异义、词类活用、通假字等现象。(1)句意:所认识的穷困的人感激我吗?得:同“德”,感恩(感激)。(2)句意:这种做法不是可以让它停止了吗?已:停止。2.翻译下面的句子。万钟于我何加焉!【答案】优厚的俸禄对我有什么益处呢?【解析】本题考查文言句子的翻译。翻译文言语句要抓住句子中的关键词汇,做到译句文从字顺,符合现代汉语语法规范。注意重点词的解释:万钟,形容位高禄厚;何加,有什么益处。3.在中华民族历史上,有无数的仁人志士都把“舍生取义”奉为人生准则,请你举出一个事例。【答案】示例:文天祥宁死不叛国,刘胡兰为保守党的秘密英勇就义,谭嗣同为变法慷慨赴死……【解析】本题考查积累。列举的人物的事例一定是表现了“舍生取义”这一人生准则的,示例:无产阶级革命家夏明翰留下“砍头不要紧,只要主义真。杀了夏明翰,还有后来人”的就义诗慷慨赴死。参考译文:鱼是我所想要的,熊掌也是我所想要的,如果这两种东西不能同时得到,那么我宁愿舍弃鱼而选取熊掌。生命是我所想要的,正义也是我所想要的,如果这两样东西不能同时得到,那么我宁愿牺牲生命而选取大义。(可是有的人)见了优厚俸禄却不辨是否合乎礼义就接受了。这样,优厚的俸禄对我有什么好处呢?是为了住所的华丽、妻妾的侍奉和熟识的穷人感激我吗?从前(有人)为了(道义)(宁愿)死也不愿接受(别人的施舍),如今(有人)却为了住宅的华美而接受了;从前(有人)为了(道义)(宁愿)死也不愿接受(别人的施舍),如今(有人)却为了得到妻妾的侍奉而接受了;从前(有人)为了(道义)(宁愿)死也不愿接受(别人的施舍),如今(有人)却为了让所认识穷困贫乏的人感激他们的恩德而接受了它。这种(行为)难道不可以停止吗?这就叫做丧失了人的天性(指羞恶廉耻之心)。二、阅读下面的文言文,完成下面小题。 十年春,齐师伐我。公将战,曹刿请见。其乡人曰:“肉食者谋之,又何间焉?”刿曰:“肉食者鄙,未能远谋。”乃入见。问:“何以战?”公曰:“衣食所安,弗敢专也,必以分人。”对曰:“小惠未遍,民弗从也。”公曰:“牺牲玉帛,弗敢加也,必以信。”对曰:“小信未孚,神弗福也。”公曰:“小大之狱,虽不能察,必以情。”对曰:“忠之属也。可以一战。战则请从。”公与之乘,战于长勺。公将鼓之。刿曰:“未可。”齐人三鼓。刿曰:“可矣。”齐师败绩。公将驰之。刿曰:“未可。”下视其辙,登轼而望之,曰:“可矣。”遂逐齐师。 既克,公问其故。对曰:“夫战,勇气也。一鼓作气,再而衰,三而竭。彼竭我盈,故克之。既克,公问其故。对曰:“夫战,勇气也。一鼓作气,再而衰,三而竭。彼竭我盈,故克之。夫大国,难测也,惧有伏焉。吾视其辙乱,望其旗靡,故逐之。”(选自《曹刿论战》)4.下列句子中,加点词的意义相同的一组是( )A.又何间焉 时时而间进B.小信未孚 愿陛下亲之信之C.忠之属也 有良田、美池、桑竹之属D.战于长勺 万钟于我何加焉【答案】C【解析】本题考查一词多义,注意根据语境确定词义。A.间:参与/间或,偶尔,有时候;B.信:诚信,信用/亲近,信任;C.属:都解释为“类”;D.于:介词,在/介词,对于;故选C。5.用现代汉语翻译下列句子。(1)肉食者鄙,未能远谋。(2)一鼓作气,再而衰,三而竭。【答案】(1)当权者目光短浅,不能深谋远虑。(2)已经穷尽了。第一次击鼓能够鼓起士气,第二次击鼓时(士气)减弱,第三次击鼓时(士气)。【解析】本题考查学生对句子的翻译能力。我们在翻译句子时要注意通假字、词类活用、一词多义、特殊句式等情况。重点词有:(1)肉食者:吃肉的人,指当权者;鄙:鄙陋,目光短浅;远谋,深谋远虑。(2)鼓:击鼓;再:第二次;竭:穷尽。6.下列对文章的理解和分析,不正确的一项是( )A.鲁庄公接见曹刿,耐心回答其“三问”,说明鲁庄公能够礼贤下士、广开言路。B.鲁庄公和曹刿的战前对话,充分说明了曹刿的军事思想是以民心向背为基础的。C.文章的脉络非常清晰,始终以曹刿的言论贯穿全文,中心突出,无一处不照应。D.文章记述的是中国历史上著名的以弱胜强的战例,着重记叙了长勺之战的具体过程。【答案】D【解析】本题考查对文章的理解和分析。D.“着重记叙了长勺之战的具体过程”有误。文章并不是重点记叙长勺之战的具体过程,而是论述曹刿关于战争的理论。全文共三段。第一段可分两层,第一层写曹刿求见鲁庄公的原因;第二层论述战前的政治准备——取信于民。第二段略写了长勺之战的经过。第三段,曹刿论述赢得战役胜利的原因,可分为两方面。一是论述了利于开始反攻的时机是彼竭我盈之时;二是论述了追击开始的时机是辙乱旗靡之时。因此“论战”是全文的重点。故选D。参考译文鲁庄公十年春天(前684年)的春天,齐国军队攻打我们鲁国,鲁庄公将要迎战。曹刿请求庄公接见。他的同乡说:“大官们会谋划这件事的,你又何必参与呢?”曹刿说:“大官们眼光短浅,不能深谋远虑。”于是进宫去见庄公。曹刿问庄公:“您凭什么跟齐国打仗?”庄公说:“衣食是使人生活安定的东西,我不敢独自占有,一定拿来分给别人。”曹刿说:“这种小恩小惠不能遍及百姓,老百姓是不会听从您的。”庄公说:“祭祀用的牛羊、玉帛之类,我从来不敢虚报数目,一定要做到诚实可信。”曹刿说:“这点诚意难以使人信服,神是不会保佑您的。”庄公说:“大大小小的案件,虽然不能件件都了解得清楚,但一定要根据自己的诚心处理。”曹刿说:“这才是尽本职的事,可以凭这一点去打仗。作战时请允许我跟您去。”鲁庄公和曹刿同坐一辆战车。在长勺和齐军作战。庄公(一上阵)就要击鼓进军,曹刿说:“(现在)不行。”齐军擂过三通战鼓后,曹刿说:“可以击鼓进军了。”齐军大败。庄公正要下令追击,曹刿说:“还不行。”(说完就)下车去察看齐军的车印,又登上车前横木望齐军(的队形),(这才)说:“可以追击了。”于是追击齐军。 打了胜仗以后,鲁庄公询问取胜的原因。曹刿说:“打仗,要靠勇气。头通鼓能振作士兵们的勇气,二通鼓时勇气减弱,到三通鼓时勇气已经穷尽了。敌方的勇气已经穷尽而我方的勇气正盛,所以打败了他们。(齐是)大国,难以摸清(它的情况),怕的是有埋伏,我发现他们的车印混乱,军旗也倒下了,所以才下令追击他们。”三、阅读下面的文言文,完成下面小题。送东阳马生序宋濂余幼时即嗜学。家贫,无从致书以观,每假借于藏书之家,手自笔录,计日以还。天大寒,砚冰坚,手指不可屈伸,弗之怠。录毕,走送之,不敢稍逾约。以是人多以书假余,余因得遍观群书。既加冠,益慕圣贤之道,又患无硕师、名人与游,尝趋百里外,从乡之先达执经叩问。先达德隆望尊,门人弟子填其室,未尝稍降辞色。余立侍左右,援疑质理,俯身倾耳以请;或遇其叱咄,色愈恭,礼愈至,不敢出一言以复;俟其欣悦,则又请焉。故余虽愚,卒获有所闻。当余之从师也,负箧曳屣行深山巨谷中。穷冬烈风,大雪深数尺,足肤皲裂而不知。至舍,四支僵劲不能动,媵人持汤沃灌,以衾拥覆,久而乃和。寓逆旅,主人日再食,无鲜肥滋味之享。同舍生皆被绮绣,戴朱缨宝饰之帽,腰白玉之环,左佩刀,右备容臭,烨然若神人;余则缊袍敝衣处其间,略无慕艳意,以中有足乐者,不知口体之奉不若人也。盖余之勤且艰若此。今虽耄老,未有所成,犹幸预君子之列,而承天子之宠光,缀公卿之后,日侍坐备顾问,四海亦谬称其氏名,况才之过于余者乎?7.解释下列加点词的含义。(1)益慕圣贤之道_____________ (2)援疑质理____________________(3)主人日再食_______________ (4)腰白玉之环__________________【答案】学说或学问 询问 供养,给……吃 腰佩,在腰上佩戴【解析】本题考查学生对文言实词的理解能力。解答时,要注意在理解句意的基础上解释词语,尤其注意通假字、古今异义、词类活用等特殊情况。(1)句意为:(我)更加仰慕古代圣贤的学说。道,学说、学问;(2)句意为:提出疑难,询问道理。质,询问;(3)句意为:旅店老板每天供应两顿饭。食,给……吃;(4)句意为:腰间挂着白玉环。腰,名词作动词,腰佩。8.请准确翻译下面句子。(1)或遇其叱咄,色愈恭,礼愈至,不敢出一言以复。(2)以中有足乐者,不知口体之奉不若人也。【答案】(1)有时遇到他斥责,我的表情更加恭敬,礼节更加周到,不敢多说一句辩解的话。(2)因为心中有足以快乐的事,不感到衣食的享受比不上其他的人。【解析】本题考查文言语句的翻译。翻译的要求是做到“信、达、雅”,翻译的方法是“增、删、调、换、补、移”,我们在翻译句子的时候要注意通假字、词类活用、一词多义、特殊句式等情况。(1)重点词:或,有时;叱咄,训斥,呵责;色,脸色、态度;愈,更加;至,周到;复,回复,这里 指辩解。(2)重点词:以,因为;足,值得;口体之奉,吃穿的供给;不若,不如。9.下列对文章内容和写法的分析,错误的一项是( )A.作者第一段逐次陈说家境贫寒给自己读书带来的困难,道出了“遍观群书”的不易。B.本文在叙事上详略有致,结构严密。详写自己年轻时求学的艰难勤奋,更好地突出了主题。C.作者写自己求学的经历,最终取得了不错的成就,只是为了展示自己求学时的艰辛曲折与自己坚强的意志。D.本文多用对比手法,如用先达的倨傲粗暴与作者的谦卑恭敬对比,突出作者一心向学;用同舍生装束豪华与作者缊袍蔽衣对比,突出作者内心强大充实。【答案】C【解析】本题考查对文章内容的分析、理解。C.根据“谓余勉乡人以学者,余之志也;诋我夸际遇之盛而骄乡人者,岂知余者哉”可知,作者勉励青年人珍惜良好的读书环境,专心治学;所以,不只是“只是为了展示自己求学时的艰辛曲折与自己坚强的意志”;故选C。10.课文与链接材料都是劝诫别人学习的文章,请结合课文与链接材料的具体内容分析,一个人学有所成,要具备哪些外部因素与内部因素? 〖链接材料〗父母养其子而不教,是不爱其子也。虽教而不严,是亦不爱其子也。父母教而不学,是子不爱其身也。虽学而不勤,是亦不爱其身也。是故养子必教,教则必严;严则必勤,勤则必成。学,则庶人之子为公卿;不学,则公卿之子为庶人。(节选自柳永《劝学文》)父母生养了子女却不很好的教育他们,就是不爱自己的孩子;即便教育了孩子却不严格要求,也同样是不爱孩子。如果说父母符合标准地教育了孩子,但子女却不自主学习,这便是他们不爱惜自己了。如果他们学习了却不勤奋努力,同样是不珍惜自身。所以,作为父母,既然生养了子女就一定有教育他们的义务,而且对子女的教育一定要严格,只有严格的教育,才可以督促孩子勤奋学习,足够勤奋就可以学有所成了。如果能利用所学的知识去通过考试,那么就算是平民子弟也会显赫一时;如果有了渊博的学识,即便是贫民子弟也有望成为将相公侯,如若不学无术,即便是王公贵族之子也可能变成贫民百姓啊!【答案】外部因素要有严格的教育,课文中“先达德隆望尊,未尝稍降辞色”,有时还会遇到他的训斥,链接材料中“虽教而不严,是亦不爱其子也”。内部因素要有自己的勤奋,课文中“手指不可屈伸,弗之怠。”链接材料里“虽学而不勤,是亦不爱其身也”。(外部因素归纳,同时结合课文或者链接材料内容分析,内部因素归纳,结合课文或者链接材料内容分析)【解析】本题考查比较阅读。根据《送东阳马生序》“余幼时即嗜学”、“家贫,无从致书以观,每假借于藏书之家,手自笔录,计日以还。天大寒,砚冰坚,手指不可屈伸,弗之怠。录毕,走送之,不敢稍逾约”、“盖余之勤且艰若此”可知,学习要勤奋刻苦;根据《送东阳马生序》“先达德隆望尊,门人弟子填其室,未尝稍降辞色。余立侍左右,援疑质理,俯身倾耳以请;或遇其叱咄,色愈恭,礼愈至,不敢出一言以复;俟其欣悦,则又请焉”可知,学习要有严格的教育;根据〖链接材料〗“父母养其子而不教,是不爱其子也。虽教而不严,是亦不爱其子也”“是故养子必教,教则必严”可知,学习要有严格的教育;根据〖链接材料〗“虽学而不勤,是亦不爱其身也”、“严则必勤,勤则必成”可知,学习要勤奋刻苦;由此可知,一个人学有所成,要具备内部因素是:学习要勤奋刻苦;外部因素是:要有严格的教育。参考译文我年幼时就非常爱好读书。家里贫穷,无法得到书来看,常常向藏书的人家求借,亲手抄录,计算着日期按时送还。冬天非常寒冷,砚台里的墨汁都结了冰,手指冻得不能弯曲和伸直,也不放松抄录书。抄写完毕后,便马上跑去还书,不敢稍微超过约定的期限。因此有很多人都愿意把书借给我,于是我能够遍观群书。成年以后,我更加仰慕古代圣贤的学说,又苦于不能与学识渊博的老师和名人交往,曾经赶到数百里以外,拿着经书向乡里有道德学问的前辈请教。前辈德高望重,门人弟子挤满了他的屋子,他的言辞和态度从未稍有委婉。我站着陪侍在他左右,提出疑难,询问道理,俯下身子,侧着耳朵恭敬地请教;有时遇到他大声斥责,我的表情更加恭顺,礼节更加周到,不敢说一个字反驳;等到他高兴了,则又去请教。所以,我虽然愚笨,但最终获得不少教益。当我外出求师的时候,背着书箱,拖着鞋子,行走在深山峡谷之中。隆冬时节,刮着猛烈的寒风,雪有好几尺深,脚上的皮肤受冻裂开都不知道。回到客舍,四肢僵硬动弹不得。服侍的人拿着热水为我洗浴,用被子裹着我,很久才暖和起来。寄居在旅店里,旅店老板每天供应两顿饭,没有新鲜肥嫩的美味享受。同客舍的人都穿着华丽的衣服,戴着用红色帽带和珠宝装饰的帽子,腰间挂着白玉环,左边佩戴宝刀,右边挂着香囊,光彩鲜明,像神仙一样;我却穿着破旧的衣服处于他们之间,但我毫无羡慕的心。因为心中有足以快乐的事情,所以不觉得吃的、穿的享受不如别人。我求学的辛勤和艰苦就是像这个样子。如今我虽已年老,没有什么成就,但所幸还得以置身于君子的行列中,承受着天子的恩宠荣耀,追随在公卿之后,每天陪侍着皇上,听候询问,天底下也不适当地称颂自己的姓名,更何况才能超过我的人呢?四、阅读下面的文言文,完成下面小题。况钟,字伯律,靖安人。初以吏事尚书吕震,奇其才,荐授仪制司①主事。迁郎中。宣德五年,帝以郡守多不称职,会苏州等九府缺,皆雄剧②地,命部、院臣举其属之廉能者补之。钟用③尚书蹇义、胡濙等荐,擢知苏州,赐敕以遣之。苏州赋役繁重,豪猾④舞文为奸利,最号难治。钟乘传⑤至府。初视事,群吏环立请判牒⑥。钟佯不省,左右顾问,惟吏所欲行止。吏大喜,谓太守暗,易欺。越三日,召诘之曰:“前某事宜行,若止我;某事止,若强我行。若辈舞文久,罪当死。”立捶杀数人,尽斥属僚之贪虐庸懦者。一府大震,皆奉法。钟乃蠲⑦烦苛,立条教,事不便民者,立上书言之。当是时,屡诏减苏、松重赋。钟与巡抚周忱悉心计画,奏免七十余万石。凡忱所行善政,钟皆协力成之。所积济农仓粟岁数十万石,振荒⑧之外,以代民间杂办及逋租⑨。(节选自《明史·况钟传》)【注】①仪制司:朝廷礼部的一个内部机构。下文的“主事”“郎中”“郡守”“巡抚”都是官职名。 ②雄剧:地位重要而事务繁重。③用:因为。④豪猾:(苏州当地的)豪强和狡猾的属吏。⑤乘传:乘坐驿车。⑥牒:文书。⑦蠲(juān):免除。⑧振荒:赈济灾荒。⑨逋(bū)租:拖欠的租税。11.解释文中加点词的含义。(1)以( ) (2)会( ) (3)诘( )【答案】以(因为) 会(适逢,恰巧遇到) 诘(诘责,责问)【解析】本题考查重点文言词语在文中的含义。解释词语要注意理解文言词语在具体语言环境中的用法,如通假字、词性活用、古今异义等现象。(1)“帝以郡守多不称职”意思是:明宣宗因为感到各地郡守大多不能称职。以,因为。(2)“会苏州等九府缺”意思是:又恰逢苏州等九府缺少知府。会,适逢,恰巧遇到。(3)“召诘之曰”意思是:况钟召集群吏责问他们道。诘,诘责,责问。12.把文中画横线的句子翻译成现代汉语。(1)吏大喜,谓太守暗,易欺。(2)钟乃蠲烦苛,立条教,事不便民者,立上书言之。【答案】(1)(这此)属吏非常高兴,说(况)太守(为人)昏昧糊涂,容易欺骗。(2)况钟于是免除烦琐苛刻的赋税,订立教民条文,有不便于老百姓的事情,立即上书提出。【解析】本题考查对文言句子的翻译能力。在翻译句子是首先要注意重点的实词、虚词、通假字、古今异义词和词类活用的情况,先要按照原句子的顺序翻译,然后按照现代汉语的习惯进行语序调整。重点词语:(1)大,非常。暗,愚昧糊涂。欺,欺骗。(2)蠲,免除。立条教,订立教民条文。13.根据选文内容,简要概括况钟治理苏州的措施。【答案】①用计处死了几个偷奸耍滑的官吏,让全府震动,奉公守法;②订立教民条文,想办法方便老百姓的生活;③想办法免减苏州府的苛捐杂税,待民以宽;④赈济灾荒,代交民间杂赋和租税。(答到三点即可)【解析】本题考查的是对文本内容的理解与分析。解答此题的关键是在理解文章内容的基础上,把握主旨,联系实际,根据题目的要求和提示的信息梳理内容,找出相关的语句,概括即可。由“越三日,召诘之曰:‘前某事宜行,若止我;某事止,若强我行。若辈舞文久,罪当死。’立捶杀数人,尽斥属僚之贪虐庸懦者。一府大震,皆奉法”可概括出:用计处死了几个偷奸耍滑的官吏,让全府震动,奉公守法。由“钟乃蠲烦苛,立条教,事不便民者,立上书言之”可概括出:订立教民条文,想办法方便老百姓的生活。由“当是时,屡诏减苏、松重赋。钟与巡抚周忱悉心计画,奏免七十余万石。凡忱所行善政,钟皆协力成之” 可概括出:想办法免减苏州府的苛捐杂税,待民以宽。由“所积济农仓粟岁数十万石,振荒之外,以代民间杂办及逋租”可概括出:赈济灾荒,代交民间杂赋和租税。参考译文:况钟,字伯律,是江西靖安人。起初为尚书吕震属吏,吕震对他的才能感到惊异,推荐授予他仪制司主事之官。后来又升为郎中。宣德五年,明宣宗因为感到各地郡守大多不能称职,又恰逢苏州等九府缺少知府,这九府都是重要难治之地,于是命令六部及都察院大臣推荐属下廉正有能力的官吏补各府的空缺。况钟由于得到尚书蹇义、胡濙等人举荐,升任苏州知府,宣宗特赐诰敕而派遣他前往。 苏州地区赋役繁重,豪强猾吏舞文弄墨以奸求利,是号称最难治理的地方。况钟乘驿站车马来到苏州府。他开始处理事务时,群吏围立在四周请他写下判牒。况钟装作不懂,向左右请教询问,一切按照属吏们的意图去办。群吏非常高兴,说知府昏暗好欺骗。过了三天,况钟召集群吏责问他们道:“前几天某件事应该办,你们阻止我;某件事不该办,你们强让我去做。你们这群人,舞文弄墨已久,罪该处死。”当即下令打死几个人,将属僚中贪婪、暴虐、庸暗、懦弱的全都痛斥一番。全府上下大为震动,全都奉法行事。于是况钟免除烦扰苛细的赋税,制定教民条文,事情有不利于老百姓的,就立即上书朝廷讲明。当时,多次下诏减轻苏州、松江的重赋。况钟和巡抚周忱精心计划,奏免赋税七十余万石。凡是周忱所推行的善政,况钟都协助大力办成。所积累的救济农民和存储的粮食每年有几十万石,除用来赈济灾荒之外,还用来代交民间杂赋和拖欠的租赋。五、阅读下面的文言文,完成下面小题。留园记①出阊门外三里而近,有刘氏寒碧庄焉。而问寒碧庄无知者,问有刘园乎,则皆曰有。盖是园也,在嘉庆初为刘君蓉峰所有,故即以其姓姓其园而曰刘园也。咸丰中,其泉石之胜,花木之美,亭榭之幽深,诚足为吴下名园之冠。及庚申、辛酉间,大乱洊至②,吴下名园半为墟莽。而所谓刘园者则岿然独存。同治中,芜秽不治。至光绪二年,为毗陵③盛旭人方伯所得,乃始修之。平之、攘之、剔之,嘉树荣而佳卉茁,奇石显而清流通。凉台燠馆,风亭月榭,高高下下,迤逦相属。春秋佳日,方伯与宾客觞咏其中,而都人士女或掎裳连袂而往游焉,于是出门者又无不曰刘园刘园云。方伯求余文为之记。余曰:“仍其旧名乎?抑肇锡④以嘉名乎?”方伯曰:“否,否。寒碧之名至今未熟于口,然则名之易而称之难也。吾不如从其所称而称之。人曰刘园,吾则曰留园,不易其音而易其字,即以其故名而为吾之新名。”余叹曰:美哉斯名乎,称其实矣!夫大乱之后、兵燹之余,高台倾而曲池平,不知凡几,而此园乃幸而无恙,岂非造物者留此名园以待贤者乎?吾知留园之名常留于天地间矣!【注】①选自《中华百年经典散文》,作者清代俞樾,有删节。②洊(jiàn)至:相继而至。③毗(pí)陵:今江苏常州市。④肇:开始:锡:通“赐”。14.留园历经岁月的变迁留存至今。下列不符合文意的一项是( )A.嘉庆年间,为刘蓉峰所以,故为“刘园”B.咸丰年间,此园经历战乱变为了废墟。C.同治年间,因得不到治理而杂草丛生。D.光绪二年,方伯修葺后改名为“留园”【答案】B【解析】本题考查理解文章内容。B.结合“咸丰中,其泉石之胜,花木之美,亭榭之幽深,诚足为吴下名园之冠”可知,选项错误。故选B。15.结合文中关于留园园名由来和园林景致的内容,说说你对最后一句“美哉斯名乎,称其实矣”的理解?【答案】“留园”与长时间以来的“刘园”称呼在读音上是保持一致的,便于人们传诵;“留园”历经战火却能侥幸保存下来,在作者看来是大自然留下这个有名的园子来等待有德行的人,使“留园”的内涵更深一层;作者更希望“留园”能长久地保存下去;从以上三个方面看,所以作者说“美哉斯名乎,称其实矣”。【解析】本题考查理解文章内容。结合“然则名之易而称之难也。吾不如从其所称而称之。人曰刘园,吾则曰留园,不易其音而易其字,即以其故名而为吾之新名”可得:“留园”与长时间以来的“刘园”称呼在读音上是保持一致的,便于人们传诵;结合“夫大乱之后、兵燹之余,高台倾而曲池平,不知凡几,而此园乃幸而无恙,岂非造物者留此名园以待贤者乎?”可得:“留园”历经战火却能侥幸保存下来,在作者看来是大自然留下这个有名的园子来等待有德行的人,使“留园”的内涵更深一层;结合“吾知留园之名常留于天地间矣!”可得:作者更希望“留园”能长久地保存下去;从以上三个方面看,所以作者说“美哉斯名乎,称其实矣”。16.下列句子中加点词的意思相同的一项是( )A.盖是园也 斯是陋室 (刘禹锡《陋室铭》)B.诚足为吴下名园之冠 朝服衣冠(《邹忌讽齐王纳谏》)C.嘉树荣而佳卉茁 清荣峻茂 (郦道元《三峡》)D.即以其故名而为吾之新名 公问其故 (《曹刿论战》)【答案】C【解析】本题考查文言词语的一词多义。A.这/判断词,是;B.第一/帽子;C.茂盛;D.旧的,原来的/原因;故选C。参考译文:出阊门外三里后,有一个刘氏寒碧庄。问有寒碧庄吗,没有知道的人,问有刘园吗,他们都说有。原来这个留园,在嘉庆初年为刘君蓉峰所有,所以就用他的姓作为这个园子的姓叫作刘园。咸丰年间,其山水之胜,花木之美,亭榭的幽深,这确实可以称为吴下名园第一名。到庚申、辛酉间,大混乱相继而至,吴地的名园有一半成为废墟。而所谓刘园则岿然独存。同治年间,荒芜不治。到光绪二年,为江苏常州市人方伯得到,于是开始整修它。平整土地、清除混乱、剔除杂草。好的树木变得茂盛而好的花卉茁壮生长,奇奇形怪状的石头显露出来而小溪也变得畅通。凉台热馆,风亭月榭,高高下下,连绵相连。春秋佳日,方伯和宾客饮酒赋诗在其中,而京都男女也有人拉着衣服连袖而去游览了。于是在这出阊门的人没有不说刘园、刘园的。方伯请求我写文章为留园做记。我说:“沿用原来名字吗?你不赐给一个吉祥的名字吗?” 方伯说:“不,不。寒碧庄知名到现在还没有被人熟知,难道不知道改名容易,被人熟知难呀。我不如跟从这个名称而称呼它。人说刘园,我就说留园,不改变他的音而改变它的字,就在他原来的名字,为我的起的新名字。”我感叹说:美啊,这些名字吗,符合实际的!等到战火焚毁破坏后,高台倾倒而曲池平,不知道有多少,但是这个园子竟然侥幸没有遭受灾祸,难道不是大自然留下这个有名的园子来等待有德行的人吗!我知道留园的名声常常停留在天地间了。 六、阅读下面的文言文,完成下面小题。大雅堂记黄庭坚丹棱杨素翁,英伟人也。其在州闾乡党有侠气,不少假借人,然以礼义,不以财力称长雄也。闻余欲尽书杜子美两川夔峡诸诗,刻石藏于蜀中好文喜事之家,素翁粲然向余请从亊焉。又欲作高屋广楹庥此石,因请名焉。余名之曰“大雅”,而告之曰:由杜子美来四百年,斯文委地,文章之士随世所能,杰出时辈未有升子美之堂者,况家室之好耶!余尝欲随欣然会意处笺以数语,终以汩没世俗,初不暇给。虽然子美诗妙处乃在无意为文,夫无意而意已至,非广之以《国风》《雅》《颂》,深之以《离骚》《九歌》,安能咀嚼其意味,闯然而入其门耶?故使后生辈自求之,则得之深矣;使后之登大雅堂者,能以余说而求之,则思过半矣。彼喜穿凿者,弃其大旨,取其发兴,于所遇林泉、人物、草木、鱼虫,以为物物皆有所托,如世间商度隐语者,则子美之诗委地矣。素翁可并刻此于大雅堂中。后生可畏,安知无涣然冰释,于斯文者乎!元符三年九月涪翁书。(出自《山谷集》,有删改)17.解释下列语句加点词。A.不少假借人( ) B.闻余欲尽书杜子美两川夔峡诸诗( )C.弃其大旨( ) D.后生可畏( )【答案】稍微 听说 主旨 敬畏,敬服【解析】本题考查对文言词语含义的理解。理解文言词语的含义要注意其特殊用法,如通假字、词类活用、一词多义和古今异义词等。(1)“不少假借人”的句意是:(他)稍微也不肯宽容。少:同“稍”,稍微。(2)“闻余欲尽书杜子美两川夔峡诸诗”的句意是:听说我想把杜甫在东西川和夔州写下的诗篇全部书写下来。闻:听说。(3)“弃其大旨”的句意是:抛弃了杜诗的主旨。旨:主旨。(4)“后生可畏”的句意是:后生学子是可以敬畏的。畏:敬畏,敬服。18.将文中画线句子翻译为现代汉语。(1)余尝欲随欣然会意处笺以数语,终以汩没世俗,初不暇给。(2)故使后生辈自求之,则得之深矣。【答案】(1)我曾经想根据我对杜诗的领会,来动笔写几句,但始终因整天被俗事羁绊,没有闲暇时间。(2)如果让青年学子自己去摸索,就会觉得很艰深吃力。【解析】本题考查翻译文言语句的能力。解答时一定要先回到语境中,根据语境读懂句子的整体意思,找出关键实词、虚词,查看有无特殊句式。尤其要注意一词多义、古今异义词、通假字等特殊的文言现象,重点实词必须翻译到位。翻译时要做到“信、达、雅”。(1)句中的“余(我)、尝(曾经)、笺(注释,这里是写)、以(因为)、暇(空闲)”几个词是重点词语。(2)句中的“使(假使,如果)、求(探求)、则(就)、深(深奥,艰深)”几个词是重点词语。19.文章着重阐述了作者对杜甫诗歌的认识,作者认为要想进入“大雅堂”应该怎样做呢?【答案】①杜诗的精妙之处在于“无意为文”,只有广泛涉猎各种典籍,博览群书,做到如杜诗般出神入化的境地;②学杜诗,不在于苟求文字和声律,而在于学习其内蕴情感,人格精神,即“无意而意已至”。【解析】本题考查对文章内容的理解与概括。在理解文意的基础上,结合文章内容概括作答。根据语句“虽然子美诗妙处乃在无意为文,夫无意而意已至,非广之以《国风》《雅》《颂》,深之以《离骚》《九歌》,安能咀嚼其意味,闯然而入其门耶?故使后生辈自求之,则得之深矣;使后之登大雅堂者,能以余说而求之,则思过半矣”可知,作者认为要进入“大雅堂”需向杜甫看齐,要达到“无意为文”,就必须广泛阅读、博览群书,做到与杜甫一般出神入化;还要学习杜甫的“无意而意已至”,就必须讲究在作诗时不再苛求文字和声律,将注意力放在其蕴含的感情上,这样才可以像杜甫一样,进入“大雅堂”。据此概括即可。参考译文:丹棱杨素翁,是一位英俊奇伟的义士。(他做事很较真),在当地乡里之间很讲豪侠之气,(对待别人的过失),(他)稍微也不肯宽容,但只凭礼义却从不依仗财势(欺凌弱者)而享誉一方。听说我想把杜甫在东西川和夔州写下的诗篇全部书写下来刻成诗碑,存放在蜀中喜好诗文者家中,素翁便欣然向我请求承担此事。并打算修一高堂广厦来陈列这些诗碑,于是请我为之命名。我替此屋取名叫“大雅堂”,并对他说:自杜甫以来四百年间,诗风颓丧,文人墨客受当世流俗影响,大都随波逐流,即使是杰出的才俊,其诗作也远未达到杜诗的境界,更不要说那些喜好诗文的一般士子了!我曾经想根据我对杜诗的领会,来动笔写几句,但始终因整天被俗事羁绊,没有闲暇时间。(即便我如愿写了),但杜诗的精妙之处乃在顺其自然,随心所欲,已达到出神入化的境地,如果不是广泛地涉猎《诗经》,熟悉《国风》《雅》《颂》,深刻领悟《离骚》《九歌》的人,怎么能咀嚼、理解杜诗的内涵和意味,从而顺利地进入到诗歌创作的正确门径呢?因此,如果让青年学子自己去摸索,就会觉得很艰深吃力;假如进入大雅堂的人,能够按照我这个观点去探寻它,就会事半功倍,省力多了。那些喜欢穿凿附会的人,抛弃了杜诗的主旨,一味地在比兴和技巧上夸夸其谈,认为杜诗中所涉及到的林泉、人物、草木、鱼虫,每一样都有所寄托,有所指代,就像当世很流行的猜诗谜的一样,那么,杜诗就毫无价值可言了。素翁可把这篇记一并刻在大雅堂中。后生学子是可以敬畏的,怎么能料定后之登大雅堂者对这碑上的诗文没有豁然贯通的人呢!元符三年九月黄庭坚(涪翁)书。七、阅读下面的文言文,完成下面小题。王生好学而不得法。其友李生问之曰:“或谓君不善学,信乎?”王生不说,曰:“凡师之所言,吾悉能识之,是不亦善学乎?”李生说之曰:“孔子云‘学而不思则罔’,盖学贵善思,君但识之而不思之,终必无所成,何谓之善学也?”王生益愠,不应而还①走。居五日,李生故寻王生,告之曰:“夫善学者不耻下问,择善而从之,冀闻道也。余一言未尽而君变色以去,几欲拒人千里之外,其善学者所应有邪?学者之大忌,莫逾自厌,盍改之乎?不然,迨年事蹉跎,虽欲改励,恐不及矣!”王生惊觉,谢曰:“余不敏,今日始知君言之善。请铭之坐右,以昭炯戒②。”(节选自《颜氏家训·勉学》)【注释】:①还(xuán):同“旋”,转身。②炯戒:明显的警诫。20.用“/”给下面句子断句。(画一处)余一言未尽而君变色以去【答案】余一言未尽/而君变色以去【解析】本题考查文言文断句。句意为:我的话还没说完,你就变了脸色离开。句子中的虚词“而”起到承接作用,根据句意和虚词,故断句为:余一言未尽/而君变色以去。21.请将文中画线的句子翻译成现代汉语。凡师之所言,吾悉能识之,是不亦善学乎?【答案】凡是老师所讲的,我全都能记下来,这难道不也是善于学习吗?【解析】本题考查文言文句子翻译。重点字词:所言:所说的话;悉:全,都;志:记;是:这。22.结合选文,谈谈你获得了哪些启示。【答案】①勤于思考,学思结合。②不耻下问,取长补短。③不自我满足,虚心接受别人的意见。【解析】本题考查道理启示。从文章中“盖学贵善思,君但识之而不思之,终必无所成”可知,学习应当善于思考,学思结合;根据“学者不耻下问,择善而从之,冀闻道也”可知,学习应当不耻下问,取长补短;根据“学者之大忌,莫逾自厌”可知,学习应当不自我满足,虚心接受别人的意见。参考译文:王生爱好学习而不得法。他的朋友李生问他说:“有人说你不善于学习,是真的吗?”王生不高兴,说:“凡是老师所讲的,我都能记住它,这不也是善于学习吗?”李生劝他说:“孔子说过‘学习,但是不思考,就会感到迷惑’,学习贵在善于思考,你只是记住老师讲的知识,但不去思考,最终一定不会有什么成就,怎么能说你善于学习呢?”王生更加生气,不理睬李生,转身就跑开了。过了五天,李生特地找到王生,告诉他说:“那些善于学习的人不把向地位比自己低的人请教当成耻辱,选择别人的优点去学习,希望听到真理啊!我的话还没说完,你就变了脸色离开,几乎要拒绝人千里之外,哪里是善于学习的人所应该具有的态度呢?学习最忌讳的事,莫过于满足自己所学的知识,你为什么不改正呢?如果你现在不改正,等你年纪大了,贻误了岁月,即使想改过自勉,恐怕也来不及了!”王生听完他的话,感到震惊,醒悟过来,道歉说:“我真不聪明,今天才知道你说得对。请允许我把你的话当作座右铭,用来警戒自己。”八、阅读下面的文言文,完成下面小题。【甲】会天大雨,道不通,度已失期。失期,法皆斩。陈胜、吴广乃谋曰:“今亡亦死,举大计亦死;等死,死国可乎?”陈胜曰:“天下苦秦久矣。吾闻二世少子也,不当立,当立者乃公子扶苏。扶苏以数谏故,上使外将兵。今或闻无罪,二世杀之。百姓多闻其贤,未知其死也。项燕为楚将,数有功,爱士卒,楚人怜之。或以为死,或以为亡。今诚以吾众诈自称公子扶苏、项燕,为天下唱,宜多应者。”吴广以为然。召令徒属曰:“公等遇雨,皆已失期,失期当斩。藉第令毋斩,而成死者固十六七。且壮士不死即已,死即举大名耳,王侯将相宁有种乎!”徒属皆曰:“敬受命。”乃诈称公子扶苏、项燕,从民欲也。袒右,称大楚。(节选自《陈涉世家》)【乙】攻一时,敌退,三保①亦自喜。婉贞独戚然,曰:“小敌去,大敌来矣。设以炮至,吾村不齑粉②乎?”三保瞿然③曰:“何以为计?”婉贞曰:“西人长火器而短技击。火器利袭远,技击利巷战。吾村十里皆平原,而与之竞火器,其何能胜?莫如以吾所长,攻敌所短,操刀挟盾,猱进鸷击④,侥天之幸,或能免乎?”三保曰:“悉吾村之众,精技击者不过百人。以区区百人,投身大敌,与之扑斗,何异以孤羊投群狼?小女子毋多谈。”婉贞微叹曰:“吾村亡无日矣!吾必尽吾力以拯吾村。”于是集谢庄少年之精技击者而诏之曰:“与其坐而待亡,孰若起而拯之?诸君无意则已,诸君而有意,瞻予马首⑤可也。”众皆感奋。(节选自《冯婉贞》,有删减)【注释】①三保:冯三保,冯婉贞的父亲,以勇而多艺被推为长,率乡民抵御入侵英法联军。②齑(jī)粉:细粉,这里用作动词。③瞿然:吃惊的样子。④猱(ná):猿猴的一种,敏捷善攀。鸷(zhì):与鹰、雕同类的一种凶猛的鸟。⑤瞻予马首:看我的马头,意即听我指挥。瞻,看。23.解释下列加点词。(1)扶苏以数谏故( )(2)楚人怜之( )(3)设以炮至( )(4)于是集谢庄少年之精技击者而诏之曰( )【答案】多次,屡次 爱戴 到 精通【解析】本题考查文言实词的理解。作答本题,重点在于文言实词的积累,同时也可以借助整个句子的意思来判断。①句意:扶苏因为屡次劝谏的缘故。数:多次。②句意:楚国人都很爱戴他。怜:爱戴。③句意:如果他们拿大炮来攻打。至:到。④句意:于是她把谢庄精通武术的少年召集起来。精:精通。24.下列加点词的意义和用法相同的一项是( )A.何以为计 俭以养德(《诫子书》)B.西人长火器而短技击 人不知而不愠(《〈论语〉十二章》)C.而与之竞火器 燕雀安知鸿鹄之志D.当立者乃公子扶苏 乃悟前狼假寐(《狼》)【答案】B【解析】本题考查一词多义。A.介词,用/连词,来;B.表转折,却/表转折,却;C.代词,西洋人/助词,的;D.是/才。故选B。25.翻译下列句子。(1)且壮士不死即已,死即举大名耳,王侯将相宁有种乎!(2)悉吾村之众,精技击者不过百人。【答案】(1)况且壮士不死便罢了,要死就该成就伟大的名声啊,王侯将相难道有天生的贵种么?(2)把我们全村的群众都算在一起,精通武术的也不过百来个人。【解析】本题主要考查点是对句子翻译。注意关键词语的翻译及句式的理解。①重点词:且,何况;宁,难道。②重点词:悉,全,都。精,精通。26.填空。两文刻画了陈胜和冯婉贞都有_____________________的特点。为了更好地突出人物的美好品质,【甲】【乙】两文都主要运用了_________________的描写方法来塑造人物,另外,【乙】文还运用了的________________描写方法。【答案】抗争精神,不甘向命运屈服,有卓越的组织领导才能(选择其中一点即可) 语言描写 神态描写【解析】本题考查对人物形象和描写方法的判断分析能力。从甲文中看出,陈胜具有抗争精神,能够把握自己的命运,组织领导才能超群。陈胜在適戍渔阳误了期限之后意识到“今亡亦死,举大计亦死”,然后奋起反抗;从乙文中看出,冯婉贞颇有远见。敌人败逃后,冯三保非常高兴,冯婉贞却头脑清醒,认为敌人不会甘于失败,还会卷土重来;后来的事实证明了她的判断准确。冯婉贞敢于担当,有领导才能。眼见父亲对自己的合理建议置之不理,冯婉贞不愿坐以待毙,动员谢庄精通武术的少年听从自己指挥,一起奋勇抗敌。从[甲]文中“陈胜、吴广乃谋曰……召令徒属曰……”可知主要运用了语言描写的方法来塑造人物;从[乙]文冯婉贞与父亲冯三保的对话中可以看出主要运用了语言描写的方法来塑造人物。从乙文“婉贞独戚然”“婉贞微叹”可以看出,【乙】文还运用了神态描写的描写方法。参考译文:【甲】恰巧遇到天下大雨,道路不通,估计已经误期。误了期限,按(秦朝的)法律都应当斩首。陈胜吴广于是商量说:“即使现在逃跑(被抓回来)也是死,发动起义也是死,同样是死,为国事而死,可以么?”陈胜说:“天下百姓受秦朝统治逼迫已经很久了。我听说秦二世是始皇帝的小儿子,不应立为皇帝,应立的是公子扶苏。扶苏因为屡次劝谏的缘故,皇上派(他)在外面带兵。现在有人听说他没什么罪,秦二世却杀了他。老百姓大都听说他很贤明,而不知道他死了。项燕是楚国的将领,曾多次立下战功,又爱护士兵,楚国人都很爱戴他。有人认为他死了,有人认为他逃跑了。现在如果把我们的人假称是公子扶苏和项燕的队伍,号召天下百姓反秦,应当会有很多响应的人。”吴广认为他讲得对。 (于是陈胜)召集并号令部属的人说:“你们诸位遇上大雨,都已误了期限,误期是要杀头的。假使仅能免于斩刑,可是去守卫边塞死掉的本来也会有十分之六七。况且壮士不死便罢了,要死就该成就伟大的名声啊,王侯将相难道有天生的贵种么?”部属的人都说:“愿意听从您的号令。”于是就假称是公子扶苏项燕的队伍,顺从人民的愿望。个个露出右臂(作为起义的标志),号称大楚。【乙】攻打了一阵,敌人退却了,冯三保非常高兴。唯独婉贞忧愁地说:“小股敌人走了,大股敌人要来的。如果他们拿大炮来攻打,我们全村不就化为粉末了吗?”冯三保吃惊地问道:“那怎么办呢?”冯婉贞说:“西洋人的长处是使用枪炮等火器,短处是不会武术。枪炮对远距离攻击有利,而武术对近身作战有利。我们村方圆十里都是平原,跟敌人较量枪炮,那怎么能取胜呢?不如用我们的长处,去攻击敌人的短处,持着刀,拿着盾,像猿猴那样敏捷地进攻,像鸷鸟那样勇猛地搏击,或许能避免这场灾祸吧?”冯三保说:“把我们全村人都算上,精通武术的不过一百来人,让这样少的人投身到强大的敌群中搏斗,这跟把一只羊孤身投到狼群里有什么不同呢?小女孩子不要多嘴。”冯婉贞微微地叹息说:“我们村庄眼看就要完了!我一定要尽全力来拯救我们的村庄。”于是她把谢庄精通武术的少年召集起来,激励他们说:“与其坐着等死,怎比得上奋起抗敌拯救我们的谢庄呢?各位如果没有这种意思也就算了,如果有这种意思的话,就听我的指挥好了。”大家都被激动得振奋起来。九、阅读下面的文言文,完成下面小题。[甲]十年春,齐师伐我。公将战,曹刿请见。其乡人曰:“肉食者谋之,又何间焉?”刿曰:肉食者鄙,未能远谋。”乃入见。问:“何以战?”公曰:“衣食所安,弗敢专也,必以分人。”对曰:“小惠未遍,民弗从也。”公曰:“牺牲玉帛,弗敢加也,必以信。”对曰:小信未孚,神弗福也。”公曰:“小大之狱,虽不能察,必以情。”对曰:忠之属也。可以一战。战则请从。(选自《曹刿论战》)[乙]士者,国之重器;得士则重,失士则轻①。今欲致天下之士,民有上书求见者,辄使诣尚书问其所言,言可采取者,秩以升斗之禄,赐以一束之帛。若此,则天下之士吐忠言,嘉谋日闻于上,天下条贯,国家表里,烂然可睹矣。夫以四海之广士民之数能言之类至众多也。然其俊杰指世陈政,言成文章,质之先圣而不缪,施之当世合时务,若此者,亦亡几人。(选自《汉书》,有删改)[注释]①重、轻:指国家的强盛和弱小。27.选出下列各句中加点词意思和用法相同的一项( )A.曹刿请见 民有上书求见者B.其乡人曰 辄使诣尚书问其所言C.小大之狱 今欲致天下之士D.忠之属也 则天下之士吐忠言【答案】C【解析】本题考查一词多义。A.接见/拜见;B.代词,他的/他们的;C.均为结构助词,的;D.尽力做好分内的事/中直;故选C。28.将选文中画线句子翻译成现代汉语。(1)肉食者鄙,未能远谋。(2)质之先圣而不缪,施之当世合时务。【答案】①当权者目光短浅,不能深谋远虑。②纵然和先代圣贤对证也没有差错,施行于当今社会合乎时务。【解析】本题考查文言句子翻译。重点词有:(1)肉食者:吃肉的人,指当权者;鄙:鄙陋,目光短浅;谋:谋划。(2)质:对质、对证;缪:差错;施:施行;合时务:合乎时务。29.请用“/”给下面的句子断句,断两处。夫以四海之广士民之数能言之类至众多也。【答案】夫以四海之广/士民之数/能言之类至众多也。【解析】本题考查文言句子划分。句意为:凭借四海之广,士人,百姓之众,能够进献忠言的人必定极多。故断句为:夫以四海之广/士民之数/能言之类至众多也。30.[甲]文中的曹刿能否称得上[乙]文中的“俊杰”?请结合语段内容分析。【答案】能。[乙]文中的“俊杰”指陈政事,出言成章,可与先代圣贤比肩,他们的言论或策略施行于当下还能合乎时务,[甲]文中的曹刿在国家遭受外敌入侵的时候,能够根据实际情况,大胆进谏,出口成章,鲁国因此打败齐国,他对国家的贡献可以和先贤比肩。【解析】本题考查内容理解概括。结合【乙】文“然其俊杰指世陈政,言成文章,质之先圣而不缪,施之当世合时务,若此者,亦亡几人”可知,【乙】文中的“俊杰”指出言成章,可与先代圣贤比肩的人,他们的言论或策略施行于当下还能合乎时务。结合【甲】文“十年春,齐师伐我。公将战,曹刿请见”等内容可知,曹刿在自己国家遭受外敌入侵时,能够根据实际情况,直言敢谏,条理清晰思路缜密地分析当前形势,鲁国因此打败齐国,他对国家的贡献不容小觑,因此曹刿称得上是“俊杰”。31.[甲]文开篇运用对比手法,体现了曹刿卫国的_____。[乙]文开篇运用对比手法,突出了贤士对于国家的_____。【答案】政治热忱(或责任感) 重要性【解析】本题考查写作手法及内容理解。结合【甲】文“十年春,齐师伐我。公将战,曹刿请见。其乡人曰:‘肉食者谋之,又何间焉?’刿曰:‘肉食者鄙,未能远谋。’乃入见”可知,将“肉食者”与自己进行对比,突出曹刿保卫国家的高度责任感;结合【乙】文“士者,国之重器;得士则重,失士则轻”可知,开篇运用对比的写法,以得到贤士和失去贤士对国家的影响进行对比,突出贤士对于国家的重要性。参考译文:【甲】鲁庄公十年的春天,齐国军队攻打鲁国。鲁庄公将要迎战。曹刿请求鲁庄公接见自己。他的同乡说:“打仗的事当权者自会谋划,你又何必参与呢?”曹刿说:“当权者目光短浅,不能深谋远虑。”于是入朝去见鲁庄公。曹刿问:“您凭借什么作战?”鲁庄公说:“衣食这一类安身的东西,不敢独自享有,一定把它分给别人。”曹刿回答说:“这些小恩惠不能遍及百姓,百姓是不会听从您的。”鲁庄公说:“祭祀神灵的牛、羊、玉帛之类的用品,我(从来)不敢虚报数目,一定按照承诺的去做。”曹刿说:“这只是小信用,未能让神灵信服,神是不会保佑你的。”鲁庄公说:“大大小小的案件,虽然不能件件都了解得清楚,但一定要处理得合情合理。”曹刿回答说:“这才是尽了本职一类的事,可以凭借这个条件打一仗。如果作战,请允许我跟随您一同去。”【乙】贤士,是国家的重要人才;得到贤士国家就强盛,失去贤士国家就弱小。如今想要招引天下的贤士,百姓有上书求见的就让他们拜访尚书,由尚书询问他们所说的内容,言论有可以采用的,就授以微薄的俸禄,赏赐一束丝织品。如果这样做,那么天下的士人倾吐忠言,皇上每天都能听到好的谋议,(这样一来)天下的条理,国家的表里,就灿烂分明,可以看得一清二楚了。凭借四海之广,士人,百姓之众,能够进献忠言的人必定极多。然而,其中的杰出之士指陈时事政务,出言成章,和先代圣贤对证也没有谬误,施行于当今社会也是合乎时务的,像这样的人,也没有几个。十、阅读下面的文言文,完成下面小题。【甲】自三峡七百里中,两岸连山,略无阙处。重岩叠嶂,隐天蔽日,自非亭午夜分,不见曦月。至于夏水襄陵,沿溯阻绝。或王命急宣,有时朝发白帝,暮到江陵,其间千二百里,虽乘奔御风,不以疾也。春冬之时,则素湍绿潭,回清倒影,绝巘多生怪柏,悬泉瀑布,飞漱其间,清荣峻茂,良多趣味。每至晴初霜旦,林寒涧肃,常有高猿长啸,属引凄异,空谷传响,哀转久绝。故渔者歌曰:“巴东三峡巫峡长,猿鸣三声泪沾裳。”(郦道元《三峡》)【乙】最奇者为石梁。长不计丈,狭仅盈尺。潜蛇窥而甲惊①,飞鸟过而魄堕。青苔十层,去履不啮;飞瀑万仞,来目未眩。遂休神于蓝桥②,啸咏于碧涧。至鱼鳖啖③其影,而步不移;猿揉摄其神,而坐不返。盖浑浑乎身世两忘焉。(节选自洪亮吉《游天台山记》。有删改)【注】①甲惊:蛇的鳞片都害怕得竖起来。②蓝桥:桥名,传说是唐装航遏仙女处。此处以蓝桥比喻石梁。③啖(dàn):吃,此处引申为“戏弄”。32.下列对两个文段中加点词语的理解和分析,不正确的一项是( )A.“夏水襄陵”中的“襄”是“冲上、漫上”的意思,写出了夏水暴涨,水势浩大的样子。B.“素湍绿潭”中的“素湍”指激起白色浪花的急流,表现了江水的明净轻快。C.“属引凄异”中的“属引”是“接连不断”的意思,强调猿猴很多,一个接一个,叫声凄惨悲凉。D.“去履不啮”中的“不啮”在这里指“不硌脚”,突出了石梁上青苔之厚。【答案】C【解析】本题考查对词语的理解和分析。C.“属引凄异”中的“属引”是“接连不断”的意思,强调声音持续不断,非常凄惨悲凉。“猿猴很多,一个接一个”错误。故选C。33.把【甲】文段中画横线的句子翻译成现代汉语。清荣峻茂,良多趣味。【答案】水清树荣,山高草盛,(实在)有很多趣味。【解析】本题考查的是理解并翻译句子的能力。解答时一定要先回到语境中,根据语境读懂句子的整体意思,然后思考命题者可能确定的赋分点,并按现代汉语的规范,将翻译过来的内容进行适当调整,达到词达句顺。此句中得分点有:清荣峻茂,水清树荣,山高草盛。良,甚,很。34.【甲】文段中“虽乘奔御风,不以疾也”一句有何表达效果?【答案】运用夸张和对比的修辞手法,侧面写出了夏季江水流动迅疾、一泻千里的特点,既使人感到惊心动魄,又使人感到豪情万丈。【解析】本题考查赏析句子。“有时朝发白帝,暮到江陵,其间千二百里,虽乘奔御风,不以疾也”意为:这时只要早晨从白帝城出发,傍晚就到了江陵,这中间有一千二百里,即使骑上飞奔的马,驾着疾风,也不如它快。作者运用夸张、对比的修辞手法和侧面描写的方法写出三峡夏季水流湍急的特点,使人感到豪情万丈,写出了三峡的奔放美。35.【甲】【乙】两个文段都有对山的描写,请从描写方法的鱼度简要分析下面句子是如何表现山的特点的。【甲】自三峡七百里中,两岸连山,略无阙处。【乙】潜蛇窥而甲惊,飞鸟过而魄堕。【答案】[甲]文段中的句子通过正面描写两岸群山连绵数百里,没有缺口,表现了山势连绵不断的特点。[乙]文段中的句子通过写“潜蛇”因害怕鳞片都会竖起来,“飞鸟”飞过也因害怕而魄落胆破,从侧面表现了石梁之险峻。【解析】本题考查理解文章内容。【甲】“自三峡七百里中,两岸连山,略无阙处”句意:在三峡七百里之间,两岸都是连绵的高山,完全没有中断的地方。作者用“自三峡七百里中”起笔,既交代了描写对象,又介绍了其总体长度。接着,作者写山,用“两岸连山,略无阙处”写山之“连”,表现了山势连绵不断的特点。【乙】“潜蛇窥而甲惊,飞鸟过而魄堕”句意:隐伏的大蛇偷看一下害怕得鳞片都竖起来,鸟儿飞过也会惊得魄落胆破。结合“长不计丈,狭仅盈尺”可知,通过写“潜蛇"因害怕鳞片都会竖起来,“飞鸟”飞过也因害怕而魄落胆破,从侧面表现了石梁之险峻。参考译文:【甲】在三峡七百里之间,两岸都是连绵的高山,完全没有中断的地方;重重叠叠的悬崖 峭壁,遮挡了天空和太阳。若不是在正午半夜的时候,连太阳和月亮都看不见。等到夏天水涨,江水漫上小山丘的时候,下行或上行的船只都被阻挡了,有时候皇帝的命令要紧急传达,这时只要早晨从白帝城出发,傍晚就到了江陵,这中间有一千二百里,即使骑上飞奔的马,驾着疾风,也不如它快。等到春天和冬天的时候,就可以看见白色的急流,回旋的清波。碧绿的潭水倒映着各种景物的影子。极高的山峰上生长着许多奇形怪状的柏树,山峰之间有悬泉瀑布,在之上飞流冲荡。水清,树荣,山高,草盛,确实趣味无穷。在秋天,每到初晴的时候或下霜的早晨,树林和山涧显出一片清凉和寂静,经常有高处的猿猴拉长声音鸣叫,声音持续不断,非常凄惨悲凉,空荡的山谷里传来猿叫的回声,悲哀婉转,很久才消失。所以三峡中渔民的歌谣唱道:“巴东三峡巫峡长,猿鸣三声泪沾裳。”【乙】(天台山)最奇异的当属石梁了。(它的)长度无法用丈来计算,窄才满一尺。隐伏的大蛇偷看一下害怕得鳞片都竖起来,鸟儿飞过也会惊得魄落胆破。梁上长满了厚厚的青苔,脱鞋赤脚行走也不觉得疼痛;万仞之高的瀑布由天上飞落,看在眼里也未曾目眩。(我)就在这蓝桥上息神养性,在碧绿的涧水旁吟啸歌咏。(在这里安坐凝神)以至观看鱼鳖在水中戏弄人的倒影,却不再移动步子;山猿吸引着人的心神,却依然静坐而不返回。大概是人已和大自然融为一体,忘了自身和客观世界的存在。十一、阅读下面的文言文,完成下面小题。【甲】至于负者歌于途,行者休于树,前者呼,后者应,伛偻提携,往来而不绝者,滁人游也。临溪而渔,溪深而鱼肥;酿泉为酒,泉香而酒洲,山有野获,杂然而前陈者,太守宴也。宴酣之乐,非丝非竹,射者中,弈者胜,觥筹交错,起坐而喧哗者,众宾欢也。苍颜白发,颓然乎其间者,太守醉也。已而夕阳在山,人影散乱,太守归而宾客从也。树林阴翳,鸣声上下,游人去而禽鸟乐也。然而禽鸟知山林之乐,而不知人之乐;人知从太守游而乐,而不知太守之乐其乐也。醉能同其乐,醒能述以文者,太守也。太守谓谁?庐陵欧阳修也。(节选自《醉翁亭记》)【乙】(轼)寻降一官,未至,眨宁远军节度副使,惠州安置。居三年,泊然无所蒂芥①,人无贤愚,皆得其欢心。又贬琼州别驾,居昌化。昌化,故儋耳地,非人所居,药饵②皆无有。初僦③官屋以居,有司犹谓不可,轼逐买地筑室,儋人运甓眷土④以助之。独与幼子过处,著书以为乐,时时从其父老游,若将终身。(节选自《宋史·苏轼传》)【注释】①蒂芥:内心不满,或不快。②药饵:药品。③僦:租赁。④运甓眷土:搬砖运土。36.下列对两个文段中加点词语的理解和分析,不正确的一项是( )A.“往来而不绝者,滁人游也”中的“绝”是断绝的意思,“不绝”表现游山的滁人很多。B.“坐起而喧哗者,众宾欢也”中的“喧哗”是声音大而嘈杂的意思,再现了太守宴上欢快热闹的情景:C.“树林阴翳,鸣声上下”中的“上下”是方位词,意思是鸟儿一会飞上,一会儿飞下,很快乐。D.“寻降一官,未至,贬宁远军节度副使”中的“寻”是不久的意思,这句话写出苏轼官职一贬再贬,处境不断变差。【答案】C【解析】本题考查文章内容理解与概括。C.“树林阴翳,鸣声上下”的意思是树林遮盖成荫,上上下下一片鸟鸣的声音。“上下”是指高处和低处的树林,是名词,而不是方位词,C选项说法不正确。故选C。37.把甲文段中画横线的句子翻译成现代汉语。醉能同其乐,醒能述以文者,太守也。【答案】醉了能够和大家一起欢乐,醒来能够用文章记叙这乐事的人是太守。【解析】本题考查学生对句子翻译能力。我们在翻译句子时要注意通假字、词类活用、一词多义、特殊句式等情况,如遇倒装句就要按现代语序疏通,如遇省略句翻译时就要把省略的成分补充完整。重点词有:同:和;述:记述;以:用。38.联系【链接材料】,结合甲文段分析欧阳修写作《醉翁亭记》的目的。【链接材料】《醉翁亭记》作于宋仁宗庆历五年(1045年),由于支持范仲淹等人推行的北宋革新运动庆历新政,欧阳修被贬放滁州。他在滁州实行宽简政治,使百姓过上和平安定的生活。【答案】①表现作者被贬后,身处逆境仍能醉情山水、怡然自得的乐观精神。②再现宽简政治下的滁州百姓和乐的情景,表现为政一方、造福一方的自得。③与民同乐的政治襟怀【解析】本题考查文章内容的理解和分析。结合链接材料“欧阳修被贬放滁州”和甲文“临溪而渔,溪深而鱼肥;酿泉为酒,泉香而酒洲,山有野获,杂然而前陈者,太守宴也”和“树林阴翳,鸣声上下”可知:①表现作者被贬后,身处逆境仍能醉情山水、怡然自得的乐观精神;结合链接材料“欧阳修被贬放滁州。他在滁州实行宽简政治,使百姓过上和平安定的生活”和“宴酣之乐,非丝非竹,射者中,弈者胜,觥筹交错,起坐而喧哗者,众宾欢也。苍颜白发,颓然乎其间者,太守醉也”可知:②再现宽简政治下的滁州百姓和乐的情景,表现为政一方、造福一方的自得;结合甲文“人知从太守游而乐,而不知太守之乐其乐也。醉能同其乐,醒能述以文者,太守也”可知:③与民同乐的政治襟怀。39.欧阳修遭贬谪后,曾表示不愿做“穷愁怨嗟”的“庸人”。苏轼是“庸人”吗?请结合乙文段中他的所作所为简要分析。【答案】他不是。①他被贬惠州三年,内心没有不满。②他和百姓相处融洽,百姓都喜欢他,帮他建屋。③他常写书为乐,与百姓一同出游,与民同乐。④他随遇而安,打算在昌化终老一生。【解析】本题考查文章内容的理解和分析。题干中“穷愁怨嗟”的意思是穷困忧愁怨恨叹息。“庸人”的意思是庸俗的人。首先回答苏轼不是这样的人。结合乙文“居三年,泊然无所蒂芥”可知:①他被贬惠州三年,内心没有不满;结合乙文“人无贤愚,皆得其欢心”和“轼逐买地筑室,儋人运甓眷土以助之”可知:②他和百姓相处融洽,百姓都喜欢他,帮他建屋。结合乙文“著书以为乐,时时从其父老游”可知:③他常写书为乐,与百姓一同出游,与民同乐;结合乙文“若将终身”可知:④他随遇而安,打算在昌化终老一生。参考译文:【甲】至于背着东西的人在路上欢唱,走路的人在树下休息,前面的人呼喊,后面的人应答,老人弯着腰走,小孩子由大人领着走,来来往往不断的行人,是滁州的游客。到溪边钓鱼,溪水深并且鱼肉肥美;用酿泉造酒,泉水清并且酒也清;野味野菜,酒杯和酒筹交互错杂;那是太守主办的宴席。宴会喝酒的乐趣,不在于弹琴奏乐,投壶的人中了,下棋的赢了,酒杯和酒筹交互错杂,时起时坐大声喧闹的人,是欢乐的宾客们。一个脸色苍老的老人,醉醺醺地坐在众人中间,是太守喝醉了不久,太阳下山了,人影散乱,宾客们跟随太守回去了。树林里的枝叶茂密成林,鸟儿到处叫,是游人离开后鸟儿在欢乐地跳跃。但是鸟儿只知道山林中的快乐,却不知道人们的快乐。而人们只知道跟随太守游玩的快乐,却不知道太守以游人的快乐为快乐啊。醉了能够和大家一起欢乐,醒来能够用文章记叙这乐事的人,是太守。太守是谁呢?是庐陵欧阳修吧。【乙】苏轼不久降一官级,还未到任,又贬为宁远军节度副使,安置在惠州。住了三年,他淡然处之毫不计较,对人不论贤才或是平常人,都能得到他们的欢心。又被贬为琼州别驾,住在昌化。昌化,是以前的儋耳,不是人所能居住的,药品都没有。苏轼起初租官房居住,有关官员还认为不可以,苏轼就买地筑屋,儋耳人就搬砖运土来帮助他。他独自和小儿子苏过一起住,把写书来当作乐趣,时常跟当地父老交往,好像要在这里终老。七、小说阅读 一、阅读下文,完成下面小题。父子之间①儿子出国留学前,提出到国内几个城市转转。二十多年间,父子各忙各的,难得有独处的时光。此次二人游,对于我俨然是一次“素质提升夏令营”,儿子则俨然是“夏令营营长”。②儿子出发之前就有点咳嗽,一路上,我提醒他按时按量服药。那边,他正忙着“消灭”麻辣串串,我嘀咕几声,便遭来一顿怒怼:知道了,这句话,已经说三遍了。末了,再给我补上一刀:能不能不像老妈一样絮絮叨叨?你们这是衰老的标志,懂吗?③路上我带着一件书画卷轴礼品,外包装足有一未长。抓在手上怕丢了,干脆将它塞在背包里,但高出了一大截。地铁里挤上挤下,儿子看不下去,一本正经地发出警告:能不能不给别人添麻烦?礼盒背在身后,很容易碰伤人家,人家都要让着你。竖着,放在自己胸前,不好吗?还有,你等车时总喜欢站在盲道上。④我像是被抓现行的“熊孩子”,苦笑着转换活题:“你能替我金着?”他接过礼盒,又鄙夷地摆下一句:你就喜欢给别人添麻烦。⑤每到一处,儿子习惯打开网络地图寻找目的地。我一般都依赖他,问东问西。问得烦了,儿子把手机仲到我面前,看到蜘蛛网一般的线条,我便有些晕头转向。在重庆地铁站,几次出站跑错方向,儿子急了:“反应太慢了,标牌上的字那么醒目;到了国外,都是外文,怎么办?”想到他妈慢我三拍,我笑了:“下次等看你们母子合作。”⑥下榻在一家连锁酒店,顶层是火锅店,离开酒店前,我们决定到火锅店一探究竟。午间,大厅却空无一人。我有些不解,随口问服务员:“住客这么多,广告天天放,怎么还是没人?”回到客房,儿子突然教训起我来:“你怎么这么说话?”我如实交代:我是感兴趣在营销广告和实际销售之间到底有着怎样的关系。儿子大声诘问:你新闻敏感,但你考虑人家的感受了?太自以为是了,大厅没人,人家不知道?……⑦和儿子一路同行,被他一路吐槽说教。作为长辈,感觉很没面子;静下心来,忽想又不无道理;仔细琢磨,似乎有点意思。⑧在职场上,你的那些小毛小病、认如问题乃至致命弱点,主管不会说,同事不愿说,下属不敢说。所以你常常会带着某些不合时宜一路狂奔,直到触犯别人底线,才有人给你一一“盘点”“算账”。⑨不过,如果家有成长中的孩子,他们不会顾忌辈分、身份、面子,会以他理解的做人做事,标准严格要求你,也会以“别人家的爸妈”提示你对标找差。当然,子女的训导,毕竟只是扯扯袖,红红脸,出出汗,并不具有“带电”的约束力,属于“家族式舆论监督”。⑩遥想当年,我给父亲也出过这样的难题,我常常懊悔自己没对父亲多些耐心,多些尊敬,哪怕是语气温和些也好。一幕一幕,恍如昨日。⑪现在,在与儿子的“较量”中,我总算有了换位的体验:痛,并快乐着。⑫毕竟,父子之间“嘴仗”的背后,无所谓“仇”与“恨”,只是世界上两个最亲密男人的较量,是双方爱的碰撞。我想,当年的父亲应该能理解。而我也早已识破我家那小子的真面目,他的咄咄逼人里,几分是得理不饶人,几分是恃宠而骄,还有几分是亲密的嗔怪。(选自《朝花时文》,有删改)1.第⑩段中加点词语“难题”在文中的意思是________________________________________2.文章第②-⑥段记叙了“我”和儿子之间的四件事,按照要求完成填空。3.对文章第①段的理解分析正确的一项是( )A.字里行间都能感受到“我”对此次旅行的畏惧。B.两个“俨然”增添了一种诙谐幽默的表达效果。C.“夏令营营长”一词表明“我”对儿子的不满。D.“素质提升训练营”突出此次旅行运动强度大。4.分析第④段画线句中加点词“熊孩子”的语言表现力。5.联系全文,结合加点词,谈谈对第⑦段画线句的理解。【答案】1.“我”也曾经吐槽说教父亲。 2.(1)“我”提醒儿子按时按量喝药;(2)“我”不会看地图老跑错方向;(3)反应迟钝;(4)自以为是 3.B 4.“熊孩子”是网络用语,多形容未受到良好的教育,调皮的孩子。这个词既表现了“我”认识到了自己的错误,也表现了“我”当时尴尬难堪的处境。 5.“感觉没面子”是指作为父亲却被儿子说教很难堪;“不无道理”表明我认同孩子所指出的错误;“有点意思”是指“我”发现两代父子相处中都有相似的“嘴仗”经历;同时体会到“较量”背后其实是父子之间爱的表达。整句话表现了作者强烈的反思意识。【解析】1.本题考查理解词语含义得的能力。依据⑩段“遥想当年,我给父亲也出过这样的难题,我常常懊悔自己没对父亲多些耐心,多些尊敬,哪怕是语气温和些也好。一幕一幕,恍如昨日”可知,“我”也曾经吐槽说教父亲。2.本题考查分析概括能力。(1)空,依据②段“儿子出发之前就有点咳嗽,一路上,我提醒他按时按量服药”可以概括为:“我”提醒儿子按时按量喝药。(2)空,依据⑤段“问得烦了,儿子把手机仲到我面前,看到蜘蛛网一般的线条,我便有些晕头转向。在重庆地铁站,几次出站跑错方向”可以概括为:“我”不会看地图老跑错方向。(3)空,依据“儿子急了:‘反应太慢了,标牌上的字那么醒目;到了国外,都是外文,怎么办?’想到他妈慢我三拍,我笑了:‘下次等看你们母子合作。’”可以概括为:“我反应迟钝。(4)空,依据⑥段“儿子大声诘问:你新闻敏感,但你考虑人家的感受了?太自以为是了,大厅没人,人家不知道?……”可以概括为:“我自以为是。3.本题考查理解辨析能力。A.依据①段“儿子出国留学前,提出到国内几个城市转转。二十多年间,父子各忙各的,难得有独处的时光”可知,字里行间都能感受到“我”对此次旅行的期待,期待同儿子独处;C.依据①段“儿子则俨然是‘夏令营营长’”中的“俨然可知,本句表现了我对儿子的怜爱之情;D.依据①段“此次二人游,对于我俨然是一次‘素质提升夏令营’”中的“此次二人游”可知,“素质提升训练营”体现了作者对此次旅游的期待;故选B。4.本题考查理解词语含义的能力。依据“熊孩子”本义可知,原指未受到良好的教育或调皮的孩子。依据④段“我像是被抓现行的‘熊孩子’,苦笑着转换活题:‘你能替我金着?’”中的”苦笑“可知,本句中是表现了“我”当时被儿子说教时的尴尬难堪的处境;依据“你就喜欢给别人添麻烦”可知,在儿子的提醒下,看出我已经意识到了自己的错误。据此,总结概括即可。5.本句考查理解句子含义的能力。依据②—⑥段内容可知,其表面意思是指和儿子一路同行,被他一路吐槽说教。作为长辈,感觉很没面子;深层含义:依据⑦段“静下心来,忽想又不无道理”可知,“不无道理”表明我认同孩子所指出的错误。依据⑩段“遥想当年,我给父亲也出过这样的难题,我常常懊悔自己没对父亲多些耐心,多些尊敬,哪怕是语气温和些也好。一幕一幕,恍如昨日”可知,“有点意思”是指“我”发现两代父子相处中都有相似的“嘴仗”经历。依据⑪段“现在,在与儿子的“较量”中,我总算有了换位的体验:痛,并快乐着”;⑫段“毕竟,父子之间“嘴仗”的背后,无所谓“仇”与“恨”,只是世界上两个最亲密男人的较量,是双方爱的碰撞”可以理解为“较量”背后其实是父子之间爱的表达。据此,总结概括可。二、阅读下面文章,完成下面小题。猫婆冯骥才①我那小阁楼的后墙外,居高临下是一条又长又深的胡同,我称它为猫胡同。每日夜半,这里是猫儿们无法无天的世界。它们戏耍、求偶、追逐、打架,吵得人无法入睡。为了逃避这群讨厌的家伙,我真想换房子搬家。奇怪,哪来这么多猫,为什么偏偏都跑到这胡同里来聚会闹事?②一天,一位视猫如命的朋友送给我一只毛线球大小雪白的小猫,缩成个团儿,小耳朵紧紧贴在脑袋上,一双纯蓝色亮亮的圆眼睛柔和又胆怯地望着我。我情不自禁赶快把它捧在怀里带回了家。③小猫一入我家,便成了我全家人的情感中心。儿子给它起了顶漂亮、顶漂亮的名字,叫蓝眼睛。每当蓝眼睛闯祸,只要一瞅它那纯净光澈、惊慌失措的蓝眼睛,心中的火气顿时全消了,反而会把它拥在怀里……我也是视猫如命了。④可是一天夜里,蓝眼睛跑出家门后再没回来。家中每个人全空了,我们房前房后去找。黑猫、白猫、黄猫、花猫、大猫、小猫,各种模样的猫从我眼前跑过,惟独没有蓝眼睛……懊丧中,一个孩子告诉我,猫胡同顶里边一座楼的后门里,住着一个老婆子,养了一二十只猫,人称猫婆,蓝眼睛多半是叫她的猫勾去的。⑤当夜,我钻进猫胡同,在黑暗里寻到猫婆家的门,我冒冒失失地拍门,非要进去看个究竟不可。门打开,一个高高的老婆子出现——这就是猫婆了。⑥我说我找猫,她立刻请我进屋去,是间阴冷的地下室。一股浓重噎人的猫味马上扑鼻而来。屋顶很低,正中吊下一个很脏的小灯泡,把屋内照得昏黄。一个柜子,一座生铁炉子,一张大床,地上几只放猫食的破瓷碗,再没别的。⑦猫婆问我:“你丢那猫什么样儿?”我描述一遍,她立即叫道:“见过见过,常找我们玩儿,多疼人的宝贝!丢几天了?”我盯住她那略显浮肿、苍白无光的老脸看,只有焦急,却无半点装假的神气。我说:“五六天了。”她的脸顿时阴沉下来,停了片刻才说:“您甭找了,回不来了!”我很疑心这话为了骗我,目光搜寻可能藏匿蓝眼睛的地方。这时,猫婆的手忽向上一指,“这都是叫人打残、摔坏的猫!我把它们拾回来养活的。您瞧那只小黄猫,孩子们想烧死它,我一把抢出来的!您想想,您那宝贝丢了这么多天,哪还有好?现在乡下常来一伙人,下笼子逮猫吃,造孽呀!”她说得脸抖,手也抖,点烟时,烟卷抖落在地。烟囱上那小黄猫,瘦瘦的,尖脸,很灵,立刻跳下来,叼起烟,仰起嘴,递给她。猫婆笑脸开花,咧着嘴不住地说:“瞧,这小东西多懂事!”⑧我还有什么理由疑惑她?面对这天下受难猫儿们的救护神,告别出来时,不觉带着一点惭愧和狼狈的感觉。⑨蓝眼睛的丢失虽使我伤心很久,但从此不知不觉我竟开始关切所有猫儿的命运。⑩转过一年,到了猫儿们求偶时节,猫胡同却忽然安静下来。⑪我妻子无意间从邻居那里听到一个不幸的消息:猫婆死了。同时——在她死后——才知道关于她在世时的一点点经历。⑫据说,猫婆无亲无故,孑然一身,拾纸为生,以猫为伴,但她所养的猫没有一个良种好猫,都是拾来的弃猫、病猫和残猫。她有猫必留,谁也不知道她家到底有多少只猫。曾有人为她找个伴儿,但结婚不过两个月,老汉忍受不了这些猫闹、猫叫、猫味儿,就搬出去住了。人们劝她扔掉这些猫,接回老汉,她执意不肯,坚持与这些猫共享着无人能解的快乐。⑬前两个月,猫婆急病猝死,老汉搬回来,第一件事便是把这些猫统统轰走。被赶跑的猫儿依恋故人故土,每每回来,必遭老汉一顿死打。⑭一种伤感与担虑从我心里漫无边际地散开,散出去,随后留下的是一片沉重的空茫。⑮入冬后,我听到一个令人震栗的故事——⑯我家对面一座破楼修理瓦顶。白天里瓦工们换瓦时活没干完,留下个洞,一只猫为了御寒,钻了进去;第二天瓦工们盖上瓦走了,这只猫无法出来,急得在里边叫。住在这楼顶层的五六户人家都听到猫叫,还有在顶棚上跑来跑去的声音,但谁家也不肯将自家的顶棚捅坏,放它出来。这猫叫了三整天,开头声音很大,很惨,瘆人,但一天比一天声音微弱下来,直至消失!⑰听到这故事,我彻夜难眠。⑱更深夜半,天降大雪,猫胡同里一片死寂,这寂静化为一股寒气透进我的肌骨。忽然,后墙下传来一声猫叫,在大雪涂白了的胡同深处,猫婆故居那墙头上,孤零零趴着一只猫影,在凛冽中蜷缩一团,时不时哀叫一声,甚是凄婉。我心一动,是那尖脸小黄猫吗?忙叫声:“咪咪!”想下楼去把它抱上来,谁知一声唤,将它惊动,起身慌张跑掉。⑲猫胡同里便空无一物。只剩下一片夜的漆黑和雪的惨白,还有奇冷的风在这又长又深的空间里呼啸。(选自《灵魂的巢》,作家出版社2005年版,有删改)6.请简要说说第①自然段在全文中的作用。7.根据表格中的提示,仿照示例,结合文中相关内容,分析猫婆不同方面的形象特点。8.请探究文中划线句“听到这故事,我彻夜难眠”蕴含了作者哪些感情?9.对于猫婆收养流浪猫的做法,她周围的人有哪些看法?你是否赞同猫婆的做法,请简要说明理由。【答案】6.写我对“猫胡同”里猫儿们的讨厌;设置悬念,激发读者的阅读兴趣,引起下文;为下文写“我”见猫婆了解这些猫的来历做铺垫(伏笔);与下文“我”也视猫如命、关切所有猫儿的命运形成对比。 7.从猫婆家中简陋生活的生活设施,可见猫婆生活贫困。 从猫婆的无亲无故、孑然一身的经历,可以看出猫婆命运坎坷、孤苦伶仃。 从小黄猫给猫婆叼烟卷或猫婆死后去故居哀叫,可见猫婆对猫非常亲近与关爱,让猫都依恋和感恩了。 8.①对猫被困在瓦内至死遭际的痛心;②对原来猫胡同内的猫寒冬雪天无处御寒的忧虑;③对人们为了自家利益而罔顾其他生命的自私行为的悲哀;④对猫婆收养受难猫儿的崇敬和追思。⑤对爱心的渴望与呼唤。 9.人们都不赞同、不理解,和她结婚的老汉更是无法忍受,非常嫌弃。我赞同猫婆的做法,因为我们要懂得尊重生命,关爱小动物,尤其是关爱弱势群体。(我不赞同猫婆的做法,因为这样养猫确实影响了家人和周边居民的生活)。【解析】6.考查分析句段作用。首段中的“为了逃避这群讨厌的家伙,我真想换房子搬家”表现了我对胡同里这些猫们的讨厌。“奇怪,哪来这么多猫,为什么偏偏都跑到这胡同里来聚会闹事”用问句的形式激发了读者的疑问,设置了悬念,引出下文对猫婆的记叙。首段提到了胡同里的那些猫,为下文我为了找自家的猫见到了猫婆,了解到这些猫的来历做了铺垫。联系第③段中的“我也是视猫如命了”和第⑨段中的“但从此不知不觉我竟开始关切所有猫儿的命运”可知,首段中写我对猫的讨厌,与下文“我”也视猫如命、关切所有猫儿的命运形成对比。7.考查语句赏析。(1)根据第⑥段中的“她立刻请我进屋去,是间阴冷的地下室。一股浓重噎人的猫味马上扑鼻而来。屋顶很低,正中吊下一个很脏的小灯泡,把屋内照得昏黄。一个柜子,一座生铁炉子,一张大床,地上几只放猫食的破瓷碗,再没别的”可知,猫婆家生活设施非常简陋。可以看出她生活的困顿。(2)根据第⑫段中的“据说,猫婆无亲无故,孑然一身,拾纸为生,以猫为伴”可知,猫婆命运坎坷,孤苦伶仃。(3)根据第⑱段中的“在大雪涂白了的胡同深处,猫婆故居那墙头上,孤零零趴着一只猫影,在凛冽中蜷缩一团,时不时哀叫一声,甚是凄婉”可知,猫婆对猫非常亲近与关爱,让猫都依恋和感恩了。8.考查语句赏析。联系第⑯段中的“但谁家也不肯将自家的顶棚捅坏,放它出来。这猫叫了三整天,开头声音很大,很惨,瘆人,但一天比一天声音微弱下来,直至消失”可得:对猫被困在瓦内至死遭际的痛心;根据第⑱段中的“更深夜半,天降大雪,猫胡同里一片死寂,这寂静化为一股寒气透进我的肌骨”可得:对原来猫胡同内的猫寒冬雪天无处御寒的忧虑;根据第⑯段中的“但谁家也不肯将自家的顶棚捅坏,放它出来”可得:对人们为了自家利益而罔顾其他生命的自私行为的悲哀;联系第⑫段中的“但她所养的猫没有一个良种好猫,都是拾来的弃猫、病猫和残猫。她有猫必留,谁也不知道她家到底有多少只猫”可得:对猫婆收养受难猫儿的崇敬和追思。联系全文内容可知,作者借猫婆照顾受伤的流浪猫的正面事件,人们不将自家的顶棚捅坏救出被困的猫的反面事件,表现对爱心的渴望与呼唤。9.第一问:考查筛选信息。根据第⑫段中的“据说,猫婆无亲无故,孑然一身,拾纸为生,以猫为伴,但她所养的猫没有一个良种好猫,都是拾来的弃猫、病猫和残猫”“但结婚不过两个月,老汉忍受不了这些猫闹、猫叫、猫味儿,就搬出去住了”可得:人们都不赞同、不理解,和她结婚的老汉更是无法忍受,非常嫌弃。考查阅读启示,开放类试题,言之成理即可。如:我赞同猫婆的做法。根据首段中的“每日夜半,这里是猫儿们无法无天的世界。它们戏耍、求偶、追逐、打架,吵得人无法入睡”和第⑫段中的“,但结婚不过两个月,老汉忍受不了这些猫闹、猫叫、猫味儿,就搬出去住了”可知,这样养猫确实影响了家人和周边居民的生活。三、阅读下文,完成下面小题。《人世间》节选梁晓声①一九八六年,周秉昆的父亲周志刚六十六岁了。②他对A市表现出了别人难以理解的深情。退休后的头一个月里,他整天骑辆旧自行车到处逛,把全市的边边角角以及四周郊区都逛遍了。他逛得特过瘾,体会却只是两句话:“哪儿都没变,哪儿都熟悉。”③他对更加脏乱差的光字片一点儿也不嫌弃,因为见过太多比光字片还要脏乱差的情形。同样的情形,是当年许多农村和城市的常态。四年里,他这位从”大三线”退休的老建筑工人,似乎把光字片当成了”小三线”,把自己家所在那条被违章建筑搞成了锯齿状的小街当成了主要工程。如何让自己的家看上去还有点儿家样,理所当然成了他心目中的重点工程他似乎要独自承担起改良的神圣使命。在春夏秋三季,人们经常见到他在抹墙,既抹自家的墙,也抹街坊邻居家临街的墙。他抹墙似乎有瘾,四年抹着薄了几把抹板。有一年,街道选举先进居民,他毫无争议地当选了,区委副书记亲自奖给他一把系着红绸的抹板。他舍不得用,钉了个钉挂在墙上。他依然是个重视荣誉的人。④他的工具不仅是抹板,还有铁锨。人们也常见他修路,铲铲这儿的高,垫垫那儿的低,填填某处的坑,像在平整自家门前的地方。见到他那么做的人有过意不去的,也有心疼他那么大年纪的,常常劝他:“拉倒吧!一条小破街,弄不弄有什么意思呢?下场雨又稀里哗啦踏烂了。”他却说:“弄弄总归好点儿,反正闲着也是闲着”或说:“我往土里掺了炉灰,再下雨不会那么泥泞不堪了。"⑤四年一晃过去,周志刚更老了。一晃多少年的“晃”字虽属民间口头语,但把那种如变脸般快的无奈感传达得淋漓尽致。周志刚完全秃顶了,脑壳左右稀疏的头发全白了。他渐渐蓄起了一尺来长的胡子,胡子倒有些许灰色,估计继续灰下去的日子肯定不会太多了。他的腿脚已不灵活,有点儿步履蹒跚,浑身经常这里痛那里酸的。当年在“大三线”工地上对体能的不遗余力的透支,开始受到必然性的制裁。别人已经称他老爷子了,而即使别人不那么称他,他也明明白白地意识到自己确实老了。⑥不论对自家房屋的维修,还是对街坊家临街墙面的义务抹平,他都感到心有余而力不足了。抹墙需几道工序,先得备下黄泥,还得有足够的麦秸或谷秸往泥里掺。和好一堆抹墙的泥很需要力气,他和不动了。黄泥也稀缺了,可挖到黄泥的地方越来越少,那种地方往往很快便出现了就地取材建起的土坯或干砸垒的黄泥小屋。当那些小屋住进了人家,如果谁还去周边挖取黄泥,常常引发严重冲突。那些人家会形成一种占山为王的领地意识,攻守同盟,态度凶悍,让企图分享公共资源者望黄泥而却步。⑦周志刚是洁身自爱的人,当然避免自取其辱。缺少了黄泥,不论是对他自家房屋的维修还是对他们那条脏街所进行的面子工程都只好停顿下来,毕竟他只是一个老迈的改良者,也只有点儿人生余力做改良者。倘要彻底改造自己家及那条脏街的面貌,需动用推土机和铲车,需有充足的建材,还需有一支建筑队——而单枪匹马的他只有一把抹板,街坊们心劲儿又不齐对他们而言,维修自家房屋是分内之事,至于那条脏街已经那样了,可以怎样改良一下不在自己考虑范围。他们认为那纯属政府的事,如果政府不觉得有失面子,他们则是特能忍受的,住在那么脏乱差的地方的人家还有面子值得在乎吗?还讲得起面子吗?讲面子起码也得有黄泥呀,连黄泥都稀缺了,就只得让面子见鬼去了。墙皮掉得太不成样子了,才趁夜到这里那里去偷黄泥。倘谁家的男人或大男孩天黑后挑着水桶走往与水站相反的方向,那么准是到什么地方偷黄泥去了,用水往回挑是为了掩人耳目,街坊们对此心照不宣。偷黄泥往往引发人身伤害事件,但由于是刚性需求,也就只能睁只眼闭一只眼了。⑧周志刚断不会做那种勾当。他连自家墙上掉下的墙皮也宝贵地留存起来,积少成多,以备用时。他不敢放在门外,怕被偷,专门放在家中一角。⑨星期日或年节假日,儿女们回来看望他和老伴时,他嘴里常常会忽然蹦出一句话:“你们谁知道哪有黄泥吗?”儿女们便都装聋作哑。⑩他是在儿女面前自尊心极强的父亲,不会问第二次的,总用自言自语缓解自己的担忧:“这个家再不修修抹抹,那就不像个家了。”【注释】①光字片:周家生活的社区叫作“光字片”是一个具有年代感的棚户区10.阅读文章,想一想第一、二段有什么作用。11.文章叙述了周志刚与“光字片”老街的故事,同时表达了他对老街的情感,请结合文章内容完成表格。12.你如何理解“当年在‘大三线’工地上对体能的不遗余力的透支,开始受到必然性的制裁”的含义?13.读完本文,你一定被周志刚老人的行为所感动,请你以第二人称的口吻为周志刚老人写一段颁奖词。要求:颁奖词中要使用到修辞手法。【答案】10.交代了人物身份和故事背景,突出周志刚对A市的深情,为后文不遗余力地修墙修路,用一已之力改造片区的行为做好了铺垫,使情节更为合理。 11.四年后 准备修墙 惦记修墙 不遗余力(竭尽全力) 12.周志刚当年超负荷工作,严重损害了身体健康,对于修墙已经力不从心。 13.结合周志刚事例,品质精神,修辞,注意整体效果。【解析】10.本题考查句段作用。第①段“周秉昆的父亲周志刚六十六岁了”交代了写作对象——周志刚,以及人物的身份和背景;第②段“他对A市表现出了别人难以理解的深情”“他整天骑辆旧自行车到处逛,把全市的边边角角以及四周郊区都逛遍了。他逛得特过瘾”写出了他对A市的一片深情;联系第③段“在春夏秋三季,人们经常见到他在抹墙,既抹自家的墙,也抹街坊邻居家临街的墙”可知,前两段的内容为后文写周志刚不遗余力地修墙修路做铺垫,推动故事情节发展。11.本题考查文章内容概括。已知信息:“刚退休”“充满深情”对应文章第②段“退休后的头一个月里,他整天骑辆旧自行车到处逛,把全市的边边角角以及四周郊区都逛遍了。他逛得特过瘾”;“头四年”“开始修墙”对应第③段“四年里,他这位从‘大三线’退休的老建筑工人……他抹墙似乎有瘾,四年抹着薄了几把抹板”;“停止修墙”“力不从心”对应第⑥段“不论对自家房屋的维修,还是对街坊家临街墙面的义务抹平,他都感到心有余而力不足了”第⑦段“缺少了黄泥,不论是对他自家房屋的维修还是对他们那条脏街所进行的面子工程都只好停顿下来,毕竟他只是一个老迈的改良者”;“休息日”“念念不忘”对应第⑨段“星期日或年节假日,儿女们回来看望他和老伴时,他嘴里常常会忽然蹦出一句话:‘你们谁知道哪有黄泥吗?’儿女们便都装聋作哑”;第①空:综合以上信息,根据第⑤段“四年一晃过去,周志刚更老了”可知,时间为:四年后;第②空:根据第③段“他对更加脏乱差的光字片一点儿也不嫌弃,因为见过太多比光字片还要脏乱差的情形。同样的情形,是当年许多农村和城市的常态”可知,周志刚看到光字片脏乱差的情形,准备修墙改造;第③空:根据第⑨段“他嘴里常常会忽然蹦出一句话:‘你们谁知道哪有黄泥吗?’儿女们便都装聋作哑”可知,周志刚年迈也依然惦记修墙;第④空:根据第③段“在春夏秋三季,人们经常见到他在抹墙,既抹自家的墙,也抹街坊邻居家临街的墙。他抹墙似乎有瘾,四年抹着薄了几把抹板”可知,周志刚在不遗余力地修墙。12.本题考查句子含义。不遗余力的意思是指用出全部力量,一点也不保留;透支的意思是所付出的金钱、心力超过所能负荷的限度;结合第③段“他这位从‘大三线’退休的老建筑工人”可知,周志刚曾经是一位建筑工人,“当年在‘大三线’工地上对体能的不遗余力的透支”是指周志刚当年在建筑工地上长期超负荷工作,透支了自己的体力;结合前句“他的腿脚已不灵活,有点儿步履蹒跚,浑身经常这里痛那里酸的”可知,“开始受到必然性的制裁”是指周志刚的超负荷工作严重损害了自己的身体健康,让现在年迈的他对修墙已经心有余而力不足了。13.本题考查颁奖词的拟写,注意结合相关示例并运用修辞。示例:那一把系着红绸的抹板,是您荣誉的象征;那一面面崭新的白墙,是您辛劳的见证;那一条宽阔平整的街道,是您精神的展现;您虽处人生暮年,却不计名利、无怨无悔,依然为社会默默做贡献,您的无私奉献,值得所有青年人学习!四、阅读下面文学类文本,完成下面小题。地铁站台的背影李小雅①“请退回黄色安全线内!”“列车马上要关门了,不要再上了!”地铁站台上的人群在列车即将关门的时候,总是特别疯狂,仿佛因挤不上列车而被迫与人分离,是一件比割肉还痛苦的事。这是“国庆黄金周”出行高峰期。拥挤的人群,密集的脚步声,生怕走散的高声叫喊,都在人们的意料之中。②“请排队有序等候列车!……”小析的叫喊声淹没于一片嘈杂声浪。他只有打开喇叭,扯着嗓门大喊,并辅之以疏导人流的指示动作,才得以被人注意。③“请排队有序等候列车!……”几个小时过去,小析已精疲力竭。他是一名地铁志愿者,每当人流量激增的节假日或周末,他都会来到站台维持秩序,保证乘客的安全。④事实上,小析的穿着十分显眼:荧光背心,红色绶带“地铁志愿者”的金字闪闪发光。他也比一般乘客高出一截,在他那个水平视线内,应该是一片黑色头顶的“一览众山小”之景。在等候列车的空当,虽身在人群之中,小析的心思此时短暂放空。他想起了他已一整年未回的遥远的家。⑤小时候,母亲总是在小析放学到家前,就做好了一桌热腾腾的饭菜。快到家门口的小析闻到饭菜香,就迫不及待地冲上楼。后来,小析的身高蹭蹭蹭地一下子超过了同龄人,并顺利入选进了市篮球队。随着训练强度的增加,学业负担的加重,小析感到烦闷无趣,想要逃离这里的一切。后来,他考到了离家很远的大学,开始了全新的生活。小析在新环境里适应良好,结交了许多新朋友,也加入了志愿者这个大家庭。一年过去了,这一年,他再没回过家。⑥“列车即将进站,请乘客们注意安全……”广播响起,把小析拉回到了现实。他赶忙打开喇叭,指引着乘客们有序等候。“先下后上,那位小伙子,请等一下!”两个方向的人流碰撞,又是一阵混乱。在忙乱之间,突然,小析看到在前一节列车排队等候的乘客里有一个熟悉的身影。⑦“妈!”⑧小析又惊又喜,那不就是一年没见的妈妈?她怎么会只身一人来到这里?难道是在找我吗?霎时间,小析的眼睛湿润了。⑨那个在人群中显得弱小的背影,那个白了头发、朴素整洁的人,就是记忆中每天做好一桌丰盛的饭菜等我回家的妈妈吗?可是眼下,这节车厢的乘客正往车内挤,在小析面前,隔着一道坚固的流动的人墙……⑩“列车即将关门,请注意安全!”小析回过神来,赶忙阻止了还在车门与站台间不甘放弃的一位大叔,“您别挤了,您的背包上不了啦!”车门关上了,小析抬眼望去,那个身影已不见。他追着车跑,但始终再也没有看见她……⑪“喂,妈!我看见你了!”小析慌忙掏出手机,打给母亲。“什么?你回来了吗,儿子?”母亲的语气里流露出惊喜。“妈,你在家?”小析听出电话里的背景并不嘈杂。“在啊,在准备午饭呢。你这孩子,回家了也不说一声!”母亲这时停下手中的活,语气中的惊喜依然不减。“妈,对不起……”小析明白过来,刚才那个背影并不是母亲,只不过是他认错了。“妈,我刚刚把别人的背影认成你了。妈,我好想你!我在当志愿者,别人放假的时候都是我最忙的时候。我过年的时候,一定回家。”此刻,小析终于明白他其实很想家,想念妈妈可口的饭菜,想念小时候妈妈温暖的怀抱和鼓励的眼神。⑫“儿子,妈妈想告诉你,你真让我骄傲。为别人付出,无私奉献,不求回报,你做志愿者,妈妈很开心。”小析听着,不知不觉流下了热泪。他知道,在每一个平凡的岗位上奋斗着的人,都是不平凡的。而正是有这么多人并肩作战,无私付出,这个社会才会更加美好。⑬小析越来越觉得,自己身上佩戴的“地铁志愿者”绶带,分量沉甸甸的……14.下列对文章的理解和分析,不正确的一项是( )A.小说的标题“地铁站台的背影”起到了设置悬念,激发读者阅读兴趣的作用。B.第④段画线句巧妙引用了杜甫的诗句,表现小析高个的特点,语言诙谐风趣。C.第⑦段独句成段,强调小析内心深处渴望见到妈妈,突出母子相见时的惊喜之情。D.这篇小说构思巧妙,情节虽然没有太多曲折,但合乎情理,读来让人颇有感触。15.根据要求,在横线处填空。(1)文章第①段对“国庆黄金周”出行高峰期地铁站台_____________的环境特点,暗示地铁志愿者工作的辛苦,侧面表现了_____________。(2)文中第⑤段从记叙的顺序来看属于____________,交代了小析一年来不回家的原因,为下文他因初次离家,思念母亲而认错背影作铺垫。16.第⑧段、⑫段划线处小析两次流泪的心理有何不同,请简要分析。17.结尾写小析觉得自己身上佩戴的“地铁志愿者”绶带,分量沉甸甸的。他为什么觉得“沉甸甸”?请简要分析。【答案】14.C 15.拥挤、嘈杂; 小析等志愿者们任劳任怨、无私奉献的高尚情怀 插叙 16.第⑧段:小析误以为见到了一年未见的妈妈,又惊又喜,同时为自己久未归家让妈妈惦念感到愧疚。第⑫段:小析为妈妈对自己的理解(鼓励)流下了感激(欣慰)的泪水。 17.“地铁志愿者”的辛勤付出让小析明白了普通劳动者虽然平凡,但无数的平凡劳动者的共同努力可以让社会变得更美好,这种社会责任感让他感到“沉甸甸”的。妈妈的赞扬鼓励和殷殷期许,也让小析感到了“沉甸甸”的分量。【解析】14.本题考查文本理解。C.错误。根据第⑪段“小析明白过来,刚才那个背影并不是母亲,只不过是他认错了”可知,母子二人并未见面;故选C。15.本题考查对文章内容的理解及记叙顺序。由第①段“地铁站台上的人群在列车即将关门的时候,总是特别疯狂,仿佛因挤不上列车而被迫与人分离,是一件比割肉还痛苦的事”“拥挤的人群,密集的脚步声,生怕走散的高声叫喊”可知,“国庆黄金周”出行高峰期地铁站台拥挤、嘈杂;结合第①段“地铁站台上的人群在列车即将关门的时候,总是特别疯狂,仿佛因挤不上列车而被迫与人分离,是一件比割肉还痛苦的事”,第②段“小析的叫喊声淹没于一片嘈杂声浪”“他只有打开喇叭,扯着嗓门大喊,并辅之以疏导人流的指示动作,才得以被人注意”,第③段“几个小时过去,小析已精疲力竭”可知,虽然出行高峰期地铁站台拥挤、嘈杂,但是小析仍坚持疏导人群上下车,侧面表现了小析等志愿者们任劳任怨、无私奉献的高尚情怀;插叙是在叙述中心事件的过程中,为了帮助开展情节或刻画人物,暂时中断叙述的线索,插入一段与主要情节相关的回忆或故事的叙述方法。由文章内容可知,第⑤段交代了小析过去一年来不回家的原因,属于插叙。16.本题考查文本内容理解。根据第⑧段“小析又惊又喜,那不就是一年没见的妈妈? 她怎么会只身一人来到这里?难道是在找我吗?”可知小析误以为见到了一年未见的妈妈,又惊又喜,同时也为自己久未归家而感到愧疚。因此,第一次落泪,是惊喜的泪,也是愧疚的泪;根据第⑫段“儿子,妈妈想告诉你,你真让我骄傲。为别人付出,无私奉献,不求回报,你做志愿者,妈妈很开心”可知,小析的妈妈非常理解并支持他。此刻的落泪,是为妈妈的理解而流下的感激的泪水。17.本题考查语句理解。根据第⑫段“他知道,在每一个平凡的岗位上奋斗着的人,都是不平凡的。而正是有这么多人并肩作战,无私付出,这个社会才会更加美好”,可知小析在听了母亲的鼓励之后,更加懂得了普通劳动者劳动的意义,虽然平凡,但只要无数的劳动者共同努力,就可以让社会变得更美好。妈妈的鼓励,让小析拥有了更加强烈的社会责任感,这种社会责任感既让他感到荣耀,也感到责任重大,因而内心“沉甸甸”的。五、阅读下面的文字,完成下面小题。老罗师傅①老罗师傅是位送水工。我也记不清老罗是从什么时候开始为我们小区送水的,他好像早已融入了我们的生活。(甲)②瘦瘦高高的老罗木讷寡言,河南人,见人只是嘿嘿一笑。因为种田难以支撑儿子上大学的开销,年过半百,进城打工。除此之外,大家对他的信息知之甚少。人们也不知道老罗公司的名称,每次见水桶快见底时,只要拨打老罗永不关机的手机,即便是夜间八九点钟,老罗也挪着不慌不忙的步子,登楼送水,撩一把汗,换上塑料鞋套,将一桶矿泉水稳稳装在饮水机上。交接十二元水费,拎着空桶,嘿嘿一笑后转身离开。有时家中临时有事,锁门不在,老罗便会将水放在门口,等下次再付费。(乙)③要不是炎热的夏天来了,小区时不时会陷入“水荒”,邻居们可能都感觉不到老罗多年如一日的存在。老罗解释道,天热山上的泉眼越来越细,取水的人越来越多,拉水车每次都需要排队好长时间,小区也疯传老罗他们公司的水以次充好。山上的泉眼终于干涸了,纯净水取代了矿泉水,价码也从十二元降成十元一桶。④小区的住户纷纷议论纯净水的水质,有沉淀了,出异味了。老罗听了,还是嘿嘿一笑,并不言声。人们也很快接受了老罗送来的纯净水,日子便又重回往日的平静。(丙)而老罗依然沉默寡言,依然及时送水,依然嘿嘿一笑。要说老罗送水有何不便,就是每次送水大家提前备好十元零钱,老罗不习惯移动支付。于是,面对小区住户的调侃,老罗依旧嘿嘿一笑,最多带着浓浓河南口音回一句话:“不中,小区里老人多,用现金支付习惯,不麻烦。”⑤老罗默默地送水,日子也静静地流淌。直到有一天,老罗的电话突然关机了,这可是从未出现的状况,人们种种猜测和抱怨。(丁)不久传来消息,老罗心梗,已经走了。这么突然!想想他的质朴和憨厚,想想他的尽职和尽责,大家叹息,好人一个。⑥老罗一走,大家只好按着水桶上的电话订购纯净水。一日,我打电话订水后,来了一位头戴棒球帽,戴耳机的年轻人,二十来岁,很帅,很酷。我想以后就要和这位小师傅打交道了,便主动索要他的电话号码。小伙子头也不抬:“水桶上有电话,订水和公司联系。”我有点尴尬,不过这只是变化的开端。一次,打电话订水后,因临时家中有事锁门外出,回来时门口却没有往常一样的水桶。第二天,在小区碰到了小伙子,问起此事,他好一顿数落,抱怨我让他白跑一趟。还说以前都是师傅老罗替我们垫资,自己不会那样做的。⑦小伙子还规定了送水的时段,且不再随叫随到,几乎一夜之间,小伙子站在了舆论风口:送水不及时,态度生硬,只通过微信收费,老人们感到非常不便。不久,公司又开通了二维码订水,只需在平台上付费订水并填写地址,然后排队等候。人们感到非常不便,但后来也渐渐接受了这种送水方式。⑧生活也再度回归平淡。只是偶尔还会忆起憨厚的老罗,在楼道里晃动着瘦高的身形,剪开熹微的光影,扛着沉甸甸的水桶缓步走来。可是又有谁想过老罗师傅曾经在心里想过些啥呢?18.请简要分析文章第①段的作用。19.文章多次将老罗师傅和小师傅进行对比,请简要概括一处对比情节并分析其作用。20.下面句子出自原文,请说说它应该放在文中甲乙丙丁的哪一处,并说明理由。一天又一天,老罗成了小区中的一员,家家离不开他。21.结合语境,请从人物描写的角度赏析下面句子的表达效果。只要拨打老罗永不关机的手机,即便是夜间八九点钟,老罗也挪着不慌不忙的步子,登楼送水,撩一把汗,换上塑料鞋套,将一桶矿泉水稳稳装在饮水机上。22.文中的老罗与杨绛笔下的老王有许多相似之处,请从两个人的品性,处境和作者的情感态度比较分析。【答案】18.结构上:照应文题,并引起下文,使文章结构浑然一体。内容上:点明本文描写对象是老罗,暗示文章主旨,表达作者对老罗的赞美和怀念之情。 19.示例:为方便老人,老罗师傅收现金;小师傅只通过微信收费;突出老罗的质朴,憨厚、尽职尽责。 20.这个句子应放在乙处。“一天又一天,老罗成了小区中的一员,家家离不开他”,这是一个总结性句子,表现了老罗对小区的重要作用;文章第②段表现出老罗为小区成员们做的事,时间长之后,自然“家家离不开他”。因此把这个句子放在乙处恰当。 21.运用了动作描写,连用“挪”“登”“撩”“换”“装”等动词,细致入微地写出了老罗送水时不慌不忙的样子和熟练的动作,表现了老罗淳朴,勤劳和为他人着想的形象。 22.品性:两个人都淳朴,待人真诚。一个送水服务到位,一个运冰货大价实。处境:两人都生活卑微,有时未得到太多的尊重。小区的人随意推断老罗公司的水以次充好,老王周围的人随意推断老王年轻时不老实。作者的情感;都表达了作者对社会中弱势群体的同情,以及对淳朴善良的社会风气的渴望与追思。【解析】18.考查分析句段作用,可从内容和结构两个角度来分析。结构上:首段中的“老罗师傅是位送水工”照应了题目“老罗师傅”,“我也记不清老罗是从什么时候开始为我们小区送水的,他好像早已融入了我们的生活”引出下文对老罗为小区送水,与大家融洽相处的倒带,引出了下文,使文章结构完整。内容上:“老罗师傅是位送水工”点明了故事的主要人物:老罗师傅。以及他的身份:送水工。“他好像早已融入了我们的生活”一句点明了主旨,表现了老罗以自己的善良淳朴与细心体贴和小区人们融洽相处,表达了作者对老罗的赞美与怀念之情。19.考查对比的写法。开放类试题,联系相关情节分析其作用即可。如第⑥段中的“第二天,在小区碰到了小伙子,问起此事,他好一顿数落,抱怨我让他白跑一趟。还说以前都是师傅老罗替我们垫资,自己不会那样做的”就运用了对比手法,老罗会为用户垫资,让人们先喝上水,而这个年轻人却不会这样做。通过对比,突出了老罗的善良淳朴与细心体贴。表达了作者对老罗的赞美之情。20.考查补充句子。“一天又一天,老罗成了小区中的一员,家家离不开他”,是一个总结性的句子。与首段中的“他好像早已融入了我们的生活”相呼应。联系乙处前面的“只要拨打老罗永不关机的手机,即便是夜间八九点钟,老罗也挪着不慌不忙的步子,登楼送水,撩一把汗,换上塑料鞋套,将一桶矿泉水稳稳装在饮水机上。交接十二元水费,拎着空桶,嘿嘿一笑后转身离开。有时家中临时有事,锁门不在,老罗便会将水放在门口,等下次再付费”可知,本段记叙了老罗为小区人们做的事,他认真负责,体贴细心。按常理推断,时间长了之后,人们自然就家家离不开他了。所以这句子放在乙处最合适。21.考查语句赏析。“挪着不慌不忙的步子”“登楼”“撩一把汗”“换上塑料鞋套”“装在饮水机上”运用动作描写,连用一列动词,再结合第②段“每次见水桶快见底时,只要拨打老罗永不关机的手机”“有时家中临时有事,锁门不在,老罗便会将水放在门口,等下次再付费。一天又一天,老罗成了小区中的一员,家家离不开他”可知,写出了老罗送水时的从容和熟练的样子,同时说明老罗送水踏踏实实,总能为他人着想的性格特点。22.本题考查对比探究人物性格特点及内容。①品性:淳朴,待人真诚。本文的老罗“结合语境,请从人物描写的角度赏析下面句子的表达效果。只要拨打老罗永不关机的手机,即便是夜间八九点钟,老罗也挪着不慌不忙的步子,登楼送水,撩一把汗,换上塑料鞋套,将一桶矿泉水稳稳装在饮水机上”,他送水服务到位;杨绛笔下的老王,运冰比别人的都大,却不加价。②处境:生活卑微,有时未得到太多尊重。本文中,老罗是一个送水工。炎热的夏天小区陷入“水荒”,小区的人没有根据地推断老罗以次充好;杨绛笔下的老王是一个三轮车夫,一只眼睛不好,被周围的人随意推断老王年轻时不老实。③作者的情感:本文末段中的“只是偶尔还会忆起憨厚的老罗,在楼道里晃动着瘦高的身形,剪开熹微的光影,扛着沉甸甸的水桶缓步走来”,表达了作者对老罗的怀念,对他所代表的社会弱势群体的同情,表现了作者对淳朴善良的社会风气的渴望与追思;《老王》一文中,作者在文末说“那是一个幸运的人对一个不幸者的愧怍”,表达了作者对老王的同情和没能让他在临终前感受家人的亲情的惭愧。同时表达了对淳朴善良的社会风气的渴望与追思。六、阅读下面的文章,完成下面小题。人力车夫①结识他,是种偶然。②应故友盛情相邀,我们夫妻俩去浙江乌镇、南浔游玩了两天。江南水乡的风韵如诗画般令人沉醉。但此行留下最深刻印象的,是一位至今仍让我难忘、感动的三轮车夫。③那日抵达南浔时已下午五点,匆匆在宾馆安顿好后,我们便急不可耐地外出欣赏黄昏时分古镇的别样神韵。初次去南浔,人生地不熟,且天色已晚,不知景点怎么走,就在疑惑犹豫间,突然瞥见在宾馆大门外,停有几辆人力三轮车在候客,于是,我们跨上了其中一辆半旧不新的三轮车,因而有幸结识了这位敦厚的车夫。④甫坐上车,他便主动热情地向我们推介两条游览路径,一声亲切的“坐稳了哦”,给人细微体贴的感受,直觉得心里暖暖的。人力车一会儿在用石板铺就的巷子里晃悠,一会儿环绕着幽静的河浜穿梭,偶尔还到罕见的廊檐下穿越,轻微地颠簸摇晃着缓慢前行,我们坐在车上却感到特别舒坦安稳。此刻的南浔,祥和而宁谧。他一边使劲蹬着车,一边不停地为我们讲解着。⑤游览了一圈,他把我们拉回宾馆。正欲下车时,一句“下车请当心,请拿好你的物品”,让人倍感殷切与关怀。当问及给多少车费时,他竟痛快而满不在乎地回道,随便你给吧。这使我十分不解——靠苦力谋生养家,却如此轻视钱财,他究竟是怎样的一个人啊。怀着半是好感半是好奇的心情,我与他相约次日清晨六时再去晨游,细细赏景。⑥第二天清晨,醒来时天已彻亮,急忙洗漱一番,疾步来到宾馆大门前。我抬腕看了一下表,离约定的钟点还差五分钟,这时他已慢悠悠地蹬着车出现在我的视野里。⑦坐上车,按我们的意愿先朝农贸市场进发。临近周边,众多摊贩已在摆摊设点,销售各种当地特产和时令果蔬。他招呼我们下来后,便知趣地将车停在一旁静候。⑧走出不远,我回眸张望,只见他人骑在车上,车把上挂着条擦汗用的海蓝色毛巾,手里捧着一本厚厚的书在专注地阅读。此后,我发现,只要有片刻工夫,他都充分利用起来,将随身携带的书拿出来读上一会儿。后来,他悄悄地告诉我,他心中有个梦想,就是想要学会英语,以更好地为海外游客服务。闻听此言,我惊讶得直视着他。无论他的想法是否切合实际,也不管他何时能够学成,至少,他那种不甘落后的上进心是真的让我尊敬和感动的。这是否该让那些时间充裕、置身优越环境却不思进取的人汗颜?他还坦诚而自信地告诉我,年底打算去考导游员。⑨这一路,他轻快地踩着车,并借景发挥,侃侃道来。我们静心听他讲解,仔细观赏沿途的古宅老店。走走停停中,我们相继来到了石拱桥和古戏台,停车上前细察、摄影,不禁被古人的精妙匠心所折服。整个游览过程中,他不仅机灵卖力地蹬车,还热情讲述相关逸闻趣事或历史掌故,劝说难得来一次的我们多看看,却全然忘记了自己用来挣钱的宝贵时间。难怪他的同行和老乡有些羡慕并略带妒忌地对我说,他的生意可是全南浔人力车里最好的。⑩其实,一个人,如果能把简单的事情做到出彩,把卑微的活儿干到极致,就十分了不起,真的不平凡了。他,姓李名贺军,52岁,是一位来自安徽阜阳质朴憨厚的进城务工者。23.文中开头写道:“但此行留下最深刻印象的,是一位至今仍让我难忘、感动的三轮车夫。”这位三轮车夫使“我”难忘和感动的原因。24.任选角度赏析文中画横线的句子。人力车一会儿在用石板铺就的巷子里晃悠,一会儿环绕着幽静的河浜穿梭,偶尔还到罕见的廊檐下穿越,轻微地颠簸摇晃着缓慢前行,我们坐在车上却感到特别舒坦安稳。25.结尾交代了人力车夫的姓名、年龄和籍贯,这一交代是否有必要?为什么?26.本文和杨绛《老王》一文的写作对象都是人力车夫。这两位人力车夫虽然生活在不同的时代,但具有相同的性格特征。试说说两人身上有哪些相同的性格特征。【答案】23.这位人力车夫不仅待客热情,服务细致周到,不计较车费,而且还具有不甘落后的上进心。 24.这句话运用了动作描写或正侧结合。描写了人力车穿行时该慢则慢、该快则快的从容情形及顾客的感受,从侧面衬托了人力车夫驾驶技术的高超和娴熟。 25.有必要因为作者在结尾交代人力车夫的姓名、年龄和籍贯,显示出本文所写内容的真实性。交代人物身份的普通来反衬出人物形象的伟大,表达出对他的敬意 26.老实厚道(质朴憨厚),心地善良,关心人,重感情。(任意答道两点即可)。【解析】23.考查筛选信息。依据④段“甫坐上车,他便主动热情地向我们推介了两条游览路径,一声亲切的‘坐稳了哦’,给人细微体贴的感受,只觉得心里暖暖的”可以概括为:他待人真诚、热情。依据⑤段“下车请当心,请拿好你的物品”“让人倍感殷切与关怀”可以概括为:他服务细致周到。依据⑤段“当问及给多少车费时,他竟痛快而满不在乎地答道,随便你给吧”可以概括为:不斤斤计较。依据⑧段“只见他骑在车上,车把上挂着条擦汗用的海蓝色毛巾,手里捧着一本厚厚的书在专注地阅读”“后来,他悄悄地告诉我,他心中有个梦想,就是想要学会英语,以更好地为海外游客服务”可以概括为:他积极进取。24.考查词句赏析。④段画线句“人力车一会儿在用石板铺就的巷子里晃悠,一会儿环绕着幽静的河浜穿梭,偶尔还到罕见的廊檐下穿越,轻微地颠簸摇晃着缓慢前行,我们坐在车上却感到特别舒坦安稳”句中的“晃悠”描写的是对人力车在巷子里行走时的状态。“穿梭”是在河浜行走时的状态。“穿越”是在廊檐下行走时的状态。生动形象地写出了在不同的道路上,人力车有不同的行走状态,该快则快,该慢则慢。可见人力车夫驾车的从容。句中的“我们坐在车上却感到特别舒坦安稳”点明了我们坐车的感受,侧面表现了人力车夫拉车技术的高超与娴熟。25.本题考查理解结尾内容及其作用能力。依据末段“他,姓李名贺军,52岁,是一位来自安徽阜阳质朴憨厚的进城务工者”可知,这交代了人力车夫的姓名、年龄和籍贯。从小说的角度理解,这表现了本文所写的人物及其事件是真实的。依据“是一位来自安徽阜阳质朴憨厚的进城务工者”可知,这是一个及其普通的劳动者。依据全文内容可知,一个身份普通的人,却做出超出平凡人的行为,令作者敬佩。所以这一交代有必要。26.本题考查分析人物形象。老王给杨绛送冰,车费减半,送钱先生去医院,不要钱,收了还不放心,临死前给杨绛送来香油和鸡蛋,可知他是一个老实厚道,心地善良,关心他人,重感情,知感恩的人。本文中的这位车夫不计较车费的多少,带着我们四处游玩,不怕耽误自己的生意,下车时提醒我们注意安全,从中可以看出他是一个老实厚道,心地善良,关心他人,重感情的人。七、阅读下面的文章,完成下面小题。遗 书①叶明之一瞬间就彻底明白,自己落入了魔掌,再也挣脱不了了。②果然,叶明之经受了十八般酷刑,双腿齐刷刷地断了,昏死了过去,醒来时已在一处黑洞洞的牢房里了。他抬抬双手,钻心的疼袭来,冷汗霎时滚了下来。③叶明之发现了一双眼睛,一眼不眨地望着自己,充满了恐惧和慌乱。他还是强忍着疼,给这双眼睛递过去了一缕淡淡的笑。④那双眼睛闪烁着躲开了,躲避在牢房门外的旮旯里。⑤叶明之第二次见这双眼睛时,是在第二天的中午。牢门打开,眼睛的主人走了进来,是来送牢饭的。他的腿断了,牢饭只能送到面前。⑥那人说:“叫我黄三吧。”声音低得像蚊子哼,目光打在脚面上。⑦叶明之饿极了,捧起瓦盆,几乎是把饭倒进肚子里的。黄三站在一边,目光还是低低的。叶明之吃完了,用手擦擦嘴,轻轻地道了声:“谢谢!”⑧黄三头也不回地走了,牢门咣的一声锁死了。⑨叶明之安静了下来,把前前后后的事想了一遍,还好,没有破绽。他是在通知完最后一个同志撤离后被捕的。出了叛徒,叶明之蹬着自行车,抢在敌人前头,通知了他的上下线,在稍微喘了口气时,被按在了地上。①不过,他嘴角却露出了不易察觉的笑意。又顿觉周身疼痛,身上没有一寸地方是好的了。⑩叶明之彻底暴露了,若不松口,死是唯一的一条路。⑪不怕死。对死,叶明之早就有了准备,参加地下党那天,他就做好了死的准备。⑫黄三是叶明之每天见到的唯一活物,送饭、送水;要大小便时喊一声,黄三就进来。只是黄三的眼睛无处放,不敢和叶明之对视。⑬叶明之有时找黄三说话,黄三躲着,叶明之就自言自语,不着边际地说一些事。他知道,黄三在听,听得静悄悄的。⑭一天夜里,叶明之听到了低低的哭声,本以为是自己做梦,但不是在梦中,叶明之听出了这哭声来自黄三。⑮早晨黄三进了门,丢下稀汤样的早饭,还丢下一句话:“我也在坐牢。”叶明之没接上话,黄三已走开了。⑯叶明之能够在牢房里挪动身体了,黄三还是将牢饭送进来,此时的黄三眼中多少有了些喜色。是为叶明之吗?叶明之没往深处想。⑰叶明之多了样事,黄三在牢房外时,他就自言自语,有时背一首诗歌,有时讲些浅显易懂的道理。他是说给黄三听的,黄三也明白是说给自己听的。俩人心照不宣,一人说,一人听。⑱他还是知道了黄三哭的原因,黄三的老母亲被人欺负了,向死里欺负。⑲身体刚刚恢复的叶明之又一次被动了大刑,这次是双手,十指被钉了竹签。⑳叶明之昏迷后醒来,黄三立在叶明之身旁,这次黄三的目光没有躲避。㉑黄三说话了:“就认了吧,说句软话。”黄三的话很轻柔,也好听。㉒“不!”叶明之吐出一个字,但硬得如铁钉。他太痛苦了,即使是一个字,都会出一身冷汗。夜里,叶明之发高烧,说胡话。黄三守在一边,有时捂叶明之的嘴,有时把他摇醒。㉓下半夜,叶明之真正睡着了。可不久,又被哭声吵醒了,叶明之听出是黄三在哭,哭声噎在嗓子眼儿里,闷闷的。㉔第二天,黄三打开牢门,这次黄三主动看叶明之的眼睛了,定定地看,看得叶明之都想躲避了。黄三的目光里有东西,湿湿的。㉕黄三说:“你梦中喊一人的名字,我捂住了。”叶明之大吃一惊,还是回了句:“谢谢!”他知道,自己喊的那个人,是自己的爱人,也是同志。㉖到了秋天,黄叶悄悄地落,叶明之算了算,春天入狱,已半年时间了。他长长地叹了口气:“牢底坐穿吧。”㉗黄三送来了好吃的,还有一壶酒。黄三脚步沉重,像是提着千斤重物。叶明之知道,自己要上路了,去好远好远的地方。内心坦然,借着黄三的力量席地而坐,理了理荒草样的乱发,将一壶酒一饮而尽。㉘黄三忍不住落了泪,然后定神将叶明之看了个遍。黄三从口袋里掏出一张皱巴巴的纸,和一支秃秃的铅笔,递给叶明之。叶明之愣了下,摇了摇头。㉙“不给她留句话?”㉚“不了,该说的,都已说过了。”②叶明之的笑从嘴角慢慢洇开。㉛黄三不舍地起身离开,但又在牢房外停住脚步,他还想听叶明之的自言自语。㉜没有,一切都死寂。黃三只听到自己心的怦怦声。㉝叶明之被活埋了,埋得不留痕迹。㉞没过多长时间,黄三消失了,从监狱不声不响地消失了。㉟黄三花了九牛二虎之力,找到了叶明之高烧时喊着的那个她。㊱她满脸泪水,问黄三:“他有遗书吗?”㊲黄三哽着嗓子回答:“我就是他的遗书。”㊳春天来了,黄三透过泪眼,看到好多花都开得红艳艳的。27.小说前两段交代了哪些内容?有什么作用?28.联系上下文,分别体会下列句中人物“笑”的意蕴。①不过,他嘴角却露出了不易察觉的笑意。②叶明之的笑从嘴角慢慢洇开。29.下面各句是对黄三眼睛的描写,分析理解错误的一项是( )A.“那双眼睛闪烁着躲开了,躲避在牢房门外的旮旯里。”(表现他内心的胆怯慌乱。)B.“这次黄三的目光没有躲避。”(在叶明之的教育下,黄三内心发生了转变。)C.“黄三的目光里有东西,湿湿的。”(他想起了自己的母亲,内心痛苦。)D.“黄三透过泪眼,看到好多花都开得红艳艳的。”(缅怀烈士,看到了希望。)30.联系全文,简要概括“我就是他的遗书”这句话的含义和作用?【答案】27.叶明之被捕;受尽各种酷刑;交代了故事发生的背景(场景),扣人心弦(激发阅读兴趣);为后文塑造人物形象作铺垫。 28.①他圆满完成了组织交代的任务(提醒同志们转移、掩护同志们转移、为党挽回损失),对自己的行动感到满意。②对妻子的思念(对革命前途的乐观),将生死置之度外。 29.C 30.叶明之对革命的坚定信念和对妻子的爱打动了黄三;他理解叶明之对他妻子的感情(同情叶明之的遭遇);他主动转达叶明之的心愿;黄三的转变是叶明之教育感召的结果。从侧面突出了革命者不朽的精神,坚强的形象;升华了主题。【解析】27.本题考查对文本内容的分析和概括。在①段“叶明之一瞬间就彻底明白,自己落入了魔掌”概括为:叶明之被捕;②段“叶明之经受了十八般酷刑,双腿齐刷刷地断了”概括为:叶明之遭受了各种酷刑。文章在开头交代了小说故事的场景,只讲了叶明之被捕,遭受酷刑并昏死了过去。开篇没有明确交代叶明之的身份,给读者留下想象空间,并为后文做铺垫。28.本题考查分析文本内容。①在⑨段“他是在通知完最后一个同志撤离后被捕的”“叶明之蹬着自行车,抢在敌人前头,通知了他的上下线”此处的笑是因为叶明之掩护同志们安全地转移,圆满顺利完成党组织交给他的任务,自己保护了战友且没有破绽后,感到安心。②在㉗段“黄三脚步沉重,像是提着千斤重物。叶明之知道,自己要上路了,去好远好远的地方。内心坦然”㉘段“黄三忍不住落了泪段”“黄三从口袋里掏出一张皱巴巴的纸,和一支秃秃的铅笔,递给叶明之。”“不给她留句话?”“不了,该说的,都已说过了。”此处的笑是完成使命后的坦然赴死,面对黄三的举动,叶明之感受到黄三的转变后的欣慰,同时也包含着对妻子深深的思念。29.本题考查对文本内容的理解。C.“黄三的目光里有东西,湿湿的”在⑰段中“叶明之多了样事,黄三在牢房外时,叶明之就自言自语,有时背一首诗歌,有时讲些一点就明的道理。叶明之是说给黄三听的,黄三也明白是说给自己听的,俩人都心照不宣着,一人说,一人听。”黄三聆听叶明之有意或无意的宣讲并深受感化,深深影响和改变了黄三。在上文㉑段㉒段中“黄三说话了:‘就认了吧,说句软话。’黄三的话很轻柔,也好听。”“不!”叶明之吐出一个字,但硬得如铁钉。叶明之坚毅的爱国情怀打动了黄三。因此C选项黄三不是因为他想起了自己的母亲而内心痛苦。C选项错误。故选C。30.本题考查对文本内容的理解。结合㉒段“夜里,叶明之发高烧,说胡话。黄三守在一边,有时捂叶明之的嘴,有时把叶明之摇醒了”可知,叶明之发高烧,说胡话,黄三就捂叶明之的嘴或者把叶明之摇醒。可见黄三知道了叶明之的一切并保守秘密,因而他就是“遗书”。结合⑰段“叶明之多了样事,黄三在牢房外时,叶明之就自言自语,有时背一首诗歌,有时讲些一点就明的道理。叶明之是说给黄三听的,黄三也明白是说给自己听的,俩人都心照不宣着,一人说,一人听”可知,叶明之在牢房内背诗歌,讲些一点就明的道理,黄三在牢房外听,一人说,一人听,俩人都心照不宣,黄三聆听叶明之有意或无意的宣讲并深受感化。可见遗书实指叶明之身后留下的精神财富,他在狱中深深影响和改变了黄三,让黄三对未来有了新的方向。结合㊲段“黄三哽着嗓子回答:我就是他的遗书。春天来了,黄三透过泪眼,好多花都开得红红艳艳”可知,“春天来了”“好多花都开得红红艳艳”喻指革命事业蓬勃发展,后继有人。可见“叶明之的遗书”象征革命先辈拋头颅洒热血的牺牲精神,在黄三的身上,他们的精神得到延续和传承。八、阅读下面文学类文本,完成下面小题。赌①出来几个月了,一直没有找到工作。所有人都因为他曾经的经历而拒绝他。渐渐的,他有些失望了。②一天,在一个建筑工地上,他看到了中学同学蚊子——朱德文。蚊子是工地上的一个小包工头,就安排他当了一个力工,吃住都在工地上。③“先干着吧,等以后有了好去处再说。”蚊子说。其实他俩的关系并不算好,上学的时候都没怎么说过话。蚊子在同学聚会的时候,听说过他犯了事。他心里很感激蚊子,无论如何,总算暂时有了一个落脚的地方。④那天晚上,天阴沉沉的,蚊子拿了5千块钱回来,说是向老板要了半年才要回来的。天太晚,已经没有客车了,要在他的工棚里将就一宿。蚊子还弄了花生米、香肠和几瓶啤酒,两个人聊起来。蚊子不胜酒力,喝了两瓶酒就有些摇摇晃晃了。他不时地盯看蚊子的包,那些藏在心底的“恶”蠢蠢欲动起来。在监狱里改造了几年,他以为那些“恶”已经连根拔除了,没想到它们还在,偷偷地生长着,使他的灵魂扭曲变形。⑤他现在太需要钱了,他想如果现在下手,蚊子没有防备,会很容易得手。于是他又给蚊子开了一瓶酒,想让蚊子醉得彻底些,那样他的成功率会更高。蚊子又喝了一大口,然后就嚷嚷着要睡觉。让他惊讶的是,蚊子竟然把钱包塞到了他的怀里,眯着眼说:“我喝多了,你替我拿着吧。”然后脸冲里,呼呼地睡着了。⑥天赐良机!握着那鼓鼓囊囊的包,他的内心惊涛拍岸。这5千块钱对他来说,诱惑是巨大的。他望了望外面,黑黑的,只有远处几点稀疏的灯光。他转眼之间就可以逃之夭夭。他悄悄地起身开门,蚊子没有反应,依然鼾声如雷,睡得香甜。⑦工地无人,夜色浓重,他庆幸着,很快融入到了黑暗里。⑧在一处昏黄的路灯下,他忽然停住了脚步,心底的“恶”有些退缩了。他想到,这几个月里,他受尽白眼,没有人信任他。所有的人都拒绝他,排斥他,只有蚊子帮了他一把,而且如此信任,对他毫无防范之心。如果自己真的拿走了这5千块钱,就是给唯一信任自己的人当头泼了冷水,让人寒心。这样做,还算个人吗?他不禁哆嗦了一下。想到这里,他折回身,重新回到棚子里,又躺到了蚊子身边。蚊子向里翻了个身,鼾声依旧排山倒海,气势非凡。⑨不过这真是一个千载难逢的好机会。躺在那里,他的“恶”并不死心,依然怂恿着他。一整夜,他被折磨得疲惫不堪,心底像压了一块大石头。⑩他终究没有拿走那5千块钱。当东方云层缝隙中射出几线柔光的时候,他把包递给了蚊子,顿时感到莫大的轻松。他的眼睛红红的,蚊子问他怎么了,他撒谎说怕钱丢了,一夜没合眼看着它。蚊子忙说对不起、对不起啊,害你遭罪了。⑪10年后,他白手起家,成为身价不菲的富商,经历可谓传奇,他的事迹常常是当地报纸的新闻。他被人称道的品质就是诚信,他的商品从不掺假。与人谈起自己成功的经历时,他总是毫不避讳自己曾经阴暗的心路历程,特别是那一个让他辗转反侧的夜晚。他说,那个夜晚,真正改变了他的命运。因为一个人的信任让他觉得自己还是一个有用的人,他不能辜负信任。他感激那个人,他会一辈子记住他的名字:朱德文。⑫看着报纸,朱德文微笑着对着报纸里的“他”说:“其实,那个夜晚,我也很担心。毕竟你曾经是个犯过事坐过牢的人,喝酒时,我看到了你眼神中的贪婪,我的钱甚至生命都处于危险之中。我就决定赌一次。我把钱给你,如果你拿走了,我也认了。那一夜,我故意装作睡得很死,其实你的每个动作我都知道。这场赌博,我赢了——不,那一晚没有输家,咱们两个都赢了。”⑬是的,那一晚两个人都赢了。(原题为《一个夜晚的赌注》有删改)31.【分析情节】根据文章内容,请选出下列选项中符合文意的一项( )A.从里面出来后,所有人都因为他曾经的经历而拒绝他,走投无路的他只能到工地去找曾经的好友朱德文安排工作。B.蚊子不胜酒力,喝了两瓶酒就有些摇摇晃晃了。他不时地盯看蚊子的包,决定趁蚊子没有防备,拿走这5000元钱。可以看出他内心的“恶”并没有完全消除,使他的灵魂扭曲变形。C.他计划着让蚊子醉得彻底些,他又给蚊子开了一瓶酒,蚊子喝了一大口,于是他趁机抢走钱,蚊子依然鼾声如雷,他融入到了黑暗里。D.多年以后,看着报纸的朱德文透露,那一晚其实自己并没有睡着,“他”的每个动作自己都知道。让我们懂得,这场赌博没有输家。32.【走进内心】如果你是文中的“他”,处于第⑩段划线句中的情景,说说你当时会有怎样的心理活动。33.【揣摩人物】“他”经历了人生起伏,请结合文章内容分析他的人物形象。34.【探究缘由】文章第⑫段中朱德文微笑着对着报纸里的“他”说:“我故意装作睡得很死,其实你的每个动作我都知道”,请你说说朱德文这样做的理由。35.【深挖内涵】富兰克林曾说:“失足,你可能立刻复站立,失信,你也许永远难挽回。”请你结合文章内容分析文末“那一晚两个人都赢了”这句话,说说两人分别赢得了什么。【答案】31.D 32.(第一人称)我想:(纠结后的庆幸)“心好累,一夜的纠结,我被折磨得疲惫不堪,还好没有背恶念操控。”(内心的善战胜了恶之后心里的轻松)“真是太轻松了,最终还是没有拿走那些钱,没有让唯一信任自己的人寒心。 33.浪子回头,知错能改(曾经犯事进过监狱,但没有好吃懒做,打工挣钱,没有拿走钱);心存善念,靠自己努力赢得尊严(最终没有辜负朋友信任,战胜自己心魔,没有拿走钱);坦诚,诚信,上进(靠诚信经营,成为身价不菲的富商,不避讳讲出自己的曾经) 34.一是为了为了自身的安全;既不信任“他”,又希望这个“赌”能救赎他,给他一个改过自新的机会。 35.“蚊子竟然把钱包塞到了他的怀里”“依然鼾声如雷,睡得香甜”,蚊子赢回了钱和生命还有对朋友的救赎;“他被人称道的品质就是诚信,他的商品从不掺假”“他白手起家,成为身价不菲的富商”,他赢回了尊严、不辜负他人的信任赢得了人生的成功。【解析】31.本题考查文章内容的理解。A. 据第②段“一天,在一个建筑工地上,他看到了中学同学蚊子——朱德文。蚊子是工地上的一个小包工头,就安排他当了一个力工,吃住都在工地上”可知,他是无意中碰见朱德文,此项“走投无路的他只能到工地去找曾经的好友朱德文安排工作”说法有误;B.据第④段“他不时地盯看蚊子的包,那些藏在心底的‘恶’蠢蠢欲动起来。在监狱里改造了几年,他以为那些‘恶’已经连根拔除了,没想到它们还在,偷偷地生长着,使他的灵魂扭曲变形”可知,这是他对自己邪恶想法的看法,但最终他并未偷走那5000元,故“可以看出他内心的‘恶’并没有完全消除,使他的灵魂扭曲变形”说法有误;C.第⑤段“他现在太需要钱了,他想如果现在下手,蚊子没有防备,会很容易得手。于是他又给蚊子开了一瓶酒,想让蚊子醉得彻底些,那样他的成功率会更高”可知,他打算趁机拿走钱,但并不是“趁机抢走钱”,此项有误;D.据第⑫段“那一夜,我故意装作睡得很死,其实你的每个动作我都知道。这场赌博,我赢了——不,那一晚没有输家,咱们两个都赢了”可知,此项表述正确;故选D。32.本题考查环境描写的作用和心理描写。“当东方云层缝隙中射出几线柔光的时候”是环境描写,环境描写的作用可以烘托人物心理。“东方云层缝隙中射出几线柔光”就如同他的内心射入了一缕阳光,让恶念消除。据“他把包递给了蚊子,顿时感到莫大的轻松”可知,看着“东方云层缝隙中射出几线柔光”,天亮了,他可以松一口气了。注意此题写你是文中的“他”,写我的心理活动,要用第一人称叙述,要写出他一晚纠结后战胜恶念后的放松。本题为开放题,答案不唯一,言之成理即可。示例:终于快天亮了,太累了,差点就要犯了大错。幸好没有被恶念操控,没有让唯一信任我的人寒心,终于可以松一口气了,以后我也要诚信做人。33.本题考查人物形象分析。据第②段“蚊子是工地上的一个小包工头,就安排他当了一个力工,吃住都在工地上”和第⑩段“他终究没有拿走那5千块钱”可知,“他”经历了人生起伏,曾经犯事进过监狱,但没有好吃懒做,打工挣钱,抵住心里的恶念,没有拿走钱,说明他浪子回头,知错能改。据第⑩段“他终究没有拿走那5千块钱。当东方云层缝隙中射出几线柔光的时候,他把包递给了蚊子,顿时感到莫大的轻松”可知,最终没有辜负朋友信任,战胜自己心魔,没有拿走钱。因此,他心存善念,靠自己努力赢得尊严。据第⑪段“10年后,他白手起家,成为身价不菲的富商,经历可谓传奇,他的事迹常常是当地报纸的新闻。他被人称道的品质就是诚信,他的商品从不掺假”可知,靠诚信经营,成为身价不菲的富商,不避讳讲出自己的曾经,说明他坦诚,诚信,上进。34.本题考查关键语句的理解。文章第⑫段朱德文的话是从另一个角度叙述这件事。从朱德文的角度思考,他“故意装作睡得很死,其实你的每个动作我都知道”的原因有两个。一来,他看到了他眼中的贪婪,认为钱甚至生命都处于危险之中,为了自身的安全考虑,故意装作睡得很死;其次,他是朱德文的同学,虽然曾经犯事,但也希望通过这个赌能救赎他,给他一个改过自新的机会。赌赢了,自己保住了钱和人身安全,也拯救了他,赌输了也只是丢了钱,但保住了人身安全。35.本题考查文本内容的理解。据第⑧段“蚊子竟然把钱包塞到了他的怀里”“依然鼾声如雷,睡得香甜”及第⑫段“那一夜,我故意装作睡得很死,其实你的每个动作我都知道。这场赌博,我赢了”可知,蚊子用装睡,把钱交给他保管的方式,赢回了5000元钱和生命,还有对朋友的救赎;据第⑪段“他被人称道的品质就是诚信,他的商品从不掺假”和“他白手起家,成为身价不菲的富商”可知,战胜了内心的恶念,他赢回了尊严,赢回了朋友的信任,还有以后做生意不辜负他人的信任,最终赢得了人生的成功。九、阅读下面的文章,完成下面小题。老妈是个菜贩子邹华卫①许朵的叛逆,是从厌烦“菜贩子老妈”开始的。②读初中时,许朵私下里叫老妈“卖菜的”,嫌她不似同学牟卉卉的老妈,举手投足,一股文化人的范儿。一想到卖菜的,许朵就巴不得离老妈远远的。③不过许朵老妈的菜摊,经营得真不错。她做买卖诚实,人又热情活络,很快就从菜摊发展到菜店,后来拥有十多家连锁店。但在许朵看来,老妈骨子里是暴发户,不管老妈的职业还是素养,都不值得敬重,以至于坚决要让自己和老妈的人生拉开距离。④读到高中,许朵坚决要求住校。老妈不同意,说:“牟卉卉跟你一个班,人家咋不住校?”许朵“哧”地冷笑一下,压根不想服从。最后老妈拗不过许朵,只好依她住校。但让许朵没有想到的是,开学时,老妈不仅自作主张执意开车送她到学校,还一见如故地和班主任聊上了。她那卖菜的大嗓门一嚷嚷,引得同学们纷纷侧目。于是开学当天,全班都认识了许朵——她妈是卖菜的,是家喻户晓的“诚信菜店”的老板娘。许朵简直沮丧透了,恨不得找个地缝钻进去。天阴沉沉的,闷得让人透不过气来,路边的小树也垂着枝条,没有一丝生气。坚持住校就是想离老妈远一点儿,可是一天没过,又被罩在她的影子里了。⑤因为卖菜的老妈,许朵不由自主地在同学面前显出自卑,甚至承受不了一些善意的玩笑。在大家聊起老爸老妈的时候,她便找借口溜出去。由此,许朵对老妈的忌恨又多了一重。⑥而老妈还是老妈,又跟学校管后勤的校长扯上,零利润为学校食堂送菜。老妈得意洋洋地对许朵说:“你们校领导说了,你吃饭不用花钱,想吃啥就去吃啥。”⑦许朵简直要崩溃了,但这次,她默然不语,只是坚决不肯再去食堂,而宁愿去学校门口那些不卫生的廉价小食店吃。结果没几天就吃坏了肚子,患了急性肠炎。⑧至此老妈终于知道许朵的抗议行为,她又气恼又意外,哆嗦着嘴唇,抬起手掌,可看着许朵苍白的小脸儿,还是无力地放了下来,叹口气,转身走了。这样的抗争之后,许朵觉得再多花一分钱就是对老妈的妥协与认输。她分外努力,只想远远地逃离老妈。⑨可到底是长身体的时候,功课越来越紧张,到了高二上学期,许朵明显出现营养跟不上的疲累。恰在这个时候,牟卉卉的老妈找到许朵,以一个母亲的急切央求道:“朵朵,卉卉也住校了,可她不许我中午送饭,这样营养怎么能跟得上呢?阿姨求你,跟她做伴儿吃饭好不好,卉卉说了,只要你同意我给你们俩送饭,她就让我送。”⑩许朵过意不去,可看着牟卉卉老妈祈求的眼神,她还是点了头。许朵想,这才是爱孩子且有素养的妈妈,不像自己老妈,是个卖菜的,永远不考虑后果,总把事情做得那样浅薄。⑪于是,牟卉卉老妈每天中午都送来丰盛的午餐,让许朵有了足够的体能,加上要远离老妈的动力,许朵的状态越来越好,高考时发挥出超常水平,可以随心所欲地挑选一所远离家乡的一流大学。⑫接到通知书的那天,许朵很兴奋,去答谢牟卉卉老妈。没想到牟卉卉老妈微笑着揽过许朵的肩膀说道:“朵朵呀,你真把你老妈逼急了,就为你的吃,来求我求卉卉想了这个法子。孩子你想想,卖菜怎么了,靠自己劳动把生意做得这么大,不值得你自豪吗……”⑬这些话,一句一句戳在许朵心窝里。⑭话语如蚕,心似桑叶。⑮真的,老妈有什么错,她不偷不抢靠自己的双手,辛勤劳动赚血汗钱,过好自己的生活,竭尽全力爱自己的孩子……⑯许朵简直想给自己一耳光。⑰那天下午,许朵第一次去了老妈的菜店,看老妈正弯着已然有些佝偻的身子,跟员工一起码菜,菜在老妈的手里是那样乖巧,服服帖帖地任由老妈侍弄。许朵愣了愣,过去接过老妈手里的菜,轻声说:“妈,你歇着去,我来吧。”(选自《意林》2022年第3期,有删改)【活动一:梳理情感的线索】36.通读全文,完成以下框架图,梳理许朵对老妈的情感变化过程。读初中时,嫌弃老妈→读到高中,①_________→高考之后,②_________【活动二:赏析语言的特点】37.从人物描写的角度赏析第⑧段画线语句的表达效果。她又气恼又意外,哆嗦着嘴唇,抬起手掌,可看着许朵苍白的小脸儿,还是无力地放了下来,叹口气,转身走了。【活动三:探究人物的形象】38.结合全文内容,试分析许朵老妈的人物形象。【活动四:领悟文章的主旨】39.简要分析许朵想要逃离老妈的原因,说说文章所要揭示的主旨。【答案】36.忌恨(怨恨)老妈 理解(心疼、敬重)老妈 37.动作描写、神态描写,生动地描绘出母亲得知许朵的抗议行为而感到气恼、意外和无奈,表现了母亲对女儿的担忧和心疼为下文母亲托卉卉妈妈给许朵送饭作铺垫。 38.母亲勤劳能干、心灵手巧,将菜摊发展为拥有十多家连锁店的菜店,菜服服帖帖任由她的侍弄母亲为人热情活络,母亲诚实经营,主动为学校食堂零利润送菜,她的店是家喻户晓的“诚信菜店”母亲关爱孩子,疼爱有方,托卉卉妈妈为孩子送饭,默默地关爱孩子。 39.许朵想逃离老妈,是因为她认为老妈所从事的卖菜职业低人一等对老妈从事的职业没有正确的认识,认为“不值得敬重”。这个故事揭示了“职业没有高低贵贱之分,只要靠自己的双手勤劳致富,都是美丽的,值得尊重的”这一主题。【解析】36.本题考查文章内容的理解与概括。划分层次,结合示例形式概括。第②段写读初中时,许朵厌烦(嫌弃、讨厌)老妈;根据题干提示“读到高中”,找到第④段“读到高中,许朵坚决要求住校”,第⑤段“许朵对老妈的忌恨又多了一重”,可知读到高中,忌恨老妈;根据题干提示“高考之后”,找到第⑪段“许朵的状态越来越好,高考时发挥出超常水平,可以随心所欲地挑选一所远离家乡的一流大学”,第⑫段“朵朵呀,你真把你老妈逼急了,就为你的吃,来求我求卉卉想了这个法子。孩子你想想,卖菜怎么了,靠自己劳动把生意做得这么大,不值得你自豪吗”,第⑰段“那天下午,许朵第一次去了老妈的菜店”“许朵愣了愣,过去接过老妈手里的菜,轻声说:‘妈,你歇着去,我来吧。’”等语句可知,许朵高考后从牟卉卉老妈那里得知其送饭原因,许朵理解了老妈,变得心疼、敬重老妈。37.本题考查从人物描写的角度赏析。“抬起手掌”“放了下来”是动作描写,“哆嗦着嘴唇”“叹口气”是神态描写,结合上文“她默然不语,只是坚决不肯再去食堂,而宁愿去学校门口那些不卫生的廉价小食店吃。结果没几天就吃坏了肚子,患了急性肠炎”“至此老妈终于知道许朵的抗议行为”可知,生动地描绘出母亲得知许朵的抗议行为而感到气恼、意外和无奈,“又气恼又意外”“看着许朵苍白的小脸儿,还是无力地放了下来,叹口气”表现了母亲对女儿的担忧和心疼。结构上,结合第⑫段“朵朵呀,你真把你老妈逼急了,就为你的吃,来求我求卉卉想了这个法子。孩子你想想,卖菜怎么了,靠自己劳动把生意做得这么大,不值得你自豪吗……”可知,为下文母亲托卉卉妈妈给许朵送饭作铺垫。38.本题考查分析人物形象。分析人物形象可从行为事件和人物描写等方面入手。结合第③段“许朵老妈的菜摊,经营得真不错。她做买卖诚实,人又热情活络,很快就从菜摊发展到菜店,后来拥有十多家连锁店”,第⑰段“菜在老妈的手里是那样乖巧,服服帖帖地任由老妈侍弄”可知,母亲勤劳能干、心灵手巧;结合第⑥段“而老妈还是老妈,又跟学校管后勤的校长扯上,零利润为学校食堂送菜”,第④段“她妈是卖菜的,是家喻户晓的‘诚信菜店’的老板娘”可知,母亲为人热情活络,诚实经营;结合第⑫段“朵朵呀,你真把你老妈逼急了,就为你的吃,来求我求卉卉想了这个法子。孩子你想想,卖菜怎么了,靠自己劳动把生意做得这么大,不值得你自豪吗……”可知,母亲关爱孩子,疼爱有方。39.本题考查主旨。根据第②段“读初中时,许朵私下里叫老妈‘卖菜的’”“一想到卖菜的,许朵就巴不得离老妈远远的”,第④段“开学时,老妈不仅自作主张执意开车送她到学校,还一见如故地和班主任聊上了。她那卖菜的大嗓门一嚷嚷,引得同学们纷纷侧目。于是开学当天,全班都认识了许朵——她妈是卖菜的,是家喻户晓的‘诚信菜店’的老板娘。许朵简直沮丧透了,恨不得找个地缝钻进去”等语句可知,许朵总想逃离老妈的原因是许朵认为老妈从事的工作“卖菜”低人一等,对卖菜职业有错误认识,认为“不值得敬重”;第⑫段牟卉卉老妈所说的话揭示主旨“孩子你想想,卖菜怎么了,靠自己劳动把生意做得这么大,不值得你自豪吗”,即职业不分高低贵贱,只要靠劳动致富,都是值得尊重、骄傲、赞美的。十、请阅读下面的文字,回答问题。桥曹春雷①一夜狂风骤雨,黎明时分天终于睛了。但她心里的狂风骤雨,还没停。②她眼前还浮现着昨日给学生们上最后一节课的情景。她缓缓地在黑板上写下最后两个字“再见”。仿佛用尽了全身力气。虚脱似的戳在那里。她不敢回头。怕一回头眼泪就汹涌而出。背后已经传来哭泣声。有女生的。也有男生的,都低低地。刻意压抑着。③她早就该告诉孩子们的,但一直拖到她要离开的前一天,她不知道该怎么跟孩子们说。这个班,从一年级到四年级都是她带的,她跟孩子们很亲。一年级时曾有孩子喊她老师时,错喊成妈妈。她也红着脸答应。④可今天。她要离开了。⑤正这样想着。村里的大喇叭响了:“老少爷们注意了啊,昨儿晚上下了一夜雨。河上的桥冲坏了,在家的男劳力都到河边去。把桥修好。今天娟子老师要到山外镇上的小学报到,可不能耽误了……”是村支书在广播。⑥桥被冲坏了?那是村里通往外面唯一的路。她想。也许是老天爷要留下我吧。这样想着,又呆立了一会儿。她还是慢慢背起行李,往河边去。⑦路上遇到很多村里人,都在往河边去。看到她都过来和她说会儿话。年轻点的喊她“娟子老师”。年纪大的喊她“妮儿”。柱子奶奶在柱子的搀扶下。也拄着拐杖颤巍巍地一步一挪地往河边去。她说:“奶奶,您去干吗啊,路上这么滑”奶奶说:“你就要走了。奶奶说啥也要送送你啊。”⑧她心里一酸。大学毕业后来到这里的四年时间里。她没少吃柱子奶奶家的饭。槐花婶、栓子大娘、宝山叔……村里五十多户人家。谁家的饭她没吃过呢?!学生们下午放学后。都抢着让她去自己家吃饭。有时连着好几天抢不到,还会闹情绪。⑨而今天。她却要离开了。⑩一路上,还有她的学生们簇拥在一起,等她。一个也不少。齐刷刷地都来了。见了她,也不多话,有孩子抢过她的行李,默默地往前走。她不知道该对孩子们说些什么。也只能默默地走。⑪河边上。已经有很多人了,似乎全村的人都来了。村支书奎山叔在指挥。河里的水都快要涌上河岸了。木桥斜塌着。十多个男人腰上拴着绳子,另一头系在岸边的树上,只穿着短裤在河里打木桩。⑫岸上的男人搬石头、抬木头;女人们抬一些细一点的木料。男孩子们也不闲着。递送一些大人们需要的物件:女孩子们则围在她身边。老人们也陪在她身边跟她聊天,不舍地说:以后常回来看看。这里是你的家。⑬是的。这里是她的家,从她第一次走进这座村子。她就认定的。她喜欢这里的山山水水,喜欢这里淳朴的“路不拾遗,夜不闭户”的民风。喜欢这里村民之间如亲人般那种氛围。她把这里当作她的“世外桃源”。⑭可现在。她要走了。这样想着。眼里就潮潮的。像是起了一层雾。⑮她又想起自己的父母。此时一定在家里乐呵呵地等着她。为了这一天。父母已经等了很久了。从她到这个村子教书开始父母就劝她回去。每次回家父母都苦口婆心:”我们年纪大了,需要照顾。你年龄也不小了在山村不好找对象。以后结婚有了孩子,教育也是问题……”⑯如今。她终于要离开这里了。⑰这时,河里的男人们把木桩都打好了,开始铺木板,但木板不够。岸上有好几个人在喊“我家有木板,去我家抬”!柱子奶奶说:“去我家,我家有。”奎山叔说:“那咋行?婶,那可是您的寿材啊!”柱子奶奶说:“寿材咋啦,以后再准备就是了。”⑱她的泪水,再也控制不住,如眼前的河水,汹涌起来。⑲……⑳四桥修好了。㉑柱子推着摩托车上了桥。是村支书安排的,要送她去镇上的学校报到。㉒她也踏上桥,缓缓向前走去,五十多米的桥,此时在她看来,那么长,仿佛一辈子都走不到尽头。她感觉得到背后无数道目光,都聚焦在自己身上,她不敢回头。㉓终于到了桥头。㉔她没有立刻走下桥。定定地站在那里,好一会儿……她突然转过身,大步往回走去。她要留在这个山村,做一座桥,让孩子们踏着,走出大山去!(选自《中国铁路文艺》2022年第11期)40.下列对小说相关内容和艺术特色的分析鉴赏,不正确的一项是( )A.第②段写孩子们刻意地压抑着哭泣声,体现出孩子们的不舍与懂事。B.第⑧段补叙孩子们热情邀请她到家里吃饭,侧面烘托出她深受孩子们的爱戴。C.她把村里当成自己的家,因为这里有淳朴的民风和视她为亲人的乡亲们。D.小说运用第三人称展开叙述,将故事娓娓道来,语言平实,感情真挚。41.请梳理小说情节,在表格中填入相应内容。42.小说开头的环境描写有什么作用?请联系全文简要分析。43.请赏析下面两段文字。(1)这样想着,又呆立了一会儿,她还是慢慢背起行李,往河边去。(从人物描写的角度)(2)她的泪水,再也控制不住,如眼前的河水,汹涌起来。(从加点词语表达效果的角度)44.请联系全文,谈谈小说以“桥”为题的妙处。【答案】40.B 41.(1)全村人都来到河边,大人、男孩子们修桥,老人、女孩子们陪她说话。(2)桥修好后,她走上桥,又走回去,决定留在村里做孩子们走出大山的桥。 42.(1)一夜的狂风骤雨,营造出她临走前压抑的氛围,烘托出她不舍又无奈的矛盾心理。(2)为下文写雨后桥被冲坏、全村的乡亲们赶来修桥,以便她能及时去镇上学校报到作铺垫。 43.(1)运用神态和动作描写,生动形象地写出她对于离开孩子们、离开村子的矛盾心理和不舍之情。(2)“汹涌”原意是指水势翻腾上涌,形容波浪又大又急,这里形容她的泪水之多,表现出她对村里人为她付出的感动之情。 44.(1)“桥”作为贯穿全文的线索,使文章脉络清晰,结构浑然一体。(2)“桥”含义丰富,既指乡亲们修好被雨水冲坏的桥,又指她甘做孩子们走出大山的桥。(3)“桥”饱含乡亲们对她的感激和爱戴之情,也表达出作者对她扎根山村,教书育人的奉献精神的赞美。【解析】40.本题考查内容理解。B.第⑧段暂时中断事件的叙述,插入她对孩子们热情邀请她到家里吃饭的事情的回忆,属于记叙顺序中的插叙。B选项中的“补叙”表述不正确;故选B。41.本题考查梳理故事情节。已知信息:“她要离开村里,可一夜的雨冲坏了村里通往外面的桥”对应第⑤段“村里的大喇叭响了:老少爷们注意了啊,昨儿晚上下了一夜雨。河上的桥冲坏了,在家的男劳力都到河边去。把桥修好。今天娟子老师要到山外镇上的小学报到,可不能耽误了……”;“铺桥的木板不够,柱子奶奶坚持用自己的寿材板铺桥”对应第⑰段“河里的男人们把木桩都打好了,开始铺木板,但木板不够”“柱子奶奶说:‘去我家,我家有。’奎山叔说:‘那咋行?婶,那可是您的寿材啊!’柱子奶奶说:‘’寿材咋啦,以后再准备就是了”;第(1)空:综合以上信息,结合第⑦段“路上遇到很多村里人,都在往河边去”,第⑫段“上的男人搬石头、抬木头;女人们抬一些细一点的木料。男孩子们也不闲着。递送一些大人们需要的物件:女孩子们则围在她身边。老人们也陪在她身边跟她聊天”,可知故事的发展可概括为:全村人都来到河边,大人、男孩子们修桥,老人、女孩子们陪她说话;第(2)空:结合第㉒段“她也踏上桥,缓缓向前走去,五十多米的桥,此时在她看来,那么长,仿佛一辈子都走不到尽头”,第㉔段“她突然转过身,大步往回走去。她要留在这个山村,做一座桥,让孩子们踏着,走出大山去”,可知故事的结局可概括为:桥修好后,她走上桥,又走回去,决定留在村里做孩子们走出大山的桥。42.本题考查环境描写的作用。“一夜的狂风骤雨”表明风大雨急,渲染了她临走前压抑的氛围,结合文章内容,可知她对于离开村子有万般不舍,而家中父母正等着她回去,“狂风骤雨”烘托出她不舍又无奈的矛盾心理。正是“一夜的狂风骤雨”,使村里通往村外的桥被冲坏,根据第⑤段“。河上的桥冲坏了,在家的男劳力都到河边去。把桥修好。今天娟子老师要到山外镇上的小学报到,可不能耽误了……”可知,环境描写也为下文写全村的乡亲们赶来修桥,以便她能及时去镇上学校报到作铺垫。43.本题考查句子赏析。(1)句中“呆立”是神态描写,“慢慢背起”是动作描写,神态和动作描写,结合前句“她想。也许是老天爷要留下我吧”可知,此句生动形象地写出她对于离开孩子们、离开村子的矛盾心理和不舍之情。(2)“汹涌”本意是指水势很大,猛烈地向上涌,这里用来形容泪水,突出泪水之多,当铺桥的木板不够时,人们纷纷喊去自己家取,柱子奶奶甚至坚持用自己的棺材板铺桥,村人们的行为让她深受感动,以至于泪水“汹涌”起来,表达了她对村里人的感激。44.本题考查标题作用。小说从雨水冲坏通往村外的桥写起,然后叙写村里人纷纷出力修桥,最后桥修好后,她决定留在村里做孩子们离开大山的桥,可以说,小说围绕“桥”展开,“桥”是贯穿全文的线索。标题中的“桥”既指乡亲们修好被雨水冲坏的桥,根据第㉔段“她要留在这个山村,做一座桥,让孩子们踏着,走出大山去”可知,又指她甘做孩子们走出大山的桥,含义丰富。乡亲们修好桥,不耽误她到山外镇上学校报到,体现出乡亲们对她的感激和爱戴之情。文章结尾“她要留在这个山村,做一座桥,让孩子们踏着,走出大山去”可知,她做出留在村里的决定,深化了主题,表达出对她扎根山村,教书育人的奉献精神的赞美之情。八、散文阅读 一、阅读下面的文章,完成后面的小题。一碗“雪花面”马海霞①那年深冬,雪下得不大,但天气却极寒,北风吹在脸上刀割一样疼。母亲去外婆家了,由我负责午饭。家里有白菜、豆腐,还有挂面。我炖了一锅白菜豆腐汤,掐着父亲快下班的点儿,煮上面条。我平时不咋做饭,煮面条时放多了挂面,煮了满满一大锅。面条剩下,坨了就不好吃了,母亲回来若看到,肯定劈头盖脸数落我一顿。②发愁时,看到修鞋的瘦大叔又来出摊了。不如将多余的面条送他,他肯定不会嫌弃。我找个大碗,盛了满满一碗面条,端到瘦大叔面前。瘦大叔放下手中的活计,客气地推却。但我哪里容得他拒绝,直接将面条放在他的三轮车上,边往回走边说:“吃吧,吃完了我再给您盛,家里还有呢。”③正说着,父亲骑车回来了,我低声对父亲说:“面条煮多了,咱俩肯定吃不了,与其到下午坨得不能吃了,不如送给那位修鞋的吃……”父亲听我说到这里,忙折回去请瘦大叔来家里吃,说外面天冷,风又大,凉风灌热气的,吃了不舒服。瘦大叔笑着说:“没事的,我天天在外面吃饭习惯了。中午下班时间来取鞋子的多,我不能离开鞋摊。”④父亲见他这么说,转身回家,让我把小桌子搬到外面,自己则盛了一碗面条,盛了一盘菜,他要和瘦大叔一起在街边吃。父亲这是犯了哪门子邪,这么冷的天,要和瘦大叔在街边吃饭?父亲不理我,把酒瓶装左口袋里,酒盅装右口袋里,一手端面一手端菜,径直出了家门,我只好搬着小桌子紧随其后。⑤瘦大叔见父亲非要和他喝两盅,也不好再拒绝。那天父亲和瘦大叔一边吃面条一边碰杯喝酒,雪花飘落在他俩身上,脸上,饭碗里、酒盅里,两人依然吃得开心,喝得尽兴。不知道的,还以为他俩是多年不见的旧友呢。⑥酒足饭饱,父亲又陪着瘦大叔聊了一会儿,直聊到要上班了,才返回家中。晚上父亲下班后,我又问起父亲中午在外面与瘦大叔喝酒吃饭的事儿,父亲打开了话匣子,说了一段往事。⑦父亲年轻时推着小车去二百里远的外县买石灰,走时奶奶给父亲准备了两顿饭的干粮和水,等买上石灰,往回赶时,却发现干粮袋和水壶都忘在了石灰厂,已经走出五里路,推着车子回去取是不现实了,半路上父亲又饿又渴,便敲开一户人家找水喝,男主人出来,听说父亲将干粮和水都弄丢了,忙邀父亲到家里吃饭。⑧父亲落座,女主人给父亲盛了一碗粥,又递给父亲一个菜窝窝。说是粥,其实里面没有几粒米,菜窝窝也是野菜多,玉米面少。女主人说,他们这里连续两年遭了水灾,只能请父亲吃这些,很是过意不去。⑨父亲说,那顿饭虽然吃得不好,但受到了客人的待遇,因为那一家人也是喝稀粥,吃菜窝窝。这让父亲很感动,若人家递给父亲一个菜窝窝让他站在外面吃,父亲便有种乞讨的感觉。⑩那顿饭吃得父亲肚子里热乎乎的,这么多年一直没忘记。⑪父亲的意思我明白了,其实我送修鞋的瘦大叔面条,初衷却是让他帮忙消灭剩饭,送给人家时也是一副大善人的模样。其实瘦大叔肯定带了午饭,也不缺我这一碗面。父亲就不同了。他是把瘦大叔当亲人当朋友,陪他在风雪中吃一碗面是情义。⑫后来,父亲和瘦大叔成了朋友。瘦大叔说,他来我们这里出摊,是他下岗后的第一份职业,说实话刚开始干,有点儿磨不开面子,怕被人瞧不起,心情也非常低落。但那天天那么冷,父亲还陪他在外面喝酒吃饭受冻,让他非常感动。那碗面让他吃得热气腾腾,寒意全无。⑬父亲说得对,一碗“雪花”面,有了情义便有了温度。1.结合上下文,按照提示,品析文中划线语句。(1)那天,是入冬后的第一场雪,雪下得不大,但天气却极寒,北风吹在脸上刀割一样疼。(从修辞角度品析句子表达效果)(2)但我哪里容得他拒绝,直接将面条放在他的三轮车上,边往回走边说:“吃吧,吃完了我给您盛,家里还有呢。”(分析加点词所表现的人物心理)2.从记叙顺序的角度,分析第⑦一⑩段的作用。3.文中父亲这一形象很鲜明,结合文章内容分析父亲的形象。4.本文以“一碗‘雪花面’”为题好在哪里?【答案】1.(1)运用比喻的修辞手法,形象生动地写出了天气的寒冷刺骨,为下文父亲请瘦大叔来家里吃面做铺垫。(2)“直接”和“吃吧”有命令意味,缺少对长辈应有的尊重,貌似真诚热情,实际表现了“我”既急于将面送出又要显出大善人的模样,而忽视了瘦大叔意愿的心理。 2.运用插叙,对父亲主动陪瘦大叔吃面的原因做了补充说明,丰富了文章内涵。 3.从父亲因天冷风寒邀请瘦大叔来家里吃面,和后来陪瘦大叔在街上一块儿喝酒吃面,可以看出父亲是一个真诚善良,懂得体谅他人、尊重他人的人。 4.示例:标题概括了文章主要内容,有设置悬念,吸引读者的作用,是本文的叙事线索,“雪花”衬托出父亲带给瘦大叔的温暖,揭示了尊重受助者的帮助才能算作善举的主题。【解析】1.本题考查句子赏析。(1)根据题干要求,这个句子需要从修辞的角度进行赏析。“北风吹在脸上刀割一样疼”一句把北风吹在脸上比作像刀割一样疼,由此可判定这个句子运用了比喻的修辞手法。用刀割的感觉来形容北风吹的感觉,由此可看出北风的寒冷刺骨。结合第③段“父亲听我说到这里,忙折回去请瘦大叔来家里吃,说外面天冷,风又大,凉风灌热气的,吃了不舒服”可知因为天冷,父亲才会邀请瘦大叔来家里吃面,因此这个句子还为下文父亲请瘦大叔来家里吃面做铺垫。(2)根据题干要求,本题需从词语的角度进行赏析。此句子主要描写的是“我”给瘦大叔多煮的面时不容拒绝的样子,“直接”叫瘦大叔吃,也没问瘦大叔是否愿意接受。再结合后文“但我哪里容得他拒绝,直接将面条放在他的三轮车上,边往回走边说:‘吃吧’”可看出“直接”和“吃吧”二字带有不可拒绝的命令意味,缺乏对长辈应有的尊重,只是为了把多余的面条送出去,体现了“我”并没有真正估计大叔的内心感受和情况,与父亲的表现有鲜明对比,突出帮人的同时更要维护对方尊严,不能盛气凌人。2.本题考查插叙及作用。⑦一⑩段记叙了父亲年轻时去外县买石灰,将干粮和水都弄丢了,在又饿又渴时,受到了一户人家的邀请进屋吃饭。解释了上文为什么父亲主动陪瘦大叔在雪中吃面,丰富了文章内涵,也进一步突出了文章主旨。3.本题考查人物形象的分析概括。根据第③段中的“父亲听我说到这里,忙折回去请瘦大叔来家里吃,说外面天冷,风又大,凉风灌热气的,吃了不舒服”和第⑤段中的“那天父亲和瘦大叔一边吃面条一边碰杯喝酒,雪花飘落在他俩身上、脸上,饭碗里、酒盅里,两人依然吃得开心,喝得尽兴”可知,他是一个善良,有情义,懂得体谅并尊重别人的人;根据第⑩段中的“那顿饭吃得父亲肚子里热乎乎的,这么多年一直没忘记”可知他是一个懂得感恩的人。根据第⑪段“父亲的意思我明白了,其实我送修鞋的瘦大叔面条,初衷却是让他帮忙消灭剩饭,送给人家时也是一副大善人的模样。其实瘦大叔肯定带了午饭,也不缺我这一碗面。父亲就不同了。他是把瘦大叔当亲人当朋友,陪他在风雪中吃一碗面是情义”可知这是父亲借机对“我”进行教育,让“我”明白情义的重要,可知父亲是一个善于教育子女的人。4.考查标题的含义和作用。首先看看标题是否用了特殊手法来吸引读者,“雪花面”到底是一种什么样的面,有设置悬念,吸引读者的作用;其次,结合文章内容可知,“雪花面”是“我”给瘦大叔的面,更是父亲在雪花飘落街边陪瘦大叔吃的面,故标题有概括文章主要内容的作用,但又不仅仅是面,这更是父亲对他的情谊、善良、温暖,以及对他的尊重,是文章的主旨所在,由此可知“雪花”衬托了善意的暖,暗示了主旨。紧接着想一下结构的作用,文章围绕“面”来写,提示了文章内容,又是文章的线索。二、阅读下面的文章,完成后面的小题。听沙丘生长的声音刘妍①哨所编号是个数字、是个标识,就如人的名字一样,但这个位于边境线上被沙丘全包围的哨所,却永远刻在我的心里。这个被大大小小的沙丘包围着的哨所,出门是沙、开窗是沙、睁眼是沙、张嘴是沙,目之所及、脚之所触,全是沙。不远处的萨吾尔山连绵不断,向无尽的远方延伸。②沙丘的沙子非常细腻,细到可以忽略。即便穿了防沙鞋套,也不知道善于“钻营”的沙子是如何实现“见缝插针”的。不一会儿,鞋里的沙粒隔着袜子也能感觉到。在所内,扫沙子比扫雪频繁、密集、时间长。扫雪只是扫一个冬天而已,扫沙子则是扫四季,春夏秋冬不停歇!院子里的沙子不断地被扫、被人移位至院外。而风,却极有恒心,它有不同的面具。时而温柔、时而凶猛、时而沉默。时而发怒,情绪状态稳定或不稳定,始终不忘夹杂着沙子攻进院子里。③沙子多了,多到有资本大大小小、高高低低、起起伏伏。沙子更知晓“团结力量大”的简单道理,它们喜欢聚拢抱团,塑造成沙丘的模样。沙丘随风移动,形态随风而变。风之精灵,随意地调侃着沙丘。④新兵马兰在工作之余,除了爱敷个面膜,就是爱对着远处的沙丘发呆,静静地听沙丘“滋滋滋”生长的声音。马兰觉得沙丘在生长,自己也在成长。她在伊犁师范大学毕业后,去年11月始,经过相关系列培训,正式成为一名光荣的护边员。第一次巡边时,她偷偷地盯着防护网看,“这世界这么多人,全国各族同胞,只有我,此时此刻能近乎零距离地接近此地。”独一无二的身份和位置,让小姑娘脸上和眼里写满了傲娇。⑤内心的傲娇很快被一次小小的事情击碎——去年12月的极寒天气,巡逻时刚好遇上生理期,她真正感受到了平常事的不同寻常。面对人生的第一次,如此这番遭遇,哭笑不得,无法分辨眼里涌出的是哭泪,还是笑泪?也是从那一刻起,她仿佛明白了作为一个女兵所需要走的路。一个月后回家,妈妈发现宝贝女儿瘦了9公斤。可怜天下父母心,妈妈和爸爸的眼里全是怜爱!可马兰却偷着乐,暗自庆幸,终于找到一条行之有效的减肥之道。⑥听沙丘生长的声音,有乐趣、有想象的空间,可双脚踏在沙丘上前行,就比水泥路上费劲多了——马兰日常巡逻,需要来来回回地走,她恨不得将全身的力气使在沙丘上……如此这般,才能提高前进的速度。沙丘上除了有人的两排脚印,还有独狼和野猪的脚印。⑦常言道:怕什么,来什么!有一次晚上执行巡逻任务,突然,听到后方有“沙沙沙”的声音,回头看,没有发现异常﹔再次回头,两米外的黑暗处,竟然有双寒中带冷的眼睛━—毫无悬念,独狼的眼睛!马兰下意识地举起手电筒直射过去,幸运的是,独狼只是狠狠地盯了马兰5秒钟便转身离去。我问马兰:“怕狼扑吗?”她的回答是肯定的,但也会用教练培训课上教的方法应急。“狼的弱点在腰部,头部超硬。”马兰说,“用护具勒紧狼的腰部。”我听了,不以为然!很显然,马兰还没有实战过,狼的獠牙凶狠极了,可不是一般人能应付的。突然间,我内心开始有点为马兰担心……没想到,马兰却觉得这是小题大做,对我的过度反应发出了“哈哈哈”的爽朗笑声。⑧马兰读的本科专业是汉语言文学,语言表达能力较强,性格很爽朗。她笑起来声音如铃铛般,常常回响在我的耳边。她说,每次执行任务只留两只眼睛,全身裹得严严实实,面膜一年四季省不得。她说完,又是一阵爽朗的笑声。“能坚持干下去吗?”我问。“这里的冬季感觉很不一般,但更加期待春、夏、秋季。”马兰回答道,“我会一直把护边员这份光荣而神圣的职责坚守下去。”上个月还在为生活琐事掉眼泪的马兰,这个月已找到了解决问题的常道,已有了巾帼不让须眉的那份豪迈和勇气!⑨沙丘随风而移动,无须愚公再世,沙山也会变沙丘,时刻被乾坤大挪移。沙丘是人的镜像,听沙丘生长的声音,仿佛听到了马兰成长的声音。(有删改)5.文中写了与马兰巡逻相关的哪些事?请简要概括。6.文章①至③段写了沙子的哪些特点?作者写这些内容的用意是什么?联系全文简要分析。7.请从修辞的角度,说说第②段画波浪线句子的表达效果。而风,却极有恒心,它有不同的面具,时而温柔、时而凶猛、时而沉默、时而发怒,情绪状态稳定或不稳定,始终不忘夹杂着沙子攻进院子里。8.第⑧段画横线句中加点的“又”“爽朗”,表现了人物的什么特点?结合文章内容简要分析。9.“镜像”在现代汉语词典中有以下两个义项,你认为第⑨段中加点的“镜像”应选择哪个?结合文章内容简要分析你的依据。①物体在镜子中的图像。②一种计算机备份,硬盘、网站等都可以做镜像,因像照镜子一样而得名。【答案】5.第一次巡边,为自己独一无二的身份和位置感到骄傲;极寒天气巡逻,遇上生理期非常尴尬,眼里涌出泪花;日常巡逻,在沙丘上行走艰难;有一次晚上巡逻,遇到独狼,毫不在意;沙丘上干燥需要包严实、经常敷面膜,不以为意。(含任四件即可) 6.无处不在,无时不有,非常之多(至少答出两个不同的特点);写出了马兰护边环境的恶劣,突出了马兰的乐观、无畏的特点。 7.用了拟人、排比的修辞,生动形象,语势强烈,写出了风变化多端,一年四季将沙吹进哨所院子的特点。 8.表现了马兰乐观、开朗、直爽的性格特点,虽然巡边环境恶劣,她却坦然面对,对自己爱美之心也毫不掩饰 9.我选义项①。沙丘随风移动,会生长;从沙丘身上看到了马兰的影子,马兰新入伍,且是女兵,在自然条件恶劣的边疆,会遇到许多新问题,她先是掉眼泪,慢慢找到了解决问题的方法,并有了不服输的勇气与豪迈,慢慢成长起来(写出变化)。【解析】5.本题考查内容理解和概括。结合第④段“她在伊犁师范大学毕业后,去年11月始,经过相关系列培训,正式成为一名光荣的护边员。第一次巡边时,她偷偷地盯着防护网看,‘这世界这么多人,全国各族同胞,只有我,此时此刻能近乎零距离地接近此地。’独一无二的身份和位置,让小姑娘脸上和眼里写满了傲娇”可概括为:第一次巡边,为自己独一无二的身份和位置感到骄傲。结合第⑤段“内心的傲娇很快被一次小小的事情击碎——去年12月的极寒天气,巡逻时刚好遇上生理期,她真正感受到了平常事的不同寻常。面对人生的第一次,如此这番遭遇,哭笑不得,无法分辨眼里涌出的是哭泪,还是笑泪”可概括为:极寒天气巡逻,遇上生理期非常尴尬,眼里涌出泪花。结合第⑥段“听沙丘生长的声音,有乐趣、有想象的空间,可双脚踏在沙丘上前行,就比水泥路上费劲多了——马兰日常巡逻,需要来来回回地走,她恨不得将全身的力气使在沙丘上”可概括为:日常巡逻,在沙丘上行走艰难。结合第⑧段“她说,每次执行任务只留两只眼睛,全身裹得严严实实,面膜一年四季省不得。她说完,又是一阵爽朗的笑声”可概括为:沙丘上干燥需要包严实、经常敷面膜,不以为意。6.本题考查内容的提炼概括。结合第①段“这个被大大小小的沙丘包围着的哨所,出门是沙、开窗是沙、睁眼是沙、张嘴是沙,目之所及、脚之所触,全是沙”,第②段“沙丘的沙子非常细腻,细到可以忽略。即便穿了防沙鞋套,也不知道善于‘钻营’的沙子是如何实现‘见缝插针’的。不一会儿,鞋里的沙粒隔着袜子也能感觉到。在所内,扫沙子比扫雪频繁、密集、时间长”,第③段“沙子多了,多到有资本大大小小、高高低低、起起伏伏”可知,沙子无处不在,无处不有,非常之多,以此写出了环境非常恶劣,但是马兰却不以为意,积极应对,体现了她的积极乐观和勇敢无畏。7.本题考查句子赏析。本句用了拟人和比喻的修辞手法,写风“极有恒心”“时而温柔、时而凶猛、时而沉默、时而发怒,情绪状态稳定或不稳定”“始终不忘夹杂着沙子攻进院子里”是用了拟人的手法,赋予了沙子以人的情感和动作,同时把沙子的不同变化比作“不同的面具”,是用了比喻的修辞手法,淋漓尽致地问写出 了风的变化多端,一年四季都会进入哨所院子的特点,非常生动形象,幽默风趣。8.本题考查语句分析。“又”体现了这不是马兰第一次发出爽朗的笑声,结合第⑦段“突然间,我内心开始有点为马兰担心……没想到,马兰却觉得这是小题大做,对我的过度反应发出了‘哈哈哈’的爽朗笑声”可知,面对恶劣的环境,面对旁人的担心,马兰并没有退缩抱怨,而是十分积极乐观,坦然面对。“爽朗”也体现了她性格十分开朗、直爽,毫不扭捏。9.本题考查内容分析。结合第⑨段“沙丘随风而移动,无须愚公再世,沙山也会变沙丘,时刻被乾坤大挪移。沙丘是人的镜像,听沙丘生长的声音,仿佛听到了马兰成长的声音”可知,沙丘随风移动,会生长,而从沙丘的身上也看到了马兰的影子。结合第⑤段“内心的傲娇很快被一次小小的事情击碎——去年12月的极寒天气,巡逻时刚好遇上生理期,她真正感受到了平常事的不同寻常。面对人生的第一次,如此这番遭遇,哭笑不得,无法分辨眼里涌出的是哭泪,还是笑泪”“可怜天下父母心,妈妈和爸爸的眼里全是怜爱!可马兰却偷着乐,暗自庆幸,终于找到一条行之有效的减肥之道”,第⑧段“上个月还在为生活琐事掉眼泪的马兰,这个月已找到了解决问题的常道,已有了巾帼不让须眉的那份豪迈和勇气”可知,马兰在自然条件恶劣的边疆,一开始也遇到了很多问题,也哭过,但是慢慢找到了解决方法,并且有了巾帼不让须眉的那份豪迈和勇气,慢慢成长了。据此分析选①。三、阅读下面的文章,完成下面小题。螺旋井张金豹①秦山以东几十里,有个掌平洼村。村外有口老井,井口是圆的,直径约18米,井台距水面26米。井壁上的台阶呈螺旋状,一圈一圈向下盘旋。台阶用乱石砌成,但严丝合缝,自成格局,仔细端详,有点像毕加索的画。台阶两侧有不规则的洞口,岩层中暗暗涌动着细如游丝的水,慢慢汇成细流,顺着洞口流进井里。当地人称之为“螺旋井”。②老井像一个历尽风霜的老人,盘坐在那里,闲看花开花落、雁去雁归,静听风声雨声、鸟叫蝉鸣。③我与这口老井相遇,纯属偶然。那天,我结束了一天的调研行程,在返城途中,见许多人围在那里游览拍照。我近前一看,不由眼前一亮,各种各样的水井我见过不少,但像这样的螺旋井,我还是第一次见到。④这时,一位大爷提着水桶,顺着石阶走下去,用水瓢将井水舀进桶里。老大爷家有自来水,但他说这井里的水甜,喝着舒服:“当年,全村几百口人豁出命来才把它打成,反过来,它又养活了全村几百口人。现在井虽然不怎么用了,但喝进肚里的水早已和身上的血混在一起,分不开了。”⑤攀谈中得知,这位人称“老榆木”的大爷,见证了老井的诞生。原来,干旱缺水一直是掌平洼村人的心病。这个村挂在半山坡上,村子周围不是峭壁就是山包。村民们种地吃饭全看老天的脸色。雨多点的年景,倒也说得过去,遇到大旱就麻烦了,轻则减收,重则绝产。到上世纪60年代,村党支部的成员们坐不住了,深感这个“家”当得有愧,当得无光。他们反复商量,提出在村子周围凿石打井。想法一传开,立刻引来议论纷纷,有的摇头,有的讥讽。村党支部书记是个硬汉子,他黑下脸:“闭眼难见三春景,出水才看两腿泥。不试怎么知道不行?”⑥他们请来水利部门的技术人员帮助寻找水源。技术员几乎跑遍了村周围的所有角落,最后选定了打井的位置。不过他说,地下水肯定有,但要打通岩石,下挖上百米,实在太难了。村党支部书记的话斩钉截铁、掷地有声:“为了吃上水,再难也得干!”⑦1967年,打井工程开工了,全村男女老少但凡有劳动能力的都上了工地。“老榆木”当时还是个毛头小伙,媳妇刚生儿子,他二话没说,拿着铁锤加入了打井的队伍。⑧那是个物质十分匮乏、生产力极其落后的年代,困难可想而知。没有挖掘机械,他们就靠锻刨锹挖,用铁锤钢钎凿石打眼,一锤下去,火星飞溅。日子一天天过去,锤头砸烂了,钢钎磨秃了,炸药用光了。怎么办?还是那句话,再难也得干!庄稼汉吐口唾沫是颗钉,洒滴汗水摔八瓣。一把把铁锤一根根钢钎,沾满了鲜红的血渍。一筐筐碎石渣,浸染着黏稠的血水和汗水。⑨就这样,一天又一天,一月又一月,一年又一年。⑩终于有一天,钢钎下面开始湿润,接着汩汩冒水,大家欣喜若狂,奔走相告,朝思暮盼的井—─打成了。⑪1967年动工,1977年竣工,用了整整10年时间,搬动土石3千多方。⑫属于老井的那个时代渐行渐远,许多人和事已化为烟云随风而去。但它留下了难以磨灭的印记,根还在,魂还在,凝结在它身上的精神永远不会消失。⑬“老榆木”很健谈,他看了看几个正在嬉戏的年轻人,意味深长地说:“这些在蜜罐里长大的孩子,只知道甜,哪知道苦﹖事不经过不知难。只有从那个时候过来的人才能体会,井水是甜的,汗水是咸的。”⑭此时,风过绿野,辽阔的天空碧蓝如洗,一团团棉絮一样的白云,轻盈地变换着不同形状。老井依然盘坐在那里,像一只天眼,凝望着风云变幻、岁月沧桑、人间百态,又似乎在向世人诉说着什么。10.村民凿石打井遇到了诸多困难,请根据提示,概括其挖井遇到的困难。(1)____________________(2)物质匮乏,生产力落后(3)____________________11.品味语言,回答下面问题。(1)结合语境,赏析第⑤段加点词语。他黑下脸:“闭眼难见三春景,出水才看两腿泥。不试怎么知道不行?”(2)结合语境,赏析第⑬段画线句。只有从那个时候过来的人才能体会,井水是甜的,汗水是咸的。12.结合全文,赏析文章第⑨段画线句的作用。就这样,一天又一天,一月又一月,一年又一年。13.文章结尾段写到,老井依然“盘坐”在那里,似乎向世人诉说着什么,请联系全文,分析结尾处“老井”向世人诉说的内容。【答案】10.示例:①地理条件恶劣,水源难找 ③工程量大,工期长 11.(1)示例:黑,意思是黑色,这里指脸色阴沉(沉下脸),神态描写,写出了支部书记面对别人嘲讽时的愤怒(生气),表达他想要带领大家凿石打井的坚定决心。(2)示例:“甜”写出了凿井成功的喜悦,“咸”写出了凿井过程的艰辛;两者形成对比,突出经过艰苦劳动后获得成功的喜悦感与自豪感。 12.示例:内容上表现了日积月累,一天天时间的流逝,用时间的漫长,表现出工程量大,工期长,突出凿石打井的难度大。也从侧面反映出村民们不放弃,坚持把井打下去的坚强决心。结构上,承上启下,既承接上文表现出的地理条件恶劣;物质匮乏工具简陋等困难,也表现了工程量巨大,为下文最终获得成功作铺垫。同时,表达了作者对掌平洼人自强不息、勇于担当、坚韧不拔精神的赞美之情。 13.示例:①先辈们当年凿石打井时的艰辛;②先辈们自强、担当、坚韧的精神;③社会的进步;④家乡的巨变;⑤生命的延续与探知;⑥文化的保护和传承。【解析】10.本题考查提炼概括信息。第一空:由第⑥段“他们请来水利部门的技术人员帮助寻找水源。技术员几乎跑遍了村周围的所有角落,最后选定了打井的位置……要打通岩石,下挖上百米,实在太难了”可概括出:地理条件恶劣;第二空:由第⑪段“1967年动工,1977年竣工,用了整整10年时间,搬动土石3千多方”可概括出:工程量大,工期长。11.①本题考查赏析词语。解答时先分析词语的本义,再结合手法分析语境义,然后再分析表达效果,最后点明情感。“他黑下脸”中的“黑”是对村党支部书记的神态描写。结合第⑤段“立刻引来议论纷纷,有的摇头,有的讥讽”可知,这一神态描写表现了村党支部书记面对别人嘲讽时内心的愤怒;结合第⑤段画线句“闭眼难见三春景,出水才看两腿泥。不试怎么知道不行”可知,这一神态描写还写出他想要带领大家凿石打井的想法,并一定要试一试的坚定决心。②本题考查赏析句子。解答时,要先明确赏析的角度,再结合内容进行分析。结合第⑬段画线句“井水是甜的,汗水是咸的”,可知将“甜”和“咸”进行了对比,运用了对比的修辞方法。“甜”既是“水”的味道,也是老一辈人内心的感觉。结合第(⑩段“终于有一天,钢钎下面开始湿润,接着汩汩冒水,大家欣喜若狂,奔走相告,朝思暮盼的井——打成了”可知,一个“甜”字,表现的是凿井成功的喜悦,“咸”字表面是汗水的味道,也包含凿井过程的艰辛。用井水的甜与汗水的咸作对比,突出老一辈人经过艰苦劳动后获得成功的自豪感。12.本题考查句段的作用。解答时,可从结构和内容两方面来分析。联系上下文语境分析即可。内容上:“就这样,一天又一天,一月又一月,一年又一年”在内容上表现了日积月累,一天天时间的流逝。结合第⑪段“1967年动工,1977年竣工,用了整整10年时间”分析,可知打井时间之漫长,表现出工程量大、工期长,也突出凿石打井的难度大;根据第⑧段“日子天天过去,锤头砸烂了,钢钎磨秃了,炸药用光了。怎么办?还是那句话,再难也得干!庄稼汉吐口唾沫是颗钉,洒滴汗水摔八瓣。一把把铁锤一根根钢钎,沾满了鲜红的血渍。一筐筐碎石渣,浸染着黏稠的血水和汗水”可知,突出凿石打井的艰难,也从侧面反映出村民们不放弃,坚持把井打下去的坚强决心;结构上,因为句子处于文章中间,所以起承上启下的作用,既承接上文第⑥段“技术员几乎跑遍了村周围的所有角落”表现出的地理条件恶劣、第⑧段“那是个物质十分匮乏、生产力极其落后的年代”的物质匮乏、第⑧段“没有挖掘机械,他们就靠锻刨锹挖,用铁锤钢钎凿石打眼,一锤下去,火星飞溅。日子一天天过去,锤头砸烂了,钢钎磨秃了,炸药用光了”表现出的工具简陋等困难,也表现了工程量巨大,为下文最终获得成功作铺垫。13.本题考查对内容主旨的理解。结合第④段“当年,全村几百口人豁出命来才把它打成,反过来,它又养活了全村几百口人。现在井虽然不怎么用了,但喝进肚里的水早已和身上的血混在一起,分不开了”可知,老井可能在诉说先辈们当年打井付出的艰辛;结合第⑥段“为了吃上水,再难也得干”可知,老井可能在诉说先辈们自强、担当、坚韧不拔的精神;结合第⑫段“属于老井的那个时代渐行渐远,许多人和事已化为烟云随风而去。但它留下了难以磨灭的印记,根还在,魂还在,凝结在它身上的精神永远不会消失”可知,老井可能在诉说社会的巨大进步,家乡的巨大变化;结合第⑬段“这些在蜜罐里长大的孩子,只知道甜,哪知道苦?事不经过不知难。只有从那个时候过来的人才能体会,井水是甜的,汗水是咸的”可知,老井可能在诉说文化的保护和传承。四、阅读下面的文章,完成后面的小题。村庄辞典叶剑秀①村字以木和寸组成,“木”为建材,“寸”意手持,整个字形表示用木材搭建定居的邑舍,意为人口聚集的自然屯落。“庄”意人口驻扎的聚居区。村与庄组合,我们便有了家。②村庄是一部厚重的辞典,记录着村庄里的沧桑巨变、风土人情、世故演绎……每个家庭是辞典里的词组或成语,每个人是辞典里的字符。③阅读村庄辞典,是需要毅力和耐心的,用一生的工夫,也未必能读完、读懂。④在村庄辞典的密解章节,从密密麻麻的文字缝隙里,仿佛可以窥见一丝隐秘。⑤打开村庄辞典,只要输入自己的名和姓,很快就知道了自己的全部。先祖何时何地迁入,家族历代纪事,人口亡故新增,成员有无荣华善举、劣迹丑闻,一目了然。每个人的生辰八字、前生造化、日月足迹、行为蜕变,记录得更加详尽。⑥每个人出生时,村庄辞典履职地给你打上一个印记。一块块不规则的图案打在臀部、腰间或胸腹,或大或小,或明或暗,那就是伴你终生的胎记。那胎记带着故乡村庄的秘方和味道,是你身世的二维码【甲】。无论你事业升腾、人生快意,或迷惘失落,都会寻到来时的路,不会迷失回家的方向。⑦村庄辞典的密解词条里,让我们隐约看到从不知晓的机密。每个人的脐带并未彻底剪断,留下一条肉眼看不见的隐形丝线,维系着与母体的链接,一种牵连和惦念永远植入我们的心怀。这或许就是那些离乡打拼、追梦路上的游子,在悲与喜的时光里,时不时冒出的一种情绪——乡愁。⑧引人入胜的是村庄辞典的旧词释义【乙】。比如炊烟:炊烟是村人用柴草、矸煤生火做饭时的呼吸,带着日子的苦难和无奈叹息,袅袅漂浮,弯曲如绵,最终绘成优美的图案,随风飘向天空。炊烟淡淡乡愁的喻义,如今已经消失,成为一种美好的怀念和永恒的记忆。比如饭市:饭市是村人相约在古树下聚餐的场所,各家端着粗茶淡饭,边吃边聊,聊尽天下大事,聊尽四季轮回,也聊家长里短,也聊猪马牛羊,也聊婚丧嫁娶。饭市是过去村人议论、传播、获取信息的聚散地,如今已被电视、手机等现代媒体替代。⑨阅读村庄辞典,结尾部分的村庄纪实,是这部典籍的核心,读来令人回味悠长。一个个平淡朴素的故事,刹那间激活消沉的因子,唤醒沦丧的传统美德。⑩阅读摘录:某年某月某日,夜深。村东刘家二梗暴病,家人疾呼,声传四邻,村人纷至沓来。老者急忙捆绑担架,村长挑拣壮实后生十六人,急送二梗奔往县城医院。三十里路,未曾停歇。蹭蹭的脚步声,溜地皮快步如飞。天亮,二梗命保。十六后生忍饥饿归,至半途,捧河水大饮,气爽。⑪某年荒春,南寨门侯家传噩耗,在外做处长的大儿子触犯法律,罪至极刑,被毙。年迈父母收尸,祈望葬埋故里。村邻不依不饶,义愤,众口拒。理由,村里保送他上大学,后步步晋升,多年来不曾回家一次,对爹娘不孝,对村人不敬,忘记亲情和背弃村庄的人,不容。老村长犯愁,思虑半天,烟袋锅敲在石碾上,脆响:他人孬,也得收留,要不他就成孤魂野鬼了,咱不能丢了祖上的本分。让他回来吧,西大坡荒凉地里,给他留个安魂的地方,活人咋能和死人一般见识。众人从,坟起,孤单成丘。⑫阅读至此,沉浸在故事的意境中,透过光阴的尘烟,仿佛看到老族长、老村长烟锅背后,一双双深邃的眼睛里,闪现着固有的村庄品性。这种高尚的品行,历久弥新,一代代在村里升华、延续。(选自《郑州日报》有改动)14.村庄是一部辞典,这部辞典里记录着哪些内容?15.有同学对第⑥段【甲】处和第⑧段【乙】处有疑问,请你解答。(1)“那胎记带着故乡村庄的秘方和味道,是你身世的二维码”中“二维码”在这里是什么意思?(2)有读者说“引人入胜的是村庄辞典的旧词释义”一句中的“旧词释义”用得不准确,请结合语境说说你的看法。16.本文语言极具表现力。请以下句为例,从词语和句式的角度分析语言的表达特点。三十里路,未曾停歇。蹭蹭的脚步声,溜地皮快步如飞。天亮,二梗命保。十六后生忍饥饿归,至半途,捧河水大饮,气爽。17.作者在最后说,一双双眼睛里闪现着村庄的品性。阅读村庄这部辞典,你读到了村庄的哪些品性?18.有人建议把本文题目改为“最美村庄”,请说说你的看法并阐明理由。【答案】14.(意近即可得分)示例:①个人和家族的身世历史。②村庄特有的地理环境、思想观念、文化特征等对人的影响。③游子对家的牵连和惦念。④村庄特有的现已消失的风物和习俗。⑤村庄平淡朴素的纪实故事。 15.①指一个人的身份标识,记载着这一个人独有的真实信息。(或答独有的印记、独有的标志也可)②示例:“旧词”是指村庄原有的物或事,“释义”是对村庄原有之物或事的解释,但是后面的内容是在说这些物或事的意义或现象已经不存在了,并不是在进行解释,所以不准确。(言之成理即可) 16.①多用短句。如“三十里路,未曾停歇”“天亮,二梗命保。”句式简短,叙事节奏明快。②用词形象。如“溜地皮快步如飞”,形象地写出了奔走的快速。③用词简练。如“大饮”“气爽”用词简练,写出了后生们猛喝水后非常舒服畅快的形象。④使用拟声词。“蹭蹭的脚步声”,生动地表现了脚步的迅疾。(答出三点即可) 17.友爱善良(友爱互助)、宽容仁慈、淳朴厚道、爱憎分明。(答出三点即可) 18.示例1:不能。本文题目为“村庄辞典”,辞典比喻村庄的记录作用,题目高度概括了全文的主要内容:村庄记录着每个人每个家族的身世历史、思想情感等,记录着村中的风俗、人情、故事……内涵厚重,与辞典的作用和特点吻合;运用修辞手法,新颖,吸引读者的阅读兴趣;“最美村庄”重在表现自然和人文之美,与本文的主要内容不符。(答出三点即可)示例2:可以改。本文写出了村里人的美好品行,所以可以改为最美村庄。【解析】14.本题考查对文章内容的梳理概括能力。此题根据提示确定相关语段,再以简洁语言进行概括。由第②段“每个家庭是辞典里的词组或成语,每个人是辞典里的字符”和第⑤段“打开村庄辞典,只要输入自己的名和姓,很快就知道了自己的全部。先祖何时何地迁入,家族历代纪事,人口亡故新增,成员有无荣华善举、劣迹丑闻,一目了然。每个人的生辰八字、前生造化、日月足迹、行为蜕变,记录得更加详尽”出概括出:个人和家族的身世历史。由第⑥段“每个人出生时,村庄辞典履职地给你打上一个印记”“无论你事业升腾、人生快意,或迷惘失落,都会寻到来时的路,不会迷失回家的方向”可概括出:村庄特有的地理环境、思想观念、文化特征等对人的影响。由第⑦段“每个人的脐带并未彻底剪断,留下一条肉眼看不见的隐形丝线,维系着与母体的链接,一种牵连和惦念永远植入我们的心怀。这或许就是那些离乡打拼、追梦路上的游子,在悲与喜的时光里,时不时冒出的一种情绪——乡愁”可概括出:游子对家的牵连和惦念。由第⑧段“引人入胜的是村庄辞典的旧词释义”“炊烟淡淡乡愁的喻义,如今已经消失,成为一种美好的怀念和永恒的记忆”“饭市是过去村人议论、传播、获取信息的聚散地,如今已被电视、手机等现代媒体替代”等语句可概括出:村庄特有的现已消失的风物和习俗。由第⑨段“阅读村庄辞典,结尾部分的村庄纪实,是这部典籍的核心,读来令人回味悠长。一个个平淡朴素的故事,刹那间激活消沉的因子,唤醒沦丧的传统美德”可概括出:村庄平淡朴素的纪实故事。15.本题考查对文章的理解能力。(1)理解词语要结合上下文内容进行,由第⑥段“每个人出生时,村庄辞典履职地给你打上一个印记。一块块不规则的图案打在臀部、腰间或胸腹,或大或小,或明或暗,那就是伴你终生的胎记”“无论你事业升腾、人生快意,或迷惘失落,都会寻到来时的路,不会迷失回家的方向”等语句可知,“二维码”在文中指一个人的身份标识,记载着这一个人独有的真实信息。(2)“旧词”是指村庄原有的物或事,“释义”是对村庄原有之物或事的解释,结合第⑧段的内容进行分析,由“炊烟淡淡乡愁的喻义,如今已经消失,成为一种美好的怀念和永恒的记忆”“饭市是过去村人议论、传播、获取信息的聚散地,已被电视、手机等现代媒体替代”等语句可知,文章并不是对该词的原有义进行解释,而是说这些物或事的意义或现象已经不存在了,因此“旧词释义”不准确。16.本题考查语句赏析能力。此题要求从词语和句式的角度着手。词语上:由“未曾停歇”“至半途”等语句可见,此句用语趋于文言,简洁准确;由“大饮”“气爽”等词语可知,用词简练,写出后生们喝水的豪饮状态和喝后舒服畅快的特征;还有“蹭蹭”拟声词,生动表现了脚步的迅疾,显得很有画面感。句式上:由“三十里路,未曾停歇”“天亮,二梗命保”“至半途,捧河水大饮,气爽”等语句可见,以短句为主,叙事节奏明快,很好地烘托出救人的紧张气氛和村民的豪爽气概。17.本题考查对文章的理解概括能力。由第⑩段“村东刘家二梗暴病,家人疾呼,声传四邻,村人纷至沓来。老者急忙捆绑担架,村长挑拣壮实后生十六人,急送二梗奔往县城医院”可知,全村深夜救人,体现了村人的善良、友爱、仁慈、助人。由第⑩段“十六后生忍饥饿归,至半途,捧河水大饮,气爽”可知,救人后忍饥饿归,体现了村人的淳朴厚道。由第⑪段“年迈父母收尸,祈望葬埋故里。村邻不依不饶,义愤,众口拒。理由,村里保送他上大学,后步步晋升,多年来不曾回家一次,对爹娘不孝,对村人不敬,忘记亲情和背弃村庄的人,不容”可知,对忘记亲情和背弃村庄的人感到义愤,体现了村人的爱憎分明。由第⑪段“给他留个安魂的地方,活人咋能和死人一般见识。众人从,坟起,孤单成丘”可知,收留忘记亲情和背弃村庄的人葬埋故里,体现了村人的宽容、厚道。据此回答即可。18.本题考查对标题的赏析。此题答案虽具开放性。首先要表明自己的观点态度,其次再阐明理由。如果倾向于使用“村庄辞典”的标题。理由可从全文围绕“村庄辞典”来写,“村庄辞典”即是文章线索,蕴含着对村庄的喜爱、赞美之情等方面来写。如果倾向于使用“最美村庄”的标题,则可从村里人的美好品行的角度阐明理由。言之有理即可。示例:不能。本文题目为“村庄辞典”,这是比喻的修辞,“辞典”的意思是记录包罗所有内容,这里就是记录村庄所有个人与家族的来龙去脉,点点滴滴,记录着村中的风俗、人情、故事……“村庄辞典”即是文章线索,全文围绕这个题目来写,也蕴含着作者对村庄的喜爱、赞美之情。而“最美村庄”重在表现自然和人文之美,与作者想表达的意图不一致。五、阅读下面的文字,完成下面小题。给自己个出路马宇龙①听闻田沛和他那条穿山而过的“愚公洞”,已经很久了。②在我的想象中,这位70多岁的老人,肯定要比一般的老年人看上去更羸弱,沧桑感更深更重。但当我在红河乡乡长的带领下,来到田赵村见到田沛时,我根本没认出他来。直到乡长介绍说这是田沛,我才回过神来——这么挺拔的身板,锐利的目光,洪亮的嗓音,哪里像个长年穿山挖洞的人?根本无法把他与我想象中的“愚公”联系起来。③“我挖这个洞子,就是给自己个出路!”④来见田沛,自然是为参观他几十年前挖的“愚公洞”。这条只凭田沛和妻子二人之力打出的“隧道”地处甘肃平凉泾川县红河乡田赵村。我钻进冰冷的洞口,感受到视觉、触觉和嗅觉带给我的奇幻感,像穿越了时光的隧道。⑤我们走得越深,来自地底下的凉意便越浓重,这种微妙的不真实感仿佛把我带回了1980年。我看到了32岁的田沛,浑身都是使不完的力气,想法多、点子多,想到就干,干就干到底。那一年,他突发奇想,跑到庆阳承包了3年苹果园。这件事现在看起来稀松平常,但在当时,土地承包到户才刚刚开始。⑥田赵村大部分土地位于岭岭子山梁南坡的台地上,大部分土地都在山顶上,山高坡陡,耕作条件很差。他的家和分到的田地之间横着一道山梁,他住在梁北,而田地在梁南,每天都要翻过岭岭子往返。岭岭子虽说不高,山前山后一个来回也要走10里羊肠小道,狭窄坡多,路上费时又危险。⑦1984年,田沛开工挖洞,一挖就是3年。开挖时,村里人得知消息,议论纷纷,劝停的,嘲笑的,什么人都有。反正就是一句话,都认为田沛夫妇瞎折腾。对此,田沛的态度是你说你的,我干我的。一路听着田沛生动的讲述,听着这些之前看过多少遍但毫无感觉的数据:全长198米,宽1.8米,高约2.3米,移动土方1000多立方米1600吨……⑧田沛清楚记得,那天是1987 年的端午节,随着他手中的镐头奋力一击,一束亮光透了进来,洞终于挖通,熬了上千个日夜的田沛终于在隧道的尽头看到了光线。那一声阳光透进来的轰响,撼动的又岂止是一片荒山坡,也让村民们板结的心思轰然洞开。⑨他有了出路,而这条路后来也成为全村人乃至全县人的一条出路。在田沛钻过山梁之后,这里的人们也开始寻求脱贫的路子,寻求一方新天地、一种新活法。从1991年到1999年,田沛干了近10年村党支部书记,他带领村干部结合村情,想办法,创条件,大胆尝试发展果品、蔬菜、花卉等特色增收产业。30多年过去了,田赵村已经成了远近闻名的省级“千村美丽”示范村。红河乡以田沛打出的愚公洞景观为中心,倾力打造出了一个生态种植、绿色养殖的特色乡村,可闻鸟语,能嗅花香……昔日的贫困村已然“蝶变”为美丽的幸福村。⑩穿过“愚公洞”,一头钻出来,柳暗花明。只见青山环绕,碧水荡漾,山上的绿色成堆成团,如烟似雾,沟底草木丛生,一片青翠,还有叫不上名的鸟儿飞起落下,叫声清越。山峦一眼望不尽,昔日的秃山穷水变成了百亩花海和水上乐园,仿佛有无限生机正静悄悄地勃发。19.下列对文章的理解和分析,不正确的一项是( )A.第⑥段介绍了田赵村的地理环境,暗示凿洞开路需要费很大功夫。B.第⑧段列举数据只为如实地反映田沛挖“愚公洞”的工程量大。C.最后一段的景物描写,呈现出脱离贫困后的田赵村的勃勃生机。D.本文以“愚公洞”为线索,叙事生动,语言流畅,富有生活气息。20.第⑤段画线句起到了什么作用?请结合全文简要分析。21.结合语境,按要求赏析。(1)昔日的贫困村已然“蝶变”为美丽的幸福村。(赏析加点词语) (2)这一个念头,就像那峭立的山头一样,在弥漫的黄土中浮现出来,越来越清晰。(赏析句子)22.文中的田沛和课文《愚公移山》中的愚公有哪些共同点?请简要分析。23.文章题目“给自己一个出路”有什么含义?请结合全文谈谈你的理解。【答案】19.B 20.这个句子起到承上启下(或:过渡)的作用,承接上文写“我”在洞内行走的感受,引出下文1980年时田沛承包苹果园的事情。(意对即可) 21.(1) “蝶变”本义指由蛹变为蝴蝶的过程,用在此处表示红河乡发生巨变,向好的方向发展,面貌焕然一新,人们的生活水平有了很大的提高。 (2)运用比喻的修辞手法,把“念头”比作“峭立的山头”,形象地写出了这个大胆的念头逐渐清晰的过程,表现了田沛敢为人先的性格特点。 22.①面对交通不便的困难,田沛想到挖洞,愚公想到移山,他们都能积极解决问题,都有敢为人先的精神;②田沛挖洞挖了3年,愚公认识到只要代代相传地干下去,就一定能把山移走,他们都有坚韧不拔的毅力。(意对即可) 23.①表层含义指田沛打通隧道,给自己一个从家到田地的出路;②深层含义指田沛带领贫困的田赵村,开拓出一条脱贫致富的出路。(意对即可)【解析】19.考查内容理解分析。B.表述不全面,第⑧段列举数据不只是为了如实反应“愚公洞”的工程量大,也突出了“我”对田沛坚持多年挖洞的震惊,从而体现了田沛专心致志、坚韧不拔、不怕苦不怕累的精神。故选B。20.考查句子的作用分析。句子的作用可以从结构上和内容上两大方面分析。第⑤段这个句子写的是我们走进隧道的感受。上文第④段“我钻进冰冷的洞口,感受到视觉、触、觉和嗅觉带给我的奇幻感,像穿越了时光的隧道”写我们进入隧道。下文“我看到了32岁的田沛,浑身都是使不完的力气。想法多、点子多,想到就干,干就干到底。那一年,他突发奇想,跑到庆阳承包了3年苹果园。这件事现在看起来稀松平常,但在当时,土地承包到户才刚刚开始”写田沛30年前承包苹果园的事情,划线句“这种微妙的不真实感仿佛把我带回了1980年”引出了田沛30年前承包苹果园的事情。据此可知这个划线句在结构上承接上文进入隧道,引起下文讲述田沛挖隧道的故事。内容上描写了进入隧道后给“我”的感受并引出田沛30年前承包苹果园的事情。21.考查词句赏析(1)“蝶变”又叫蜕变,是指像毛毛虫等变态发育的昆虫在茧中经过一个不食不动的阶段而变形为成虫的过程。一般指在蛰伏中向更好或更完美的方面极大蜕变。此句中“蝶变”指红河乡发生了翻天覆地的化,人们的生活水平和居住环境得到了巨大改善。(2)题干要求从修辞的角度进行赏析。作者将“念头”比作“俏丽的山头,由此可知运用了比喻的修辞手法。俏丽的山头给人的感觉是高高的,巍峨的,此处把田沛要在山下钻个洞打一个通道出来的念头比作俏丽的山头,可看出田沛的念头是大胆的,突破常人且高于常人的,表现出田沛敢为人先的精神,表达了作者对田沛的钦佩和赞赏。22.考查人物形象分析。结合文章第⑧段“一路听着田沛生动的讲述,听着这些之前看过多少遍但毫无感觉的数据:全长198米,宽1米,高约2米,移动土方1000多立方米、1600吨……”可知挖隧道十分艰难,但田沛坚持把隧道挖通了,由此可看出他和愚公一样坚韧不拔、意志坚强、敢作敢为;结合文章第⑩段“从1991年到1999年,田沛干了近10年村党支部书记,他带领村干部结合村情,想办法,创条件,大胆尝试发展果品、蔬菜、花卉等特色增收产业”可看出他和愚公一样坚持不懈,持之以恒。23.考查标题含义分析。分析标题含义我们可以分析其表层含义和深层含义,注意它是否有象征意义。表层含义:结合第③段“我挖这个洞子,就是给自己个出路!”可知“给自己一个出路”表层上来看是田沛打通一条隧道,给家到田地通路。深层含义:结合文章第⑨段“那一声阳光透进来的轰响,撼动的又岂止是一片荒山坡,也让村民们板结的心思轰然洞开”、第⑩段“他有了出路,而这条路后来也成为全村人乃至全县人的一条出路。在田沛钻过山梁之后,这里的人们也开始寻求脱贫的路子,寻求一方新天地、一种新活法”等内容可知,“给自己一个出路”的深层含义是指田沛带领贫困的田赵村走出一条脱贫致富的出路。六、阅读下面的文章,完成后面的小题。腊月①腊月一到,天地就显得空旷起来。雪要是在晚上降临,规模又不小,就能给人们十足的惊喜。夜里,那几声竹折的声音,已经把梦的暖意加重了。清晨,打开门,偌大的雪光马上席卷了你,你的思绪飞扬在群山和原野。无论小雪和大雪,腊月的雪总是喜气的,所以都叫做“瑞雪”,表达着一种憧憬的欢喜。腊月是农历的最后一个月,既然是“最后”,就必须要有一个了结,如此,一场纷纷扬扬的大雪,不妨看作是一场对季节盛大的祭扫,扫掉尘垢,扫掉阴霾,扫掉烦忧。②在腊月,花信风也吹着。瑞香、兰草、水仙、山茶……这些花骨朵,要么雅致,要么野性,叫腊月有了些许的浪漫。尤其是梅花,因为文人骚客的渲染和寄托,似乎把腊月的风姿都占尽了,其实,对于寻常百姓家,倒是特别惦记着腊梅的。腊梅比梅花来得早些,属于灌木一类。在墙角,在篱边,在园子那头,不管寒冷天气的意见如何,腊梅自顾自就动了灵气,于光溜溜的枝头笑出声来。开始是一丝丝的浅笑,后来一棵树、两棵树都笑开了去。腊梅的出身在低处、在偏僻、在旮旯,她无所谓,那些花瓣即使被雪裹着、被冰割着,也总是金灿灿地呼吸着,幽幽的清香 。腊梅,不想风光,也不甘沉沦寂寞,真是一个顽强的精灵。“疏林冻水熬寒月,唯见一株在唤春”“枝横碧玉天然瘦,恋破黄金分外香”,这样的诗句写腊梅,也算贴切,但又觉得有点矫情了:腊梅的品性,本比这更超然。③想来想去,腊月的许多生机,也是聚在菜园子里的。“小寒大寒,冻作一团”,天再冷,园子冻不死,那一园子的青葱总是鲜活着。“冰冻响,萝卜长”,萝卜坚决和严寒叫板,把自己往高处举、往土里深掘,最后,身板被锤炼得结结实实,而甜嫩脆的品质,更叫我们充满了期待——萝卜炖羊肉,馋人哟。④菠菜、矮脚白、包菜、黄心菜、芹菜、芫荽、胡萝卜、大蒜、冬苋菜、香葱、韭菜……也都在勇敢地和季节抗衡,这些寻常的蔬菜,其实是腊月里蛰伏的许多小小的幸福,不惊天动地,却把每一个日子点缀得不凝滞、不呆板。当然,它们能够在腊月里天天向上,长得滋润,还是因为农人们日常细心呵护照料的结果。每一户农家,对菜园子的拾掇,其实和对待四季里的每一项劳作是一样的,都含了实实在在的念想,手心里攥着绵绵的劲道。⑤年年岁岁,村里的柴烟延续着平常的日子,到了腊月,柴烟的身姿和内涵就要丰富很多,因为一个最大的节日在等着:年。⑥“腊七腊八,腌鱼腌鸭。”腊月一项要紧事,就是准备吃货。月初,鱼鸭就差不多腌透了,晴好的日子,每家每户的屋檐下,油汪汪晾着那一串一串的,受用着阳光温和的咂摸。整整一个月,农家的灶间难得消停,磨豆腐、蒸米酒、做糍粑,弥漫的香味香气,抵消了寒冷侵袭,人心,被烘得暖暖的。⑦这当口,女人们除了准备关于吃的一切,还要扫扬尘、纳鞋底、购年货、照顾母鸡抱窝。农事也没停,男人们忙着积肥、造肥、修仓、修理农具,等等。“人勤春早”,大伙儿明白,好日子都是顶着风霜雨雪干出来的,腊月,每一个人都在尽最大的本事,把凛凛的冬天过成一个热烈的期盼。⑧大年三十晚,腊月只剩下很短的时光。所有的念想都沉淀下来:平安是福,团圆是福,一家乐呵是福。这个晚上,过得快,也过得慢,要祭祖,要放开门炮仗,要守岁,要给晚辈压岁钱,当然,高潮就是年夜饭了。⑨不管年景咋样,也要把年夜饭弄得欢实起来:要凑成双数的碗,至少要有八个菜,寓意发达;肘子是有的,下了许多功夫,蒸成一大碗,滑嫩,溜光,如琥珀,香气逼人;鱼是有的,红烧为主,红辣椒把鱼身子盖满了,看着就想出汗;鸡肯定也是有的,大都用炖的做法,汤黄亮亮的,点缀着桂圆、荔枝、红枸杞。马上,一家人围成一桌,吃上个把小时,吃得热气腾腾、热汗淋漓,吃得满面红光光、油光光,满屋子尽是快活,尽是喜气。⑩“野兴疏,冬寥落,炉前沉醉酒一壶。”是啊,腊月已经来了,春天也会捂不住的,不如,今夜,在梦里,潜回老家的山村去,守着一盆炭火,陪着父母,听他们唠叨你的小名,唠叨整整一个晚上……(陈爱民/《人民日报》 2020年01月18日08 版)24.文章题目改为《年》可以吗?说说理由。25.结合你的阅读积累,说说你对第②段“腊梅的品性,本比这更超然”这句话的理解。26.第③段说“想来想去,腊月的许多生机,也是聚在菜园子里的”“也”字可以去掉吗?为什么?27.⑥⑦段两处引用有什么作用。28.品读第⑨段,自选角度做两处赏析。 29.文章第⑩段表达了怎样的情感?【答案】24.从内容来讲,文章叙述描写了整个腊月的特点和一系列活动,高潮和重点是准备过年和过年。用《年》做题目,不能涵盖全部内容,也消减了铺垫的作用。 25.第②段的两句诗“疏林冻水熬寒月,唯见一株在唤春”用对比手法表现腊梅在寒月中心怀春天的昂扬积极;“枝横碧玉天然瘦,恋破黄金分外香”,表现的是腊梅的清香袭人,清气入骨。其实,腊梅的品性比这更超然,“零落成泥碾作尘,只有香如故”“待到山花烂漫时,她在丛中笑”,腊梅还有一种孤高、不染尘俗,独立高洁的品性。 26.“也”是“不止……还”的意思,表示递进,承接上文的内容,腊月的生机在飞扬的瑞雪中、在浪漫的花朵中,这里引出下文:也在菜园子里天天向上的各种蔬菜中。这里起过渡的作用。 27.引用俗语、谚语,“腊七腊八,腌鱼腌鸭。”引出腊月的要紧事——准备吃货;“人勤春早”朴素的语言蕴含生活的哲理:好日子都是顶着风霜雨雪干出来的。俗语谚语让文章语言平实朴素,更接地气。 28.示例1:“欢实”这个词带有口语色彩,有热闹丰富热气腾腾的感觉,写出了年夜饭的喜气;示例2:一组排比句,写出了年夜饭的丰富、丰盛,有香、有色、有味、有气氛;示例3:善用叠词,黄亮亮写色泽诱人、热气腾腾写暖人心、满面红光光、油光光写人们脸上洋溢的喜气。在内容上恰如其分地烘托气氛;增强语言的韵律感、节奏感,读起来朗朗上口,富有音乐美; 29.表达了对于春天的憧憬和希冀,对老家人情风貌的怀恋,对于传统味浓郁的年文化的温情缅怀。【解析】24.本题考查分析文章标题。结合第①段“腊月一到,天地就显得空旷起来”,第②段“在腊月,花信风也吹着”,第③段“想来想去,腊月的许多生机,也是聚在菜园子里的”,第⑤段“年年岁岁,村里的柴烟延续着平常的日子,到了腊月,柴烟的身姿和内涵就要丰富很多,因为一个最大的节日在等着:年”,第⑥段“腊月一项要紧事,就是准备吃货”,第⑦段“腊月,每一个人都在尽最大的本事,把凛凛的冬天过成一个热烈的期盼”,第⑧段“大年三十晚,腊月只剩下很短的时光”,第⑨段“不管年景咋样,也要把年夜饭弄得欢实起来”,第⑩段“是啊,腊月已经来了,春天也会捂不住的”可知,从内容上看,文章叙述描写了整个腊月的特点和一系列活动,高潮和重点是腊月人们准备过年和过年的情景,故用“腊月”做标题可以涵盖全部内容;相反,用“年”做题目,不能涵盖全部内容,也消减了铺垫的作用。25.本题考查理解句段。结合第②段“腊梅比梅花来得早些,属于灌木一类。在墙角,在篱边,在园子那头,不管寒冷天气的意见如何,腊梅自顾自就动了灵气,于光溜溜的枝头笑出声来”可知,腊梅具有不畏严寒的品性;结合第②段“腊梅的出身在低处、在偏僻、在旮旯,她无所谓,那些花瓣即使被雪裹着、被冰割着,也总是金灿灿地呼吸着,幽幽的清香不动声色、不卑不亢、不绝如缕”,“腊梅,不想风光,也不甘沉沦寂寞,真是一个顽强的精灵”可知,腊梅具有不畏艰险、努力进取、顽强拼搏、不卑不亢的品性;结合第②段“疏林冻水熬寒月,唯见一株在唤春”可知,诗句用对比的手法,突出了腊梅在寒冷的冬天仍坚持对春天的向往;结合“枝横碧玉天然瘦,恋破黄金分外香”可知,这两句诗是要表现腊梅的香气袭人,清气入骨;结合“这样的诗句写腊梅,也算贴切,但又觉得有点矫情了:腊梅的品性,本比这更超然”可知,作者认为上述两处诗句不足以表现出腊梅的品性,结合上述腊梅的品性的分析,我们可以联想到的写梅的诗句有:“零落成泥碾作尘,只有香如故”,“冰雪林中著此身,不同桃李混芳尘”写出了腊梅的孤高、不染尘俗,独立高洁的品性;“前村深雪里, 昨夜一枝开”,“墙角数枝梅, 凌寒独自开”写出了腊梅不畏严寒的品性。26.本题考查赏析词句。结合第③段“想来想去,腊月的许多生机,也是聚在菜园子里的”分析,“也”是“不……还”的意思,表示递进,承接上文第②段“在腊月,花信风也吹着。瑞香、兰草、水仙、山茶……这些花骨朵,要么雅致,要么野性,叫腊月有了些许的浪漫。尤其是梅花”,腊月的生机在飞扬的瑞雪中、在浪漫的花朵中,这里引出下文“天再冷,园子冻不死,那一园子的青葱总是鲜活着”“萝卜坚决和严寒叫板,把自己往高处举、往土里深掘”“菠菜、矮脚白、包菜、黄心菜、芹菜、芫荽、胡萝卜、大蒜、冬苋菜、香葱、韭菜……也都在勇敢地和季节抗衡”,告诉读者腊月也在菜园子里天天向上的各种蔬菜中;“也”在这里起过渡的作用。27.本题考查分析引用的作用。结合第⑥“‘腊七腊八,腌鱼腌鸭。’腊月一项要紧事,就是准备吃货”可知,引用“腊七腊八,腌鱼腌鸭”的俗语,引出腊月的要紧事是准备吃货;结合第⑦段“‘人勤春早’,大伙儿明白,好日子都是顶着风霜雨雪干出来的,腊月,每一个人都在尽最大的本事,把凛凛的冬天过成一个热烈的期盼”可知,“人勤春早”是一句谚语,引用这句谚语,说明“大伙儿明白,好日子都是顶着风霜雨雪干出来的”这个道理,朴素的语言蕴含生活的哲理:好日子都是顶着风霜雨雪干出来的。综上分析可知,引用俗语谚语让文章语言平实朴素,更接地气。28.本题考查赏析语句。可从词语口语色彩、修辞、叠词、句式等方面批注。示例:结合“马上,一家人围成一桌,吃上个把小时,吃得热气腾腾、热汗淋漓,吃得满面红光光、油光光,满屋子尽是快活,尽是喜气”可知,第⑨段在句式选择上,多用短句,结构简单,句义明晰,简洁明快。29.本题考查分析作者情感主旨。结合第⑩段“是啊,腊月已经来了,春天也会捂不住的”可知,这里表达了作者对于春天的憧憬和希冀;结合“不如,今夜,在梦里,潜回老家的山村去,守着一盆炭火,陪着父母,听他们唠叨你的小名,唠叨整整一个晚上”可知,这里表达了作者对老家人情风貌的怀恋,表达了作者对于传统味浓郁的年文化的温情缅怀。七、阅读下文,完成下面小题。岁月的忧伤①我在门前站了很久,还是忍不住伸出手指,轻轻去抹蓝色门牌上岁月的痕迹。五曲巷15号,窄巷尽头一户平常的人家。几年不见,它已经明显衰老,我温暖的眼神依然捕捉到了它那老人才有的忧郁和苍凉。“最是人间留不住,朱颜辞镜花辞树”,在连房屋街巷都变得老迈的今天,我柔软的手指能阻挡什么呢?我一遍遍试图还原它昔日的光亮,不过想更切近地知道,曾经真真切切属于这里的,那些幸福的人,那些快乐的事,那些蓬勃的日子,到底都到哪里去了。②二十多年前,我坐在简陋的小饭桌边,看正午的阳光穿过一大碗清水,投射到炊烟熏黑的墙壁上去,明亮的光圈在一家人的头顶上一闪一跳,像放电影。几只母鸡咕咕叫着在我们脚下觅食,铝制的房门把手在太阳下散发着温润的光。我悄悄把这个中午的这些细节记了下来。这是我对时间的第一次留意,我想知道,如果把生活的一瞬印进心里,时间的脚步是不是就可以走得慢些。据父母说那是我们家最为清贫的一段时期,四个上学的孩子加上不轻的债务,让他们的日子非常拮据。可我搜遍童年的记忆,回想起来的却只有大片黄黄紫紫开得茂盛的丝瓜花、扁豆花和一张张朝气蓬勃的脸。我记得父亲曾异想天开地在院子里种满小麦,还用花纹蛤皮装饰了门前的巷子,母亲则每晚在缝纫机前一边干活一边哼唱。她心灵手巧,除了衣服做得好,蒸馒头时为我捏的面喜鹊也是栩栩如生。那时父母正在盛年,日子在他们手上每天都新鲜且富有朝气。我们在这样的日子里成长,从没有因为贫穷自卑过,相反,我们一直以为自己是天底下生活得最好的人。③后来我知道这种方法不对。因为无论怎样努力,时间还是疾步如飞地走向一个又一个明天。它每天的变化极其细微,等我们忽然觉察,已经有好大一串日子被甩到身后了。而且,它是有策略的,它从不一味傻跑,而是在行程中不断给我们制造悬念或者惊喜,使我们愿意甚至盼望新一天的到来。比如那天我放学回家,发现二哥手里新添了一把吉他。清澈的音乐混合着枣花的香气在院子里流淌,全家人因此变得喜气洋洋。那一刻,我轻易就原谅了时间的匆匆,因为是它让哥哥长大,使他可以像父亲那样工作、挣工资,使热爱音乐的一家人终于可以在自己的院子里而不是在信号不佳的收音机里听到悦耳的琴声。④再后来,我们空阔的院子被越长越大的枣树浓荫遮起来。我们陆续长大,然后像分叉的树枝把生命伸向另一片天空。树下听琴的日子和我偷读第一封男生来信的过往,在更新鲜、更繁忙、更美好的日子扑面而来的时候,马上就被我忽略了。时间像一把巨大的扫帚,转眼就把那些生动的声音、表情、情节或者眼神打扫得干干净净。我们追随着时间的脚步,觉得有更多更好的日子等在前面,所以,大家一路匆匆,努力前行,很少顾上回头。⑤直到有一天,母亲永远退出我们的生活。⑥这才知道,属于家庭的最完美的日子,已经一去不返了。那些觅食的母鸡,盛着清水的粗瓷大碗,母亲的哼唱,还有满院的麦子,再也回不到我的生活中来了。那天,我看哥哥握着母亲的手,与她约定来世再做母子,忽然觉得掉进了巨大的时间黑洞。我不知道在神秘的时空中,要经过怎样繁琐的组合,曾经的一家人,才能再成为一家人。这显然是个渺茫的愿望,这一醒悟,让我心凉如水。⑦现在,我与巷子四目相对,默默无语。我每迈出一步,都踏在厚厚的时间的尘土上。沉寂多年的情景不断纷扬起来,它们一片又一片落在我薄瘦的肩膀上,使我痛楚地感觉到了岁月的重量。(作者:陈晓霞。有删改)30.梳理“我”对时间态度变化的脉络,根据提示,补全下面横线处的内容。(1)二十多年前,希望时间可以走得慢些,_________。(2)_________,因为时间总是给我制造悬念或者惊喜。(3)_________,因为觉得有更多更好的日子等在前面。(4)直到有一天,猛然醒悟,心凉如水,_________。31.第②段画线句子在文中有什么作用?32.文中的数量词运用很有特点。请简析下面句子中加点词语的表达效果。(1)我一遍遍试图还原它昔日的光亮……那些蓬勃的日子,到底都到哪里去了。(2)沉寂多年的情景不断纷扬起来,它们一片又一片落在我薄瘦的肩膀上,使我痛楚地感觉到了岁月的重量。33.结合全文,简要回答选文标题中“忧伤”的具体含义。【答案】30.因为父母把日子过得新鲜且富有朝气 后来,愿意甚至盼望新一天的到来 再后来,追随着时间的脚步匆匆前行 因为母亲去世,让“我”明白岁月会带走属于家庭的最完美的日子 31.描述二十年前看到的家中情境,营造出安静祥和、温馨和美的家庭生活氛围。这些记忆中的细节,是童年美好生活的代表,解释了“我”童年时期想让“时间的脚步”“走得慢些”的原因:为后文“最完美的日子”“一去不返”的醒悟和光阴易逝、岁月无情的感慨作了铺垫。 32.(1)“一遍遍”表现“我”抹拭门牌次数之多,体现“我”想找回昔日美好时光的急切心情。(2)“一片又一片”指的是“沉寂多年的情景”,运用比喻使无形无质的回忆“情景”有了形状,有了重量,有了数量,仿佛可以看见、可以触摸,形象生动地表现了“我”的痛楚和感慨。 33.“忧伤”的具体含义有:对属于家庭的那些幸福的人、快乐的事、蓬勃的日子的怀念与追寻;对属于家庭的温馨美好时光不再的遗憾与迷茫;对时光匆匆无法挽留的痛楚与感叹;对认清岁月流转无情的无奈与无助。【解析】30.考查内容理解概括。第一空,结合选文第②段“那时父母正在盛年,日子在他们手上每天都新鲜且富有朝气”可得:因为父母把日子过得新鲜且富有朝气;第二空,结合选文第③段“而且,它是有策略的,它从不一味傻跑,而是在行程中不断给我们制造悬念或者惊喜,使我们愿意甚至盼望新一天的到来”可得:后来,愿意甚至盼望新一天的到来;第三空,结合选文第④段“我们追随着时间的脚步,觉得有更多更好的日子等在前面,所以,大家一路匆匆,努力前行,很少顾上回头”可得:再后来,追随着时间的脚步匆匆前行;第四空,结合选文第⑥段“那天,我看哥哥握着母亲的手,与她约定来世再做母子,忽然觉得掉进了巨大的时间黑洞。我不知道在神秘的时空中,要经过怎样繁琐的组合,曾经的一家人,才能再成为一家人。这显然是个渺茫的愿望,这一醒悟,让我心凉如水”可得:因为母亲去世,让“我”明白岁月会带走属于家庭的最完美的日子。31.考查句段作用。由“看正午的阳光穿过一大碗清水,投射到炊烟熏黑的墙壁上去,明亮的光圈在一家人的头顶上一闪一跳,像放电影”“几只母鸡咕咕叫着在我们脚下觅食,铝制的房门把手在太阳下散发着温润的光”等内容可知,此处回忆了童年时日常生活中的某一场景,属于环境描写,渲染了宁静祥和、安逸温馨的生活画面,解释作者希望童年时光“走得慢些”的原因;同时也为后文作者抒发“完美日子”一去不复返的感慨和无奈之情做铺垫。32.考查词语赏析。(1)“一遍遍”写出“我”反复擦拭门牌的情景,体现出“我”渴望找回昔日美好时光的迫切之情,体现出“我”对过去美好时光的留恋。(2)“一片又一片”将无形的已经消逝的情景具体化、有形化,生动形象地写出“我”回忆往事时内心的痛楚和无奈。33.考查标题理解。结合选文第②段“我记得父亲曾异想天开地在院子里种满小麦,还用花纹蛤皮装饰了门前的巷子,母亲则每晚在缝纫机前一边干活一边哼唱。她心灵手巧,除了衣服做得好,蒸馒头时为我捏的面喜鹊也是栩栩如生。那时父母正在盛年,日子在他们手上每天都新鲜且富有朝气”可知,标题“岁月的忧伤”中的“忧伤”表达对过去美好时光的怀念;结合选文第⑥段“这才知道,属于家庭的最完美的日子,已经一去不返了。那些觅食的母鸡,盛着清水的粗瓷大碗,母亲的哼唱,还有满院的麦子,再也回不到我的生活中来了”等内容可知,标题“岁月的忧伤”中的“忧伤”表达对温馨美好时光逝去的感叹和怅惘,同时流露出对岁月无情流逝的无奈和感伤。八、阅读下文,完成各题。午休的父亲①那位在楼梯拐角水泥地上午休的父亲睡着了。②九月的北京处于近年少有的高温时节。特别是中午,一丝风也没有,每一片树叶都静止着,看上去皱巴巴的。人若置身户外,如在桑拿房中,片刻便会出汗,感到缺氧似的。医生们频频出现在电视中,提醒民众做好防暑降温的自我保护。③我住的小区正进行老旧小区的楼房改造。我家住的那幢楼刚搭完脚手架。我因颈椎病重,不敢享受空调,所以不但开窗,连门也开着。“嗨,吃了没?我也吃过了!”一天中午,我在家边吃饭边看电视。我有点耳背,不知不觉便将电视声调得挺大。然而我竟听到了门外一个男人大声所说的话,遂将电视声调小。受好奇心驱使,起身走到门口,探头向外看——但见一个裸着上身的四十余岁的男人仰躺在楼道拐角那儿,身下垫着片由废旧纸箱拆开的纸板,纸板上铺着脏兮兮的工作服。他头枕一块泡沫,一边的额角贴着创可贴,不是那种窄窄的小长条形的,而是有三四个那么宽的方形的;他一手拿手机,一手扇风凉,一小片扇形的纸板,分明是从身下那块大纸板上撕下来的;他脏兮兮的裤子的裤筒卷到了膝部,小腿布满褐红色的墙漆点子。而他周围,遍地碎墙渣子。工作尚未结束,没有清扫的必要。我关了电视。④“老婆,我那摩托要推到棚子里,别下雨淋了它。不会下雨?老天爷听你的?再旧不是还能骑吗?不也是钱买的吗?我也想家行了吧?女儿在左右吗?快让女儿跟我通话……”走廊拢音,那男人的话声,我听得更清楚了。“好女儿,每次听到你的声音,老爸的心情就是开心!还不能返校?那就更要把网课听好。学习这事,靠的就是自觉。不是为老师学的,也不是为我和你妈学的。你刚高一,人生还长呢,知道这个道理就好。钱不是问题!爸还是那句话,你将来能考到什么份儿上,爸妈就有能力供你到什么份儿了。不许!别改视频!我不许!又不是几年没见了,视频费流量呢!你非视频我可关机了啊!聊会儿就行。认真听着,老爸得嘱咐你几句。你妈也在上班,你要心疼她,有空儿,屋里屋外的活多干点儿,就当替老爸干了。你奶奶岁数大了,腿脚不灵便了,她出门你都要挽着她。我这儿一切都好,别牵挂我。热!北京也热,老爸这会儿在午休呗。我们有临时工棚嘛。怎么可能每人一张床,没那么好的条件。但有通铺,铺的新凉席,每人都有睡的地方。还有大风扇,凉快得很,特解乏……”⑤我想我再听下去似乎是一个偷听者了,便去关门,却不料见到了这样一幕——楼上念小学的姗姗正与妈妈上楼。姗姗看着那男人的样子吃惊不小,呆住在一级台阶上。姗姗妈也不由得“呀”了一声,却立刻对女儿说:“上楼啊,叔叔是热的。”那男人旋即坐起,慌忙往身上披工作服,连说:“见笑见笑。”姗姗妈说:“理解,有什么可见笑的呀。”她边说边牵着姗姗的手上楼去了。而那男人却站也不是,躺也不是,手足无措,样子恓惶其极了。⑥我关上门,听到有人敲门。开门一看,见是那男人。我问:“有事儿?”他语无伦次地说:“没事儿,可也有事儿。就是拜托您替我向那位女同志表示一下歉意,刚才我那样子是很不礼貌的……”我笑道:“没关系,她不会在意的,我也不会。”“多谢多谢,这天真是的,热得人没处躲没处藏的,水泥地不是凉快嘛。”他窘窘地退回了楼道拐角。⑦我仍然敞着门,但见姗姗拎着塑料袋在门外看我。⑧我刚要开口,姗姗将手指压在自己唇上,接着指塑料袋。我走到门口,她小声说,里边的东西本是她妈让她送给“午休的叔叔”的。我也小声说:“那你送过去呀。”她细声细气地说:“叔叔睡着了,爷爷您过会儿替我送给他吧。”我扭头看去,见那位午休的父亲,背朝楼梯,蜷着双腿,已睡实了。他的工作服也不垫在身下了,不知怎么被弄成一团搂在怀里了。⑨我接过塑料袋一看,装的是两瓶矿泉水,一瓶可乐,一个很水灵的刚洗过的大桃子。“爷爷您轻点儿关门。”姗姗说完此话,踮起脚尖,蹑手蹑脚地往楼上迈。在楼梯上冲我一脸烂漫地笑。⑩有好邻居是种造化。34.文章情节感人,请在横线上填写恰当的内容。“我”听到一位建筑工人师傅在楼道里和老家的亲人通电话。→(1)_______________→ 师傅请“我”替他向姗姗母女致歉。→(2)____________________35.阅读第⑤段画线句,请从表达效果的角度写一条批注。36.第④段中父亲不肯和女儿视频通话的真实原因是__________37.下列对文本内容的分析理解,错误的一项是( )A.第②段渲染天气的炎热,为下文建筑工人没穿工作服午休、“我”家开着门的情节作铺垫。B.第④段只写了那位工人父亲的话,略去了他女儿说的话,但读者也能从中体会到父女情深。C.文章语言质朴传神。从姗姗和“我”的对话中可以看出,姗姗虽然年幼,但也非常懂事。D.文章采用顺叙的叙述方式,以“我”的视角,展现了发生在小区改建过程中的温馨一幕。38.关于本文的主旨有以下几种理解,你认同哪一种?请简述理由。A.赞美建筑工人 B.赞美小区居民 C.赞美和谐的人际关系【答案】34.姗姗母女上楼时撞见没穿工作服的师傅。 姗姗请“我”转送师傅水和水果 35.运用人物的动作描写和语言描写。通过“旋即坐起”“慌忙披”等词语,写出了那位工人师傅当时的慌乱、无措。反复“见笑见笑”,表明他充满歉意。这一系列人物描写凸显那位工人师傅的朴实、诚恳。 36.怕女儿看到他受伤的模样和简陋、艰苦的工作环境而心疼、担心。 37.D 38.示例一:B。文中用大量篇幅来写建筑工人,目的是为了引出下文姗姗母女、“我”对其行为的体谅、理解,对小区建设者的尊重、关爱。文章结尾画龙点睛,表明了作者的写作意图,是对好邻居的赞美。示例二:C。建筑工人不文明的午休方式是囿于客观环境,当他意识到后主动道歉;姗姗母女和“我”对其行为充分体谅、理解,主动关爱。这体现出人与人之间和谐的人际关系。【解析】34.本题考查文章情节梳理。结合第⑤段“楼上念小学的姗姗正与妈妈上楼。姗姗看着那男人的样子吃惊不小,呆住在一级台阶上……那男人旋即坐起,慌忙往身上披工作服,连说:‘见笑见笑。’”可知,①概括为:姗姗母女上楼时撞见没穿工作服的师傅。结合第⑧段“叔叔睡着了,爷爷您过会儿替我送给他吧”和第⑨段“我接过塑料袋一看,装的是两瓶矿泉水,一瓶可乐,一个很水灵的刚洗过的大桃子”可知,②概括为:姗姗请“我”转送师傅水和水果。35.本题考查句子赏析。第⑤段划线句“那男人旋即坐起,慌忙往身上披工作服,连说‘见笑见笑。’”中,“坐起”“披上”运用了动作描写,“见笑见笑”运用语言描写,“旋即”“慌忙”体现出男人的害羞着急以及反应速度快。通过“旋即坐起”“慌忙披”等词语,写出了那位工人师傅当时的慌乱、无措。反复“见笑见笑”,表明他脱下衣服本是为了凉快一些,没想到碰上了正上楼的姗姗和妈妈,所以感到不好意思,充满歉意。这一系列人物描写凸显那位工人师傅的朴实、诚恳。36.本题考查文章内容分析。由第③段“纸板上铺着脏兮兮的工作服”“他那同样脏兮兮的裤子的裤筒卷到了膝部,小腿布满褐红色的墙漆点子”可知,他的工作大多是体力活,劳动强度较大;由“仰躺在二楼和三楼之间的拐角那儿”“而他周围,遍地碎墙渣子”可知,他的工作条件很简陋;由“他一边的额角贴着创可贴,不是那种窄窄的小长条形的,而是有三四个那么宽的方形的”他的工作还很容易受伤,具有一定的危险性。所以父亲不肯和女儿视频通话的真实原因怕女儿看到他受伤的模样和简陋、艰苦的工作环境而心疼、担心。37.本题考查理解分析文章内容。D.根据第①段“那位在楼梯拐角水泥地上午休的父亲睡着了”和第⑧段姗姗说的话“叔叔睡着了,爷爷您过会儿替我送给他吧”可知,文章开头的“父亲”就正在午休,作者以倒叙的方式回忆炎热的这天,“我”碰巧听到装修师傅在给女儿打电话,而他因为天气太过炎热工作时脱下了上衣,没想到碰到了正巧上楼的姗姗和她的妈妈,装修师傅“手足无措,样子恓惶其极了”,但姗姗和妈妈对其行为表示体谅、理解,还对小区建设者表示尊重,因为“午休的父亲睡着”,所以请“我”转送师傅水和水果,最后让作者发出感叹“有好邻居是种造化”的故事。所以本篇文章采用的是倒叙的叙述方式,并不是选项中说的“顺序”。故选D。38.本题考查主旨把握。这是一道开放性试题,答案不唯一。文章通过“我”听到了门外,一个农民工午休时给自己的家人打电话,尤其是给女儿打电话时的样子、神情等,表现了表现了一个“父亲”的体贴与(担当),表现了作者对他的理解与赞赏,意在引起人们对农民工生存状况的关注。同时通过写姗姗和她妈妈理解师傅因为天气炎热而脱掉上衣工作的行为,还他送去水果和水,体现出大家对城市建设者的理解、体谅,他们的言行意味着我们的这个社会不乏善良、包容和温暖。根据文章的最后一段“有好邻居是种造化”可知,文章结尾画龙点睛,表明了作者的写作意图,是对好邻居的赞美。据此选择你认同的本文主旨,结合文章简述理由即可。示例:我选C。文中写了午休时间,干体力活的父亲与女儿的通话,在通话中,表现出作为父亲对女儿的关心、爱护与鼓励,父亲将所有的累和苦扛在自己肩上,尽量为女儿创造条件。文章以描写午休农民工的特写镜头,表现出作者对这类吃苦耐劳的农工的赞美,也引发人们对他们生存状况的关注。九、阅读下面的文章,完成小题。走过村庄胡启涌①风把我与一地的落叶吹向村庄,步履窸窣,叶子窸窣。②河流紧贴着村庄,娴静的河水湛蓝如练。风拂过时,河中清波粼粼,两岸也随着波纹荡漾开去。几棵有些年数的柳树,痴守着河岸,风急柳乱,河风任意侍弄着柳树的长发,河面也变得影影绰绰起来。③这个隐在山间的小村庄,现在只剩下一片宁静了,人们离开了这里,集体迁到了远处更为便利的小镇。我无数次走过这里,书写过这里的草木人间,我在这里能快速地找到内心的故乡。现在,河边没了挑水的人,没了洗衣的人,没了放牛的人。整条河流已交给了几只黑水雉,黑水雉是一种文静的水鸟,如一串草甸子浮在水面,时而将藏在翅膀下的头抬起,看一下四周后又把头埋下。它熟悉这条河流,只需要抬头一瞥,就会读懂河流的每一个细节。④整个村庄都是青瓦木房,石墙护院,推开虚掩的木门,屋里的烟火已远去,抬手触摸旧物件,润润的凉凉的,就像受潮的记忆,时时处于破壳发芽的状态。只要随意一摸,手印就会真实地印上去,指纹与木纹清晰地重叠着,一个完整的村庄顿时在心中温暖起来。⑤房顶上,道道瓦沟和排排瓦垄上的湿湿印迹告诉我,一场小雨刚刚来过。排排青瓦,是村庄一架铺陈开去的大号钢琴,风吹过琴声响起,轻时如低诉,急时似号鸣。檐角上的一撮松土上,一根蒿草在风中时俯时仰,指挥着村庄的露天音乐盛会。⑥瓦檐下的木房已无人居住,一码一码的柴火堆在房前房后。曾经奔忙在田土间的铁跸光亮不再,在岁月的销蚀中斑驳暗沉。挂在木梁上的一把锄头,紧紧贴着墙壁,与脱了齿的犁耙、断了柄的钉耙,在时光中保持着一致的静谧。一个完整的背篼搁在屋角,还配有一对精致的棕丝背带,里面装着一些包谷壳,算是对屋里残缺旧物的一个补充。门外一口石水缸,底部已陷在了泥土中,古意苍苍。在曾经的时光里,这座瓦房主人的每一餐饭,每一次浆洗,都从这口水缸开始演绎。还有随处可见的石猪槽,没入瓦砾中的石磨盘,干爽如昔的地窖……待到春天,草芽陆续冒出地面,几阵春雨后,疯长的绿意就会再次把它们覆盖,会将这些故园中的旧物件,精心做成一个又一个的绿色包裹,存放在村庄这里,耐心地等待取件人的到来。⑦一堵半人高的老墙,像村庄的一道浓眉,由脸盆大小的砾石摆砌而成,大约是村民们从村外河里拾来,光滑的表面有水流的痕迹,也有村民的体温。石缝中生长着坠盆莲和金边吊兰,缀有水珠,透明喜人。满墙缠绕的枯瘦南瓜藤,恰似一幅章草书法作品中的几道焦墨,恣意地交织在墙上。几个南瓜无人摘回,如几滴硕大的墨点滴落在墙头。南瓜上有几个小洞,走近探看,里面的瓜子和瓜瓤早被松鼠和鸟儿啄去,只剩下一个个空壳,写意地悬挂在墙头。一群麻雀在墙头翻飞,蹦上跳下,叽喳不休。一只流浪猫突然出现,麻雀“噗”的一声飞向瓦房。瓦房是鸟儿最安全的家,麻雀总能在檐下的檩条间找到舒惬住处。它们与燕子为邻,同在屋檐下,燕子的旧日泥巢还空着,一个挨一个的“挂”在檐下。等到草绿花开的日子,燕子归来,它们就会同守屋檐下,共话杏花春雨。⑧我披着轻寒,一个人走过村庄。这里的瓦房依山而建,一条青石满铺的路,顺着山势随弯就弯地穿过村庄。其实村庄不大,就十几户人家,人们离开后也很少回来了,把带不走的瓦房和过去留在了这里,交给了四季的风雨管理。时间愈久这里就愈安静,适合一个人孤独地走过。⑨还好,河流不会离开这里,始终以母性的温情拥抱着村庄。我与瓦房、石板,还有那些静躺在光阴里的旧物件,一起依偎在河流的臂弯里,用最暖心语言慰藉彼此,想起沈从文那一句:我就这样一边看水一边想你。(选自《光明日报》2022年11月02日16版)39.通读全文,理清思路,完成下列填空。文章以①______为线索,写出自己在村庄的所见所闻所感。文章开头以“落叶”交代了时间,作者走向了村庄。然后描写②______,接着介绍③______,最后写④______。40.有人认为文章第②段是赘笔,应该删去。你是否同意,请简要说明理由。41.文章第⑦段运用多种景物描写的方法,将村庄老墙及屋檐处写得生机勃勃,情趣盎然。请写出一种景物描写的方法并结合内容具体分析。42.请结合文章内容谈谈你对文章结尾引用沈从文“我就这样一边看水一边想你” 这句话的理解。【答案】39.走过村庄(或作者行踪) 紧贴村庄,所见河流及河岸之景(走近村庄) 走进村庄所见景物(或青瓦木房、屋里、房顶上、瓦檐下、老墙处等具体景物) 走过村庄 40.不同意。结构上:与结尾“河流一直温暖村庄”相呼应,使文章结构完整。内容上:营造出朦胧的诗意氛围,奠定全文感情基调。主旨表达上,“几棵有些年的柳树,痴守着河岸”表达了作者对村庄的依恋,突出主旨。 41.示例一:动静结合。前部分写静物:老墙是由“脸盆大小的砾石摆砌而成”、“石缝中生长着坠盆莲和金边吊兰,缀有水珠”、“满墙缠绕的枯瘦南瓜藤”及被吃空的南瓜;后部分写到动物,麻雀、猫、燕子等在此间的活动,动静结合,相映成趣。示例二:虚实结合。在写实景的同时,“光滑的表面有水流的痕迹,也有村民的体温”、“等到草绿花开的日子,燕子归来,它们就会同守屋檐下,共话杏花春雨”等处虚写,增强文章感染力,给读者留下丰富的想象空间。 42.沈从文这句话诗意地表达了无时无刻不在的思念与热爱。本文则表达了作者对村庄的喜爱、依恋,对内心故乡的找寻,对城市化进程下农村旧时生活的怀念与守望。作者引用沈从文的话,收束全文,有助于突出主旨表达情感,又增添了文学性。【解析】39.本题考查文章内容概括。第②空:根据第②段“河流紧贴着村庄,娴静的河水湛蓝如练……几棵有些年数的柳树,痴守着河岸,风急柳乱,河风任意侍弄着柳树的长发”可概括为:紧贴村庄,所见河流及河岸之景;第③空:根据第④段“整个村庄都是青瓦木房,石墙护院,推开虚掩的木门,屋里的烟火已远去”、第⑤段“房顶上,道道瓦沟和排排瓦垄上的湿湿印迹告诉我,一场小雨刚刚来过。排排青瓦,是村庄一架铺陈开去的大号钢琴”、第⑥段“瓦檐下的木房已无人居住,一码一码的柴火堆在房前房后”、第⑦段“一堵半人高的老墙,像村庄的一道浓眉,由脸盆大小的砾石摆砌而成”可概括为:走进村庄所见景物;第④空:根据第⑧段“我披着轻寒,一个人走过村庄”“时间愈久这里就愈安静,适合一个人孤独地走过”可概括为:走过村庄。第①空:综合以上信息可知,作者行文按照走近村庄——走进村庄——走过村庄的游踪顺序展开,故文章一作者的游踪为线索,书写自己的所见所闻所感。40.本题考查句段作用。本段中“河水湛蓝如练”“风拂过时,河中清波粼粼,两岸也随着波纹荡漾开去”“河风任意侍弄着柳树的长发,河面也变得影影绰绰起来”对河流及沿岸景色的描写,营造了一种幽美、朦胧的意境,奠定文章的感情基调;“几棵有些年数的柳树,痴守着河岸”写柳树痴守河岸与第③段“人们离开了这里,集体迁到了远处更为便利的小镇”写人们离开小镇形成鲜明对比,表达作者对村庄的依恋;结合第⑨段“还好,河流不会离开这里,始终以母性的温情拥抱着村庄”可知,第②段与第⑨段形成照应,使文章结构更加严谨,更利于表现文章主题。41.本题考查环境描写的技巧。运用修辞:“一堵半人高的老墙,像村庄的一道浓眉”运用比喻写老墙,把老墙比作一道浓眉,写出了老墙的厚重;“满墙缠绕的枯瘦南瓜藤,恰似一幅章草书法作品中的几道焦墨,恣意地交织在墙上。几个南瓜无人摘回,如几滴硕大的墨点滴落在墙头”运用比喻写南瓜藤和南瓜,写出南瓜藤的恣意和南瓜的硕大;动静结合:“一堵半人高的老墙”“石缝中生长着坠盆莲和金边吊兰,缀有水珠,透明喜人”“满墙缠绕的枯瘦南瓜藤”“几个南瓜无人摘回”“南瓜上有几个小洞”属于静景描写,“一群麻雀在墙头翻飞,蹦上跳下,叽喳不休”“一只流浪猫突然出现”是动景描写,本段动静结合,能使文章活泼生动,使所写事物活灵活现,直观形象地给读者以栩栩如生的印象;虚实结合:“光滑的表面有水流的痕迹,也有村民的体温”“等到草绿花开的日子,燕子归来,它们就会同守屋檐下,共话杏花春雨”是虚写,语段中写老墙、坠盆莲、金边吊兰、南瓜藤、南瓜、麻雀、猫燕子的泥巢等是实写,虚实结合大大丰富了文章内容,开拓了文章的意境,为读者提供广阔的审美空间,充实人们的审美趣味。42.本题考查句子含义。结合第③段“这个隐在山间的小村庄,现在只剩下一片宁静了,人们离开了这里,集体迁到了远处更为便利的小镇。我无数次走过这里,书写过这里的草木人间,我在这里能快速地找到内心的故乡。现在,河边没了挑水的人,没了洗衣的人,没了放牛的人”可知,作者来到村庄,看到现在的村庄因村民的集体搬迁而变得落寞。作者由此想到过往的热闹生活,再结合尾段“还好,河流不会离开这里,始终以母性的温情拥抱着村庄。我与瓦房、石板,还有那些静躺在光阴里的旧物件,一起依偎在河流的臂弯里,用最暖心的语言慰藉彼此”可知,“一边看水一边想你”的“你”实际上指的是作者在怀念过去热闹的村庄生活,同时,也反思着当下城镇化进程中偏远山村的发展去向。最后,引用名人的话语,能够增加文章的文化底蕴,更加深化文章主旨。十、阅读下面的文章,完成后面的小题。艾草,香如故肖朵朵①每当院子里那丛野生的迎春花爆出第一朵灿黄的小花时,我就知道,采艾叶的时节到了。②在那个几乎没有零食的童年,当我的上下牙齿终于可以沉溺在柔软、香甜的艾草米果中,那些柔软中丝丝缕缕牵扯出的艾叶会在我的口腔里跳舞,让我因冬天阴寒而凝滞的血液开始温暖地奔腾起来。妈妈说过:“艾草米果是养胃的。”③阳春三月,妈妈带着我们三姐妹走五里路去城郊采艾叶。直到现在,闭上眼睛,我仍能想起城郊那马尾松林拂过的松香,那时天上有一两只飞鸟掠过。我们把细嫩的艾草叶尖掐下以备做米果,绿色汁液染上我们的指尖。懂中医的妈妈轻声念叨着:“艾,在《尔雅》里叫冰台、艾蒿,在《名医别录》里叫医草,在《中药大辞典》里叫艾蓬、香艾……”这些富有草木气息的名字美得如此纯净,就像此时天空里大朵大朵的绵软白云。④艾的气息萦绕在我成长的岁月里。夜晚,我把双脚浸入艾草水中,让热气沿着穴位一路上扬。我在氤氲的热气中,走进艾草编织的大千世界。《诗经》中记载:“彼采艾兮!一日不见,如三岁兮!”道出古人对艾的炽热依恋:一日不见,如隔三秋;屈原在《离骚》里说:“户服艾以盈要兮,谓幽兰其不可佩。”讲述古人将艾缠绕于腰际,甚至比高洁的幽兰还惹人亲近;孟子更是直接宣扬艾叶的治病疗效:“犹七年之病,求三年之艾也。”这些叙述,像夏夜轻灵的萤火虫一样,在我的脑子里飞舞。只因在我的成长过程中,妈妈总是一边帮我悬灸,一边将这些向我娓娓道来。⑤点燃的艾灸,升起一束笔直的青烟,慢慢弥散,燃烧艾条的热气暖暖地渗透进皮肤。我也这样为我的孩子做艾灸。生命的轮回和传承如此微妙,竟然是这不起眼的艾草,教会了我外婆熏禳毒气、教会了我母亲强身健体、教会了我祛病延年。不知道,我的儿子以后会不会喜欢艾,他生命中还会不会有艾相伴。但我一定会告诉他,艾是这片厚重土地上平淡而执着的存在。而他或许还会记得,年少时艾灸的热气像羽毛划过身体,淡淡的,暖暖的,犹如母爱。一如我边帮他艾灸边念着自己写的小诗:“在袅袅的艾烟中,你睡着了,你不知道,我坐在你身旁守候你的梦话……”⑥那年春天,我们开车回到故乡,回到故乡那片我小时候采艾叶的马尾松林。我们下了车,雨下得又轻又软,墨绿的群山全被雨水濡湿成深青色,青草簌簌地拱动着生长,一层一层绒毛般的艾草已经布满山野。⑦雨过天晴时,我带儿子去那里采艾叶。笔直的松树树干直指天宇,山风吹拂,鸟语啁啾,如雾如岚的水汽依旧弥漫在林中,阳光透射而过,碎金点点,营造出那个我曾那么熟悉的世界。光阴流转,一户户人家已像蒲公英的种子一样播撒四方,好在,我们的根还扎在这故乡的土地上。而那一片片绿茸茸的艾草,总是在那里目送我们、等待我们。⑧云过高天,正引领我们从天的一头走向遥远的另一头。而我们脚下的艾草,静默不动,香如故。(选自2022年06月06日《解放日报》,有删改)43.选文回忆了与艾草有关的哪些生活场景?请简要概括。44.按要求品析语言。(1)这些富有草木气息的名字美得如此纯净,就像此时天空里大朵大朵的绵软白云。(从修辞的角度)(2)但我一定会告诉他,艾是这片厚重土地上平淡而执着的存在。(品析加点词语的表达效果)45.选文第④段引用了多篇描写“艾”的诗文,有何作用?46.选文标题意蕴丰富,请简要赏析。【答案】43.夜晚“我”用艾草泡脚;妈妈帮“我”悬灸;“我”为孩子做艾灸。 44.(1)运用比喻的修辞方法,生动形象地表现了艾草的美丽的名字给我带来的舒爽、优美的心理感受。(2)“平淡而执着”这个短语赋予了艾以人的美好品格,鲜明地表现了艾顽强的生命力和不追求奢华的美好品格。 45.引用艾的诗文,一方面介绍了艾的悠久历史、艾的美感和艾的功用,另一方面也表现了“我”对艾的热爱与痴迷;另外还使文章显得更为典雅,更有说服力。 46.“艾草,香如故”作为文章的题目,小的方面是说“我”对艾的热爱从小到大一直如故,从未改变;大的方面是说艾的历史悠久,传承久远,其“香如故”;这样使文章的内涵更为丰富,可读性更强。【解析】43.本题考查文章内容概括。根据第④段“夜晚,我把双脚浸入艾草水中,让热气沿着穴位一路上扬。我在氤氲的热气中,走进艾草编织的大千世界”,可以概括为:夜晚“我”用艾草泡脚;根据第④段“这些叙述,像夏夜轻灵的萤火虫一样,在我的脑子里飞舞。只因在我的成长过程中,妈妈总是一边帮我悬灸,一边将这些向我娓娓道来”,可以概括为:妈妈帮“我”悬灸;根据第⑤段“点燃的艾灸,升起一束笔直的青烟,慢慢弥散,燃烧艾条的热气暖暖地渗透进皮肤。我也这样为我的孩子做艾灸”,可以概括为:“我”为孩子做艾灸。44.本题考查句子赏析。(1)句中“像此时天空里大朵大朵的绵软白云”把“艾草的美丽的名字”比作“天空里大朵大朵的绵软白云”,是运用了比喻的修辞手法,“大朵大朵的绵软白云”形容非常柔软舒适,生动形象地表现了艾草的美丽的名字给“我”带来的舒爽、优美的心理感受,抒发了“我”对艾的喜爱之情。(2)“平淡而执着”形容人平凡而又坚韧顽强的精神,是运用了拟人的修辞手法,赋予了艾以人的美好品格,“平淡”写艾草生长在茂密的树林中,不甚起眼,表现了艾甘于平凡,不追求奢华的品质,“执着”是写艾草不管风吹日晒,依然顽强生长,表现了艾顽强的生命力,表达了作者对艾草的赞美。45.本题考查对文中内容的理解。①根据“《诗经》中记载:‘彼采艾兮!一日不见,如三岁兮!’道出古人对艾的炽热依恋:一日不见,如隔三秋;屈原在《离骚》里说:‘户服艾以盈要兮,谓幽兰其不可佩”讲述古人将艾缠绕于腰际,甚至比高洁的幽兰还惹人亲近;孟子更是直接宣扬艾叶的治病疗效:‘犹七年之病,求三年之艾也’”可知,引用诗文说明了艾的悠久历史,艾的美感和艾的功用;②“这些叙述,像夏夜轻灵的萤火虫一样,在我的脑子里飞舞”运用比喻的修辞手法,生动形象地表现了我对艾的喜爱之情;③引用诗文,语言典雅,增强了文章的文采。46.本题考查标题含义。根据第②段“在那个几乎没有零食的童年,当我的上下牙齿终于可以沉溺在柔软、香甜的艾草米果中,那些柔软中丝丝缕缕牵扯出的艾叶会在我的口腔里跳舞”、第④段“艾的气息萦绕在我成长的岁月里。夜晚,我把双脚浸入艾草水中,让热气沿着穴位一路上扬。我在氤氲的热气中,走进艾草编织的大千世界”、第⑤段“点燃的艾灸,升起一束笔直的青烟,慢慢弥散,燃烧艾条的热气暖暖地渗透进皮肤。我也这样为我的孩子做艾灸”、第⑥段“那年春天,我们开车回到故乡,回到故乡那片我小时候采艾叶的马尾松林”可知,从小到大,我都对艾草充满喜爱,还要告诉我的孩子艾草的平淡与执着,由此可知,“艾草,香如故”的表层含义是指我对艾草的执着热爱一如既往,一直未变;根据第④段“《诗经》中记载:‘彼采艾兮!一日不见,如三岁兮!’道出古人对艾的炽热依恋:一日不见,如隔三秋;屈原在《离骚》里说:‘户服艾以盈要兮,谓幽兰其不可佩。’讲述古人将艾缠绕于腰际,甚至比高洁的幽兰还惹人亲近;孟子更是直接宣扬艾叶的治病疗效:‘犹七年之病,求三年之艾也’”、第⑤段“生命的轮回和传承如此微妙,竟然是这不起眼的艾草,教会了我外婆熏禳毒气、教会了我母亲强身健体、教会了我祛病延年”可知,艾草很早就在古书中有记载,有关功用和历史等,同时我们家从外婆到母亲到我这一代都在对艾草进行传承;由此可知,“艾草,香如故”的深层含义是艾草历史悠久,传承久远,它的影响、它的功用一直未变;标题内涵深刻,可以增强文章可读性,吸引读者。九、说明文阅读 一、阅读下面的文字,完成下面小题。生物处理技术出手,垃圾成资源李禾固体废物被转化成生物能或有机肥①我国每年产生的畜禽养殖废弃物近40亿吨、主要农作物秸秆约10亿吨、一般工业固体废物约33亿吨、大中城市生活垃圾约2亿吨。固体废物产生量巨大,已成为环境的重要污染源。生物处理技术是利用微生物分解固体废物的处理技术,主要是处理废物中的有机成分。生物处理技术不仅能实现有机固体废物的减量化,还能变废为宝,将其转化为有机肥料以及生物质天然气等能源产品。②生物处理技术主要包括好氧堆肥技术、厌氧发酵技术和生物转化等。③好氧堆肥技术是指有氧的条件下,细菌、真菌、放线菌、纤维素分解菌、木质素分解菌等好氧微生物分泌在细胞内合成在细胞外起作用的酶——胞外酶,将固体废物中的有机成分分解为可溶性的有机质。这些有机质再渗入微生物细胞中,参与新陈代谢,从而实现固体废物向腐殖质转化,最终达到腐熟稳定,成为有机肥料或有机土壤等。而好氧堆肥反应会产生热量和二氧化碳。④厌氧发酵技术在无氧的条件下,利用厌氧或者兼性厌氧微生物降解有机固体废物,并花得甲烷和二氧化碳。该技术不仅能修实现垃圾无害化和减量化,还可以被得生物能甲抗和氢气,实现国体废物的资源化利用。⑤不过,在将有机物转化为沼气(即甲烷和二氧化碳)时,还会产生沼渣、沼液。厌氧发酵产生的沼渣、沼液富含未降解的有机物和矿物质,根据我国相关法规,还需要进行另一阶段的处理后进行回收。这是一个连续的过程。厌氧发酵工艺目前是处理厨余垃圾的主要工艺,近年来,在垃圾分类收集政策实施后,该技术在北京、上海等主要城市已获得逐步发展和应用。⑥生物转化是利用昆虫等有机废物转化为蛋白质和肥料的生物过程。整个过程产生的高黄量蛋白质,可作为动物饲,满足动物生长的营养需求。生物处理固废技术取得良好效果⑦近年来,我国在生物处理固体废物方面有大量的技术创新和成功案例。⑧江苏连云港市推出的餐厨垃圾处理项目,利用有机废物或厌氧发酵后的副产物沼渣养殖蝇蛆等昆虫,从而获得蛋白和肥料。即通过自动化上料设备,将菜有蝇蛆卵的养殖盒送入育雏单元房进行养殖。经3-4天的养殖孵化后,分盒加料后送到快速生长单元房养殖,养殖约4天后再通过筛分设备将成虫和虫粪分离。分离后的成虫是各类饲料蛋白的理想原料,可供应给禽类养殖场、水产、宠物等行业;虫粪是有机肥,可用于有机绿色农产品生产中。生物处理技术存在的诸多挑战⑨生物处理技术仍存在诸多挑战,比如好氧堆肥法占地面积大,邻避效应强,在中大型城市难以推广。厌氧发酵法经济性差、产气率低,生物质天然气的能源转换效率低,能直接将天然气并网的项目较少,大部分是转换为电能,存在较高的能量损失;副产物利用率低,大部分项目产生的沼渣、沼液不能资源化利用,还需投入较高水处理与焚烧处理成本;预处理技术有待进一步提高,国内垃圾分类尚处于起步阶段,收运的有机垃圾中会含有大量杂质,对厌氧发酵影响较大。(选自2022年2月29日《科技日报》,有删改)1.下面对选文的理解与分析错误的一项是( )A.生物处理技术是利用微生物分解固体废物的处理技术,固体废物中的所有成分都能用生物处理技术进行处理,将垃圾变废为宝。B.生物处理技术仍存在诸多挑战,预处理技术有待进一步提高,我们推行垃圾分类,可以减少有机垃圾中的杂质,对厌氧发酵有益。C.本文介绍了生物处理技术的三种方式、处理后能达到的效果以及运用生物技术处理固体废物面临的挑战三部分内容,按逻辑颜序安排材料,条理清晰。D.“在垃圾分类收集政策实施后,该技术在北京、上海等主要城市已获得逐步发展和应用”一句中,如果去掉“逐步”一词,将与原文意思不符,表达就不准确了。2.文中例举的江苏连云港市推出的餐厨垃圾处理项目运用了哪两种生物处理技术?请概述出该项目中两种处理技术的转化对象及其生成物。3.第①段划线句子主要使用了哪种说明方法?有何作用?【答案】1.A 2.厌氧发酵技术和生物转化。利用厌氧发酵技术转化餐厨垃圾生成沼渣,养殖昆虫;利用生物技术将昆虫成虫转化为蛋白,虫粪转化为有机肥。 3.列数字。用具体数字准确(或直观地)说明了我国固体废物产生量巨大,已成为环境的重要污染源,进一步说明利用生物处理技术处理固体废物的必要性。【解析】1.本题考查对文章内容的梳理、辨析。A.由第①段“生物处理技术是利用微生物分解固体废物的处理技术,主要是处理废物中的有机成分。生物处理技术不仅能实现有机固体废物的减量化,还能变废为宝,将其转化为有机肥料以及生物质天然气等能源产品”可知生物处理技术主要是处理废物中的有机成分,而不是“废物中的所有成分”,所以,理解有误;故选A。2.本题考查对内容的理解。第②段“生物处理技术主要包括好氧堆肥技术、厌氧发酵技术和生物转化等”和第③段“好氧堆肥技术是指有氧的条件下,细菌、真菌、放线菌、纤维素分解菌、木质素分解菌等好氧微生物分泌在细胞内合成在细胞外起作用的酶——胞外酶,将固体废物中的有机成分分解为可溶性的有机质”和第④段“厌氧发酵技术在无氧的条件下,利用厌氧或者兼性厌氧微生物降解有机固体废物,并花得甲烷和二氧化碳”和第⑥段“生物转化是利用昆虫等有机废物转化为蛋白质和肥料的生物过程”,可知生物处理的三个技术;由第⑧段列举的江苏连云港市推出的餐厨垃圾处理项目中,“利用有机废物或厌氧发酵后的副产物沼渣养殖蝇蛆等昆虫”,这是厌氧发酵技术;由第⑧段“分离后的成虫是各类饲料蛋白的理想原料,可供应给禽类养殖场、水产、宠物等行业;虫粪是有机肥,可用于有机绿色农产品生产中”可知,利用生物技术将昆虫转化为蛋白质,将虫粪转化为有机肥,这是生物转化技术;可据此作答。3.本题考查说明方法辨析及其作用。第①段划线句子“我国每年产生的畜禽养殖废弃物近40亿吨、主要农作物秸秆约10亿吨、一般工业固体废物约33亿吨、大中城市生活垃圾约2亿吨”中“40亿吨”“10亿吨”“33亿吨”“2亿吨”,运用了大量数据,这是列数字的说明方法;由第①段“固体废物产生量巨大,已成为环境的重要污染源”说明固体废物已成为环境的重要污染源;由第①段“生物处理技术不仅能实现有机固体废物的减量化,还能变废为宝,将其转化为有机肥料以及生物质天然气等能源产品”,说明生物处理技术的重要性;由此可知,第①段划线句子运用列数字的说明方法,具体准确的说明固体废物产量巨大,已成为环境的重要污染源,从而引出本文说明对象:生物处理技术处理固体废物的必要。二、阅读下面的文章,完成下面小题。神舟十三号凯旋,得益于这些“黑科技”①2022年4月16日上午,一朵“红白伞花”缓缓降落在内蒙古的东风着陆场,神舟十三号载人飞船成功着陆,3名航天员安全回家。在返回过程中,“黑科技”设备也顺利完成任务,护送航天员乘组平安“回家”。②4台着陆反推发动机是神舟十三号飞船上的重要设备,着陆反推发动机能否成功点火和正常工作,是决定航天员能否安全回家的“最后一棒”。返回舱的着陆过程对于航天员来说是个不小的挑战。在经历灼烧、黑障、开伞减速等程序后,返回舱仍然以近9米/秒的速度下降。而此时航天员背朝下面朝天坐在返回舱里,如此高的着陆速度将损伤航天员的颈椎。为了确保他们的安全,必须进一步减少冲击。而这一关键的“刹车”过程就由4台反推着陆发动机完成。为此,设计人员设计了一套出色的“刹车”动作:在返回舱距离地面1米时,4台反推着陆发动机必须在10毫秒内同时点火,大量燃气的积聚将在燃烧室内形成高压,最终从尾部的喷口中喷出,以反推力来减缓落地速度。每台发动机都能在瞬间产生大约3吨的巨大推力,这股巨大的反推力有效地抑制了返回舱的下坠势头,大大降低了飞船的下降速度。③从飞船与空间站分离开始到精准降落在东风着陆场,整个飞行过程都离不开惯性导航设备。惯性导航设备包括光纤惯性测量单元和二浮惯性测量单元。光纤惯性测量单元是飞船GNC分系统的关键单杭,用于测量飞船的角速度和加速度,通过给出准确测量信息,为宇航员准确返回着陆场提供关键数据信息,助力飞船成功进入返回轨道,确保飞船精准落地。二浮惯性测量单元位于飞船返回舱内,是飞船空间稳定运行和安全返回的关键单机,通过实时测量飞船的运动信息,精确控制飞船的姿态和速度,为飞船稳定运行和安全返回提供可靠保障。④“打开降落伞稳稳落地”是保障航天员安全“回家”的重要程序。在返回过程中,伞舱盖打开后,先拉出引导伞,再拉出减速伞。减速伞工作数秒以后会和返回舱分离,并拉出主伞,通过主伞,返回舱的落地速度会逐渐降低。此次从天而降的“红白伞”的主伞面积达1200平方米,由1900多块伞衣拼接而成,全部展开后可以覆盖3个篮球场,拉直长度近70米,能够横跨足球场,是世界上最大的环帆伞。4.下列对文章的理解和分析,不正确的一项是( )A.文章所说的三个“黑科技”分别是:反推着陆发动机、惯性导航设备和“环帆伞”。B.护送航天员乘组平安“回家”过程中,4台反推着陆发动机完成关键“刹车”动作。C.惯性导航设备在整个飞行过程中都起着非常重要的作用,为飞行提供安全保障。D.伞舱盖打开后,先拉出引导伞,再拉出减速伞,减速伞能使返回舱落地速度逐渐降低。5.下列对文章的理解与分析,不正确的一项是( )A.本文以先总后分的顺序介绍了神舟十三号载人飞船返回时使用的“黑科技”。B.“瞬间产生大约3吨的巨大推力”中“大约”一词体现说明文语言的准确性。C.光纤惯性测量单元主要是精确控制飞船的姿态和速度,保障飞船安全返回。D.文末列举了一组数字,并与篮球场作比较,突出环帆伞的面积是世界最大的。6.原文第②——④段是按技术含量的高低顺序排列的,现请选择一种恰当的说明顺序给它排序并说明理由。【答案】4.D 5.C 6.我会按时间顺序,以3、4、2的顺序来安排。因为第3段写导航,这是飞船与空间站分离时就开启,然后是第4段打开降落伞,最后是第2段,返回舱距离地面1米时发动机点火反推。【解析】4.此题考查内容理解。由第④段“伞舱盖打开后,先拉出引导伞,再拉出减速伞。减速伞工作数秒以后会和返回舱分离,并拉出主伞,通过主伞,返回舱的落地速度会逐渐降低”这句话可以得知,应该是主伞能使返回舱落地速度逐渐降低。此项“减速伞能使返回舱落地速度逐渐降低”理解有误。故选D。5.此题考查信息筛选与辨析。依据文章第③段“二浮惯性测量单元位于飞船返回舱内,是飞船空间稳定运行和安全返回的关键单机,通过实时测量飞船的运动信息,精确控制飞船的姿态和速度,为飞船稳定运行和安全返回提供可靠保障”可知,选项中的“光纤惯性测量单元”应该是“二浮惯性测量单元”。故选C。6.考查对文章内容的理解和顺序的安排。常用的说明顺序有:时间顺序、空间顺序、逻辑顺序等。本文第②段主要介绍返回舱在距离地面1米时,4台反推着陆发动机点火反推降速,是飞船返回的最后阶段;第③段主要介绍的是惯性导航设备保障飞船稳定运行、精准定位和安全返回,是飞船返回时的最初准备阶段;第④段介绍的是飞船打开降落伞返回落地前的阶段。据此可按飞船返回时的时间先后来安排(时间顺序),即:③④②。三、阅读下面的文字,完成下面小题。分子育种技术让水稻更高产①米饭是中国人餐桌上最常见的主食,但是我们今天习以为常的能吃饱饭,是在最近几十年才得以实现的。②水稻增产与育种、水利、肥料、农药等诸多因素有关,而起到核心作用的是育种。据中国科学院院士李家洋介绍,育种对我国农业增产的贡献率约为40%。③育种的目标,是希望获得各种特征都符合种植者与消费者需求的作物。作物具备的与农业生产相关的特征,被称作农艺性状。而所有的农艺性状,归根结底都是由其遗传物质DNA上的特定碱基序列——基因决定的。④认识到这一点后,育种学家开始有意识地运用诱变和杂交这两大武器来改造作物。诱变是通过运用物理或化学的办法来处理种子,提高作物基因改变的概率。将这些基因突变的种子种植下去后,再观察挑选其中的有益变异并逐步培育出新品种。而杂交则是将两个亲缘关系较远的品种进行人工授粉交配,再将其后代中发生了基因重组并具备某种优良性状的植株挑选出来,经过多代自交,最终培育出性状优良、遗传性状稳定的新品种。⑤尽管诱变和人工杂交技术让人们摆脱了对自然界中缓慢发生的变异和杂交的依赖,但无论是自然变异还是人工诱变,基因的变异都是随机发生的。而杂交育种则需要经过多代的筛选、会交和自交,耗费的时间常常要以十年计。显而易见,要想进一步提高育种的效率,精准改变基因是其关键。⑥在育种学家看来,高效精准育种的第一步,就是需要找到调控关键农艺性状的基因。不过水稻有4万多个基因,几乎是人类基因数量的两倍,一个一个基因去研究费时费力。加之基因和性状不是一一对应,而是既存在多个基因调控同一个性状,也存在一个基因影响多个性状的情况,基因和性状之间构成复杂的调控网络。为了解决这一难题,李家洋团队与合作者在水稻中率先应用了全基因组关联分析的方法。⑦全基因组关联分析方法是指:首先,记录大量不同品种植株的各种性状;随后,对这些植株开展基因组测序;接下来,对比不同品种间的差异,寻找其中的规律。假如若干植株均具有株高较高这一特征,而它们的某个基因又有着相同的变化,这个基因的作用很可能就是调控株高。⑧确定了控制性状的关键基因(对性状影响较大的基因,也称主效基因)后,下一步就是将这些基因按照育种学家的设计进行组合。传统的杂交技术不变,但是筛选不再是大海捞针,因为基因已经确定。当杂交的植株在实验室中刚出苗的时候,利用基因组测序或者利用对特定基因的标记追踪,就能很方便地找到具有理想组合的植株。通过对特定基因的分子标记追踪,能够获得将多种位于不同品种植株的优异基因集于一身的新品种,将传统育种不可能达到的目标变为可能。这就是李家洋提出并首先实践的“分子设计育种”。⑨那么,分子设计育种如何提高产量呢?这需要先确定哪种农艺性状与产量关系密切,然后寻找控制这种农艺性状的基因。⑩育种学家认为,与产量关系最为密切的农艺性状是水稻的株型。具体而言,就是水稻的茎秆基部分枝(称为分蘖)的数目和角度、植株的高度、叶片和穗的形态等。⑪意识到了株型对产量的意义后,育种学家提出来水稻“理想株型”或“新株型”的概念。即认为育种的目标之一是让作物具有最合适的株型,来最大限度地提高光合作用的效率,增加经济产量。李家洋解释道:“理想株型应当具备株高1.2米左右,4~7个分蘖,具有一定的分蘖角度、茎秆粗壮倒伏、根系发达、穗大粒重等特征。”⑫在接下来的十多年里,李家洋团队接连发现了多个与分蘖相关的基因。其中,于2010年发现的IPAI最为关键。李家洋表示:“通过调控IPAI的表达量,不但能够让水稻具有合适的分蘖,还让茎秆粗壮、根系发达。它甚至还能提高水稻的抗病能力,影响穗和粒的形态。”⑬李家洋团队很快将IPAI用于分子设计育种。通过杂交将具有IPAI优异变异的基因导入其他水稻中,可以让水稻获得理想株型状态,使产量提高约20%。基于这一进展,IPAI被誉为“新一代绿色革命基因”。⑭“农业已经进入了高速发展的基因组时代,育种科学正在发生翻天覆地的变化。”李家洋院士说。(选自2021年第11期《科学世界》,有删改)7.下列说法不符合文意的一项是( )A.农艺性状是指作物具备的与农业生产相关的特征,它是由基因决定的。B.农业进入了高速发展的基因组时代,诱变和杂交已经被新的育种科学技术淘汰。C.全基因组关联分析能够帮助育种学家确定控制性状的关键基因。D.分子设计育种提高产量既要确定与产量关系密切的农艺性状,还要寻找控制这种农艺性状的基因。8.概括什么是分子设计育种。9.简析第⑦段划线句子的说明方法及作用。假如若干植株均具有株高较高这一特征,而它们的某个基因又有着相同的变化,这个基因的作用很可能就是调控株高。10.简析下面句子中加点词的表达效果。理想株型应当具备株高1.2米左右,4~7个分蘖,具有一定的分蘖角度、茎秆粗壮不倒伏、根系发达、穗大粒重等特征。【答案】7.B 8.确定控制性状的关键基因,利用基因组测序或者利用对特定基因的标记追踪,获得将多种位于不同品种植株的优异基因集于一身的新品种。 9.举例子。通过列举发现某个调控株高的基因这一过程,具体说明了什么是全基因组关联分析方法。10.“一定”表示某种程度,在这里起限制作用。说明理想株型的分蘖角度适中,体现了说明文语言的准确性、严密性。【解析】7.本题考查理解文意的能力。B.依据⑭段“农业已经进入了高速发展的基因组时代,育种科学正在发生翻天覆地的变化”可知,本文只提到了农业进入了高速发展的基因组时代,育种科学正在发生巨大变化,并未提到“诱变和杂交已经被新的育种科学技术淘汰”这一说法。故选B。8.本题考查概括说明对象的能力。依据⑧段“确定了控制性状的关键基因(对性状影响较大的基因,也称主效基因)”“利用基因组测序或者利用对特定基因的标记追踪”“获得将多种位于不同品种植株的优异基因集于一身的新品种”即可概括什么是分子设计育种。9.本题考查分析说明方法及其作用的能力。依据“假如”可知,这是运用举例子的说明方法。依据“首先,记录大量不同品种植株的各种性状;随后,对这些植株开展基因组测序;接下来,对比不同品种间的差异,寻找其中的规律。”可知,本句通过列举发现某个调控株高的基因这一过程,具体说明了什么是全基因组关联分析方法。10.本题考查赏析说明语言的能力。“一定”表示某种程度的意思。依据“理想株型”“分蘖角度、茎秆粗壮不倒伏、根系发达、穗大粒重等特征”可知,这里说明的是理想株型的分蘖角度适中的标准和特征,从而体现了说明文语言的准确性和严密性。四、阅读下面的文章,完成小题。“月宫”过“一天”,人间过多久?记者:王珏玢 邱冰清①一直以来,人们对月亮有无尽的好奇与向往。宋代苏轼曾写下“不知天上宫阙,今夕是何年……追问月亮上的时间。那么,月亮上也有白天和黑夜的变化吗?月亮上的“一天”是多久呢?天文科普专家为您揭秘。②中科院紫金山天文台科普主管王科超介绍,地球上之所以有昼夜交替,主要是因为地球在不停地自转,同一个地点有时候照得到太阳,有时候照不到。天文学上,描述地球昼夜周期有一个专门的概念:太阳日。太阳连续两次经过同一子午线的时间间隔即为一个太阳日,平均为24小时。③与地球类似,月球也在不停地自转,也会产生昼夜交替现象。月球自转周期大约是27天,考虑到公转等影响,月球昼夜更替的时间比27天略长。如果类似太阳日,将月球上太阳连续两次经过同一子午线的时间间隔定义为“一天”,其平均长度是29.53天,也就是地球上一个朔望月的长度。④“月上‘一天’地上一月并不是巧合。事实上,朔望月和月球‘一天’反映出的,都是月面受到阳光照射的变化周期。只不过这一周期,在地球上看来是月相盈亏之变,而在月球上,则是长达半月的白天和长达半月的黑夜。”王科超说。⑤我国神话故事中常将月亮称为“广寒宫”,这一点并不为过。王科超说,月球的昼夜温差可达300摄氏度。白天,月球被太阳直射的地方可能高达120摄氏度:夜晚,月球表面温度可能降至零下180摄氏度。“这对 ‘五兔二号’月球车和嫦娥四号着陆器来说是极大的考验,也因此它们有月昼工作和月夜休眠两种模式。”⑥月球还是观星的好地方。由于没有大气层,月球上无论何时都能清楚地看到星星,且星星不会“眨眼”。“五兔二号”月球车曾在社交媒体上告诉网友“我看了超多的星星”。元宵节,也许当我们吃着元宵赏着月,“五兔二号”月球车和嫦娥四号着陆器也在月球上看星星呢!(来源:新华社,略有改动)11.下列说法不符合文意的一项是( ) A.地球和月球都有昼夜交替现象,主要是因为它们都能自转形成的。B.月上“一天”就是地上一月,在这一周期里,我们无论是在地球上还是月球上看到的都是月相盈亏之变。C.“玉兔二号”月球车和嫦娥四号着陆器有月昼工作和月夜休眠两种模式,是因为月球昼夜温差过大。D.在地球观星不如在月球上效果好,是因为地球有大气层,而月球没有。12.指出文章划线句子运用的说明方法,并分析其作用。【答案】11.B 12.运用了列数字和作比较的说明方法,科学准确地强调了月球上昼夜温差之大。【解析】11.本题考查信息的筛选和辨析。B.结合文章第④段“‘月上‘一天’地上一月并不是巧合。事实上,朔望月和月球‘一天’反映出的,都是月面受到阳光照射的变化周期。只不过这一周期,在地球上看来是月相盈亏之变,而在月球上,则是长达半月的白天和长达半月的黑夜。’”可知,在这一周期里,月球上看到的是长达半月的白天和长达半月的黑夜,B选项说法有误;故选B。12.本题考查说明方法及其作用。常见的说明方法有:举例子、分类别、下定义、摹状貌、作诠释、打比方、列数字、列图表、引用说明等。解答此题要根据说明文的常用说明方法及特征,然后判断文段内容,说明了事物什么特征,说明什么问题,根据这些来表达其作用。结合画线句中“300摄氏度”“120摄氏度”“零下180摄氏度”这些数据可判断出运用了列数字;结合“白天,月球被太阳直射的地方可能高达120摄氏度:夜晚,月球表面温度可能降至零下180摄氏度”可知将月球白天和黑夜的温度进行对比,这是运用了作比较的说明方法;运用列数字和作比较,语言具体准确,写出了月球昼夜温差大的特点,具有说服有力。五、阅读下文,完成各题。这就是ChatGPT①能跟人对答如流,会写策划方案和调研报告,也能撰写邮件、论文、脚本,制定商业提案,创作诗歌、故事,甚至让编写代码、检查程序错误都变得易如反掌……最近,这个名为ChatGPT的智能聊天机器人全网火爆,上线2个月活跃用户就轻松破亿。②之前,人们曾津津乐道的TikTk,拥有1亿用户用了9个月;而Facebk达到这一规模足足用了4年半之久。正如微软CEO纳德拉所言,“在我从事技术工作的30年里,ChatGPT是我从未见过的技术扩散。”③ChatGPT是人工智能公司OpenAI于2022年11月推出的聊天机器人,能通过学习理解人类的语言并进行对话,在提高办公和学习效率方面表现惊人。“ChatGPT可不是简单的智能聊天机器人,它是一个以自然语言为界面的智能机器人。”文渊智库研究员王超的解释虽然只有几字之差,但两者完全不同。④人机的自然语言通信能让机器具备听、说、读、写、译等人类具备的语言能力,让人机对话如同人际交流般顺畅丝滑。依靠庞大的资料库和含有1750亿个参数的语言大模型,ChatGPT可以把结构化的信息转译成自然语言,能深度理解人类语言,流畅地与人类对话。在进行讨论、被诘问、被辩难时,其回应( )符合上下文逻辑,( )还能模拟演示思考的过程,这是以往聊天机器人并不具备的能力。⑤有人认为ChatGPT是颠覆性的技术创新,不过,也有专家并不认同,觉得它只是一种交互方式的革新。ChatGPT本质上只是一个基本的自然语言处理流程,它或许更智能、更博学,但依然是尚处于原型阶段的聊天机器人。要靠它彻底解决难题,去替代需要大量专业知识或人工的工作,仍然为时过早。目前,ChatGPT最大的优势是效率,它能让很多事情变得高效,这无疑将影响就业市场。⑥不具备专业知识的普通人依然会认为ChatGPT“越来越像人了”,甚至还会联想到《流浪地球2》中不断给人类制造危机的MOSS,担心人类会被替代。全新的ChatGPT背后仍然是AI概念,而AI“替代和威胁人类”这一担忧早就存在,连霍金都有过类似的预言。这就难怪普通人会忧心忡忡。不过,我们暂时不需要有这样的担心。因为ChatGPT仅仅是一款智能工具,并没有传言中那么无所不能。它距离“完美”有着不小的距离,用户反映最多的问题就是它经常“一本正经地胡说八道”,存在事实性错误、知识盲区和常识偏差等诸多问题。原因之一大概就是,ChatGPT就像镜子,如果镜子前的信息鱼龙混杂,结果就可能一塌糊涂。⑦所以,当下人们要担心的,不是被替代,而是ChatGPT 广泛应用所带来的伦理和安全问题,学术造假、技术滥用、舆论安全等风险不容忽视。专家指出,针对这些问题,就需要在发展技术的同时,对ChatGPT应用边界加以管控,建立对人工智能生成内容的管理法规,同时加强治理工具的开发,通过技术手段识别人工智能生成内容。⑧有人问,既然ChatGPT是双刃剑,那我们为什么还要发展?其实,历史早已给出了答案。(节选自《科普时报》及“搜狐网”,有删改)13.联系上下文,以下填入第④段括号处的关联词,最恰当的一项是( )A.有时,有时B.不但,而且C.要么,要么D.由于,因此14.以下对上文的分析与理解,不正确的一项是( )A.第②段用TikTk、Facebk和ChatGPT作比较,突出ChatGPT扩散速度之快。B.作为以自然语言为界面的智能机器人,ChatGPT与人类的对话更加的顺畅自然。C.目前ChatGPT只是一款智能工具,还存在着问题,因此对就业不会有任何影响。D.“双刃剑”之说是客观而理性的,它指出了ChatGPT给人类带来的优势和担心。15.结合第③至⑤段,ChatGPT和以往的聊天机器人相比,其突破和优势在于:(1)__________________;(2)__________________。16.文章结尾,面对“既然ChatGPT是双刃剑,那我们为什么还要发展?”的问题,作者说“历史早已给出了答案”。你认为这一答案是:_______________(请完整表述)。科技史上能够印证这一答案的例子有:_____________。(举出一例,并用一句话说明)【答案】13.B 14.C 15.以自然语言为界面,符合逻辑,模拟演示思考过程; 让很多事情变得高效。 16.示例一:科技发展是必然趋势,我们可以用其优势造福人类。 塑料,虽然也造成了白色污染,但无数塑料制日用品的诞生给人们的生活带来便利。(示例二:科技发展对人类有利有弊,但大多数问题是发展中的问题,只能靠持续发展和进步才能解决。汽车给我们带来了便利,也污染了大气,新能源汽车就是依靠发展和进步,解决汽车污染问题。示例三:新的科技发明,本身没有问题,有问题的是使用技术的人。互联网将人类带入信息社会,而不法分子在网上的交易也更加隐秘,但这不是互联网的问题,而是人的问题,互联网是无辜的。)【解析】13.本题考查关联词的辨析。由第④段“这是以往聊天机器人并不具备的能力”可知,“符合上下文逻辑”与“还能模拟演示思考的过程”之间应该是递进关系,强调ChatGPT功能的强大。A.表示并列关系;B.表示递进关系;C.表示选择关系;D.表示因果关系;故选B。14.本题考查对文章内容的理解与分析。C.由第⑦段“它能让很多事情变得高效,这无疑将影响就业市场”可知, ChatGPT在某些领域会带来影响,故选项“对就业不会有任何影响”错误。故选C。15.本题考查对文章内容的理解与概括。由第③段“ChatGPT可不是简单的智能聊天机器人,它是一个以自然语言为界面的智能机器人”,第④段“其回应不但符合上下文逻辑,而且还能模拟演示思考的过程,这是以往聊天机器人并不具备的能力”可知,以自然语言为界面,符合逻辑,模拟演示思考过程;由第⑤段“目前,ChatGPT最大的优势是效率,它能让很多事情变得高效,这无疑将影响就业市场”可知,让很多事情变得高效。16.本题考查对文章内容的理解与拓展。开放性试题,答案不唯一,围绕“双刃剑”可知,ChatGPT等科技产品有利有弊,在造福人类的同时,也会带来一些问题,但科技本身没有问题,且全球科技化的发展趋势势不可挡,就此可举例来谈。示例:当今科技日新月异,科技向前发展的趋势势不可挡,我们可以用科技来造福人类。如空调的发明,空调使用的制冷剂,尤其是氟利昂制冷剂,对大气层不但有温室效应,而且会破坏臭氧层,形成臭氧层空洞。空调在使用过程中,会产生冷凝水,排水处理不当会造成各种污染、漏水等影响。但空调能使人们在适宜的温度下生活,使心情、睡眠、工作、学习都处在正常状态。六、阅读下文,完成下面小题。开启人工智能新纪元①ChatGPT 是由人工智能研究实验室OpenAI在2022年11月30日发布的全新聊天机器人模型,它能够通过学习和理解人类的语言来进行对话,还能根据前后的聊天内容轻松进行互动,真正像人类一样交流。②为什么ChatGPT一经发布就火爆得一塌糊涂?③它拥有超过人类的翻译、语言理解和表达、推理,甚至是编程和创意生成能力,并且有超强的学习意识和学习能力。随着用户使用的更深入和模型的进一步扩大,未来这些能力很可能会更强。其目前的状态犹如金庸小说《天龙八部》中没有开窍的段誉,六脉神创功力会越来越强。④众所周知,人类有举一反三的特质,但是过去的机器是没有的。过去为了完成一个特定任务,需要将大量样本“喂”给机器进行训练才可以实现。但现在ChatGPT模型可以用很少的样本甚至仅仅表达出我们的任务意图,就可以照葫芦画瓢,并做得非常好。这种类比能力是以往的人工智能所不具备的。不仅如此,情境学习的能力也使它脱颖而出。比如我们可以让模型去类比唐朝诗人李白、杜甫等人的诗句风格,再放上几个相关例子,它们可能就会造出带有这些诗人风格的全新诗句。⑤极为缜密的逻辑推理能力也为它添彩增色。举例说,如果提问ChatGPT模型一个非常专业的问题,它不仅会给出答案,而且逻辑和推理非常清晰,看上去很有道理。甚至对于一些奥数题目,它可以给出精准的答案和详细的推理过程。代码的超长上下文依赖,以及各种变量和模块的嵌套与组合功能的不经意习得,使得ChatGPT模型拥有了超强的“往前翻屏”去寻找、归纳和总结信息的能力。⑥它还拥有关似智序的高级技能。ChatGPT模型上市后,最有危机感的是主打搜索引攀的公司,搜索引擎提供的是一堆各种各样的答案片段,需要查询者自己去梳理和整理。但是ChatGPT模型能给出特别详细的一份调查报告,最让人吃惊的是其只用了一个模型,没有借助外部的其他系统。⑦虽然能力超群,但是ChatGPT模型还存在一些明显的缺陷,并不是无坚不摧。⑧它会产生一种莫名其妙的幻觉,会脑补出一些东西,进行一本正经的胡说八道。比如说,会把发生在某个人身上的故事说得有鼻子有眼地安放到另外一个人身上。而且,它依然不能避免传统人工智能的缺陷,受制于数据的时效性,会犯一些常识性的错误,因为ChatGPT模型受训的数据截止于2021年9月。同时,因为受制于大数据的质量和模型归纳偏见,会产生一些伦理偏差,特别是宗教、肤色、年龄性别等的偏见,还会无底线地讨好对话者,对于一些常识性的问题容易产生动摇。⑨ChatGPT模型是否会颠覆各大产业?ChatGPT模型的本质还是一种人机协同方式,对话者给机器一个主题,其会形成一个大纲,给出一系列的建议。这种功能在营销和文案写作领域有巨大的应用空间,但在传媒领域,它依然无法替代传统记者和编辑的工作;在教育领域的颠覆作用也没有想象得那么巨大,虽然它会给出答案,但是依然是作为参考,不同的学生对于答案有不同的理解和推理过程,其最后给出的作业和论文还是会有所区别。(选自《上海科技报》有删改)17.阅读文章第②-⑨段,回答以下问题。这几段内容,作者依次介绍了ChatGPT的三个方面:第一个方面:火爆的原因第二个方面:_______第三个方面:_______对于ChatGPT火爆的原因,作者说明了以下四点:(一)它拥有超过人类的多种能力和超强的学习意识与能力;(二)(3)__________(三)(4)____________(四)它拥有类似智库的高级技能。18.第③段加点词“很可能”体现了说明文语言的准确性,请具体分析:____________19.下列材料放在 段和 段之间合适?为什么?该模型一现身,仅用了5天时间就达到了100万名用户,推出2个月后用户已经过亿人。以色列总统艾萨克•赫尔左林甚至在今年2月1日发表了部分由人工智能撰写的演讲,成为首位公开使用ChatGPT的国家领导人。【答案】17.明显的缺陷 能否颠覆各大产业 它拥有举一反三的特质 它有极为缜密的逻辑推理能力 18.“很可能”表示推测,意思是在未来ChatGPT的能力有可能更强,也有可能不强,“很可能”体现了说明文语言的准确性。 19.放在第一段与第二段之间,该材料讲的是ChatGPT在短时间内用户大增,连总统也在其列。与第二段的内容相互承接。【解析】17.本题考查文章内容的梳理与概括。解答此题,首先要找到文章关键句,对选定文章进行分层,再梳理概括相应内容。①空,联系第⑦段“虽然能力超群,但是ChatGPT模型还存在一些明显的缺陷,并不是无坚不摧”可知,文章⑦⑧段重点介绍了ChatGPT明显的缺陷;②空,联系第⑨段“ChatGPT模型是否会颠覆各大产业?”可知,文章第⑨段介绍了ChatGPT能否颠覆各大产业;③空,阅读文章可知文章第②至⑥段,介绍了ChatGPT火爆的原因,联系第④段“众所周知,人类有举一反三的特质,但是过去的机器是没有的”“但现在ChatGPT模型可以用很少的样本甚至仅仅表达出我们的任务意图,就可以照葫芦画瓢,并做得非常好”可知,原因二可概括为:它拥有举一反三的特质④空,联系第⑤段“极为缜密的逻辑推理能力也为它添彩增色”可知,原因三可概括为:它有极为缜密的逻辑推理能力。18.本题考查说明文语言。“很可能”表示推测,说明事物按照预先设想的发展的趋势很大。在文中指在未来ChatGPT的能力变得更强的可能性很大,但也有可能不强,体现出说明文语言的严谨性和准确性。19.本题考查说明材料的回归。阅读材料可知,这段话主要讲了ChatGPT在极短的时间内,就收获了大量的用户,甚至连以色列的总统这样的社会精英都在其中,充分证明了ChatGPT的火爆程度。联系文章可知,本文在第②段提出了“为什么ChatGPT一经发布就火爆得一塌糊涂?”的问题,所以应放在第①段与第②段之间,来引出后文。据此总结概括即可。七、阅读下文,完成下面小题。噪音会伤害植物吗?①噪音危害,是一种让人类头痛的生态污染,这已经是广为人知的科学常识了。不论是汽车喇叭的刺耳尖叫,还是建筑工地的机械咆哮,都深深地折磨着城市的人类居民。同样,城市附近野生动物稀少,主要原因也在于它们怕吵,以至于被噪音赶走。②也许,世界上唯一不惧噪音的生物只剩下没有听觉、没有神经系统的植物了?事实并非如此。科学家发现,虽然噪音不会直接影响植物的生理活动,但可以通过改变动物的行为,间接影响植物的生长和繁殖。甚至,在噪音污染被消除后,这种影响仍然会持续影响很长一段时间。③从2005年开始,科学家们在美国新墨西哥州的响尾蛇峡谷进行了长达15年的一系列生态调查,专门研究噪音对于生物群落的影响。④响尾蛇峡谷的地理位置有些特别,一方面,该地区远离都市,物种丰富,有很多动物和植物。另一方面,该地区也是美国重要的天然气产地,这里开发了数千个天然气井,井口旁有很多嘈杂的机械,产生大量的噪音。⑤在这个峡谷,科学家挑选了115个相同面积的安静或者嘈杂的区域,分别观察这些区域中植物的生长情况。结果发现,植物群落确实会受到噪音污染的影响。2007年的研究数据显示,与远离天然气井的安静区域相比,天然气井附近的嘈杂区域的植物多样性更低,而且植物幼苗少得多。比如,平均每片安静区域中大约会有55棵矮松幼苗和29棵刺柏幼苗。而嘈杂区域平均只有13棵矮松幼苗和4棵刺柏幼苗。⑥为何嘈杂区域的植物会“减产”?科学家分析,这与栖树鸟类对噪音的反应有关。比如,北美灌丛松鸦与矮松的关系非常密切,它们以矮松的果子为主要食物。灌丛松鸦非常聪明,养成了埋藏食物的习惯,这样可以保证冬天也不会挨饿。他们有时候会忘记了埋藏食物的地方,那些被遗忘的矮松种子就成了幸运儿,在春暖花开的时候发育成小树苗。于是,灌丛松鸦无意中也成了矮松的“播种者”。⑦灌丛松鸦讨厌噪音,嘈杂的区域,就很少飞过去。即使那里有矮松种子,它们顶多速去速回。这样一来,嘈杂区域的矮松获得播种的机会就越来越少,最终导致了群落的衰落。⑧噪音对于矮松更大的危害还在于,噪音污染带来的负面影响会持续很长时间。灌丛松鸦非常聪明,会记得被噪音折磨的地点和情景,所以噪音即使消除,灌丛松鸦也不会立即飞回先前的嘈杂区域。2007年之后,响尾蛇峡谷的天然气井设施得到了改造升级,噪音污染得到了有效治理。2019年这里已经变得很安静了,科学家对这里的植物进行了考察,但是他们发现鸟类没有返回,幼苗也依旧数量稀少。⑨科学家认为噪音对于矮松群落的负面影响时间可能比人们想象的要长得多,人类必须开始重视噪音对于植物的影响,这样才能保护好地球复杂的生态环境。(选自2022年第3期《大科技》,有删改)20.下列对选文的理解和分析,不正确的一项是( )A.响尾蛇峡谷曾经物种丰富,后来这里开发了数千个天然气井,物种数量也随之改变。B.北美灌丛松鸦以矮松的果子为主要食物,灌丛松鸦无意中也成了矮松的“播种者”。C.噪音通过改变动物的行为,间接影响植物的生长和繁殖,人类要重视治理噪音污染。D.有效治理响尾蛇峡谷的噪音污染后,尽管幼苗数量依旧稀少,但灌丛松鸦开始返回。21.第⑤段主要运用了作比较的说明方法,有何作用?22.襄江公园里,晨练的人往往都长时间放着高分贝音乐。为制止这类行为,请结合本文知识,代公园管理处拟写一条标语。【答案】20.D 21.运用作比较的说明方法,具体准确地说明了噪音对于生物群落有着严重的影响,会导致植物的“减产”。 22.示例:用虔诚的态度对待生活,用安静的方式对待他人。【解析】20.考查辨析信息。D.根据第⑧段中的“科学家对这里的植物进行了考察,但是他们发现鸟类没有返回,幼苗也依旧数量稀少”可知,本项“但灌丛松鸦开始返回”有误。故选D。21.考查说明方法。联系第⑥段中的“为何嘈杂区域的植物会‘减产’?科学家分析,这与栖树鸟类对噪音的反应有关”可知,“平均每片安静区域中大约会有55棵矮松幼苗和29棵刺柏幼苗。而嘈杂区域平均只有13棵矮松幼苗和4棵刺柏幼苗”运用作比较的说明方法,具体准确地说明了噪音对于生物群落有着严重的影响,会导致植物的“减产”。22.考查拟写标语。开放类试题,结合文本相关知识,语言简洁,呼吁人们保持安静,保护生态即可。如:还鸟儿以清静,给自然以空灵。八、阅读《飞花令的前世今生》一文,完成下面小题。飞花令的前世今生李定广①飞花令是我国古代的酒令游戏之一。古代宴会中,为了佐饮助兴,推一人为令官,其余的人听其号令,轮流做指定的游戏,或说出指定的诗文,或对对子,或讲故事,或说笑话,或唱曲子等,违令或输的人饮酒。②酒令种类成百上千,汉唐两朝尤其繁盛,其中文化素养较高的人喜欢玩诗文一类的酒令游戏,称为“雅令”。③唐宋时期“雅令”盛行,由此产生了一大批唐宋“小令词”,就是我们所说的“唐宋诗词”的“词”,比如《调笑令》《三台令》。唐代诗人王建的《调笑令》曰:“团扇,团扇,美人病来遮面。玉颜憔悴三年,谁复商量管弦。弦管,弦管,春草昭阳路断。”酒宴上创作出这样诙谐又经典的“小令词”,无疑会给酒宴带来浓厚的文化气息。④明清以来,人们常玩的“雅令”为飞花令。“飞花”一词出自唐代诗人韩翃《寒食》诗首句“春城无处不飞花”。飞花令早期只以“花”字为令,规定每人说一句嵌有“花”字的诗词,说不出或者说错了诗句要罚酒。若在座的都是高手,则增加难度,对“花”字的位置作出严格要求,一般是依次出现,一轮完成,不断循环。“飞花令”在明代越玩越火,后来逐渐不局限于“花”字,诸如“春”“月”“红”等这些古诗词里出现频率较高,且又有美好寓意的字,也成为飞花令的常用令字。⑤近半个多世纪以来,飞花令骤然沉寂,无论是在现实酒宴中还是在大众读物里,基本消失了踪影。直到2017年《中国诗词大会》第二季播出,飞花令如天女散花般回归了,带给人们“春城无处不飞花”的惊艳感受。《中国诗词大会》专家团队发掘出这一古老的文化品种,并对旧式飞花令进行了三点改造,让其适应新时代的文化土壤。⑥首先,让飞花令大众化。旧式飞花令主要活跃在文人雅士的宴会中,现在要让它真正“飞入寻常百姓家”,最重要的是降低难度门槛。如不限定令字在诗句中的位置,这样就能让全民都“玩得起”。⑦其次,创造新的有“难度梯队”的玩法。为了适应比赛,也为了一些诗词高手的需求,《中国诗词大会》创造出三个难度等级的飞花令:单字飞花令、双字飞花令、超级飞花令。“超级飞花令”主要指由类与类组合的飞花令,如“数字+季节”“数字+酒器”“颜色+植物”等,也包括成语飞花令等难度较大的玩法。2021年播出的《中国诗词大会》第六季又升级并更名为:横扫千军、组合飞花令、超级组合飞花令。“横扫千军”是选手一人以“双字飞花令”对战百人团。“超级组合飞花令”的“超级”体现在两个方面:一是用类与指定字的组合,如植物+指定字“红”;二是偶然性组合,在滚动字牌中选择。⑧再次,让大众感受到飞花令的美感。旧式飞花令一般只说一句诗词,改造后的飞花令要求必须说两句。中华诗词的美感优势之一就是上下句节奏、对称和平仄相对而形成的节奏美、整齐美(参差美)以及抑扬美。⑨玩新式飞花令,颇能体现“腹有诗书气自华”的才情,又能表现敏捷和机智,更能展现中华审美的魅力,深受人们喜爱。新式飞花令,已成为当下大众喜闻乐见的诗词游戏,也成了人们学习诗词的方式之一。(选自2022年01-02期《读者·读点经典》,有删改)23.下列关于“飞花令”的理解,不正确的一项是( )A.飞花令是我国古代宴会用来佐饮助兴的酒令游戏之一。B.古代酒令形式多样,可以对对子,讲故事,说笑话,唱曲子等。C.飞花令中的“飞花”一词源自韩翃诗句“春城无处不飞花”。D.飞花令必须以“花”字为令,规定每句诗词嵌有“花”字。24.下列对文章第⑦段运用说明方法的理解,不正确的一项是( )A.运用了作诠释的说明方法,便于读者对内容的理解。B.运用了举例子的说明方法,使得说明更具体。C.运用了作比较的说明方法,来说明《中国诗词大会》创新的三种玩法。D.本段综合运用了多种说明方法,更加具体详细地说明了《中国诗词大会》对旧式飞花令改造中创造出的新式玩法。25.下列对文章的分析,不正确的一项是( )A.文章首先点出说明对象,再以时间为序对飞花令的前世今生一一进行说明,最后指出飞花令的作用。B.文章第⑤~⑧段运用了总分的结构,第⑤段总说,⑥⑦⑧段分别用“首先”“其次”“再次”进行分说,条理清晰。C.第⑧段中的加点词语“一般”,说明了旧式飞花令多数情况下说一句诗词,不排除特殊情况,体现了说明文语言的严谨准确。D.本文是一篇事理说明文,依次介绍了飞花令的玩法、由来及《中国诗词大会》对旧飞花令的改造等内容。【答案】23.D 24.C 25.D【解析】23.本题考查文本信息的筛选与辨析。D.根据第④段“飞花令早期只以‘花’字为令,规定每人说一句嵌有‘花’字的诗词”以及“‘飞花令’在明代越玩越火,后来逐渐不局限于‘花’字,诸如‘春’‘月’‘红’等这些古诗词里出现频率较高,且又有美好寓意的字,也成为飞花令的常用令字”可知,选项中“飞花令必须以‘花’字为令,规定每句诗词嵌有‘花’字”的表述与原文不符。故选D。24.考查对说明方法的理解与辨识。根据第⑦段“‘超级飞花令’主要指由类与类组合的飞花令”“‘横扫千军’是选手一人以‘双字飞花令’对战百人团。‘超级组合 飞花令’的‘超级’体现在两个方面:一是用类与指定字的组合,如植物+指定字‘红’;二是偶然性组合,在滚动字牌中选择”分别解释了什么是“超级飞花令”,什么是“横扫千军”,以及超级组合飞花令的“超级”指什么,可知运用的是作诠释的说明方法,使得说明更具体。根据第⑦段“‘超级飞花令’主要指由类与类组合的飞花令,如‘数字+季节’‘数字+酒器’‘颜色+植物’等,也包括成语飞花令等难度较大的玩法”中关键词“如”的提示,可知运用了举例子的说明方法,使得说明更具体、生动。第⑦段“2021年播出的《中国诗词大会》第六季又升级并更名为:横扫千军、组合飞花令、超级组合飞花令”只是进一步说明难度升级之后的内容,并没有使用作比较的说明方法。据此,答案为C。25.考查对文章内容的理解分析。D.本文是一篇事物说明文,文章先介绍了飞花令的由来,之后介绍了玩法以及《中国诗词大会》对旧飞花令的改造等内容。故选D。九、阅读下面选文,完成下面小题。冬奥会的冰,咋制成?①在冬奥会冰雪项目高水平竞技中,光滑稳定的高质量冰面至关重要。运动员在冰雪世界的精彩表现,更是离不开这一“硬核”基础条件。②冬奥会的冰面制作对平坦度、光滑度的要求十分严苛。如在冰壶比赛中,冰面仅仅1毫米的平坦度偏差,就会对冰壶的走向产生很大影响,进而影响选手的发挥。那么北京冬奥会的冰,是怎样制成的呢?③冰面的制作过程很复杂。整个过程大概分为制作冻底冰、喷白、封白、画线、贴lg、封线、涨水制作表面冰几个步骤。首先在赛场浇筑纯净水,制冰师需要用喷棒均匀地把水喷洒在赛场上,并确保它和混凝土地面完全贴合;待水结冰并达到一定温度后,开始喷白漆,使赛场成为公众较为熟恐的奶白色;待白凑固定后,进行贴lg和画线;再进行涨水制作表面冰,最后用扫冰车打磨和找平,制冰工作才算真正完成。这个过程中,每一寸、每一层冰面都需要制冰师傅的精心雕琢,这样才能为赛会制作出高质量的冰面。④除了制冰师的匠心,制冰工作还有“黑科技”的加持。相比于传统的建冷式冰场,本届冬奥会中,我国首次大规模采用了二氧化碳跨临界直冷技术。该技术能够将冰面的温度差控制在0.5℃以内,远低于国际滑联冰面温差不超过1.5℃的标准;据了解,二氧化碳跨临界直冷制冰技术是当前冬季运动场馆最先进、最环保、最高效的制冰技术。它的应用不仅能够为比赛提供温度、厚度均衡的冰面,同时碳排放量也接近于零,有助于打造出最快、最环保的冰面。⑤ 不同的冰上运动对冰面有着不同的要求。如被举为“冰上舞蹈”的花样滑冰要软橘的冰面助力起跳,冰面温度要求-4℃到-3℃:短道速滑则需要脆硬的冰面助力提速,冰面温度为-7℃到-6℃。历届比春中,这两个项目几乎都被安排在同一个场馆,有时二者的举行时间也相对接近,北京冬奥会也不例外,按照国际奥委会的要求,从“最快”到“最美”的冰场转换要在三小时内完成,而我国采用的制冰效能更高的二氧化碳制冰法在两小时内就能完成冰场转换。实现冰面转化后,冰面监测系统可以测到冰面不同区域的硬度和温度,确保冰面质量达到竞赛的需求。(选自“学习强国”学习平台,有删改)26.下面的说法不符合原文意思的一项是( )A.冬奥会的冰面制作对平坦度、光滑度的要求很严苛。B.冰面的制作过程很复杂,每一寸、每一层冰面都需要制冰师傅的精心雕琢。C.二氧化碳跨临界直冷技术的应用碳排放量等于零,有助于打造出最快、最环保的冰面。D.不同的冰上运动对冰面有着不同的要求。27.以下哪个选项不是二氧化碳跨临界直冷制冰技术的“最先进、最环保、最高效”具体体现( )A.能够将冰面的温度差控制在0.5℃以内,远低于国际规定的温差标准。B.碳排放量接近于零。C.两小时内就能完成冰场转换,比国际奥委会要求的时间快一个小时。D.在赛场浇筑纯净水,制冰师需要用喷棒均匀地把水喷洒在赛场上,并确保它和混凝土地面完全贴合。28.下面对本文的理解分析,不正确的一项是( )A.这篇说明文首先介绍了冬奥会上制冰过程的步骤,接着介绍了根据比赛需要而对制冰工作的要求,最后介绍了制冰工作中的黑科技,表达了对制冰师的赞美之情。B.第②段中加点词语“仅仅”,强调了冰壶比赛对冰面的质量要求高,体现了说明文语言的准确、严谨。C.第④段中画线句子起承上启下的过渡作用。承接第三段制冰师高超的工艺,引领下文制冰工作中的科学技术。D.第⑤段中画线句子运用了举例子、列数字的说明方法,具体准确地说明了不同的冰上运动对冰面有着不同的要求。【答案】26.C 27.D 28.A【解析】26.本题考查对文章内容的理解与分析。C.由文章第④段“同时碳排放量也接近于零,有助于打造出最快、最环保的冰面”可知,原文说的是“接近于零”,而非选项说的“等于零”。故选C。27.本题考查对文章内容的理解与分析。D.由第③段“冰面的制作过程很复杂”“首先在赛场浇筑纯净水,制冰师需要用喷棒均匀地把水喷洒在赛场上,并确保它和混凝土地面完全贴合”可知,这是冰面的制作过程,而非“二氧化碳跨临界直冷制冰技术”。故选D。28.本题考查对文章内容的理解与分析。A.由第④段“除了制冰师的匠心,制冰工作还有‘黑科技’的加持”和第⑤段“不同的冰上运动对冰面有着不同的要求”可知,文章先介绍制冰工作中的黑科技,然后介绍了根据比赛需要而对制冰工作的要求,选项表述顺序有错。故选A。十、阅读下面的文章,完成下面小题。大脑中的热力学刘恺大脑与熵(shāng)①为什么热量总是从较热的物体传递到较冷的物体?为什么凋落的树叶不会重回枝头,枯萎的花朵不会重新绽放?②自然界中宏观事物的发展过程都有一个优先的方向,遵循的规律便是热力学第二定律,也称“熵增加原理”,它指的是在一个孤立系统里,如果没有外力做功,其无序的程度(即熵)会不断增大。(A)③人的生命也如此,从物理层面的运动到精神层面的思考,都会消耗能量并产生熵,打破原有的平衡。我们的大脑也是如此,当我们思考时,大脑会消耗能量,产生熵。(B)④热力学第二定律,有几种常用的表述方式。德国物理学家克劳修斯将其表述为:热量不能自发地从温度低的物体传递到温度高的物体。英国物理学家威廉·汤姆逊(开尔文勋爵)和德国物理学家马克斯·普朗克将其表述为:不可能从单一热源吸取热量,使之完全变成功,而不产生其他影响。熵增加原理表述为:孤立系统的熵永不自动减少,熵在可逆过程中不变,在不可逆过程中增加。思考越高级熵越多⑤人类的大脑每天需要接收、记忆各种信息,学习、理解各种知识,分类、处理各样事务。这意味着大脑的熵在不断增加,若不及时优化、排序,大脑就会越来越无序。(C)⑥研究人员利用功能性核磁共振成像(FMRI)技术,观测到大脑不同认知功能的区域消耗的氧气量,从而计算出大脑不同时间、不同位置的能量消耗情况,并通过模型,计算出其产生的熵。⑦研究发现,当大脑处于休息状态时,各个脑区处于不同状态且不断转换,但各个状态之间的转换是随机、可逆的。我们认为这是一种“细致平衡(detailed balance)”状态。⑧而当大脑处于思考状态时,各个脑区在不同状态间的转换存在方向性,这被称为“细致平衡破缺(brken detailed balance)”。也就是说,在思考的过程中,神经活动的“细致平衡”被打破了,这时表示混乱程度的熵也增加了。⑨研究人员发现,在情绪处理、工作、社交、语言、理性思考、风险决策、运动执行等7种不同类型的大脑认知活动中,运动执行和风险决策等活动中的熵产生最明显,而语言、情绪处理等活动中的熵产生较少。也就是说,进行越高级、越复杂的认知活动,大脑产生的熵越多。挖掘大脑的潜能⑩结合这一科学研究与自己用脑学习时的情况想一想、观察一下,前文中说的与自己的情况是不是一样?利用这项研究可以让我们在未来学得更好或变得更聪明吗?我们又要如何对抗大脑熵的增加呢?⑪研究表明,当学习的速度慢下来时,神经元中熵的增加会减缓,从而使我们的学习效率不降反升。⑫对于每个人来说,大脑是我们身体的总指挥部,对抗大脑熵增加的方法就是科学用脑,不断挖掘大脑的潜能。例如,在学习方面,可以不断地重复、慢慢雕琢、反复练习,要知道“欲速则不达”“慢工出细活”反而学习效率更高。或者去看一场电影、读一本书、与朋友进行一次畅谈,从中探索到新鲜的信息、知识或智慧,为大脑“减熵”。⑬人类对自身大脑的认知与探索神秘而迷人的宇宙一样,充满未知却又其乐无穷。想要真正解锁人类大脑蕴藏的玄机还有很长的路要走,但相信总有一天,人们能解开大脑的终极奥秘!知识链接——“贪吃”的大脑⑭在人体中,大脑的重量仅占体重的2%,但是消耗的能量却达到了整体消耗的20%~30%,大脑“贪吃”葡萄糖的习惯使其成为全身耗能最大的器官。(选自《知识就是力量》2023-01-12,有删改)29.下列各项与原文内容一致的是( )A.热力学第二定律,也称“熵增加原理”,它指的是在一个孤立系统里,其无序的程度(即熵)会不断增大。B.情绪处理类型的大脑认知活动比风险决策活动产生的熵少。C.不断地重复、慢慢雕琢、反复练习,当学习的速度慢下来时,神经元中不产生熵,从而使我们的学习效率不降反升。D.当我们思考时,大脑会消耗能量,产生熵。什么都不想,就不会产生熵。30.请将下列语句放到文中最恰当的位置(在A、B、C中选一处),并说说这些文句在文中能起到怎样的作用。比如,整洁的桌面,如果我们不随时整理、清洁,使用一段时间后,会变得杂乱无章。桌面的混乱程度,就是熵。若我们一直不整理(即没有外力做功),桌面就会越来越乱(即熵会不断增大)。31.简要分析“知识链接——‘贪吃’的大脑”语段的语言特点。【答案】29.B 30.C处。运用打比方的说明方法,把大脑的及时优化排序比成随时整理清洁桌面,生动形象地说明如果没有外力做功,熵就会不断增大,大脑就会越来越无序,通俗易懂。 31.该语段语言既准确严谨,又生动。列举了准确的数据,体现语言的准确严谨;大脑“贪吃”,运用拟人,生动形象地说明大脑消耗葡萄糖地特点,体现出语言的生动。【解析】29.本题考查内容理解和辨析。A.结合第②段“遵循的规律便是热力学第二定律,也称‘熵增加原理’,它指的是在一个孤立系统里,如果没有外力做功,其无序的程度(即熵)会不断增大”可知,其无序的程度(即熵)会不断增大的前提条件是如果没有外力做功,因此该选项有误;C.结合第⑫段“研究表明,当学习的速度慢下来时,神经元中熵的增加会减缓,从而使我们的学习效率不降反升”可知,当学习的速度慢下来时,神经元中也产生熵,但是增加会减缓,因此该选项有误;D.结合第③段“人的生命也如此,从物理层面的运动到精神层面的思考,都会消耗能量并产生熵,打破原有的平衡”和第⑦段“研究发现,当大脑处于休息状态时,各个脑区处于不同状态且不断转换,但各个状态之间的转换是随机、可逆的。我们认为这是一种‘细致平衡(detailed balance)’状态”可知,该选项说法错误;故选B。30.结合第⑤段“人类的大脑每天需要接收、记忆各种信息,学习、理解各种知识,分类、处理各样事务。这意味着大脑的熵在不断增加,若不及时优化、排序,大脑就会越来越无序”可知,该段讲的是要及时处理大脑中的信息,不然会混乱无序,而语句“比如,整洁的桌面,如果我们不随时整理、清洁,使用一段时间后,会变得杂乱无章。桌面的混乱程度,就是熵。若我们一直不整理(即没有外力做功),桌面就会越来越乱(即熵会不断增大)”,讲的也是要及时整理清洁,与选段内容一致。并且该语句是用了打比方的说明方法,把大脑的及时优化排序比成随时整理清洁桌面,生动形象地说明如果没有外力做功,熵就会不断增大,大脑就会越来越无序,更加通俗易懂。故放在C处。31.本题考查说明文语言特点结合链接材料“大脑的重量仅占体重的2%,但是消耗的能量却达到了整体消耗的20%~30%”可知,本段材料列举了准确的数据,体现了说明文语言的准确性。结合链接材料“大脑‘贪吃’葡萄糖的习惯使其成为全身耗能最大的器官”可知,本段材料将大脑拟人化,生动形象地说明大脑消耗葡萄糖地特点,体现出语言的生动。据此可分析,该语段语言既准确严谨有生动形象。十一、阅读下文,完成下面小题。耐寒植物如何过冬李耕拓①植物生长活动的最低温度通常是 0℃。秋天之后,有些植物特别是很多一年生草本植物纷纷枯萎。到了更为寒冷的冬季,冰封的大地上几乎看不到红花绿叶,但也有些“英雄好汉”是不怕严寒的。②通常而言,即便是同一种植物,冬季和夏季的抗冻能力也不一样。在夏季活 动期多不耐寒,在冬季休眠期则更为耐寒。这是因为春夏季节,植物生长旺盛,养分消耗多于积累,因而其抗冻能力较弱。如北方的梨树,在-30~-2℃低温下能平安越冬,在春天却抵挡不住微寒的袭击;松树的针叶,冬天能耐-30℃的严寒,夏天如果人为地降温到-8℃就会冻死,就是这个道理。③到了秋天,情形就变了,秋季白昼温度高日照强,叶子的光合作用旺盛;而夜间气温低,树木生长缓慢,养分消耗少,积累多,于是树木越长越“胖”,变粗壮并木质化,树叶里合成了更多的脱落酸(休眠素),这种植物激素被输送到植物枝梢的尖端和侧芽后,这些部位的新陈代谢会受到抑制,从而进入休眠状态,不再萌芽生长,植物体也停止生长。这意味着植物的物质和能量消耗大大减少,养分因此被积蓄起来,树木逐渐有了抵御寒冷的能力,即使叶子在冬天被冻掉,小枝依旧完好无损。④耐冻植物都有休眠的特点,它们常使用“沉睡”的妙法来对付冬寒。一般而言,处于休眠状态的植株抗寒力强,并且植株休眠越深,抗寒能力越强。事实上,多年生植物的季节性休眠是长期自然选择的结果,是植物应对不利环境的一大绝招。⑤此外,每一棵树木都有一副“甲胄”,保护它们娇嫩的组织不受寒气侵袭。这副“甲胄”就是木栓层。每年夏天,树木都在树干和树枝的皮下储存木栓组织—— 死的间层。木栓既不透水,也不透气。停滞在其气孔中的空气能够阻挡树木的热量向外散发。树木年龄越大,木栓层越厚。因此,老树、粗树的抗寒能力比枝嫩干细的小树强。⑥另外,植物还常常会通过细胞膜脱水或合成液态抗冻有机物来增强细胞的抗冻性。有的植物会通过降低自身含水量以适应低温环境,安全过冬。具体来说,就 是将水从细胞内排到细胞外,防止细胞内的水结冰。⑦植物之所以能够这样做, 是因为其体内既有自由水, 也有结合水, 二者的“脾气”大相径庭:自由水在 100℃沸腾,0℃时结冰;结合水要高于 100℃才沸腾, 在比 0℃低得多的温度下才会结冰。冬天,植物体内的自由水减少后结合水所占比例则相对增加,使细胞液浓度增高,不易结冰,植物因此可以忍受更低的温度。⑧当然, 如果以上方法还不足以抵抗严寒, 一些植物还会通过增加糖或蛋白质、脂肪的含量,或者增强生物膜系统结构的稳定性来练就更高更强的御寒本领。(有删改)32.下面有关本文的理解分析,不恰当的一项是( )A.第①段加点词“几乎”表示对程度的界定,体现了说明文语言准确、严密的特点。B.第③段加点的“越长越胖”,生动形象地写出了树木积累养分越来越多的样子。C.第⑤段中加点的“甲胄”,是指树木在树干和树枝的皮下储存的木栓组织。D.本文是一篇事理说明文,作者按照时间顺序介绍了一些植物耐寒抗冻的方法。33.通读全文,概括本文主要介绍了耐寒植物过冬的哪些方法。34.简要分析第②段划线句子所使用的说明方法。【答案】32.D 33.①使用“沉睡”休眠的方法来对付冬寒(答“进入休眠状态,减少物质和能量消耗”或“蓄积物质和能量,减少消耗”也可);②形成“甲胄”——木栓层,保护它们的组织不受寒气侵袭;③通过细胞膜脱水或合成液态抗冻有机物来增强细胞的抗冻性。 34.示例一 举例子。作者列举北方的梨树和松树的针叶这两个例子,具体、形象地说明了植物不同季节抗冻能力不一样的特点。示例二 作比较。为了说明植物在不同季节抗冻能力的差异,作者运用作比较的方式,以冬天低温和春天微寒北方梨树不同的状态作比较,以夏天-8℃的低温和冬天-30℃的严寒松树针叶的不同反应作比较,给读者以清晰的印象。示例三 列数字。为了准确地说明“即便是同一种植物,冬季和夏季的抗冻能力也不一样”,作者采用列数字的方式,如“-30~-2℃”“-30℃”“-8℃”等,让说明严密、可信。 【解析】32.考查内容理解。D.本文首段引出说明对象:植物的耐寒。第②段和第③段介绍了植物冬季比夏季更为耐寒的原因。第④段到文末介绍了植物几种对付冬寒的方法。这是逻辑顺序。故选D。33.考查筛选信息。根据第④段中的“耐冻植物都有休眠的特点,它们常使用‘沉睡’的妙法来对付冬寒”可得:使用“沉睡”休眠的方法来对付冬寒。根据第⑤段中的“每一棵树木都有一副‘甲胄’,保护它们娇嫩的组织不受寒气侵袭。这副‘甲胄’就是木栓层”可得:形成“甲胄”——木栓层,保护它们的组织不受寒气侵袭。根据第⑥段中的“另外,植物还常常会通过细胞膜脱水或合成液态抗冻有机物来增强细胞的抗冻性”可得:通过细胞膜脱水或合成液态抗冻有机物来增强细胞的抗冻性。34.考查说明方法。画线句列举了北方的梨树和松树的针叶两个实例,这是举例子。以冬天低温和春天微寒北方梨树不同的状态作比较,这是作比较。句子中有准确的数字,这是列数字。联系本段首句“通常而言,即便是同一种植物,冬季和夏季的抗冻能力也不一样”可知,三种说明方法综合运用,具体准确地说明了“即便是同一种植物,冬季和夏季的抗冻能力也不一样”。十、议论文阅读 一、阅读下文,回答问题。“咬牙”是一种修炼向贤彪①宝剑锋从磨砺出,梅花香自苦寒来,干事创业,往往需要一番“咬咬牙”的坚持。②党史上,有位干部因“咬牙”而闻名。1943年,冀南抗日根据地斗争异常艰苦与残酷:战斗频繁,严重旱灾,庄稼颗粒无收,痢疾、霍乱盛行,时任冀南区党委书记、行署主任兼冀南军区政治委员的宋任穷,一边坚持对敌作战,一边组织生产自救,还深入敌后做群众工作,累得多次吐血,凭着顽强的意志和乐观主义精神,他率领冀南军民咬紧牙关度过了最艰难的时期,因此得名“咬牙干部”。③“咬牙”体现的是坚韧。人生之路难以一帆风顺,惟有不畏艰险、直面挑战,坚持不懈、持之以恒,才能采撷成功的果实。习近平总书记当年在梁家河插队时,什么活儿都干,开荒、种地、铡草、放羊、拉煤、挑粪……从不惜力,磨砺出不畏艰险、百折不挠的意志品格。成功往往只奖赏那些坚韧的人。一件工作、一项事业干到最艰难的时候,往往最需要咬紧牙关。而一旦坚持下来,就容易突出重围、打开局面。④“咬牙”彰显的是智慧。京剧《沙家浜》中,18名新四军伤病员被困芦苇荡,因连续多日面临日伪军的“扫荡”,体力和毅力几近于极限。指导员郭建光激励大家:有利的情况和主动的恢复,往往产生于再坚持一下的努力之中。最终,他们以“再坚持一下”的顽强精神,迎来了大部队的反“扫荡”。咬紧牙关的坚持精神,不仅需要不畏艰难的勇敢,也需要透过现象看本质的智慧。咬紧牙关坚持下去,就能变被动为主动。⑤“咬牙”蕴含的是担当。在重压和困难面前“咬牙”坚持,强健的是人的内心。“拼命黄郎”黄大年,平均每年出差130多天,一回来就一头扎进办公室。他的研究首次推动我国快速移动平台探测技术装备研发,攻克了技术瓶颈。正是“咬牙”背后的担当精神,成就了黄大年无悔的人生,⑥行百里者半九十。越接近成功就越艰难,越艰难就越要坚持,否则就会前功尽弃、半途而废。从某种意义上说,“咬牙”是成功的序曲。没有松骨峰战斗中志愿军的“咬牙”,就没有以“气”胜“钢”的功绩;没有谷文昌一次次面对失败后的“咬牙”,就没有沙海变绿洲的奇迹;________,________。拿出不达目的誓不罢休的执着,砥砺千磨万击还坚劲的意志,知难而进、久久为功,多经历几次“咬牙”,一个人必能闯关夺隘、化险为夷,用奋斗之犁开辟前行之路。⑦惟其艰难,更显勇毅。“咬牙”是一种修炼,在一次次“咬牙”中,软弱将变得坚强,稚嫩将变得成熟,徘徊将变得坚定。1.文章主要是从哪三个方面论述“咬牙”精神的?2.第②段主要运用了什么论证方法?有什么作用?3.简要分析第③段的论证思路。4.模仿第⑥段划线句的句式,给本段补充一个论据。【答案】1.①“咬牙”体现的是坚韧;②“咬牙”彰显的是智慧;③“咬牙”蕴含的是担当。 2.举例论证,列举宋任穷率领冀南军民咬紧牙关度过抗战最艰难的时期的例子,有力地证明了本文的论点。 3.首先提出“咬牙”体现的是坚韧这一分论点,然后通过道理论证和举例论证,列举习近平总书记的例子论证这一观点,最后得出只有坚持才能突出重围、打开局面的结论。 4.示例:没有王进喜用身体堵住井喷的“咬牙”,就没有大庆油田的传奇。(或答:“没有爱迪生面对失败的一次次“咬牙”,就没有发明成功后的喜悦。”“没有司马迁在酷刑面前的“咬牙”,就没有史学巨著《史记》的问世。”等)【解析】1.本题考查对文章内容的概括能力。解答此题,需读文段,找出每段的中心句。③~⑤段每段的首括句就是段落的中心句。抓住这三个中心句,进行综合即为答案:③段“咬牙”体现的是坚韧;④段“咬牙”彰显的是智慧;⑤段“咬牙”蕴含的是担当。2.本题考查论证方法及作用。常见的论证方法有举例论证、对比论证、比喻论证、道理论证。根据第②段“党史上,有位干部因‘咬牙’而闻名”,后详细叙述了其在冀南抗日根据地咬紧牙关度过了最艰难的时期的事迹,具体生动、有说服力地论证了第①段的论点,也就是本文的中心论点:即干事创业,往往需要一番“咬咬牙”的坚持。3.本题考查论证思路。分层,从论点、论证方法、论据、总结等角度分层。表述时要有层次性。(1)论点角度。首先,提出了分论点:“咬牙”体现的是坚韧。(2)论证方法或者论据:习近平总书记当年在梁家河插队时,从不惜力,磨砺出不畏艰险、百折不挠的意志品格。成功往往只奖赏那些坚韧的人。运用举例论证的方法证明了分论点。(3)最后总结:一件工作、一项事业干到最艰难的时候,往往最需要咬紧牙关。而一旦坚持下来,就容易突出重围、打开局面。4.本题考查围绕论点选择事实论据的能力和仿写能力。认真读第⑥段划线句子,首先结合分论点“‘咬牙’是成功的序曲”,其次分析句子特点:两个分句内容上都是列举事实,我们补充的论据也应该是事实论据;两个句子结构上都采用“没有……就没有……”的句式,我们仿写的句子也应该采用这种句式。示例:没有越王勾践在失败面前的“咬牙”,就没有最后灭掉吴国的反转。二、阅读文章,完成下面小题。涵养几分静气①一则故事,读来耐人寻味。父亲丢了一块表,抱怨着四处寻找,可怎么也找不到;等他出去后,儿子悄悄进屋,不一会儿就找到了。父亲很好奇,儿子则回答:我就安静地坐着,听到滴答滴答的声音,表就找到了。②这就是“心静”的力量。“静而后能安,安而后能虑,虑而后能得”。一事当前,苦恼抱怨无济于事,烦躁惊慌也于事无补,唯有稳住阵脚、静下心来、凝神细思,方能寻觅解决之道。诸葛亮在《诫子书》中一语道破“静”的重要,“静以修身,俭以养德,非淡泊无以明志,非宁静无以致远”。坚守内心的宁静,保持清醒的头脑,才能见天地之精微,察万物之规律。那么,“静心功夫”从何处得来?③涵养静心功夫,离不开严格的自律。毛泽东同志在湖南一师求学时,特意到最喧闹的地方读书,目的就是锻炼意志,让自己能够胸怀静气,心绪不受外界所扰。为了让自己安静下来,有人给自己定下“静坐”的“日课”,坚持每天独坐两小时,终于改掉了身上浮躁的毛病。严格的自律,助人逐渐养成宁静、专一的心态,让人虽得意而不忘形、处困顿而不沉沦、临诱惑而不摇摆,也因静而明志,因静而生慧。④有人说,每临大事有静气。大事当前的静心功夫,往往得益于艰难困苦的历练。苏轼《书舟中作字》记载,一次乘船途中,滩险舟危,舟中士子面无人色,唯独他“作字不少衰”。这份从容盖因其“更变亦多矣”,所以能临事不惧,处变不惊。可见,不惮于在难事、烦事、急事、苦事上多磨炼,就能将内心打磨得成熟豁达,沉稳有定力,不再拘泥于一时一事的得失;多经历几番雨疏风骤的洗礼,耐得几回兴衰荣辱的打磨,就能在大事来临之时,气定神闲、沉着冷静,抵达“不以物喜,不以己悲”的境界。⑤读书以养性,书画以养心。“静心功夫”,还可从读书中来。寂寞忧愁时读一阙《定风波》,心会变得豁达敞亮;委屈不平时读一卷《宽容的哲学》,能收获“既忍且耐”的智慧。身处低谷之时,从书中汲取前行的力量,纵使“山重水复”也终将迎来“柳暗花明”;得意之时,时常读书以自省,也才能“不忘初心,方得始终”。端起沉甸甸的书本,走进书中的世界,浮躁之气便能荡涤一空,久而久之,自然能寻得“此心安处”。⑥古人云:“人心多从动处失真。若一念不生,澄然静坐,云兴而悠然共逝,雨滴而冷然俱清,鸟啼而欣然有会,花落而潇然自得。何地无真境,何物无真机。”涵养几分静气,绝非暮气沉沉,而是多一些沉潜、少一些浮躁,多一些从容不迫、少一些进退失据。“静心”的功夫,助人眼界开阔,胸襟豁达,挺过如磐风雨,包容万千气象,寻得生命真谛。5.下面对文章的理解不正确的一项是( )A.第①段通过父与子不同心境下寻找手表的故事,引出文章遇事需要涵养静气而非苦恼抱怨的中心论点,同时将抽象的道理具体化,激发读者的阅读兴趣。B.第②段运用了举例论证的论证方法,论证了坚守内心的宁静,保持清醒的头脑,才能见天地之精微,察万物之规律的论点。C.本文先用一个故事引出论点,再从三个方面论证静心功夫从何而来,最后引用古人的话得出结论——“静心”的功夫,助人眼界开阔,胸襟豁达,挺过如磐风雨,包容万千气象,寻得生命真谛。D.第⑤段列举了几个读书养性的例子,形成排比,告诉我们“静心功夫,还可从读书中来”的道理。6.下列与第②段中的“静心功夫”含义不符合的一项是( )A.董仲舒严于自律,三年不窥园,潜心向学,学业精进,终成大儒。B.在淝水之战中,谢安作为东晋一方的总指挥,面对号称百万的前秦军队,临危不乱,镇定地指挥,从容应对,以少胜多,为东晋赢得几十年的安定和平。C.现代作家路遥克服饮食的匮乏,生活用品的不足,在艰苦的环境中耕耘着精神家园,创作出《平凡的世界》。D.陶渊明好读书,心有静气,在纷繁世事中能认清自己内心的需要,才能不为五斗米折腰,回归自己的“旧林”。【答案】5.B 6.C【解析】5.本题考查对文章内容的理解。要求学生仔细阅读文本,然后分析选项。B.有误,根据“静以修身,俭以养德,非淡泊无以明志,非宁静无以致远”可知,“运用了举例论证的论证方法”错误,这里采用的是“道理论证(或引用论证)”;故选B。6.本题考查对论点论据的匹配。根据第②段“一事当前,苦恼抱怨无济于事,烦躁惊慌也于事无补,唯有稳住阵脚、静下心来、凝神细思,方能寻觅解决之道”“坚守内心的宁静,保持清醒的头脑,才能见天地之精微,察万物之规律”可知,“静心功夫”能够让人镇定,保持清醒头脑,寻找到解决方法;选项C“路遥克服饮食的匮乏,生活用品的不足,在艰苦的环境中耕耘着精神家园”的主要意思是“克服物质上的匮乏”,与“静心功夫”无关,故不相符;故选C。三、阅读下面议论性文本,完成下面小题。磨砺始得玉成①冰清玉洁、精金良玉、玉振金声、玉树临风……在中国传统文化里,“玉”常常被用来形容人的优秀品质,彰显着东方美学的韵味。凝视一块美玉,看到的是柔美和温润;触摸一块美玉,感受到的是坚实和高洁。中国人从美玉身上,寻找到了修身立德的追求,发现了为人处世的智慧。②《荀子•法行》中记载,子贡询问“君子贵玉”的原因,孔子答曰:“夫玉者,君子比德焉。”谦谦君子,正如温润之玉,光华敛于内而不炫于外,清辉藏于身而气韵自生,给人以温暖和力量。方志敏在狱中面对敌人的百般诱降和严刑逼供,依旧正气凛然,坚贞不屈,这是革命先烈“宁为玉碎,不为瓦全”的坚韧气节。黄文秀为了让基层群众过上好日子,把韶华留在广西百色的大山深处,这是当代年轻人“艰难困苦,玉汝于成”的责任担当。③玉石藏于山中、埋于谷底,与普通石头不分轩轾,如何成为众所敬仰的美玉?坚固细密的玉石,必得经过一番艰苦的切、磋、琢、磨,才会露出质朴面目,显出纯美底色。“玉不琢,不成器”,器与人皆通此理。只有历经雕琢磨砺,唤醒内心的道德律,淬炼信仰的主心骨,才能成为一个有益于国家和人民的人。大凡成就大业者、贡献杰出者、勇赴使命者、舍己为人者,无不是经历几番潜心学习、艰苦锻炼、奋斗打拼,才为世所识、为人所赞。磨砺始得玉成,笃行方能致远。拿出“燕子垒窝”“老牛爬坡”的劲头,在工作实践中战风雨、斗寒霜,在严峻复杂的斗争中砥砺心性、强壮筋骨、增长才干,才能琢磨成器、百炼成钢,最终有所成就。④习近平总书记强调:“广大青年人人都是一块玉,要时常用真善美来雕琢自己,不断培养高洁的操行和纯朴的情感,努力使自己成为高尚的人。”每块玉都有独特的形态与气质,每个青年也都有独特的禀赋与追求。对玉的雕琢不能千篇一律,对青年人才的培育也讲究因材施教。因材因性,把雕琢之功施于无形,方能获得出乎意料的效果。广大青年也需要在干事创业的路途中,心怀真善美的追求,以不负韶华之朝气、激越沧海之胆气,将为人民服务的意识镌刻在灵魂深处,清白做人,踏实做事,让青春之花在人民最需要的地方绽放。胸怀锦绣,腹隐珠玑,时时砥砺,事事精进,必能成为栋梁之才。⑤“言念君子,温其如玉”。正如美玉经得起岁月的淘洗,美德懿行耐得住时间的检验和人心的评判。以人比玉,由玉及人,我们每个人都需要更加勤勉,用时间和汗水雕琢自己,努力成为一块“美玉”,永远散发洁净通透的光泽。(《人民日报》2022年11月9日)7.下列对文章的理解和赏析,错误的一项是( )A.开篇点题,点明中国人有“修身立德的追求,为人处世的智慧”这一中心论点。B.成语及引文的使用,使本文语言严密,有说服力的同时又不失文雅。C.引用习近平主席的话属于道理论证,鼓励青年人应努力把雕琢自己的品质作为重要的人生课题。D.本文力求引导我们立足自己的禀赋和追求,不断磨砺,最终成为对人民有用的人。8.本文论证严密,请简析本文第③段的论证思路。9.青年人应如何将自己磨砺成一块温润美玉?请根据④⑤段内容分条概括。【答案】7.A 8.首先提出问题——如何成为众所敬仰的美玉,然后论证玉石需经过切磋琢磨才能成为美玉;接着由玉及人,人玉同理,只有艰苦锻炼、奋斗打拼,才能为世所识、为人所赞;最后发出号召,在工作实践中不断磨炼,才能有所成就。 9.用真善美雕琢自己,培养高洁操行和纯朴情感,做一个高尚的人;青年人要清楚自己的禀赋和追求,发展自己的个性和特长;清白做人、踏实做事,时时砥砺,事事精进;更加勤勉,用时间和汗水雕琢自己。【解析】7.考查对文本内容的理解与分析。A.理解和赏析错误。根据①段“……中国人从美玉身上,寻找到了修身立德的追求,寄托着为人处世的智慧”论述中国人贵玉(从美玉身上,寻找到了修身立德的追求,寄托着为人处世的智慧)、②段“《荀子·法行》中记载,子贡询问“君子贵玉”的原因……”论述贵玉的原因、③段“玉石藏于山中、埋于谷底,与普通石头不分轩轾,如何成为众所敬仰的美玉”论述玉何以贵(必得经过一番艰苦的切、磋、琢、磨,才会露出质朴面目,显出纯美底色)、④段“广大青年要时常用真善美来雕琢自己,不断培养高洁的操行和纯朴的情感,努力使自己成为高尚的人”论述希望青年人能成为玉(努力使自己成为高尚的人)、⑤段“我们每个人都需要更加勤勉,用时间和汗水雕琢自己,努力成为一块‘美玉’,永远散发洁净通透的光泽”总结全文,得出结论:人只有经历磨练,才能有所成就。可知,本文是归纳论证,结论部分,才是作者的中心论点之所在。因此,中心论点是:人只有经历磨练,才能有所成就。首段“修身立德的追求,为人处世的智慧”并非中心论点。故选A。8.本题考查分析论证思路。第③段首句“玉石藏于山中、埋于谷底,与普通石头不分轩轾,如何成为众所敬仰的美玉”提出问题:如何成为众所敬仰的美玉。第二句“坚固细密的玉石,必得经过一番艰苦的切、磋、琢、磨,才会露出质朴面目,显出纯美底色”论证玉石需经过切磋琢磨才能成为美玉。三四五句“‘玉不琢,不成器’,器与人皆通此理……大凡成就大业者、贡献杰出者、勇赴使命者、舍己为人者,无不是经历几番潜心学习、艰苦锻炼、奋斗打拼,才为世所识、为人所赞”由玉及人,玉石经过琢磨才能成为美玉,人只有经过磨砺才能有所成就,得出观点:磨砺始得玉成,笃行方能致远“笃行方能致远……才能琢磨成器、百炼成钢。六句“拿出……最终有所成就”发出号召,号召人们在工作实践中不断磨炼,才能有所成就。9.本题考查内容概括。根据第④段“要时常用真善美来雕琢自己,不断培养高洁的操行和纯朴的情感,努力使自己成为高尚的人”可概括为:用真善美雕琢自己,培养高洁操行和纯朴情感,做一个高尚的人;根据第④段“每个青年也都有独特的禀赋与追求。对玉的雕琢不能千篇一律,对青年人才的培育也讲究因材施教”可概括为:青年人要清楚自己的禀赋和追求,发展自己的个性和特长;根据第④段“清白做人,踏实做事,让青春之花在人民最需要的地方绽放。胸怀锦绣,腹隐珠玑,时时砥砺,事事精进,必能成为栋梁之才”可概括为:清白做人、踏实做事,时时砥砺,事事精进;根据第⑤段“我们每个人都需要更加勤勉,用时间和汗水雕琢自己”可概括为:更加勤勉,用时间和汗水雕琢自己。四、阅读下文,回答问题。青年是时代的受益者也是贡献者张 凡①日前,国务院新闻办公室发布《新时代的中国青年》白皮书。作为新中国历史上首部关于青年发展的白皮书,《新时代的中国青年》提出新时代中国青年生逢盛世,面临中华民族发展的最好时期,号召中国青年勇挑重担,以更加自信开放的姿态与世界各国青年共同构建人类命运共同体。②“江山代有才人出,各领风骚数百年。”社会进步离不开青年,国家发展需要青年,每一代中国青年都有自己的责任与使命,每一代青年都有自己的际遇和机缘,都要在自己所处的时代条件下谋划人生、创造历史。当代青年作为整个社会力量中最为积极、最有生气的中坚力量,是建设新时代中国特色社会主义事业的有力接班人,更是实现中华民族伟大复兴的先锋力量。③青春逢盛世,新时代的中国青年是时代的受益者。随着物质文明环境的不断优化、精神成长空间与可能的不断拓宽、国家制度与公共政策的不断完善和健全,当代中国青年,尤其是大学生、农民工及新兴社会阶层三类青年群体,在教育学习、择业就业、人生道路等诸多方面有着更为广阔的成长空间和更为多元的发展选择。2020年,外出农民工总数近1.7亿人,其中多数为青年;青年常住人口城镇化率达71.1%,比10年前增加15.3个百分点,农村青年发展选择更加自主;2021年,全国九年义务教育巩固率95.4%,高中阶段毛入学率91.4%,高等教育毛入学率57.8%,青少年接受知识文化教育权益保障有序。新时代的中国青年享受着党和国家的种种政策与红利,拥有前所未有的良好的成长成才环境。④青年当有为,新时代的中国青年亦是时代的贡献者。不论是奥运赛场还是抗疫战场,不论是脱贫攻坚还是乡村振兴,不论是城市建设还是为国戍边……新时代的中国青年,都在以不懈的努力在不同“考场”上完成属于他们的青春答卷。2021年东京奥运会421名中国运动员中,“95后”“00后”运动员占比近60%;2022年北京冬奥会1.9万名赛会志愿者中35岁以下青年占94%;新冠肺炎疫情发生以来,32万余支青年突击队、550余万名青年奋战在抗击疫情一线,为打赢疫情防控的人民战争作出贡献;在助力脱贫攻坚和乡村振兴中,47万名“三支一扶”人员参加基层支教、支农、支医和帮扶乡村振兴,以及数百万名青年大学生参与“三下乡”社会实践,为新时代乡村建设提供了青年力量与青年智慧。⑤一代人有一代人的长征,一代青年有一代青年的使命与担当。新时代的中国青年已经在新时代的大舞台上发光发亮,绽放着他们火热的青春。奋斗路上他们始终砥砺前行,追梦途中他们依旧踔厉奋发。虽不免要直面压力与难题,但他们从不畏惧,也没有被困难所吓倒,更不为失败而惶惶终日,他们始终心怀憧憬,向往美好与未来。期待新时代的中国青年笃行致远,用自己的本领与实干,向世界讲好中国故事、从历史继承优良传统,勇做走在时代前列的奋进者、开拓者、奉献者。(选自2022年4月28日《光明日报》)10.下面对文章内容的理解有误的一项是( )A.第①段提到的“国务院新闻办公室发布《新时代的中国青年》白皮书”引出了论题。B.第②段引用诗句,论证了社会进步,国家发展离不开青年的观点。C.第③段列举“青年农民工的择业就业空间更为广阔、农村青年的发展选择更加自主、青少年接受知识文化教育更有保障”等事例,论证了本段的分论点。D.“黄文秀研究生毕业后,放弃大城市的工作机会,毅然回到家乡,在脱贫攻坚第一线倾情投入、奉献自我。”这一事例能够论证本文的论点。11.请你根据文章的论证思路,用“//”将本文分成三个部分并阐明文章的主要观点。① ② ③ ④ ⑤ 主要观点:12.本文语言准确严密,请结合第④段划线句子加以分析。13.请你结合本文内容,解释“江山代有才人出,各领风骚数百年”的现实意义。【答案】10.B 11.①//②③④//⑤ 文章主要观点:青年是时代的受益者也是贡献者。 12.画线句运用举例论证,有力地论证了“新时代的中国青年是时代的贡献者”。 13.现实意义:社会进步离不开青年,国家发展需要青年,每一代中国青年都有自己的责任与使命,都有自己的际遇和机缘,都要在自己所处的时代条件下谋划人生、创造历史。【解析】10.本题考查理解分析文章内容。B.理解不正确。结合第②段“‘江山代有才人出,各领风骚数百年。’社会进步离不开青年,国家发展需要青年,每一代中国青年都有自己的责任与使命,每一代青年都有自己的际遇和机缘,都要在自己所处的时代条件下谋划人生、创造历史”可知,第②段引用诗句,论证了每一代中国青年都有自己的责任与使命,每一代青年都有自己的际遇和机缘,都要在自己所处的时代条件下谋划人生、创造历史的观点。故选B。11.本题考查论证思路以及中心论点。结合第①段内容可知,文章开头提出国家号召新时代青年勇挑重担,更自信、开放地与各国青年共同努力的观点;结合第②③④段内容可知,作者接着从三个方面进行分析:第②段引用诗句,证了“每一代中国青年都有自己的责任与使命”的道理,第③段论述了“青春逢盛世,新时代的中国青年是时代的受益者”,第④段论述了“青年当有为,新时代的中国青年亦是时代的贡献者”;结合第⑤段内容可知,作者最后总结全文,赞美了新时代中国青年,期待他们在时代前列奋进、开拓、奉献。据此分析,文章结构可划分为①//②③④//⑤;结合题目可知,本文的主要观点是“青年是时代的受益者也是贡献者”。12.本题考查论证方法。结合第④段画线部分分析,这里列举出在冬奥会、抗疫一线、新时代乡村建设等各个领域勇担使命的青年力量,属于举例论证的方法,充分有力地论证了“新时代的中国青年是时代的贡献者”这一分论点,从而进一步地论证了中心论点。13.本题考查理解句子。结合第②段“‘江山代有才人出,各领风骚数百年。’社会进步离不开青年,国家发展需要青年,每一代中国青年都有自己的责任与使命,每一代青年都有自己的际遇和机缘,都要在自己所处的时代条件下谋划人生、创造历史”可知,“江山代有才人出,各领风骚数百年”的现实意义在于社会进步离不开青年,国家发展需要青年,每一代中国青年都有自己的责任与使命,每一代青年都有自己的际遇和机缘,都要在自己所处的时代条件下谋划人生、创造历史。五、阅读下面文本,完成下面小题。咬定目标积小胜为大胜①不久前,天舟四号货运飞船发射升空,与空间站组合体完成自主快速交会对接,正式开启了中国空间站在轨建造阶段。遥想新中国成立之初,“一辆汽车、一架飞机、一辆坦克、一辆拖拉机都不能造”过去儿十年,包括航空航天在内的中国工业和中国科技由弱到强,逐一突破各道难关,终于迈入世界前列。年年有成,积小胜为大胜,正是事业成功的必经之路。②走远路必从近处开始,登高山必从低处起步。做任何事情都要积小成大、坚持不懈。好高骛远,操之过急,可能欲速不达。不积跬步,无以至千里;不积小流,无以成江海。我们干事创业,就需要咬定目标、脚踏实地,埋头苦干、久久为功,一步一个脚印,铺就成功之路。③不舍寸功才能善作善成。00后奥运射击冠军杨倩表示,练习中有一次打得不好的话,就会主动去要求自己,一定要把这一枪打好,不能放弃。所谓“冠军品质”,其实就是从一个个简单的动作开始,一招一式反复练习,最终成就卓越。每一块奥运奖牌都是训练汗水凝结成的,每一个鲜活人物角色都是由一个个细节塑造出来的,每一次科研突破都是反复试验酝酿的……聚沙成塔、积少成多是事业成功的必由之路。新中国的发展成就,也是从一块块砖垒起来的,从一条条路建起来的,从一件件产品造出来的。创造业绩,就必须打好地基、做好积累。④精益求精才能更进一步。有人曾计算,假设火箭有5万个零部件,若要使整体安全系数达到99.99%,那每个零部件的安全系数则要达到99.9999999‰。正是因为有中国航天人对细节的一丝不苟、对质量的“锱铢必较”,才有了长征系列运载火箭实现400余次发射的成果。在脱贫攻坚伟大实践中,8年近1亿人脱贫,这一人类减贫史上的奇迹,凭借的也是精准到户、精准到人的铢积寸累、精益求精。成功从来不是一蹴而就的,而是需要把每一件事情做细做好,把重复的事情做专做精,在守正创新中追求卓越,在精雕细琢中实现跨越。⑤久久为功才能行稳致远。从“一五”计划到“十四五”规划,从解决人民温饱问题到全面建成小康社会,正是靠着锲而不舍的拼搏奋进、驰而不息的艰苦奋斗,党和人民创造了“当惊世界殊”的历史成就。“ 。”在实践中不断砥砺、经受考验,不断激发热情、超越自我,才能日有所进,最终取得一番成就。“不求近功,不安小就。”脚踏实地把每件平凡的事做好,不驰于空想、不骛于虚声,一切平凡的人都可以获得不平凡的人生,一切平凡的工作都可以创造不平凡的业绩。⑥每一个孜孜不倦的奋斗身影背后,都有一颗不服输、不放弃的追梦之心。立志当高远,立志还需躬行,扎扎实实干,点点滴滴做,才能干有所成。新时代是实干家、奋斗者的时代,脚踏实地、艰苦奋斗,行而不辍、不弃微末,一切美好的东西都能够创造出来,每个人都可以抵达向往的远方。14.阅读文章,概括本文的中心论点。15.若在第五段横线上填入一句话,你认为“万事从来贵有恒”和“九层之台,起于累土”,哪句更恰当?请说明理由。16.品析第三段划线句子。17.请结合“链接材料”和文章内容,说说苏翊鸣的哪些做法值得你学习。【链接材料】滑雪冠军苏翊鸣4岁起接触单板滑雪,从一个个简单的动作开始,通过积累慢慢做到更好。为了滑雪,他可以不睡懒觉,即使耳朵被冻伤也坚持上山,日复一日的刻苦训练对于他来说是一种享受。为了解锁一个新动作,有很多细节,即使教练已经满意,他还是会要求再来一遍……2021年1月,他实现了5圈翻转(1800度),成为中国首位完成这一极高难度动作的选手。没有蹴而就的成功,但有水滴石穿的奇迹【答案】14.我们干事创业,就需要咬定目标、脚踏实地,埋头苦干、久久为功,一步一个脚印,铺就成功之路。 15.“万事从来贵有恒”。“万事从来贵有恒”意为做事最重要是持之以恒,“九层之台,起于累上”意为日积月累才能有所成就;第⑤段的分论点“久久为功才能行稳致远”意为持之以恒才能走得稳走得远“万事从来贵有恒”作为道理论据,可论证分论点。 16.运用排比的修辞手法,条理清晰阐释了聚沙成塔、积少成多是事业成功的必由之路的道理;或运用举例论证,有力论证了不舍寸功才能善作善成的观点。 17.答案示例:我要学习苏翊鸣的不舍寸功、精益求精、久久为功的做法。他从一个简单的动作开始,通过积累做到更好,是不舍寸功的表现;他克服困难,日复一日的刻苦训练是久久为功的表现;他为解锁新动作,对细节丝不苟,是精益求精的表现。【解析】14.本题考查论点概括。根据第①段“年年有成,积小胜为大胜,正是事业成功的必经之路”可知,本段通过列举中国航天事业取得的一系列成功离不开积少成多,持之以恒的奋斗,提出“积小胜为大胜”的话题;第②段引用名言,并进一步分析,提出“我们干事创业,就需要咬定目标、脚踏实地,埋头苦干、久久为功,一步一个脚印,铺就成功之路”的观点;第③~⑤段分别从“不舍寸功才能善作善成”“精益求精才能更进一步”“久久为功才能行稳致远”等三方面进行具体论述;第⑥段“新时代是实干家、奋斗者的时代,脚踏实地、艰苦奋斗,行而不辍、不弃微末,一切美好的东西都能够创造出来,每个人都可以抵达向往的远方”总结全文,并发起呼吁,号召大家脚踏实地,艰苦奋斗;由此分析可知,本文的中心论点是:我们干事创业,就需要咬定目标、脚踏实地,埋头苦干、久久为功,一步一个脚印,铺就成功之路。15.本题考查语句衔接。“万事从来贵有恒”意思是不管做什么事情,最重要的从来都是坚持下去。“九层之台,起于累土”意思是指九层高台是从一筐土开始堆积起来的。引喻做事是从最基本开始,经过逐步的积累,才能有所成就。根据第⑤段“久久为功才能行稳致远”“在实践中不断砥砺、经受考验,不断激发热情、超越自我,才能日有所进,最终取得一番成就”可知,本段强调的是持之以恒才能行稳致远,与“万事从来贵有恒”表达的意思一致,故“万事从来贵有恒”填入横线处最恰当。16.本题考查赏析句子。画线句中“每一块……每一个……每一次……”运用了三个句式相同的分句,属于排比的修辞手法,语势铿锵,起到突出强调的作用。根据前句“所谓‘冠军品质’,其实就是从一个个简单的动作开始,一招一式反复练习,最终成就卓越”后句“聚沙成塔、积少成多是事业成功的必由之路”可知,运用排比突出强调了聚沙成塔、积少成多是事业成功的必由之路,进而论证“不舍寸功才能善作善成”。17.本题考查人物形象。结合链接材料中的“从一个个简单的动作开始,通过积累慢慢做到更好”、结合第⑤段“坚持在实践中不断砥砺、经受考验,才能日有所进,最终取得一番成就”可知:他脚踏实地,持之以恒,从简单动作开始积累能力,不断做到更好,这种不舍寸功的表现值得我们学习;结合材料中“为了滑雪,他可以不睡懒觉,即使耳朵被冻伤也坚持上山,日复一日的刻苦训练对于他来说是一种享受”可知:他严以律己,为了滑雪不睡懒觉,即使耳朵冻伤也坚持训练,这种久久为功的精神值得我们学习;结合材料中“为了解锁一个新动作,有很多细节,即使教练已经满意,他还是会要求再来一遍”以及文章第④段“精益求精才能更进一步”“成功从来不是一蹴而就的,而是需要把每一件事情做细做好,把重复的事情做专做精,在守正创新中追求卓越,在精雕细琢中实现跨越”可知:他精益求精,不断追求更高的境界值得我们学习。六、阅读下文,完成各题。视频弹幕“辣眼睛”,我们没理由“束手无策”①随着互联网全民创作时代的到来,大量用户上传的UGC(用户生产内容)成为不少视频网站内容的主要来源之一,用户“畅所欲言”的弹幕也作为新生文化现象和新兴的互动形式而备受关注。②据多家媒体报道,某知名视频弹幕网站又一次因出现低俗动漫内容而被曝光。而另一时尚生活分享平台则被爆出存在大量烟草营销文章。目前,问题动漫已被删除,搜索烟草关键词也不再显示相关内容。可是,对于已经观看过这类内容的未成年人来说,其造成的伤害无法量化也很难弥补——面对这些“辣眼睛”内容,我们没有理由“束手无策”。③相关视频网站真的拿“辣眼睛”内容“束手无策”?稍加分析便不难发现,所谓“束手无策”不过是一种不作为的借口。④号称系统算法的所谓自动推荐机制,帮助这些内容获得动辄百万的点击量。由于缺乏有效的惩戒机制,即便遭遇举报后,不少上传不良内容的账号会用更为隐晦的字眼持续传播相关内容,屡禁不止。用户即时发送的弹幕似乎更成为了某些低俗粗俗、攻击性内容狂欢的天地。而打着“鼓励创作”“追求流量”的旗号,很多平台“睁一只眼闭一只眼”,一旦被举报曝光,也只是做下架删除处理。⑤长此以往,这些“辣眼睛”内容所产生的负面影响将贻害无穷。要破解难题,相关政府部门必须强化管理。眼下,中国网络视听节目服务协会发布了《网络短视频平台管理规范》和《网络短视频内容审核标准细则》。这些条例的出台,制约了互联网平台的行为。《规范》规定视频上传后须有后台“审核”进行一定的前期过滤。同时将弹幕与视频标题、简介、评论等一并列入“先审后播”的范畴。《规范》特别强调了网络短视频平台应当建立未成年人保护机制。⑥互联网平台也要主动作为。平台方不能总是抱着“事后补救”的侥幸心态被动灭火,应该防患于未然。技术是中性的,______成为不良内容滋生蔓延的温床,_______进一步加大审核技术的研发,收紧内容审核的“网眼”。做到审核筛查“零死角”。⑦与此同时,要优化推荐机制,更好地引导青少年观看、上传优质健康的视频内容。近年来,不少网站致力于向青少年用户传播传统文化、弘扬社会正能量。《我在故宫修文物》《十年逐梦路》等纪录片由此走红,引发无数网友对国宝守护工匠的“弹幕致礼”。还有普及中国历史,回望峥嵘岁月的内容,也让“此生无悔入华夏,来生还在中华家”的评论成为刷屏金句。最近主办的“中国华服日”活动更是围绕国风音乐、汉服、传统戏曲等向青少年推送优质内容。⑧然而,这些正面内容却无法抵消少数低俗内容所造成的负面影响。这也时刻警示我们:越是面向青少年提供文化娱乐内容的平台,越要依规行事,越要从源头规范和引导用户的内容生产。⑨让低俗内容在互联网平台上销声匿迹,让更多的面向青少年提供文化娱乐内容的平台风清气正,成为传递正能量、引领健康的文化娱乐消费的有效工具。18.第④段中加点词“狂欢”在文中的意思是_________________________19.根据上下文,填入第⑥段画线处的词语正确的一项是( )A.即使……也要B.与其……不如C.因为……所以D.只要……就要20.文章的中心论点是__________________。围绕中心论点,文章层层深入,指出互联网平台“束手无策”的实质是______________,作者认为应对的具体措施是:(1)__________(2)_________21.第⑧段删去更好还是保留更好?请简述你的理由。【答案】18.肆无忌惮、不受约束地任意发送 19.B 20.对于影响青少年健康成长的网络平台中的不良内容,我们要有所作为。 “束手无策”不过是一种不作为的借口。 强化管理,出台管理条例。 互联网平台加大审核技术的研发,优化推荐机制。 21.此段既是对前文内容的总结,又再次重申了网络平台中的低俗内容对青少年成长的危害,强调网络平台应尽的责任,自然引出下文的呼吁和号召。如删去,就起不到突出强调作用,与第⑨段的衔接也比较生硬。因此保留更好。【解析】18.本题考查词语理解。第④段“用户即时发送的弹幕似乎更成为了某些低俗粗俗、攻击性内容狂欢的天地”中的加点词“狂欢”本意是尽情地欢乐,结合上文“由于缺乏有效的惩戒机制,即便遭遇举报后,不少上传不良内容的账号会用更为隐晦的字眼持续传播相关内容,屡禁不止”可知,这里是肆无忌惮、不受约束地任意发送上传不良内容。19.本题考查关联词语的使用。A.“即使……也要”表示假设关系;B.“与其……不如”表示选择关系;C.“因为……所以”表示因果关系;D.“只要……就要”表示条件关系;根据“成为不良内容滋生蔓延的温床”与“进一步加大审核技术的研发,收紧内容审核的‘网眼’”两句是选择关系,应使用“与其……不如”。故选B。20.本题考查内容理解与提炼。(1)根据①②段从互联网发展的现状,引出“对于已经观看过这类内容的未成年人来说,其造成的伤害无法量化也很难弥补——面对这些‘辣眼睛’内容,我们没有理由‘束手无策’”的观点——对于影响青少年健康成长的网络平台中的不良内容,我们要有所作为,③至④点明“辣眼睛”内容的危害,⑤至⑧段指出解决问题的办法,⑨段发出号召——让低俗内容在互联网平台上销声匿迹,可知,均是围绕“对于影响青少年健康成长的网络平台中的不良内容,我们要有所作为”来展开的,因此这就是本文的中心论点。(2)根据题干“指出互联网平台‘束手无策’的实质”的提示,找到③段“相关视频网站真的拿‘辣眼睛’内容‘束手无策’?稍加分析便不难发现,所谓‘束手无策’不过是一种不作为的借口”可提炼为:“束手无策”不过是一种不作为的借口;(3)根据题干“作者认为应对的具体措施是”的提示,找到⑤段“要破解难题,相关政府部门必须强化管理。眼下,中国网络视听节目服务协会发布了《网络短视频平台管理规范》和《网络短视频内容审核标准细则》。这些条例的出台,制约了互联网平台的行为”,可概括为:强化管理,出台管理条例。(4)根据题干“作者认为应对的具体措施是”的提示,和对措施一的概括“相关政府部门必须强化管理”,找到⑥段“要互联网平台也要主动作为……进一步加大审核技术的研发……”和⑦段“与此同时,要优化推荐机制,更好地引导青少年观看、上传优质健康的视频内容”,可概括为:互联网平台加大审核技术的研发,优化推荐机制。21.本题考查句段作用。内容上,根据第⑧段“然而,这些正面内容却无法抵消少数低俗内容所造成的负面影响。这也时刻警示我们:越是面向青少年提供文化娱乐内容的平台,越要依规行事,越要从源头规范和引导用户的内容生产”,结合③至④点明“辣眼睛”内容的危害,⑤至⑦段指出解决问题的办法,可知,此段既是对前文内容的总结,又再次重申了网络平台中的低俗内容对青少年成长的危害,强调网络平台应尽的责任;结构上,结合⑨段“让低俗内容在互联网平台上销声匿迹,让更多的面向青少年提供文化娱乐内容的平台风清气正,成为传递正能量、引领健康的文化娱乐消费的有效工具”可知,自然引出下文的呼吁和号召;因此,不能删去。七、阅读下面的文章,完成小题。技术时代的阅读作者:张炜①随着科技的进步,在电子数字时代能够节省出大量时间,好像做什么事情都方便多了;可越是节省时间,越是觉得没有时间,几乎还没来得及好好着手干一件事,一天、一周、一个月甚或一年便飞逝而去。就在这种空前的匆忙中,传统的纸质阅读被手机阅读所替代,深入的欣赏与领悟被碎片化的浏览所击溃。人们开始不停地看手机,随时随地,乘车、散步、吃饭、等车、候机,甚至坐在马桶上,都在阅读手机。可见不是没有阅读,而是方式变了,完整的时间被一再切割,割成很小的单位。②这种切割是有代价的。任何一种事物的进步,一定伴随着许多退步。电子阅读的确方便了我们,可是对于视网膜的伤害是很重的,对记忆力和专注力的破坏是长久的。我们的审美领悟力、对语言艺术的敏感判断力,都会在这个过程中慢慢流失。③显而易见,电子阅读无法从根本上取代传统的纸质阅读。为了开发、提炼和巩固思想与创造力,许多时候还是要倚重书籍。要从事文学写作和学术研究,就更不可能放弃传统阅读,尤其还要专注于经典阅读。经典的力量在于积累了人类最强大的生命力和创造力,是精神的结晶。这是人类流传了几千年的经验的保存和传递方式,蕴藏着人类文明的遗传密码。④阅读不仅仅是一个方法和形式的问题,而必定与内容紧密相关。事实上闪烁和冷漠的荧屏很难容纳和沉淀深邃厚重的思想,它需要一个人在相对安静沉默中,甚至是孤寂中,面对一个朴实的文本。这个过程是一个整体,不能分解和割裂。那些经过人类长期摸索而产生的生命感悟,那一刻的汇聚和凝结,当转化为一些电子符号被接受时,会发生不同程度的耗损和衰减。对于许多人而言,他们仍然无法从屏幕中触摸那些炽热柔软的心灵。现代读者并不拒绝数字阅读,而只想利用和转化,使之成为有益的辅助。这作为一个问题横亘在每个人面前,谁都无法回避。⑤我们身处一个技术时代,会因为离得太近而看不清它的局限、它与人文的关系。所有杰出科学家的人文修养都很高,都是一个时代里最富有人文情怀的人。他们那种强大的创造力也来自强大的人文关怀力。⑥我们可以读爱因斯坦文集,从他极具文学价值的演讲与其他文字中,感受和领略其高不可攀的境界。他是一个业余的文学写作者,但他的表达力和深邃度都不是一般作家所能抵达的。⑦居里夫人是唯一一位得过两次诺贝尔奖的科学家,她女儿的回忆录中写到其感人至深的最后岁月:由于镭的辐射,手指甲裂开,不停地流血,拿不住东西,但直到最后还坚持读书,读得最多的就是文学经典。⑧原来杰出的科学家是如此锻造淬炼出来的,他们都不是简单的技术主义者,而是一些具有广博气度和超常审美力的人,始终高度关注人类的精神状况。⑨我们的眼睛牢牢地盯紧技术,盯住芯片、医药、空间技术之类,却不能忽略产生这些的人文土壤。这是一个长期培植的工作,是一个族群的精神信仰以及原动力,尤其要重视全民的阅读状况。以色列是一个人口与面积很小的国家,却是阅读力极强的民族,这片土地上产生了那么多科学家、思想家和作家。一个族群的人文素养如何,从人的神情气质、举手投足间便展露无遗。一个族群的基本素质存在问题,即使拥有一流的技术也不能持久,更不能转化为巨大的创造力。提高整个社会的文明程度,这既是民族尊严所在,也是一个民族创造力的基础。我们进入了硅时代,这个时代将对人文水准提出更高的要求。(来源:《思维的锋刃》)22.下列对文章内容的理解和分析正确的一项( ) A.本文的中心论点是:技术时代,电子阅读无法从根本上取代传统的纸质阅读。B.电子阅读有百害而无一利:它对于视网膜的伤害很重,对记忆力和专注力的破坏长久,还会造成人们审美领悟力、对语言艺术的敏感判断力流失。C.毋庸置疑,现代读者不应拒绝数字阅读,可以利用和转化,使之成为有益的辅助。D.文章论述经典文学阅读的作用,从对个人的影响到对族群的影响,采用举例论证和对比论证。23.本文开头有怎样的特点,并说说这样写的好处。【答案】22.A 23.示例:从生活现象写起。这样写:(1)引出论题;(2)同时也是文章的论据;(3)增强文章趣味性、吸引读者,激起读者的阅读兴趣;引发读者思考。【解析】22.本题考查文章内容理解。B.根据第②段“电子阅读的确方便了我们,可是对于视网膜的伤害是很重的,对记忆力和专注力的破坏是长久的”可知,电子阅读并不是有百害而无一利;C.根据第④段“现代读者并不拒绝数字阅读,而只想利用和转化,使之成为有益的辅助”可知,语段指出的是现象,而选项则转化为了观点;D.根据⑥~⑨段的内容可知,在论证的过程中没有用到对比论证,运用了举例论证,都是一些正面事例;故选A。23.本题考查句段作用。开头段中“人们开始不停地看手机,随时随地,乘车、散步、吃饭、等车、候机,甚至坐在马桶上,都在阅读手机”是现阶段我们的一些生活现象,文段从生活现象入手,可以增加文章的趣味性,可以吸引读者的阅读兴趣,进而反观自身;“不是没有阅读,而是方式变了,完整的时间被一再切割,割成很小的单位”引发读者思考,阅读方式的改变带给了我们哪些影响?从而引出文章论题:电子阅读和纸质阅读;进一步引出本文观点:电子阅读无法从根本上取代传统的纸质阅读。同时开头所涉及的生活现象也作为事实论据证明了文章的观点,虽然电子阅读为我们提供了便利,但是最终架空了我们的审美能力,鉴赏能力,要想真正品鉴文章,有所收获,最终还是要回到纸质阅读中去。八、阅读《一起做“读书种子”》一文,完成下面小题。一起做“读书种子”向贤彪①“读书种子”一词的“发明权”,当属宋代的黄庭坚。他在《山谷别集》中说:“四民①皆当世业②,士大夫家子弟能知忠信孝友,斯可矣,然不可令读书种子断绝,有才气者出,便名世矣。”②做一粒“读书种子”,让阅读成为一种力量,推动文化传统薪火相传,可以说正是读书人的自我期许。从“每有会意,便欣然忘食”的陶渊明,到“读书破万卷,下笔如有神”的杜甫,再到“不是老夫朝不食,半山绝句当朝餐”的杨万里,无不是“读书种子”的精彩写照。读书滋养美好心灵,可以遇见更好的自己,看到更美的世界。重视读书、重视传承,千百年来已融入中国人的血脉里,成为中国特有的文化禀赋。③在老一辈无产阶级革命家中,不少人是“读书种子”。彭德怀带兵打仗“横刀立马”,平时非常注重抓学习,而且还经常督促身边工作人员养成读书习惯。他常用吕蒙“士别三日,即更刮目相待”的故事激励大家,乐于把自己收藏的书拿出来共享。为了检查大家读书的情况,彭德怀用饭粒把书中的页码粘起来,如果发现有人读书做样子,没有把粘饭粒的书页打开,他就会提出严厉的批评。在彭德怀的关心督促下,他身边的工作人员都养成了读书习惯,终身受益。④“童心便有爱书癖,手指今馀把笔痕。”植物种子是有形的,延续繁衍,生生不息;“读书种子”则是无形的,不知不觉中变化气质、增长才干,承接弘扬中耳濡目染、潜移默化。正因为这样,文明薪火不断发扬光大。做一粒“读书种子”,静下心来读一点书、做一点学问,因为其中有对优秀传统文化的承袭,对高雅生活的追求,对时代潮流的引领。⑤研究植物种子的钟扬不拘泥于书本知识,在科学上敢于“奇思妙想”,在被认为无法种植红树林的上海滩涂中栽种成功。这是钟扬的故事。他的故事所展示的,不只是知识的力量,更有实践的力量、创新的力量,赋予“读书种子”以新内涵。世人常常说“知识就是力量”,但知识必须与实践结合,使之变成能力或本领之后,才能产生力量。“读书种子”与实践“沃土”的紧密结合,必能孕育壮苗、结出硕果。⑥今天,我们提倡有质量有分量的阅读,注重阅读率的同时更加注重“阅读力”,正需要像“读书种子”一样痴迷知识学问之中。少一点对物欲的追求,多一点对知识的渴求;少一点无谓的应酬,多挤一点时间读书;少一点人云亦云的跟风,多一点独立思考的精神,人生境界就能达到崭新高度。(有删改)【注】①四民:士、农、工、商。②世业:世代相传的事业。24.下面的说法不符合原文意思的一项是( )A.植物种子是无形的,生生不息;“读书种子”则是有形的,延续繁衍,承接弘扬中变化气质、增长才干。B.世人常常说“知识就是力量”,但知识必须与实践结合,使之变成能力或本领之后,才能产生力量。C.“读书种子”与实践“沃土”的紧密结合,必能孕育壮苗、结出硕果。D.少一点对物欲的追求,多一点对知识的渴求;少一点无谓的应酬,多挤一点时间读书;少一点人云亦云的跟风,多一点独立思考的精神,人生境界就能达到崭新高度。25.下面对第①段在文中作用的分析,正确的一项是( )A.交代“发明权”“思考”两词的由来,引出议论的话题;引用杨万里的话,起到了论据的作用,很有说服力,层层深入地阐发了对“每有会意,便欣然忘食”的深刻认识。B.交代“读书种子”一词的由来,引出议论的话题;引用黄庭坚的话,起到了论据的作用,很有说服力;引用黄庭坚的话,激发读者的阅读兴趣。C.交代“读书种子”一词的由来,引出议论的话题;引用陶渊明的话,起到了论据的作用,很有说服力,层层深入地阐发了对“不是老夫朝不食”的深刻认识。D.交代“发明权”“思考”两词的由来,引出议论的话题;引用杜甫的话,起到了论据的作用,很有说服力,层层深入地阐发了对“手指今馀把笔痕”的深刻认识;引用杜甫的话,激发读者的阅读兴趣。26.下面对本文的理解分析,不正确的一项是( )A.做一粒“读书种子”,让阅读成为一种力量,推动文化传统薪火相传,可以说正是读书人的自我期许。读书滋养美好心灵,可以遇见更好的自己,看到更美的世界。B.第③段中画线句子运用了举例论证的论证方法,举彭德怀用饭粒粘书页督促身边工作人员读书的例子,具体论证了老一辈无产阶级革命家中不少人是“读书种子”的观点,增强了说服力。C.钟扬的故事所展示的,不只是知识的力量,更有实践的力量、创新的力量,赋予“读书种子”以新内涵。D.从全文看,作者的主要观点是提倡有质量有分量的阅读,注重“阅读力”的同时更加注重阅读率,正需要像“读书种子”一样痴迷知识学问之中,从“读书破万卷,下笔如有神”的杨万里,到“半山绝句当朝餐”的杜甫,无不是“读书种子”的精彩写照。【答案】24.A 25.B 26.D【解析】24.本题考查文章内容的理解与辨析。A.由文章第④段“植物种子是有形的,延续繁衍,生生不息;‘读书种子’则是无形的,不知不觉中变化气质、增长才干,承接弘扬中耳濡目染、潜移默化”可知,选项中“植物种子是无形的,生生不息;‘读书种子’则是有形的”理解有误;故选A。25.本题考查文章内容的理解和分析。A.由第①段“‘读书种子’一词的‘发明权’,当属宋代的黄庭坚。他在《山谷别集》中说”可知,本段话借“读书种子”一词的发明权引出论题,引用的是黄庭坚在《山谷别集》中的话,并非杨万里,选项中“交代‘发明权’‘思考’两词的由来,引出议论的话题;引用杨万里的话”理解有误;C.由文章第②段“从‘每有会意,便欣然忘食’的陶渊明,到‘读书破万卷,下笔如有神’的杜甫,再到‘不是老夫朝不食,半山绝句当朝餐’的杨万里”可知,引用陶渊明的话并非在第①段,且“不是老夫朝不食”是杨万里的画,选项理解有误;D.由第①段“‘读书种子’一词的‘发明权’,当属宋代的黄庭坚。他在《山谷别集》中说”可知,本段话借“读书种子”一词的发明权引出论题,且引用杜甫的话在第②段的“到‘读书破万卷,下笔如有神’的杜甫”,选项表述有误;故选B。26.本题考查文章内容的理解分析。D.由第⑥段“我们提倡有质量有分量的阅读,注重阅读率的同时更加注重‘阅读力’”可知,选项中“注重‘阅读力’的同时更加注重阅读率”理解有误;故选D。九、阅读下文,回答问题。琴声和棋力之上陈鹏举①古往今来,琴棋书画,人人都不免遇见。这里说说排列前两位的琴和棋。②先说琴。中国古琴留下来的总共也就几十首曲子吧。只是这几十首曲子,胜却无数。古琴曲子,每个人弹出来都不一样,不是水平高低的不一样,而是一首曲子,一百人弹,就是一百个模样。所谓琴声,在古琴里,不单是琴声,更是弹琴人的心声。这是古琴伟大的地方,也是古琴足以遗世独立的地方。③俞伯牙和钟子期是知音。钟子期听俞伯牙弹“高山流水”,听出了俞伯牙的心声。人生在世,所经历的悲欢荣辱,只有自己最清楚,就像鱼在水中,冷暖自知。俞伯牙把这宣泄在琴声里,居然被另一个人听清楚、听明白了。这就是俞伯牙和钟子期的缘分。④白居易在《琵琶行》里详尽地写了琵琶的弹奏过程,是叙事诗的需要,也是他对琵琶弹奏的精熟了解,更是他对音乐不可名状的敏感和欣赏能力。这段有关琵琶弹奏的文字极为美妙,同时也十分精当地写出了他在琵琶声里听到的悲伤。他与琵琶女“同是天涯沦落人”的伤感,也就随之而来。⑤苏东坡写过一首《琴诗》:“若言琴上有琴声,放在匣中何不鸣?若言声在指头上,何不于君指上听?”他说,琴声从哪里来?说是来自琴吧,那琴在琴匣里的时候,怎么没声音?说是来自弹琴的手指吧,那么离开琴的手指,怎么没声音?这个绝顶聪明的人,用了似乎玄虚的诗句,说出了再浅显不过的道理:琴声是人弹出来的。⑥再说棋。这个世界上,最具棋的意义的许是中国围棋。围棋黑白对垒,每颗棋子下在棋枰上,只占一个点,不能移动,一如星斗满天,胜似山崩海啸,却又生机无限。说起来,棋是要分胜负的,分了胜负,棋才算终局。只是棋的最终意义,不在分胜负,不是下棋人的棋力,而是下棋人的心力。⑦人生在世,总希望获得成功,下棋人也是。只是历来下棋的大人物、大棋手,并不这么想。他们要的是伟大。伟大是什么?伟大是壮志、是梦想,是为实现壮志和梦想所竭尽的心力。⑧东晋谢安正和客人下围棋,侄儿谢玄书信到了。他看完信,得知前秦苻坚已被击败,淝水之战已经获胜,他放下信,脸上不露喜色。等客人问了,他才缓缓说:“小儿辈大破贼。”说话时,神情举止与之前没什么变化。直到下完棋,他回内室。过门槛的时候,木屐上的齿碰折了,他也没发现。史书的记录自始至终没写棋局本身,连胜负也没提及,只是写谢安的神情举止,还有缓缓说出的一句话。淝水之战,东晋以少胜多,这一战将决定历史的走向,战前东晋并无取胜的十分把握,谢安下棋时的心情,是可以想见的。他和来客下棋时,翻江倒海的心力,非常人所能及。⑨最好的棋手,都不是“胜负师”。当代棋手中,有的风格华丽,有的风格唯美,有的甚至第一手棋就直落天元。真正的棋手,对棋的渴望,是对未知的渴望,这个未知,不仅指棋,还指人生和世界。⑩下棋的意义是否在于分胜负?到今天,这个问题已无须再争。阿尔法狗出现了,人类最好的棋手难求一胜。下棋还有意义吗?仍然有。棋还会下吗?仍然会下。缘由是,阿尔法狗争的仅仅是胜负,而人类棋手不是。人类棋手在棋枰上倾注的心力,不是阿尔法狗可以理解的。(选文有删改)27.下列关于本文思路或结构的说法不正确的一项是( )A.文章采用了先总后分的结构,先总起,再分别叙述琴与棋。B.写琴时依时间顺序提到俞伯牙和钟子期、白居易、苏东坡。C.作者谈到最具棋的意义的中国围棋,思绪跨越过去和现在。D.文章以琴、棋为线索,传递了古今中外文人对琴、棋的热爱。28.品读第⑧段中的画线句,试结合其中细节分析当时谢安的心情及人物性格特点。29.请结合全文,谈谈你对文章标题“琴声和棋力之上”的理解。30.读完本文,发挥想象,描绘你在王维《竹里馆》里听到的“琴声”。竹里馆[唐]王维独坐幽篁里,弹琴复长啸。深林人不知,明月来相照。【答案】27.D 28.示例一:从文中的“缓缓说”“木屐上的齿碰折了,他也没发现”,可见谢安内心其实是激动狂喜的,但他却能强装镇定,不喜形于色,这份过人的定力是非常罕见的。示例二:接到“淝水之战”的捷报,谢安内心其实是激动狂喜的,以至于过门槛的时候,他“木屐上的齿碰折了,他也没发现”。但他却能在外人面前压抑内心的翻江倒海,不喜形于色,只“缓缓”言说,这份心力非常人所能及。 29.示例:“琴声”是人弹出来的,“琴声”不单是乐器所发出的声音,更是弹琴人的心声;棋本是要分胜负的,“棋力”本指下棋人的功力,但在作者看来,棋的最终意义不在分胜负,不在下棋人的棋力,而在下棋人的心力。琴声与棋力承载了人们对于这个世界和人生的理解,这种文化的内涵超出了娱乐或博弈本身,这也是琴和棋最伟大的地方。 30.示例:幽深的竹林里不时传出几声琴音,琴声悠扬,时有时无,时远时近。月光倾泻,可以想见诗人坐在蒲团上抚琴的闲适样子。【解析】27.本题考查信息的筛选和辨析。D.“文章以琴、棋为线索,传递了古今中外文人对琴、棋的热爱”表述错误,线索是贯穿一篇文章始终的,在文章的不同段落中都有所体现,但这篇文章的琴与棋是分开叙述的,不能称之为线索。故选D。28.本题考查细节描写及作用。找出具体的细节描写,联系语境分析其作用。此句的细节描写有两处:一处是对客人“缓缓说”淝水之战大获全胜的消息。接到“小儿破贼”的捷报,谢安的内心应该是狂喜不已,可是他能镇定自如,不表露出来,体现其过人的定力;二处是“过门槛的时候,木屐上的齿碰折了,他也没有发现”。说明谢安沉浸在内心的激动喜悦之中,没把别的事放在心上。但这种喜悦仍没有表现于外,这份心力非常人所能及。29.本题考查标题含义。标题含义要理解表层含义,还要挖掘深层含义。表层含义:文章第②段关于“琴声”作者要表达的观点是,琴声不仅仅是指乐器发出的声音,更多的是传递出了弹琴者的心声;文章第⑥段作者提到“棋的最终意义,不在分胜负,不在下棋人的棋力,而是下棋人的心力”。深层含义:文章第⑨段“真正的棋手,对棋的渴望,是对未知的渴望,这个未知,不仅指棋,还指人生和世界”一句的提示可知,作者写“琴”与“棋”想要表达的是这些器具作为一种载体已经超越了琴与棋本身的功能,承载了人们对于这个世界和人生的理解,这也是对题目“之上”二字的诠释,体现了作者的独到感悟。30.本题考查迁移能力。要充分理解文本内容,结合题干要求展开想象,组织语言作答。《竹里馆》的大意是:我独自坐在幽深的竹林里,一边弹琴一遍高歌长啸。没有人知道我在竹林深处,只有明月相伴静静照耀。依据题干的要求,要展开合理的想象,描绘静夜之下“琴声”空灵与悠扬,体现诗人隐居时的现实生活与情趣。例如:清幽寂静的竹林中传出悠扬动听的琴声,时断时续,悠远绵长,似有清溪静静流淌,流向了远方。那弹琴之人,一定是独坐月下,轻拢慢抚,悠闲自在。十、阅读下文,回答问题。读书的理由押沙龙①老家有一位长辈,属于国学爱好者。在我小时候,他曾拿着树枝在地上一边画字一边给我讲解,“城外为廓,廓外为郊,郊外为甸”,说得非常郑重,好像是告诉我一个极大的秘密。他也读过《易经》,没事了还给人算命。我上大学后,有次寒假回家碰到他,他兴奋地拉着我这个“学问人”问长问短,还说要出个上联考考我。上联大致是“走马灯,灯走马,灯熄马停步”之类的东西。我想了半天对不出来,只好说我不会,他既失望又惊诧,意思是你连这都不会,还上什么大学堂呢?②仔细想来,他的这些想法其实很有传统根据的。过去民间对才子的最常见表扬就是会对对联会写诗。在中国民间,自古以来就把知识当成一种类似巫术的东西,而且非常相信文字本身的魔力。中国民间最崇拜的知识分子可能就是诸葛亮了。鲁迅说《三国演义》的“诸葛多智而近妖”,以为是败笔,其实正因为他“近妖”,才更贴近民间对大知识分子的遐想。这传统一直延续到现代小说《白鹿原》,里面的朱先生也是这个路子。③当然,文字本身也是有魔力的。李太白醉草的吓蛮书可以吓跑外国入侵者,韩退之的祭鳄文可以赶走水里大鳄鱼。古代文化精英对这些故事多是笑而存之,但底层百姓,尤其是底层知识分子,对此是很愿意相信的。而且他们很执拗地认为,一个人能写出好文字来,自然就会治理国家。他们这么想,原因也很简单,那就是他们崇拜知识,但并不真知道知识有什么用。人们对自己崇拜而又不懂的东西,就容易有这种感觉。这就像我们现在看《最强大脑》,大部分观众并不太理解科学上的真正强大思维是怎么一回事,但知道水下盲拧魔方很难,所以容易把这种脑力杂技当成“最强大脑”。这其实跟古代老百姓崇拜对对联的才子有点像。④中国传统文化谱系里缺少一种“爱智”的传统。文化精英们强调文化知识的功用性,普罗大众则信仰它的神秘性。对于前者,知识像是一种教化的工具;对于后者,知识像是一种神秘的巫术。经过现代文明的“祛魅”之后,教化和巫术一起坍塌,大家就有点无所适从了。⑤那么读书到底有什么用呢?⑥读书无非是一种交流。书籍就像一个个漂流瓶,作者把他的感受、经验、思考放到这个瓶子里,然后你把它打捞起来,满足你健旺的好奇心、贪婪的求知欲,驱散你特有的孤独感。它们也许能让你在现实中做决定时更聪明一些,也许不能。读书能否让你变得更聪明,往往取决于你其他的一些特质。我见过很多聪明的读书人,也见过不少愚蠢、偏执的读书人,还见过不少如果少读点书也许更好的人。读书只是给你一个机会,让你有可能活在一个更大一点的世界里。但如果你对此不在乎,那也真的不是什么太大不了的事儿。31.文章题目是《读书的理由》,为什么要从老家长辈写起?32.文化知识的神秘性表现在哪几个方面?33.阅读下列句子,回答括号中的问题。(1)知识像是一种神秘的巫术。经过现代文明的“祛魅”之后,教化和巫术一起坍塌,大家就有点无所适从了。(加点的“祛魅”具体指什么?) (2)我见过很多聪明的读书人,也见过不少愚蠢、偏执的读书人,还见过不少如果少读点书也许更好的人。(选择一处加点词,从初中三年指定阅读的名著、三年里学过的课文中找出与加点词相对应的人物,并阐述理由。)34.以下是几种关于读书的理由,请结合本文观点,选取其中一种理由谈谈你的看法。A.古话说得好:“书中自有颜如玉,书中自有黄金屋。”所以我要好好读书。B.初中三年,除了教材,我只读教材规定的名著,因为中考要考名著阅读呀!C.教材指定的名著并非全部都是精品,值得一读的书也不是只有文学作品一种类型,大境界的人生需要有大阅读的思维。多读书,读好书,才是写好“人”字的基础。【答案】31.文章从老家长辈写起,通过写他把国学看得神乎其神(很神秘),对我对不出对联感到失望惊诧等事情(关于长辈的事讲出一点即可),体现了中国民间对知识的态度,引出作者对读书的理由的思考(或下文对读书理由的议论),激发读者兴趣。 32.文化知识的神秘性表现在自古以来人们把知识当成一种类似巫术的东西,而且非常相信文字本身的魔力。 33.(1)“祛魅”指的是现代文明让文化知识显得不再神秘,也弱化了它的功用性。(在现代文明背景下人们的知识崇拜不复存在,教化功能弱化。意思相近即可。)(2)聪明的读书人示例一:我认为《儒林外史》中的王冕是聪明的读书人。他上知天文下知地理,而且对科举制度有非常清醒的认识,不死读“八股文”,有自己的爱好和独立的人格。示例二:我认为王选是一个聪明的读书人:他既是“汉字激光照排系统之父”,又是“有市场眼光的科学家”,更难能可贵的是他对于名气、权威有着非常清醒的认识,敢讲真话,讲实话,并在功成名就之后能够大力扶持年轻人。愚蠢、偏执的读书人示例三:我认为《儒林外史》中的马二先生是愚蠢、偏执的读书人。他读书只认定“八股文”,一生追求举业,丝毫没有自己的思考,显得迂腐、偏执。(周进、范进、鲁编修等人皆可。)示例四:我认为孔乙己是愚蠢、偏执的读书人。他热衷科举,却连“半个秀才”都捞不到,被人嘲笑、欺辱,还一直穿着他那件象征读书人身份的破长衫不肯脱下,显得迂腐可笑又可怜。如果少读点书也许更好的人示例五:我认为《儒林外史》中的匡超人是个少读书也许更好的人。少年时期的匡超人淳朴忠厚,孝敬父母,到杭州结识假名士后思想蜕变,跟潘三混在一起后更是用胸中一点笔墨包揽讼词,代人参加科举考试,读书所学尽用来做毫无礼义廉耻的事情了。(严贡生、王县令、汤公等人皆可)《孔乙己》中的丁举人都可以,情节清楚,理由分析到位即可。)(现实中古今中外读书人的例子都可以举,只要言之成理即可。) 34.我选择A。“书中自有颜如玉,书中自有黄金屋”这种读书理由是极端功利主义的表现。本文作者告诉我们,读书是一种交流,它在满足人的好奇心、求知欲的同时更重要的是能充实人的内心,提升人的精神境界。我选择B。只读考试要考的书,不读其他,显然是对读书概念的窄化与浅化,(这种读书理由是极端功利主义的表现。)名著只是引领,目的是让我们真正爱上阅读,懂得阅读,真正的阅读应该像本文作者那样,把读书当做是一种交流,在阅读中去满足自己的好奇心、求知欲,去丰盈自己的内心,提升自己的精神境界。我选C。教材指定的名著不乏经典之作,但受制于编选的主客观条件,难免有不如人意的地方。在浩瀚的书籍海洋中也肯定有许多的经典值得我们去拜读。阅读的范围也不要仅仅局限于文学作品,可以把视野放到整个人类文化的层面,在与书本交流的过程中去丰富感受,汲取经验,学会思考,丰盈内心,提升精神境界,用大视野去书写人生的大格局。【解析】31.本题考查的是文章开头的作用。议论文的开头通常有两个作用:一是通过事例引出论题或论点。二是激发读者阅读兴趣。本文开头,从老家长辈要“我”对对联写起,对“我”这个大学生感到失望和惊诧写起,引出民间把知识看得很神秘的传统,体现了中国民间对知识的态度,从而引出下文对读书理由的议论,增强了文章情趣,能激发读者的阅读兴趣。32.本题考查文章信息的筛选和提炼。文章第①段“好像是告诉我一个极大的秘密”,说明了文化知识在民间有神秘性,具体表现在第②段中的“在中国民间,自古以来就把知识当成一种类似巫术的东西,而且非常相信文字本身的魔力”。33.(1)本题考查词语的理解。“祛魅”本义指“是指对于科学和知识的神秘性、神圣性、魅惑力的消解”。句子“经过现代文明的‘祛魅’之后,教化和巫术一起坍塌,大家就有点无所适从了”中的“教化和巫术”是指上句中的“文化精英们强调文化知识的功用性,普罗大众则信仰它的神秘性”。“祛魅”意思是弱化了“文化知识的功用性”,“坍塌”说明“在现代文明背景下人们的知识崇拜不复存在”。据此可知,“祛魅”这里是指在现代文明背景,人们的知识崇拜不复存在,知识不再神秘,知识的教化功能也大大弱化。(2)本题考查文章阅读的拓展应用。“聪明的读书人”“愚蠢、偏执的读书人”“如果少读点书也许更好的人”,这三类人物,在课文及名著中有很多代表,如《儒林外史》中的各色知识分子,鲁迅笔下的孔乙己、寿镜吾、藤野先生等。示例:①“我见过很多聪明的读书人,也见过不少愚蠢、偏执的读书人,还见过不少如果少读点书也许更好的人”句中“聪明的读书人”指的是那些会读书,真正从书中得到智慧,并学有所成的人。《朝花夕拾》中的鲁迅,1902年,他东渡日本,开始在东京弘文学院补习日语,后来进入仙台医学专门学校。他之选择学医,意在救治像他父亲那样被庸医所害的病人,改善被讥为“东亚病夫”的中国人的健康状况。“匿名信事件”和“看电影事件”,鲁迅受到极大的刺激。这使他认识到,精神上的麻木比身体上的虚弱更加可怕。对国人有非常清醒的认识,弃医从文学,回到本国从事文艺工作,希望用以改变国民精神。②“愚蠢、偏执的读书人”指的是那些被科举制度毒害迂腐的读书人。《儒林外史》中的马二先生,是一个为举业而耗尽终生的受害者,但他始终把举业视为“神圣不可侵犯的事业”,他甚至迷信八股到这样的程度,显得迂腐、偏执。③“如果少读点书也许会更好的人”,指那些本来有些智慧,却因曲解读书的目的而一事无成的人。《孔乙己》中的孔乙己,是清朝末年的知识分子,他信奉万般皆下品,唯有读书高的教条,但是他连秀才都没有考上,有没有功名利禄,生活过得非常穷酸。孔乙己有着封建教育制度影响下的书生特有的清高本质,如果他能少读书,不端着读书人的架子,就不会深受科举制度的毒害,就不会落个悲惨的结局,命运也许会更好。34.本题属于开放性试题。结合文章观点,选取三个选项中的任一理由,结合实际来谈。语句通顺,言之成理即可。例如:选择A,“颜如玉”和“黄金屋”是古人对读书意义的理解,这是一种物质上的追求。但文中提到读书“满足你健旺的好奇心、贪婪的求知欲,驱散你特有的孤独感”,这是精神世界的满足,从而丰盈自己的内心,提升自己的精神境界。选择B:“只读教材规定的名著”对于这种目的性极强的读书,必然会使读书的面缩小,我们读书是为了是自己在多方面有提升,吸收提升自己的知识层面,思想境界的提高等,而不仅仅是为了成绩的需求,“满足你健旺的好奇心、贪婪的求知欲”。选择C:“读书无非是一种交流”,“驱散你特有的孤独感”,“给你一个机会,让你有可能活在一个更大一点的世界里”,所以要多读书,在读书中体验快乐,在交流中提升自我。在与书本交流的过程中去丰富感受,汲取经验,学会思考,去丰盈自己的内心,提升自己的精神境界。十一、非连续性文本阅读 一、阅读下面的材料,完成后面的小题。【材料一】“天问一号”探测器由环绕器和着陆巡视器(含“祝融号”火星车)两部分组成。前期,环绕器是搭载“祝融号”火星车的星际专车。在将“祝融号”火星车安全送达火星后,环绕器上升至中继轨道,此后数月它将变身为通信器、建立起“祝融号”火星车与地球之间的中继通信。“祝融号”火星车在着陆初期阶段,它与地面的通信通过环绕器采用UHF频段+X频段两种中继通信方式进行。X频段深空应答机负责环绕器与地面的通信,UHF频段收发信机负责“祝融号”火星车与环绕器之间的通信,再加上25米口径的大天线,共同构成一个主体通信网。(摘编自中央纪委国家监委网站,有删改)【材料二】探索火星的愿望美好,难度却极大。火星与地球相距遥远,每隔780天左右才会合一次,会合时带来的理想探测窗口期仅一个月左右。不仅如此,由于探测器飞行线路会受到太阳系内多个天体的扰动,还有太阳风、空间辐射等的复杂影响,轨道设计的难度可想而知。“天问一号”被火星捕获后,在环绕火星的三个月飞行中,拍摄了大量火星表面照片,由科学家们反复研究和比较分析,最终选出符合探测任务要求的着陆地点。要完成降落火星的任务,还需要解决一系列更复杂的问题。“天问一号”从火星大气层外缘通过软着陆的方式降落到火星表面,整个降落过程大致分为“进入、减速、软着陆”三步。从开始踏上进入点的那一刻起,“天问一号”就迎来了此次探测旅程中最为凶险的“恐怖9分钟”。在这短暂的时间内,“天问一号”要从约2万千米/小时的速度降至0米/小时,与此同时还要完成一系列的复杂动作,整个过程,不能有丝毫差错。而且由于火星与地球距离遥远、信号微弱、通信延时等原因,地面人员没有干预的机会,完全靠“天问一号”自主完成:着陆后,“天问一号”携带的“祝融号”火星车,驶向火星地面,开始工作。(摘编自《科技日报》有删改)【材料三】从人类目前对火星的探测结果看:火星大气层很薄,氢气极少,同时充满尘埃。地表温度白天可达28℃,夜晚可低至-132℃,平均-57℃。相比于地球,火星完仓不具备人类生存的条件。哪怕地球环境再恶劣,从必要性来说,改造地球上的沙漠,也比改造火星容易得多。那又何谈“移民火星”?(摘编自《科学认识探测火星的重要意义》1.下列对材料的理解正确的一项是( )A.“天问一号”探测器由环绕器和“祝融号”火星车组成,环绕器既是星际专车又是中继通信器。B.火星探测难度极大,原因之一是火星与地球会合带来的理想探测窗口期相对较短C.“天问一号”探测器在飞行和着陆火星的整个过程,都必须完全独立自主地完成相关工作。D.“祝融号”火星车在着陆初期阶段,X频段深空应答机负责环绕器与火星车的通信,UHF频段收发信机负资地面与环绕器之间的通信。2.说说材料二中划线句子所用的说明方法并分析其作用。3.如果你是一位科幻小说的作家,请在三则材料的基础上构思人类成功“移民火星”的创举。【答案】1.B 2.运用列数字的说明方法,列举“约2万千米/小时”“0米/小时”,准确具体科学严谨,强调“天问一号”需要在短时间内急剧降速,突出了“天问一号”着陆难度大本领强。 3.言之有理,符合文本内容即可。【解析】1.本题考查信息筛选与判断。A.有误,结合材料一“天问一号探测器主要由环绕器和着陆巡视器(含·祝融号”火星车)两部分组成”可知,选项“由环绕器和祝融号火星车组成”说法不准确;C.有误,根据材料二“在这短暂的时间内,‘天问一号’要从约2万千米/小时的速度降至0米/小时,与此同时还要完成一系列的复杂动作,整个过程,不能有丝毫差错。而且由于火星与地球距离遥远、信号微弱、通信延时等原因,地面人员没有干预的机会,完全靠“天问一号”自主完成”可知,“天问一号”探测器着陆火星的整个过程中是“完全独立自主地完成相关工作”,并不是“飞行”时也是“完全独立自主地完成相关工作”。D.有误,根据材料一“X频段深空应答机负责环绕器与地面的通信,UHF频段收发信机负责‘祝融号’火星车与环绕器之间的通信”可知,选项“X频段深空应答机负责环绕器与火星车的通信,UHF频段收发信机负责地面与环绕器之间的通信”说法不正确。故选B。2.本题考查说明方法及作用分析。材料二划线句子“在这短暂的时间内,要从约2万千米/小时的速度降至0米/小时”中,“约2万千米/小时”和“0米/小时”运用的是列数字的说明方法,具体准确地列举了“天问一号”着陆时速度的变化,借此说明了“天问一号”着陆难度大,更容易使读者信服,突出了“天问一号”本领强。3.考查拓展能力。开放性试题,答案不唯一。由材料一可知,“天问一号”探测器可以与地球之间建立通信联系,为“移民火星”解决通讯问题。由材料二可以“天问一号”可以环绕火星三个月飞行,又突破技术难题,成功着陆火星,为“移民火星”解决核心技术问题。由材料三可知,火星大气层很薄,氢气极少,同时充满尘埃。地表温度白天可达28℃,夜晚可低至-132℃,平均-57℃,虽然改造困难,但并不是不可能。但聪明的人类会通过制造氧气,利用空调系统等方法改造火星条件,创造具备人类生存的条件,让人类成功“移民火星”。示例:100年后,C国“天问100号”诞生,“天问100号”探测器功能强大,火星与地球通信顺畅。“天问100号”可以环绕火星飞行长达4个月,由于通讯问题解决了,100年前最为凶险的“恐怖9分钟”着陆问题早已攻克,5分钟安全着陆是常态,经过治理,火星空气质量达标,适合居住的特殊的“火星仓”研制成功了,“移民火星”计划不再是梦想。二、阅读下面几则材料,完成下面小题。材料一:①1949年以来,考古工作者在河南安阳、郑州商代遗址中都发现了一种青釉器。这些青釉器究竟属于陶器还是瓷器呢?为了解决这个问题,考古文物界、陶瓷科技界的学者携手合作,对这个问题展开了深入研究。中国科学院上海硅酸盐研究所古陶瓷专家李家治等人采集自商代中期至春秋战国的青釉器文物标本40多件,进行了测试。对胎釉测试的结果是青釉器胎中氧化硅含量都在75%左右,氧化铝含量也在15%左右,加起来在90%左右。瓷土的主要成分是硅酸铝,所以其中氧化硅和氧化铝的含量较高。而一般陶器的氧化硅含量在70%以下,加上氧化铝,总含量也仅在80%左右,这一数据表明,青釉器胎体所用原料已摆脱了制陶所用的易熔黏土,应是采用了较粗放的瓷土。在青釉器的胎体中,氧化钙、氧化镁等碱性氧化物的含量明显下降,大都在1%以下,而一般陶器中,大都在3%以上,青釉器的氧化铁的含量一般小于3%,而一般陶器中氧化铁的含量约为6%。②由于青釉器胎体上酸性氧化物含量增加,碱性氧化物含量明显减少,所以要求在较高温度(即1200℃左右)下才能烧结,烧结后,质地坚硬,开口的气孔大大减少,所以吸水率明显降低。化学组成及由它引起的烧成温度和性能不同,就使青釉器在质地上明显不同于一般陶器。③再从青釉器的显微结构来看,它胎中的石英颗粒大小不一,还有一定量的莫来石(莫来石是煅烧后的普通陶瓷中含有的一种最重要的 SiO2·A12O3二元系统的矿物晶体)和相当比例的玻璃相,并有一定量的气孔,这种结构与泥质陶器有着本质的区别。青釉器表面的釉的显微结构比较简单,一般已经完全熔为玻璃,非常透明。(节选自周嘉华《文物中的化学》,有删改)材料二:唐代人陆羽在《茶经》中,将青瓷比作玉,比作冰,他的比喻代表了中国士大夫们的审美观,是对自己君子行为操守的炫示,其核心观念是君子要有温文尔雅的风度,做事要举行中庸的原则。晚唐五代最著名的瓷器品种是越窑秘色瓷,唐代诗人陆龟蒙《秘色越器》中有“九秋风霉越窑开,夺得千峰翠色来”一句,说的是秋天越窑开窑了,那釉色如同融入“千峰翠色”,足见其精美。宋代的青釉,依然追求表现瓷器如玉般的质感。明代中叶,商品经济迅速发展,一种强烈要求摆脱旧思想旧道德的束缚,公开谈论物质享受的思潮不可遏制。晚明清初,实学思潮勃兴,一些儒学家也认为,人的需要应该是全面的,既要有精神生活,也要有物质生活。于是,人们不再拘泥于虚幻的自我表现形式,瓷业生产也张开双臂,迎接五彩缤纷、精细艳俗的彩瓷时代的到来。(节选自霍华《恰如灯下故人: 谛听中国瓷器妙音》,有删改)材料三:①2013年9月,习近平在哈萨克斯坦纳扎尔巴耶夫大学发表演讲时表示,各国可以利用创新的合作模式,共同建设“丝绸之路经济带”,由此提出了“一带”建设思想。同年10月,习近平在印尼国会发表演讲时表示,中国愿同东盟国家加强海上合作,共同建设21世纪“海上丝绸之路”,由此提出了“一路”建设战略。“一带一路”倡议继承古丝绸之路开放包容、兼收并蓄的精神,因此和古丝绸之路有相似之处。②中国灿烂的陶瓷文化是传统民族文化高度发展的产物,是民族文化的结晶。而景德镇通过美轮美奂的陶瓷艺术、精湛卓越的制瓷工艺,确立了其在中国乃至世界的“瓷都”地位。在现今的时代背景下,景德镇陶瓷可以重振往日辉煌。(节选自郭杰忠《海上丝绸之路:陶瓷之路》,有删改)4.下面是一份有关商代青釉器的探究任务单,请你根据材料一和已有内容,进行完善。5.材料二中,从唐宋至明清,人们在瓷器色彩及审美观念上出现了怎样具体的变化?6.在习近平主席提出“一带一路”重大战略决策十周年之际,苏州某学校进行了一次“瓷都”实地游学活动,同学们深感景德镇瓷器在现代可以走得更远。请你根据材料内容,并结合生话实际,为景德镇瓷器在“一带一路”中更好地走出国门提出可行的具体建议。【答案】4.青釉器究竟属于陶器还是瓷器? 商代青釉器的原料是什么?商代青釉器的成分含量如何? 商代青釉器的施釉情况如何?商代青釉器的结构怎样? 90%左右 表面的釉的显微结构比较简单,一般已经完全熔为玻璃,非常透明。 瓷器 5.瓷器色彩:(1)唐代:追崇如玉如冰般的青瓷。(2)晚唐五代:喜欢越窑秘色瓷。(3)宋代:追求如玉般的质感。(4)明清之际:人们青睐于五彩缤纷、精细艳俗的彩瓷。审美观:(1)唐代:追求温文尔雅的风度和做事的中庸原则(2)晚唐五代:追求精美。(3)宋代:追求质感。(4)明朝中叶:摆脱旧思想旧道德束缚,谈论物质享受的思潮(5)晚明清初:人既要有精神生活,也要有物质生活。人们不再拘泥于虚幻的自我表现形式。 6.(1)自身品牌意识:打造景德镇品牌,弘扬文化自信。在坚持景德镇瓷器的核心陶冶技术的基础上,吸纳更多国家的元素。让更多的外国人喜欢中国的瓷器。举办“丝路瓷行”系列文化展演活动,线上线下同步进行。完善运输技术,做好售后服务。(2)宣传层面:拓宽渠道,创新景德镇瓷器的宣传途径:借助主流媒体,展示千年瓷都的风采。主动加强与国内外媒体的合作,通过各种宣传和纪录片不断加大宣传力度。(3)政府层面:促融合,推动不同文明交流互鉴:借助官方微博、微信公众号等新媒体加强宣传。建设文创街区、艺术景区、科教园区等“筑巢引风”,成立招才引智局等,让越来越多的海外陶瓷爱好者和艺术家选择在景德镇学习、创作和生活,形成了独特的“洋景漂”现象。(4)商业层面:举行拍卖会或者博览会等商业活动。【解析】4.本题考查材料的提炼与概括。提出问题:结合材料一“1949年以来,考古工作者在河南安阳、郑州商代遗址中都发现了一种青釉器。这些青釉器究竟属于陶器还是瓷器呢?为了解决这个问题,考古文物界、陶瓷科技界的学者携手合作,对这个问题展开了深入研究”可概括为:青釉器究竟属于陶器还是瓷器?探究问题:参照问题解决的角度,结合探究任务单中解决问题的第一和第二列内容为“器物种类”和“原料”,据此可提出问题:商代青釉器的原料是什么?商代青釉器的成分含量如何?结合探究任务单中解决问题的第四列内容为“施釉情况”,和材料一第③段“再从青釉器的显微结构来看”可以据此提出问题:商代青釉器的施釉情况如何?商代青釉器的结构怎样?结合材料一第①段“对胎釉测试的结果是青釉器胎中氧化硅含量都在75%左右,氧化铝含量也在15%左右,加起来在90%左右”可概括为:90%左右;结合材料一第③段“再从青釉器的显微结构来看,它胎中的石英颗粒大小不一,还有一定量的莫来石(莫来石是煅烧后的普通陶瓷中含有的一种最重要的 SiO2·A12O3二元系统的矿物晶体)和相当比例的玻璃相,并有一定量的气孔,这种结构与泥质陶器有着本质的区别。青釉器表面的釉的显微结构比较简单,一般已经完全熔为玻璃,非常透明”可概括为:表面的釉的显微结构比较简单,一般已经完全熔为玻璃,非常透明;综合“商代青釉器”的原料和烧成温度以及施釉情况来看,它不是一般陶器,而应该是比较初级的瓷器,所以得出结论:青釉器是瓷器。5.本题考查材料内容的提炼和概括。瓷器色彩上:结合材料二“唐代人陆羽在《茶经》中,将青瓷比作玉,比作冰”可概括为:唐代:追崇如玉如冰般的青瓷。结合材料二“晚唐五代最著名的瓷器品种是越窑秘色瓷,唐代诗人陆龟蒙《秘色越器》中有‘九秋风霉越窑开,夺得千峰翠色来’一句,说的是秋天越窑开窑了,那釉色如同融入‘千峰翠色’”可概括为:晚唐五代:喜欢越窑秘色瓷;结合材料二“宋代的青釉,依然追求表现瓷器如玉般的质感”可概括为:宋代:追求如玉般的质感。结合材料二“晚明清初”和“瓷业生产也张开双臂,迎接五彩缤纷、精细艳俗的彩瓷时代的到来”可概括为:明清之际:人们青睐于五彩缤纷、精细艳俗的彩瓷。审美观:结合材料二“唐代人陆羽在《茶经》中,将青瓷比作玉,比作冰”和“其核心观念是君子要有温文尔雅的风度,做事要举行中庸的原则”可概括为:唐代:追求温文尔雅的风度和做事的中庸原则;结合材料二“晚唐五代最著名的瓷器品种是越窑秘色瓷”和“那釉色如同融入‘千峰翠色’,足见其精美”可概括为:晚唐五代:追求精美;结合材料二“宋代的青釉,依然追求表现瓷器如玉般的质感”可概括为:宋代:追求质感;结合材料二“明代中叶,商品经济迅速发展,一种强烈要求摆脱旧思想旧道德的束缚,公开谈论物质享受的思潮不可遏制”可概括为:明朝中叶:摆脱旧思想旧道德束缚,谈论物质享受的思潮;结合材料二“晚明清初,实学思潮勃兴,一些儒学家也认为,人的需要应该是全面的,既要有精神生活,也要有物质生活。于是,人们不再拘泥于虚幻的自我表现形式”可概括为:晚明清初:人既要有精神生活,也要有物质生活。人们不再拘泥于虚幻的自我表现形式。6.本题考查提建议。属于开放性试题,结合题干可知,根据材料内容,并结合生话实际,为景德镇瓷器在“一带一路”中更好地走出国门提出可行的具体建议。结合材料一和材料二内容可知,我国的瓷器历史悠久,结构精美;结合材料三可知,“景德镇通过美轮美奂的陶瓷艺术、精湛卓越的制瓷工艺”“在现今的时代背景下,景德镇陶瓷可以重振往日辉煌”,我国可以与其他国家合作,共同建造“一带一路”,将瓷器文化重振光彩。可以从四个角度,分别是瓷器自身角度、政府角度、宣传方面、商业层面。示例:自身角度:打造景德镇品牌,提升自身质量,同时吸纳国外现代元素,进行文化展览,做好售后服务;政府角度:政府进行基础设施建设,建设相关园区,要吸引海外投资力量和陶瓷方面艺术家进行交流合作,借助抖音,微博等新媒体进行宣传,让更多的人了解景德镇瓷器文化;加强市场监管,培养产权意识;加强人才培养,鼓励文化交流;宣传层面:在传统媒体如《人民日报》上刊登专栏介绍,央视类投放广告,拍摄纪录片,在主流媒体循环播放等等;商业层面:举行大型博览会,抓住互联网发展契机,电商平台进行线上规模销售。三、阅读下面的材料,完成下面小题。【材料一】春联这种文学样式以工整、对偶、简洁、精巧的文字描绘时代背景,抒发美好愿望。春联的内容有许多讲究。譬如,“春好禾苗壮,人新稻谷丰”是体现农民对新的一年祈望与祝福的春联。不同的人家,不同的行业,都会有不同于他人的祈望与祝福。春联的内容,不仅应符合作者自身的特点,更应体现出一种健康的审美趣味和追求。如“一夜连双岁岁岁如意,五更分二年年年称心”这样的春联表达了对未来的美好追求和向往。【材料二】春联在形式上有诸多限制。首先,上下联字数要相等。其次,上下联词性要相对。如“新春富贵年年好,佳岁平安步步高”,“新春”对“佳岁”,“富贵”对“平安”,“年年好”对“步步高”。此外,上下联平仄要相调。现在的一声二声大致相当于古代的平声,三声四声大致相当于古代的仄声。对联讲究平仄,末尾最严。上联末尾字字音必须为仄声,下联末尾字字音必须为平声。【材料三】“脱单脱贫不脱发,高个高颜求高分”……春节将至,年轻人把不加班、不脱发、不出BUG这些接地气的愿望写进春联,还融入了音符、代码、英文、化学元素或互联网中的流行文化,写成了别有一番风味的“现代春联”,让年味有了浓浓新意。(选自《广州日报》2022年1月8日,有删改)7.根据材料,下列不属于春联的一项是 ( )A.春临大地百花艳 节至人间万象新B.爆竹声声辞旧岁 红梅朵朵迎新春C.冬去山川齐秀丽 春来桃里共芬芳D.新厦落成增瑞气 华门安居进财源8.志愿者送了一副对联给敬老院,小闽不知道以下两联哪个是上联,哪个是下联。请你根据材料中对联的相关知识,正确写出上下联并说明理由。9.有人担心材料三中这种“万物皆可写春联”的做法,会拉低春联的门槛,消费春联的传统蕴意。你对此怎么看?请表明自己的观点,结合材料展开论述。【答案】7.D 8.上联:天伦有乐乐驻心头歌晚景;下联:岁月含情情融孝道沐春晖理由:根据材料二可知,一声二声为平声,三声四声为仄声;上联末字必须为仄声,下联末字必须为平声(或:对联要求仄起平收)。“景”是第三声,相当于仄声;“晖”是第一声,相当于平声。 9.示例一:我认为没有必要担心。春联无论是高雅还是通俗,其本质都是反映人们对未来生活的期盼,寄寓美好的祈望和祝福。其次,春联不只是人们撰写新年愿望的工具,还可以是一个展现个性的平台。与其千篇一律地使用内容重复多年的春联,不妨给现代春联一些包容和空间。示例二:我认为有必要担心。春联的内容有许多讲究,不仅应符合作者自身的特点,更应体现出一种健康的审美趣味和追求。这种数千年流传下来的象征吉祥、具有传统蕴意、表达人们对美好生活的向往的民族风俗应该得到继承并发扬光大,而不是一味地“万物皆可写春联”。【解析】7.本题考查内容理解。D.不属于春联。根据“新厦落成增瑞气,华门安居进财源”中的“新厦落成”“华门安居”可知,本联表达的是新房子刚刚建成。乔迁新居之意,与春联无关。故选D。8.本题考查区分上下联。根据【材料二】“上下联平仄要相调。现在的一声二声大致相当于古代的平声,三声四声大致相当于古代的仄声。对联讲究平仄,末尾最严。上联末尾字字音必须为仄声,下联末尾字字音必须为平声”可知,区分上下联,关键是看尾字的平仄,上联尾字是仄声,也就是现在的一声二声,下联是平声,也就是现在的三声四声。据此查看尾字“景”是三声属于仄声,“晖”是一声属于平声,因此可以断定“天伦有乐乐驻心头歌晚景”是上联,“岁月含情清融孝道沐春晖”是下联。9.本题考查看法。从“会或不会拉低春联的门槛,消费春联的传统蕴意”中任选一种观点,结合材料,说出理由即可。示例:我认为不会拉低春联的门槛,消费春联的传统蕴意。根据【材料一】“春联的内容,不仅应符合作者自身的特点,更应体现出一种健康的审美趣味和追求”可知,春联的内容,只要符合春联特点,体现健康的审美趣味和追求即可。另据【材料三】“春节将至,年轻人把不加班、不脱发、不出BUG这些接地气的愿望写进春联,还融入了音符、代码、英文、化学元素或互联网中的流行文化,写成了别有一番风味的‘现代春联’,让年味有了浓浓新意”可知,“万物皆可写春联”的做法,别有一番风味的“现代春联”,让年味有了浓浓新意。因此我们应该提倡,而不必担心会拉低春联的门槛,消费春联的传统蕴意。四、阅读下面的材料,完成后面的小题。材料一①天下之本在国,国之本在家,家之本在身。自古以来,“家”对中华民族而言有着无与伦比的影响力,一篇篇流传至今的诫子文章和家训家诰,大多是将个人的自强不息、立志勤学等优秀品质与奉献牺牲、爱国主义等中华文化的要素紧密联系在一起。正是依靠这种家国同构、家国天下的共同意识和民族精神,我们的民族才能历经数千年磨难而不断创造出新的辉煌。孟母三迁、_____________、_____________……这些流传千古的家风建设故事不一而足,激励了一代代中国人努力成为贤才干将,为中华民族精神传承与发扬提供了最深厚最滋养的土壤。②家庭是社会的基础,家风汇聚而成国风。在此次抗击新冠肺炎疫情的过程中,同样涌现出很多以家风家教为载体的感人事迹。钟南山院士出身医学世家,他的父亲对他说过:“一个人,要在世界上留下点东西,那他在世界上就不算白活了。”诚实、和蔼、执着,这些钟家优良传统深深地影响了这位老者,在此次疫情中,他依然赤诚敬业、心念人民,始终冲在最前线。不仅如此,还有母亲曾在17年前抗击非典,女儿如今加入了援鄂医疗队;作为重症医学专家的父亲请缨奔赴武汉前线,不忘叮嘱儿子努力学习,继续从医救人……家与国交融互通,个人与时代紧密相连,他们都将良好家风凝化为民族精神。③今天,我们汇聚力量共抗疫情;明天,我们将继续毫不动摇向着民族复兴的伟大目标前进。(节选自《光明日报》,有改动)材料二①陶侃曾在浙江海阳做县吏,监管渔业,常有下属送东西给他。有一次,一位下属送了一坛鱼鲊(腌鱼)给陶侃,孝顺的陶侃马上念及一生贫居乡间的慈母,便嘱托乡人带给母亲。谁知母亲却原封不动地将这一坛鱼鲊退了回来,并在信中写道:“尔为吏,以官物遗我,非惟不能益吾,乃以增吾忧矣。”陶侃收到母亲退回的鱼鲊和回信,大为震动,更愧疚万分。他下定决心,一生遵循母亲教导:清白做人,廉洁为官。②岳飞十五六岁时,北方的金人南侵,宋朝当权者腐败无能,节节败退,国家处在生死存亡的关头。岳飞的母亲大义凛然,主动励子从戎,精忠报国。她为了让儿子永远铭记大丈夫当“精忠报国”的训诫,用绣花针把这四个字刺在岳飞的背上!岳飞投军后,奋勇杀敌、义无反顾,屡建战功。后来岳飞成为一代有名的抗金英雄,为历代人民所敬仰。(摘自“百度百科”)材料三①英国网友@Gareth Phillips:中国的行动力鼓舞人心。我想象不到世界上任何一个国家有这样的意志和能力,可以在这么短的时间内建成医院。②美国网友@Frankie Taylr:我敬佩那些冲上疫情防控一线、全力遏制疫情的大无畏的中国人,还有迅速启动全国疫情防控体系并建设专门收治医院的中国速度。③印度网友@Naveed Dar:勤奋的中国人民团结互助,全社会上下一心、互相配合,强有力的政府采取一系列有效措施保护民众和救治患者。④南非网友@Dean Olley:中国人民在抗击疫情中展现出的勇气令人敬佩!面对突发疫情,中国政府反应迅速、行动果断。⑤阿曼网友@Shaum Rein:长期来看,中国的恢复能力很强。中国人民总是非常团结,他们在疫情期间,仍旧能保持乐观,彼此互帮互助。中国人民的奉献精神和勇往直前感染着我。(节选自《人民日报》)材料四疫情期间,某课题组对部分90后、00后青年就有关问题展开调查。下面是部分调查统计结果。图一 未参与到一线防控工作的青年,采用其他方式参与战疫的占比。图二 疫情对当代青年价值观的影响调查结果(社会责任感方面)(数据来源于新华网)10.从以上材料看,下列理解和判断,不正确的一项是( )A.流传至令的诫子文章和家训家诰,都将个人的优秀品质与中华交化的要素紧密联系在一起。B.岳飞在母亲的鼓励下投身军旅,并屡建战功,成为有名的抗金英雄。C.中国在抗击新冠疫情中表现出的行动力鼓舞人心,感动了许多海外网友。D.近一半的受访者以志愿服务的方式参与了战疫。11.根据材料二的内容,仿照“孟母三迁”的形式,在材料一的横线上分别补上四字短语作为论据。12.材料一的第二段主要运用了什么论证方法?有什么作用?13.中国抗击新冠疫情取得重大战略成果的原因有哪些?请结合以上材料加以概括。【答案】10.A 11.陶母教子(侃母退鲊);岳母刺字(岳母励子) 12.举例论证。举钟南山受家风影响冲在抗疫最前线等事例,具体有力地论证了家风汇聚成国风(良好家风能凝化为民族精神)的观点。 13.中国民族汇聚一心;中国家风家训的熏染;英雄人物的贡献;青年人的奉献。【解析】10.考查信息的筛选辨析。A.根据材料一①段中“一篇篇流传至今的诫子文章和家训家诰,大多是将个人的自强不息、立志勤学等优秀品质与奉献牺牲、爱国主义等中华文化的要素紧密联系在一起”中的“大多”一词,可知本项“都”语义表述过于绝对;故选A。11.考查根据材料补写论据。题干要求根据材料二的内容,仿照“孟母三迁”的形式补写论据。材料二第①段介绍的是陶侃的母亲教育陶侃为官之道,可概括为“陶母教子”;材料二第②段介绍的是岳飞的母亲在岳飞的背上刺字,教育他要心系国家,精忠报国,可概括为“岳母刺字”。12.考查对论证方法的理解及作用分析。论证方法主要有举例论证、道理论证、对比论证、比喻论证、引用论证。解答时,首先根据句子内容判断论证方法,再结合语境分析其作用。结合“钟南山院士出身医学世家,他的父亲对他说过:‘一个人,要在世界上留下点东西,那他在世界上就不算白活了。’诚实、和蔼、执着,这些钟家优良传统深深地影响了这位老者,在此次疫情中,他依然赤诚敬业、心念人民,始终冲在最前线。不仅如此,还有母亲曾在17年前抗击非典,女儿如今加入了援鄂医疗队;作为重症医学专家的父亲请缨奔赴武汉前线,不忘叮嘱儿子努力学习,继续从医救人……”的内容可知,本段列举了钟南山院士在钟家优良家风的影响下,无私奉献,以耄耋之年冲锋在抗疫的第一线,这是举例论证的论证方法。这一真实事例,可以证明家风家训对人性格形成的重要影响。举例论证的应用,使文章的论证更有说服力。13.考查材料的提炼概括。根据材料三③段中的“勤奋的中国人民团结互助,全社会上下一心、互相配合,强有力的政府采取一系列有效措施保护民众和救治患者”可得:中国民族汇聚一心;根据材料一①段“一篇篇流传至今的诫子文章和家训家诰,大多是将个人的自强不息、立志勤学等优秀品质与奉献牺牲、爱国主义等中华文化的要素紧密联系在一起。正是依靠这种家国同构、家国天下的共同意识和民族精神,我们的民族才能历经数千年磨难而不断创造出新的辉煌”中对于家风家训的阐述可得:中国家风家训的熏染;根据材料一②段“钟南山院士出身医学世家,他的父亲对他说过:‘一个人,要在世界上留下点东西,那他在世界上就不算白活了。’诚实、和蔼、执着,这些钟家优良传统深深地影响了这位老者,在此次疫情中,他依然赤诚敬业、心念人民,始终冲在最前线。不仅如此,还有母亲曾在17年前抗击非典,女儿如今加入了援鄂医疗队;作为重症医学专家的父亲请缨奔赴武汉前线,不忘叮嘱儿子努力学习,继续从医救人……”中对于钟南山院士在抗疫过程中做出突出贡献可得:英雄人物的贡献;根据材料四的表格可知,当代80%以上的青年在受到战疫英雄事迹激励表示能扛起社会责任,并且愿意在战疫斗争有需要的时候奉献自己的力量。据此可得:青年人的奉献。五、阅读下面的材料,完成后面的小题。(材料一)生物质是指一切通过绿色植物的光合作用所形成的有机物质,包括微生物、植物和动物,及其排泄物、垃圾及有机废水等源自生物体的有机物质。废弃生物质是人类在利用生物质的过程中生产和消费产生的废弃物,它仍然属于生物质的宏观范畴,但是能量密度、可利用性等都有显著的降低。根据不同的来源可以将废弃生物质分为三类:城市生物质废物(主要包括家庭厨余垃圾、餐厨垃圾、城市粪便以及城镇污泥);农作物废物(主要以玉米秸、麦秸和稻秸为主);禽畜粪便。废弃生物质作为固体废物的一种,是人们必须妥善处理的环境污染物,若处理处置不当,将会导致严重的环境污染。废弃生物质在造成环境污染的同时,也是重要的可再生资源和能源。合理高效地将废弃生物质资源化不仅可以充分利用生物质能这一可再生清洁能源,而且对于 CO2 减排也具有重要意义。(节选自百度百科)(材料二)图表:中国2007年生物质原料资源农林废弃物的状况与2030年增量2007年实际产能 2030年预测产能增量(图表来自江瀚咨询)(材料三)废弃生物质变废为宝一直是个国际难题。近日,中国科技大学江鸿教授课题组与俞汉青教授课题组合作,分别成功制备了高热值且稳定的固相生物煤和高性能的石墨烯、碳纳米管等材料,为实现废弃生物质热解技术商业化应用提供了重要的技术支撑。热解是废弃生物质资源化利用的重要技术之一。通过缺氧条件下的生物质热解,可以得到可再生的生物油、生物炭和一部分热解气。国内外学者一直致力于研究生物油的催化提质和分离,期望获得高附加值的化学品或优质燃料。然而,生物油的成分复杂且不稳定,通常包含数百种有机化合物。在催化过程中,部分有机物发生缩合、脱水、结焦等反应,会导致催化剂失效,使催化提质过程难以持续。研究人员发现,通过常压蒸馏过程参数控制,实现生物油快速结焦可以得到一种新的固体燃料,研究人员将其命名为生物煤。分析显示,不同生物质原料得到的生物煤热值与商用煤热值相当。此外,生物煤还具有性能稳定、低含硫量、不含重金属等环境友好特性。另外,热解过程产生的高温气体中包含小分子碳有机物,且热解气温度较高,是制备碳纳米材料的潜在前体。研究人员通过优化热解条件,利用化学蒸汽沉积方法制备3D石墨烯,还通过改变热解沉积条件,得到了碳纳米线。这些高附加值碳材料在污染物去除和储能方面展示了良好性能,利用生物质热解气合成石墨烯具有更小的环境影响和能量消耗。(节选自《科技日报》)14.下列理解和判断不正确的一项是( )A.废弃生物质虽然会造成环境污染,但作为重要的可再生资源和能源,我们应该合理高效地利用它。B.有了中国科技大学提供的技术支撑,废弃生物质热解技术就将投入商业化应用。C.国内外学者期望获得高附加值的化学品或优质燃料,但催化提质过程难以持续。D.像碳纳米线这样的高附加值碳材料利用生物质热解气合成石墨烯具有更小的环境影响。15.材料一主要运用了哪两种说明方法,请说明其作用。16.请用简洁的语言概括材料二图表的内容。17.废弃生物质资源化利用是大势所趋,作为中学生的你,在日常生活中有没有变废为宝的小妙招呢?请分享两条。【答案】14.B 15.材料一主要运用了下定义和分类别两种说明方法。对于“生物质”和“废弃生物质”这两个概念,材料一中运用了下定义的说明方法,科学、准确、简洁的解释,让读者易于理解;根据来源的不同对废弃生物质进行了分类,条理清楚,一目了然。 16.中国2007年生物质原料资源农林废弃物的实际产能和2030年预测产能增量均呈上升趋势。 17.①用茶叶渣消除臭味,用过期的牛奶擦皮鞋、地板,用蔬果渣清除油污,用吃剩的面包清楚墙壁污垢、除臭,秸秆还田做肥料。②废旧毛笔巧除家电缝隙的灰尘:用布不宜擦净,可将废旧的毛笔用来清除缝隙里的灰尘,非常方便。③用废药瓶上的橡皮盖子制作实用搓衣板:将废药瓶上的橡皮盖子搜集起,按纵横交错位置,将橡皮盖子一排排钉在一块长方形木板上(钉子要钉在盖子凹陷处)。【解析】14.本题考查信息的筛选与辨析。B.“就将投入商业化应用”说法太绝对,与原文材料三“近日,中国科技大学江鸿教授课题组与俞汉青教授课题组合作,分别成功制备了高热值且稳定的固相生物煤和高性能的石墨烯、碳纳米管等材料,为实现废弃生物质热解技术商业化应用提供了重要的技术支撑”不符;故选B。15.本题考查说明方法及其作用的分析。常见的说明方法有:举例子、分类别、下定义、摹状貌、作诠释、打比方、列数字、列图表、作引用等。解答时首先要了解说明文的常用说明方法及特征,然后根据文段内容从说明了事物什么特征,说明什么问题方面来表达其作用。料一第一段“生物质是指一切通过绿色植物的光合作用所形成的有机物质,包括微生物、植物和动物,及其排泄物、垃圾及有机废水等源自生物体的有机物质”“废弃生物质是人类在利用生物质的过程中生产和消费产生的废弃物,它仍然属于生物质的宏观范畴,但是能量密度、可利用性等都有显著的降低。”可知运用的是下定义的说明方法,用科学、简洁、明确的语言对“生物质”“废弃生物质”的本质特征作概括的说明,使读者对这两个事物有个明确的概念;由材料一第二段“根据不同的来源可以将废弃生物质分为三类:……”可知是运用的是分类别的说明方法,按照来源的不同将废弃生物质分为三类,并一类一类地加以说明,将复杂的事物条理清楚地进行说明。16.本题考查图表分析。这类题一般问从该图中得出什么观点或结论。答题要认真审题,明确要求。要注意表头和表脚的文字,弄清说明对象和比较角度。答题格式为思考比较数字后得出的结果+调查的目的或调查得出的结果所蕴含的意义。答案用语不能离开调查的对象、目的,可优先选用题干(表题)中的词语。此题表头是“中国2007年生物质原料资源农林废弃物的状况与2030年增量”,抓住2007年实际产能与2030年预测产能增量图形变化可以看出,2007年实际产能和2030年预测产能增量明显呈上升趋势。17.本题考查拓展延伸。开放性试题,答案不唯一。示例:泡茶后剩下的茶叶的多用途方法:浸泡水中数天后,浇在植物根部,促进植物生长;把残茶叶晒干,放到厕所或沟渠里燃熏,可消除恶臭,具有驱除蚊蝇的功能;还可以铺撒在潮湿处,能够去潮;还可装入枕套充当枕芯,非常柔软;用残茶叶擦洗木、竹桌椅,可使之更为光洁。六、阅读下列非连续性文本,完成下面小题。【材料一】2021年4月16日发布的《2020年度中国数字阅读报告》(以下简称《报告》),从多个角度呈现了后疫情时代中国数字阅读产业的现状、特点以及趋势。《报告》指出,2020年中国数字阅读产业规模达351.6亿元,增长率达21.8%;数字阅读用户规模达到4.94亿,增长率5.56%;人均电子书阅读量9.1本,人均有声书阅读量6.3本。与此同时,人均纸质书阅读量6.2本,同比去年减少2.6本。2020年用户平均单次电子阅读时长为79.3分钟,有声阅读时长为62.8分钟,17﹣22点是阅读的“晚高峰”。值得一提的是,儿童也成为阅读的重要用户,儿童数字阅读付费用户规模增速56.5%,日均数字阅读时长29分钟,每晚20﹣22点是儿童数字阅读的高峰。(摘自人民网)【材料二】2021年4月23日,中国新闻出版研究院发布第十八次全国国民阅读调查成果。与数字化阅读相关的图表如下:(摘自“传媒”微信公众号)【材料三】自二十一世纪以来,人们借助电脑、电子阅读器、手机等电子终端工具,以及各种数据库、在线教育、电子书包等新兴数字出版平台浏览信息和获取知识,已经成为信息社会生活的一种常态。于是,一些人士对于“碎片化阅读”忧虑之甚,认为读者在相对较短的空闲时间内进行的所谓“阅读”,以及浏览微博、短信等零碎的、片段化的内容,“不易形成系统的、深度的知识体系”。但我们也应该看到,这一趋势是数字化时代人们阅读方式变迁的必然结果。其实,碎片化阅读的语境,是相对于长达千余年的纸本阅读方式而言的。当前,有些人对文字内容缺乏整体性阅读及系统性理解,阅读注意力不够集中,乃至难以进入深度思考、有所创新的佳境。针对上述现象,笔者曾提出“左书右网,前语后文”及“开卷读书,启屏求知”等阅读方法论建议:“网络的普及,正在逐渐改变着人们的阅读习惯……电子书的出现,拓宽了书的范围,丰富了书的形式,让每一个热爱文字和书籍的人,更多了一种选择。阅读的本质,在于阅读内容,不管是纸质阅读还是数字阅读,都是对内容的阅读。”全民阅读推广名师之一、知名出版家、作家聂震宁先生也提出,“在繁忙的生活中,人们不妨利用碎片时间在手机上、电子阅读器上,读一些自己喜欢读的东西,同时,一定要挤出空闲时间读些纸书”。他认为,一个成熟的“阅读社会”,应该“善待一切阅读方式,坚守人类阅读认知规律,推动传统阅读与新兴阅读(方式)的融合”,让人们愿意将“读文读图”“看屏看书”视为现代化的“快乐生活”方式之一,以提升其精神生活质量。(摘自人民论坛网)18.根据以上材料,下列理解和判断不正确的一项是( )A.2020年,我国数字阅读产业规模和用户规模相较前一年增幅不大。B.近两年来,我国数字阅读的主要方式是手机阅读和网络在线阅读。C.我国数字化阅读人群分布显现出人群比例随年龄增长递减的趋势。D.纸质阅读、数字阅读,都是对内容的阅读,它们的本质是一样的。19.材料一画线句主要运用了________的说明方法,说明了___________________。20.请根据材料一和材料三,概括“数字阅读”的主要特点。【答案】18.A 19.列数字 2020年我国儿童成为数字阅读的重要用户。 20.阅读内容的数字化;阅读主体的年轻化;阅读媒介的数字化;阅读方式的碎片化。(意思对即可)【解析】18.本题考查材料内容辨析。A.“增幅不大”有误,根据材料一第二段“2020年中国数字阅读产业规模达351.6亿元,增长率达21.8%;数字阅读用户规模达到4.94亿,增长率5.56%”可知,2020年,我国数字阅读产业规模和用户规模相较前一年增幅大;故选A。19.本题考查说明方法及作用。材料一画线句中的“56.5%”“29分钟”“每晚20-22点”三组数据,可知此句运用了列数字的说明方法,由画线句“值得一提的是,儿童也成为阅读的重要用户”可知,具体准确地说明了2020年我国儿童成为数字阅读的重要用户的情况,使说明内容更具说服力。20.本题考查材料内容概括。结合材料一第三段“2020年用户平均单次电子阅读时长为79.3分钟,有声阅读时长为62.8分钟”可知,阅读内容数字化;结合材料三第一段“人们借助电脑、电子阅读器、手机等电子终端工具,以及各种数据库、在线教育、电子书包等新兴数字出版平台浏览信息和获取知识”可知,阅读媒介数字化;由材 料三第一段“读者在相对较短的空闲时间内进行的所谓‘阅读’,以及浏览微博、短信等零碎的、片段化的内容”可知,阅读方式碎片化;由材料一第三段“儿童也成为阅读的重要用户,儿童数字阅读付费用户规模增速56.5%,日均数字阅读时长29分钟”可知,阅读主体年轻化。据此概括作答即可。七、阅读下面的文字,完成下面小题。【材料一】2021年11月4日,第四届中国国际进口博览会开幕式在上海举行,国家主席习近平通过视频发表主旨演讲,举办中国国际进口博览会是中国坚定支持贸易自由化和经济全球化,主动向世界开放市场的重大举措,有利于促进世界各强经贸交流合作、促进全球贸易和世推动开型世界经济发展。【材料二】第四届中国国际进口博览会的吉祥物主题形象为中国的“国宝”大熊猫,吉祥物取名为“进宝”,既有进口博览会之宝”的涵义,也是“进博”的谐音,还暗含着“招财进宝”吉祥寓意。【材料三】人类发展当前面临重大挑战。新冠疫情反复延宕,地政治局势紧张,全球治理严重缺失,如何破解发展难题?中国倡导推动更加包容、更加普惠、更有韧性的全球发展,与国际社会当前和长远的需求高度、深度契合,受到国际社会广泛认同和赞誉。中国积极打造高水平对外开放体系。中国已成为140多个国家和地区的主要贸易伙伴;迹续举办5届中国国际进口博览会,让各国企业更好地在中国市场寻求发展机遇,也让中国本土企业向世界展现价值;建设高标准市场体系、深化要素市场化改革、持续放宽外资准入、不断改善管商环境……中国以扎实行动推动建设开放型世界经济。德国柏林普鲁士协会名誉主席福尔克尔·恰普克表示,各国利益相互交织,和平相处、合作共赢是应对共同挑战唯一可行的方式,这也正是中国一直致力于实现的目标。中国提出的全球发展倡议为全球实现更加普惠的发展擘画蓝图。一年多来全球发展倡议之友小组”成员已逾60国。中国不仅是全球发展倡议的提出者,也是落实这一倡议的行动派:中欧班列铺画82条运输线路,通达欧洲24个国家200个城市;“一带一路”倡议提出以来,已有149个国家、32个国际组织同中国签署200多份合作文件……21.【理解推断】根据上述材料,下列说法正确的一项是( )A.2021年11月4日,第四届中国国际进口博览会在上海举办。B.进口博览会的吉祥物主体形象为中国的“冰墩墩”,取名为“进宝”,既有“进口博览会之宝”的涵义,也是“进博”的谐音。C.中国已成为140多个国家和地区的主要贸易伙伴。D.新冠疫情反复延宕,地缘政治局势紧张,全球治理严重缺失。中国已经破解人类发展当前面临的重大挑战。22.【迁移运用】结合材料说一说中国连续举办5届中国国际进口博览会的意义。【答案】21.C 22.①人类发展当前面临重大挑战。中国倡导推动更加包容、更加普惠、更有韧性的全球发展。②各国利益相互交织,和平相处、合作共贏是应对共同挑战唯一可行的方式,这也正是中国一直致力于实现的目标。③中国积极打造高水平对外开放体系,连续举办5届中国国际进口博览会,让各国企业更好地在中国市场寻求发展机遇,也让中国本土企业向世界展现价值。④举办中国国际进口博览会有利于促进世界各国加强经贸交流合作,促进全球贸易和世界经济增长,推动开放型世界经济发展。【解析】21.考查辨析信息。A.根据材料一中的“2021年11月4日,第四届中国国际进口博览会开幕式在上海举行”可知,本项有误;B.根据材料二中的“第四届中国国际进口博览会的吉祥物主题形象为中国的‘国宝’大熊猫”可知,本项有误;D.根据材料三中的“人类发展当前面临重大挑战。新冠疫情反复延宕,地政治局势紧张,全球治理严重缺失,如何破解发展难题?中国倡导推动更加包容、更加普惠、更有韧性的全球发展,与国际社会当前和长远的需求高度、深度契合,受到国际社会广泛认同和赞誉”可知,本项表述过于绝对;故选C。22.考查筛选信息。根据材料三首段中的“人类发展当前面临重大挑战。新冠疫情反复延宕,地政治局势紧张,全球治理严重缺失,如何破解发展难题?中国倡导推动更加包容、更加普惠、更有韧性的全球发展”可得:人类发展当前面临重大挑战。中国倡导推动更加包容、更加普惠、更有韧性的全球发展。根据材料三第二段中的“各国利益相互交织,和平相处、合作共赢是应对共同挑战唯一可行的方式,这也正是中国一直致力于实现的目标”可得:各国利益相互交织,和平相处、合作共贏是应对共同挑战唯一可行的方式,这也正是中国一直致力于实现的目标。根据材料三第二段中的“中国积极打造高水平对外开放体系。中国已成为140多个国家和地区的主要贸易伙伴;迹续举办5届中国国际进口博览会,让各国企业更好地在中国市场寻求发展机遇,也让中国本土企业向世界展现价值”可得:中国积极打造高水平对外开放体系,连续举办5届中国国际进口博览会,让各国企业更好地在中国市场寻求发展机遇,也让中国本土企业向世界展现价值。根据材料一中的“举办中国国际进口博览会是中国坚定支持贸易自由化和经济全球化,主动向世界开放市场的重大举措,有利于促进世界各强经贸交流合作、促进全球贸易和世推动开型世界经济发展”可得:举办中国国际进口博览会有利于促进世界各国加强经贸交流合作,促进全球贸易和世界经济增长,推动开放型世界经济发展。八、阅读下面非连续性文本材料,完成下面小题。材料一:新时代劳动教育的内涵较之以往有了很大的发展。首先,体现在价值观方面,人不同于其他生物的根本特征在于人需要存在的理由,即价值感和意义感,而劳动正是使人获得价值感和意义感的一种独有的、自觉的对象性活动。新时代的劳动教育要引导学生懂得劳动最光荣、劳动最崇高、劳动最伟大、劳动最美丽的道理,形成劳动创造美好生活、劳动不分贵贱的积极劳动价值观。其次,体现在内容观方面。随着新型劳动的不断涌现,劳动教育内容必须主动扩容以涵盖变化着的劳动现实,针对劳动新形态,深化产教融合,改进劳动教育方式,树立发展的内容观。再其次,体现在消费观方面。人们从生产走向消费,要求劳动教育关注基本的生产劳动外,更应关注作为消费的劳动。当前青少年普遍存在攀比与过度消费的现象,侧面反映出消费教育在劳动教育中的缺失。最后,体现在闲暇观方面。斯宾塞认为,在生产方式达到圆满、劳动力得到最大化节约时,闲暇时间会大量增加且占据重要地位。随着人们劳动时间的减少和闲暇时间的增多,身心合一的、主动的、创造性的劳动是使人从“沉重的肉身”和“横流的欲望”的奴役状态中解放出来的现实载体,劳动教育必须涵盖正确的闲暇观。材料二:漫画《“体贴”的爸爸》材料三:劳动教育与体育虽具有不同的表现形式和目的追求,但本质上都是增强人的体质的收要内容。劳动教育可以从身心两方而促进学生的发展,其中最基本的就是促进学生的生理健康,起到强身健体的作用。在参加劳动教自的过程中,可以让:肌肉骨骼得到锻炼发展,增大肺活量,促进心脏的发展,通过劳动也可以调节脑力活动,促进学生脑部神经系统的发展,有利于消除学习带来的压力和负担,对学生的心理健康有重要的促进作用。2018年教育部“全国学生体质与健康调研”数据显示,全国6岁儿童近视率达14.5%、小学生达36%、班中生达71.6%、高中生达81%此外,还有肥胖率增高,征兵体检不合格率居高丕下等情况。其根本原因就是缺乏锻炼,忽视劳动教育。所以说劳动教育对市少年的体质与健康有着重要意义。材料四:劳动教育缺位成为我国当前教育的严峻现实,中小学生劳动教育现状不容乐观,出现了一些学生不会劳动、轻视劳动,不珍惜劳动成果的现象。从学校来讲,劳动与技术课程经常被占用,师资、场地、经费缺乏,劳动教育无计划、无考核:有的把劳动当惩罚手段,劳动多教育少,忽视劳动观念和劳动习惯培养。从家庭来讲,体力劳动和生产劳动在家庭教育中被忽视,家长往往只关心孩子的学业成绩,只要学习好,什么都不用干。从社会来讲,一夜暴富、不劳而获的思想有所蔓延,体力劳动和生产;劳动被淡化;好逸恶劳、看不起劳动、轻视劳动人民的现象客观存在。这与党的教育方针和教育的根本任务不相符合,影响了青少年学生的健康成长。23.材料一从哪些方面阐释新时代劳动教育内涵?24.结合材料,谈谈我国的劳动教育缺位的原因。25.材料三中的划线句子运用了哪些说明方法?有什么作用?26.请你合理使用材料内容,劝导漫画中的“体贴”爸爸。【答案】23.价值观、内容观、消费观、闲暇观 24.①家长因只关注孩子成绩、溺爱等原因对孩子劳动教育的忽视。②学校忽视劳动观念和劳动习惯的培养。③社会上存在好逸恶劳、看不起劳动、轻视劳动人民等现象。 25.举例子,列数字,具体有力准确地说明了劳动教育对青少年的体质与健康有着重要意义。 26.示例:爸爸,谢谢您对我的关心。但是我们适当地参加一些劳动可以利于消除学习带来的压力和负担,对我们的心理健康有重要的促进作用,我们的学习也需要劳逸结合呀,您说呢?结合材料内容,语气委婉(有称呼或使用礼貌用语),有理有据即可。【解析】23.本题考查材料信息筛选。根据材料一“首先,体现在价值观方面,人不同于其他生物的根本特征在于人需要存在的理由,即价值感和意义感,而劳动正是使人获得价值感和意义感的一种独有的、自觉的对象性活动”可概括为:价值观;根据“其次,体现在内容观方面。随着新型劳动的不断涌现,劳动教育内容必须主动扩容以涵盖变化着的劳动现实,针对劳动新形态,深化产教融合,改进劳动教育方式,树立发展的内容观”可概括为:内容观;根据“再其次,体现在消费观方面。人们从生产走向消费,要求劳动教育关注基本的生产劳动外,更应关注作为消费的劳动”可概括为:消费观;根据“最后,体现在闲暇观方面。斯宾塞认为,在生产方式达到圆满、劳动力得到最大化节约时,闲暇时间会大量增加且占据重要地位”可概括为:闲暇观。24.本题考查材料内容理解。根据材料四“劳动教育缺位成为我国当前教育的严峻现实,中小学生劳动教育现状不容乐观,出现了一些学生不会劳动、轻视劳动,不珍惜劳动成果的现象。从学校来讲,劳动与技术课程经常被占用,师资、场地、经费缺乏,劳动教育无计划、无考核;有的把劳动当惩罚手段,劳动多教育少,忽视劳动观念和劳动习惯培养”可概括为:学校对劳动教育的忽视,忽视培养劳动观念和劳动习惯;根据材料四“从家庭来讲,体力劳动和生产劳动在家庭教育中被忽视,家长往往只关心孩子的学业成绩,只要学习好,什么都不用干”可概括为:家长因只关注孩子成绩、溺爱等原因对孩子劳动教育的忽视;由材料四“从社会来讲,一夜暴富、不劳而获的思想有所蔓延,体力劳动和生产劳动被淡化;好逸恶劳、看不起劳动、轻视劳动人民的现象客观存在”可概括为:社会上轻视劳动、不劳而获、好逸恶劳等不良风气的影响。25.本题考查说明方法的辨析与作用。画线句中列举了“14.5%”“36%”“71.6%”“81%”等一系列数据,属于列数字的说明方法,同时,这些数据也是全国中小学生的现实健康状况,所以也是举例子的说明方法,结合尾句“所以说劳动教育对市少年的体质与健康有着重要意义”可知,运用这两种说明方法,准确科学地说明了劳动教育对请少年的健康和体制有着重要意义。26.本题考查语言表达。由材料三“劳动教育可以从身心两方面促进学生的发展,其中最基本的就是促进学生的生理健康,起到强身健体的作用”、“在参加劳动教育的过程中,可以让肌肉骨骼得到锻炼发展,增大肺活量,促进心脏的发展,通过劳动也可以调节脑力活动,促进学生脑部神经系统的发展,有利于消除繁重学习带来的压力和负担,对学生的心理健康有重要的促进作用”可知,劳动能从身心两方面促进青少年的发展,还可以调节脑力活动,对学生的心理健康有重要的促进作用。据此进行劝说。示例:爸爸,像您说的劳动确实可能会让我们感到劳累,但它能从身心两方面促进我们的发展,在参加劳动教育的过程中,可以让肌肉骨骼得到锻炼发展,增大肺活量,促进心脏的发展,也有利于消除繁重的学习压力,劳逸结合对我们的心理健康有重要的促进作用,您觉得呢?九、阅读下面两个文本,完成下面小题。【文本一】①我的朋友圈平均两天出现一个的“YYDS(永远的神)”,最近有被更新的流行语“破防了”赶超的趋势。如实说,朋友圈那些惯常用流行语的人并无恶意,使用流行语的目的,也主要是努力显得时兴,铆着劲儿让自己显得有趣。可问题就在于,复读机式的有趣并不是真的有趣,甚至相当乏味。当一个热词反复出现在聊天记录、综艺字幕、电梯广告、视频弹幕里时,它的诙谐和巧妙之处就被一次次稀释了。更严重的是,这种毫无技术含量的表达和缺乏趣味的文字,正在逐渐消磨我们的语言能力。②根据《中国青年报》社会调查中心对2002名受访者的调查,76.5%的人感觉自己的语言越来越贫乏。在豆瓣成立的“文字失语者互助联盟”里,有22万参与者求教怎么形容冰糖葫芦好吃、有什么能代替“颜文字”。③“失语症患者”离文字越来越远,症状是热衷于使用各种热词流行语。还容易大惊小怪,抛弃句号,因为句号意味着冷漠、不满和谴责的语气,只有感叹号才显得热情。他们也放弃遣词造句,需要表达时就打开图库贴一张“说大事专用图”“不配图我总觉得不专业”“我是来凑图滴”:写文章要三行插入两个表情包,文字不再用来讲逻辑和故事,只呈现情绪和状态。渐渐地,我们连100字的点评也凑不满了。阅读和书写长文更显吃力,能够领悟别人所表达的意思,并且精确表达自己的感受,成为一种逐渐稀缺的能力。④摧毁我们的语言能力的并不是流行语和各种感叹号、图库和表情包,而是我们不再思考,在看似有趣的流行语外衣下,包裹着干瘪的灵魂。哲学家维特根斯坦在《逻辑哲学论》一书中写道:“我的语言的界限意味着我的世界的界限。”复制流行语是容易的,精确地形容思想、感受、味觉、气味,用别人未曾用过的表达,去触碰那些存在却未被注意和书写的地带,才是有价值的,也是永恒的。⑤语言是活的,流行语们自有归处,我们宁可发明些辞趣,也别让自个儿的大脑成了别人思想的跑马场。(作者:杨杰。有删改)【文本二】①网络流行语,到底是不是一种语言上的“病症”?这个话题似乎已争论多年。如果说它们都是语言的“毒瘤”,显然有一棍子打死之嫌,因为毕竟有一些网络流行语已在口耳相传中“扶正”,得以登上大雅之堂,成为语言丰富性的有益补充,如“给力”“蛮拼的”等等。但无论如何,过度使用网络流行语对于我们的一大影响是,终究患上了“语言贫乏症”。②有人曾对古人的表达和我们的网络流行语做了一番对比:古人形容人漂亮可以用“貌比潘安”“玉树临风”“顾盼神飞”,我们只会说“高富帅”“白富美”;古人形容人难看可以用“东施效颦”“獐头鼠目”,我们只会说“矮穷矬”“颜值低”:古人表达悲伤用“我心伤悲,莫知我哀”,我们只会用“蓝瘦香菇”……也许这种对比有些极端,但如果有一天你张口闭口只会习惯性地说“高富帅”“矮穷矬”,会不会也被自己表达的匮乏惊呆了?③网络世界无穷尽,现代传媒的传播作用和影响不可小觑。一些网络热词之所以迅速被传播,其实很大程度上取决于网民的搬运作用。一些热衷于炒作网络热词的推手也正是利用这一点,让一些粗鄙的“网络流行语”蔓延网络。④或许我们应该相信语言的“自净能力”,不用过于担心害怕,因为从古至今语言一直在融合、创新、淘汰,最终沉淀下来的才是精华。但毕竟语言也是一种文化,对于个人来说,选择使用什么样的语言会潜移默化地影响我们的文化内涵乃至思想深度,而聪明的人,总会在泥沙俱下的“潮流”中挑选出自己想要的东西,而不是放任自己陷入“流行”的漩涡。(作者:长余。有删改)27.下列对文本一和文本二的理解和分析,不正确的两项是( )A.文本一“它的诙谐和巧妙之处就被一次次稀释了”一句中的“稀释”是一种比喻的说法,意思是“诙谐和巧妙之处”变淡了。B.文本一中“失语症患者”离文字越来越远,其症状就是大量使用流行语。C.文本一原题“有话好好说”,实际上是作者对“失语症患者”的提醒或要求。D.文本二第②段用古人的表达和网络流行语的表达做对比,证明“过度使用网络流行语会使我们语言贫乏”的观点。E.两个文本的作者都认同对“流行语”应采取顺其自然的态度。28.请简要分析文本一的论证思路。29.两个文本都谈到“流行语”,其作用有何异同?请结合两个文本简要分析。【答案】27.BE 28.首先(第①段)由生活中的流行语现象引出“毫无技术含量的表达和缺乏趣味的文字,正在逐渐消磨我们的语言能力”的观点;然后(第②③两段)用《中国青年报》的调查数据和“失语症患者”的症状及危害来论证上述观点;接着(第④段)引用维特根斯坦的话并简析,指出使用流行语导致的“不再思考”才是“摧毁我们的语言能力”的深层原因;最后(第⑤段)呼吁我们“宁可发明些辞趣,也别让自个儿的大脑成了别人思想的跑马场”。 29.相同点:在两个文本中,“流行语”都是议论的话题:两文对“流行语”的观点也基本上都是否定的。不同点:文本一开头列举的“流行语”现象,还起到了引出观点、使观点有了现实意义的作用:“流行语”也只是要批判的“不好好说话”的种种导致“失语症”的原因之一。【解析】27.考查内容理解辨析。B.有误,结合【文本一】第③“‘失语症患者’离文字越来越远,症状是热衷于使用各种热词流行语”可知,选项表述有误;E.有误,两个文本的作者都认同对“流行语”应采取全面辩证的态度,并非“顺其自然”;故选BE。28.考查论证思路。结合第①段“我的朋友圈平均两天出现一个的‘YYDS(永远的神)’,最近有被更新的流行语‘破防了’赶超的趋势”可知,通过列举日常生活中常见的网络用语引出“这种毫无技术含量的表达和缺乏趣味的文字,正在逐渐消磨我们的语言能力”这一观点;结合选文第②段“根据《中国青年报》社会调查中心对2002名受访者的调查,76.5%的人感觉自己的语言越来越贫乏”、第③段“症状是热衷于使用各种热词流行语。还容易大惊小怪,抛弃句号,因为句号意味着冷漠、不满和谴责的语气,只有感叹号才显得热情”等内容可知,用《中国青年报》的调查数据和“失语症患者”的症状及危害来论证上述观点;由第④段“哲学家维特根斯坦在《逻辑哲学论》一书中写道:‘我的语言的界限意味着我的世界的界限。’”等内容可知,引用维特根斯坦的话并简析,指出使用流行语导致的“不再思考”才是“摧毁我们的语言能力”的深层原因;最后结合选文第⑤段“语言是活的,流行语们自有归处,我们宁可发明些辞趣,也别让自个儿的大脑成了别人思想的跑马场”可知,作者总结归纳,进一步论证观点。29.考查内容概括。相同点:“流行语”是两篇文章共同的论题,两文对“流行语”的态度基本都持否定观点。不同点:文本一主要论述了“流行语”是导致“失语症”的重要原因,并由此提出“摧毁我们的语言能力的并不是流行语和各种感叹号、图库和表情包,而是我们不再思考”这一观点,强调了思考的重要性;文本二主要论述“流行语”对我们日常生活的影响和冲击,意在论述我们要选择性接受“流行语”,辩证对待“流行语”这一观点。十、阅读下面的材料,完成后面的小题。材料一:芯片是半导体元件产品的统称,是集成电路的载体,由晶圆分割而成。常常是计算机或其他电子设备的一部分。简单来说,芯片就是把我们随处可见的电阻电容等电子器件以及由它们所组成的电路,集成封装到一个很小的颗粒里。芯片是非常精密的仪器,其单位为纳米。芯片的种类有很多,一系列相互关联的芯片又可以组合成芯片组,它们相互依赖组合在一起能发挥更多的作用。比如智能手机里的信号发射和接收模块,就是由多个芯片组合在一起的更大的集成电路。(摘编自“人民网”)材料二:芯片的集成度越来越高,制造工艺越来越复杂。现在的高端芯片所含的晶体管数目动辄就有100亿个以上,比如Intel旗舰Arc处理器拥有217亿个晶体管。如此大规模的晶体要在小小的芯片上生产出来,难度可想而知。晶体管的数目越多意味着每个晶体管的尺寸就要小,现在最高端的工艺已经可以生产3nm大小的芯片了。想象一下,人的头发的直径是40~50微米,也就是40000nm,那么3nm就比头发直径的万分之一还要小。为了制造这么小的晶体管,需要用到最核心的设备高端光刻机,光刻机被认为是芯片制造业的皇冠,设计与制造光刻机本身的难度就很高,需要借助全球科技的力量。现在最高端的EUV光刻机只有荷兰的ASML公司才能生产,售价大于1亿美元。只有EUV光刻机才能生产7nm的芯片。(摘编自“搜狐网”2023年1月5日)材料三:目前,我国半导体产品主要集中在半导体材料、晶圆制造和封装测试等中低端领域。半导体产能也主要集中在28 nm以上的成熟制程。技术水平差异导致我国需要大量进口中高端半导体产品。其中CPU、GPU、存储器等领域几乎全部依赖进口。据中国海关总署统计,我国半导体设备国产化率不足20%。2021年的进口额度高达4325亿美元。在国际上。美国政府力图将美国半导体企业迁至美国本土、中国台湾、日本以及韩国等控制力所及的地区,并联合盟友遏制中国半导体发展。(摘编自《环球时报》2023年1月31日)材料四:经过近几年的发展,我国在芯片设计技术方面进步很快。目前中国不仅拥有全球数量最多的设计公司。而且水平也达到了相当高度,能够设计出一批优秀产品。然而,中国还存在设计工具方面的“短板”。芯片设计是在电子设计自动化工具(EDA)软件平台上来完成的。EDA工具非常昂贵。搭建一整套IC设计流程的工具。需要花费几百万到几千万然而。目前能提供该软件服务的主要是外国公司。在芯片制造领域,目前中国芯片制造厂的装备需要从国外进口。其中难度最大的、超高端的7nm、5nm使用的光刻机。只有ASML能生产,由于美国控制着ASML的出货。中国大陆的企业想买EUV光刻机。目前几乎没有可能。芯片产业所面临的人才瓶颈。突出表现为行业高端人才稀缺。芯片产业高端人才并非单纯的技术研发人才,而是能够集技术创新与产业创新于一体的复合型人才,这导致芯片岗位人才大量稀缺《中国集成电路产业人才发展报告(2020-2021年版)》预测。到2023年前后,全行业人才需求将达到76.65万人左右。其中人才缺口将达到20万。(摘编自“数科邦”平台2023年1月18日)30.下列对以上材料的理解和判断,不正确的一项是( )A.芯片是非常精密的仪器,它常常是电子设备的一部分。其单位是纳米。B.制造芯片最核心的设备是光刻机。而光刻机只有荷兰的公司才能生产。C.我国半导体设备国产化率较低。中高端半导体产品需要大量依赖进口。D.目前,我国芯片设计技术的发展已取得较大进步,但仍存在一些短板。31.材料二第一段运用了多种说明方法。请列举其中两种并分析各自的表达效果。32.请结合材料三概括中国半导体产业发展面临哪两大挑战。33.请结合上述材料,为中国半导体芯片产业的发展提出三点建议。【答案】30.B 31.(1)运用举例子的说明方法,举Intel旗舰Arc处理器所拥有的晶体管数目多,具体有力地说明了芯片制造的难度大,增强了说服力。(2)运用列数字的说明方法,列举具体的数字,具体科学地说明芯片含有的晶体管数目多,高端芯片的尺寸小的特点,增强了说服力。(3)运用作比较的说明方法,把高端芯片的大小与头发直径作比较,突出了高端芯片尺寸小的特点,增强了说服力。 32.(1)半导体产品主要集中在中低端领域,高端技术水平落后。(2)美国及其盟国对我国半导体发展的遏制。 33.(1)重视和加强国际合作。(2)加快自主研发步伐。(3)大力培养芯片高端人才,积极引进外来人才。【解析】30.本题考查对材料的理解和判断能力。解答此题,要仔细阅读各选项,找出与原文不同之处,再作出判断。B.由材料二第二段“现在最高端的EUV光刻机只有荷兰的ASML公司才能生产”可知,只有荷兰公司才能生产的不是普通光刻机,而是最高端的EUV光刻机。因此选项表述不正确;故选B。31.本题考查辨析说明方法及其作用。常见的说明方法有:举例子、列数字、打比方、下定义、作诠释、作比较、分类别、摹状貌、引用等。答题时,先判断说明方法,然后分析其作用。由材料二第一段中的“比如Intel旗舰Arc处理器拥有217亿个晶体管,如此大规模的晶体要在小小的芯片上生产出来,难度可想而知”可知,运用举例子的说明方法,其作用是为了有力地说明芯片制造的难度大。由“100亿”“217亿”“3nm”等可知,运用了列数字的说明方法,其作用是说明芯片含有的晶体管数目多,高端芯片的尺寸小。由“人的头发的直径是40~50微米,也就是40000 nm,那么3nm就比头发直径的万分之一还要小”可知,运用了作比较的说明方法,把高端芯片的大小与头发直径作比较,突出了高端芯片尺寸小的特点,增强了说服力。32.本题考查概括材料内容。题干要求结合材料三概括,阅读材料三,由“目前我国半导体产品主要集中在半导体材料、晶圆制造和封装测试等中低端领域,半导体产能也主要集中在28nm以上的成熟制程。技术水平差异导致我国需要大量进口中高端半导体产品,其中CPU、GPU、存储器等领域几乎全部依赖进口”等,可以概括挑战一为:半导体产品主要集中在中低端领域,高端技术水平落后。由“美国政府力图将美国半导体企业迁至美国本土、中国台湾、日本以及韩国等控制力所及的地区,并联合盟友遏制中国半导体发展。”可概括挑战二为:美国及其盟国对我国半导体发展的遏制。33.本题考查逻辑思维能力。针对材料二第二段“光刻机被认为是芯片制造业的皇冠,设计与制造光刻机本身的难度就很高,需要借助全球科技的力量”等,可提出建议:重视和加强国际合作。针对材料四第一段“芯片设计是在电子设计自动化工具(EDA)软件平台上来完成的,EDA工具非常昂贵,搭建一整套IC设计流程的工具,需要花费几百万到几千万,然而,目前能提供该软件服务的主要是外国公司”及材料四第二段内容,可提出建议:加快自主研发步伐。针对材料四第三段内容,可提出建议:大力培养芯片高端人才,积极引进外来人才。十一、阅读下面的材料,完成后面的小题。【材料一】大山里的女校张桂梅,筹建了全免费的公办女子高中——云南丽江华坪女子高级中学。该校学生大多来自贫困山区,建校12年,1645名女孩从这里走进大学。张柱梅一直坚守教育岗位,把文化摆既贫困的理念带进大山。2019年该校的综合升学成绩位居当地第一。中央电视台《面对面》节目的记者对她进行了专访。记者:您为什么想筹建这样的学校?张桂梅:我在民族中学时,常有女生从课堂上消失,这些姑娘被家里定下婚事,要出嫁了。华坪儿童之家成立,收养的孩子中有一部分是被遗弃的健康女婴,这让我萌生了筹建免费的女子高中的想法。记者:为什么不男女生一起招收呢?张桂梅:女孩子受教育后,可以改变三代人。如果地有文化,她会把孩子丢掉?我的初衷就是解决低素质母亲和低素质孩子的恶性循环。(画外音)从那之后,张桂梅以校为家,成了学生们眼中的“魔鬼”,从洗敝、吃饭到自习,每件事都被张桂梅严格限制在规定时间内。张桂梅:我们要付出的远超过一般学校的老师。老师结婚,办完了婚礼,第二天就回来上课。我们的孩子开始刷题,人家说这个方式对孩子不好,我们没办法呀,尽管苦一点、累一点,但大山里的学生也可以考到名校,这一切都值了。【材料二】《山海情》情动90后前不久,扶贫剧《山海情》在一片好评声中收官,火得让许多年少不知“贫”滋味的90后和00后热泪盈眶。《山海情》取材于真实的生活,讲述了二十世纪九十年代以来,宁夏西海固的移民在国家扶贫政策的支持和兄弟省市的对口帮扶下,攻坚克难,将黄沙弥漫的“干沙滩”建设成为寸土寸金的"金沙滩"的故事。昔日贫困的农村、一心干实事的扶贫干部、走向田间地头的教授专家……这些镜头里的人和事,也曾真实地出现在老百姓身边。没有偶像滤镜,没有玄幻情节,无须眼花缭乱的特效,饱含生活气息的细节和充沛的情感,已经足够引起观众的情感共鸣。【材料三】国家优先发展教育党中央、国务院始终把教育摆在优先发展的战略位置,始终做到规划优先、投入优先、资源优先。党的十八大以来,国家财政性教育经费占国内生产总值比例连续保持在4%以上,优先向农村地区、边疆民族地区、革命老区、边远贫困地区教育发展倾斜。截至2020年底,全国义务教育阶段辍学学生由台账建立之初的60多万人降至682人,其中20多万建档立卡辍学学生实现动态清零。目前,全国中小学互联网接入率从2012年的25%上升到100%,拥有多媒体教室的学校比例从48%上升到95.3%。近年来,我国“特岗计划”累计招聘教师95万名,“国培计划”培训中西部乡村学校教师校长近1700万余人次,连片特困地区乡村教师生活补助惠及8万多所乡村学校127万名教师,19万名教师选派到边远贫困地区、边疆民族地区和革命老区支教……一支“下得去、留得住、教得好”的乡村教师队伍正在成长起来。最新数据显示,乡村教师队伍的整体素质大幅提升,本科以上学历占51.6%,中级以上职称占44.7%。34.下列对相关内容分析不正确的一项是( )A.面对华坪女中目前取得的成绩,张桂梅并不满意,她希望孩子们能进最好的学校,这是她创建免费女子高中的初衷。B.电视剧《山海情》取材于真实的生活,记录了二十世纪九十年代以来,宁夏西海固移民在国家扶贫政策支持下逐渐脱贫的故事。C.党的十八大以来,国家财政性教育经费优先向农村地区、边疆民族地区、革命老区、边远贫困地区教育发展倾斜。D.截至2020年底,全国义务教育阶段辍学率急剧下降。35.结合三则材料分析,中国为脱贫事业采取了哪些具体措施?(至少答三点)36.假如你是中央电视台《面对面》节目主持人,请为下面的采访提纲补写一个问题,并简单阐述这样提问的理由。采访提纲采访问题:___________提问的理由:_________________________【答案】34.A 35.(1)优先发展教育,向农村地区、边疆民族地区、革命老区、边远贫困地区教育发展倾斜:(2严控适龄儿童辍学;(3)改善学生学习环境;(4)提升乡村教师整体素质;(5)兄弟省市的对口帮扶贫困地区:(6)教授走向田间地头指导生产;(7)培养一心干实事的扶贫干部,深入一线,脱贫攻坚;(8)拍摄制作精良的扶贫影视剧,对扶贫事业进行大力宣传。 36.既然是免费的学校,那开办学校的费用哪里来? 一方面是对学校如何运作感到好奇;另一方面,可以挖掘张桂梅办学背后的艰辛,让新闻内涵更丰富【解析】34.本题考查对内容的分析。A.有误,根据材料一第三段“我的初衷就是解决低素质母亲和低素质孩子的恶性循环”可知,“面对华坪女中目前取得的成绩,张桂梅并不满意,她希望孩子们能进最好的学校,这是她创建免费女子高中的初衷”与原文不符,表述错误;故选A。35.本题考查材料内容概括。根据材料三首段“优先向农村地区、边疆民族地区、革命老区、边远贫困地区教育发展倾斜”直接提炼;根据材料三第二段“截至2020年底,全国义务教育阶段辍学学生由台账建立之初的60多万人降至682人,其中20多万建档立卡辍学学生实现动态清零”可提炼:严控适龄儿童辍学;根据材料三第二段“目前,全国中小学互联网接入率从2012年的25%上升到100%,拥有多媒体教室的学校比例从48%上升到95.3%”可提炼:改善学生学习环境;根据材料三第三段“最新数据显示,乡村教师队伍的整体素质大幅提升,本科以上学历占51.6%,中级以上职称占44.7%”可提炼为:提升乡村教师整体素质;根据材料二“《山海情》取材于真实的生活,讲述了二十世纪九十年代以来,宁夏西海固的移民在国家扶贫政策的支持和兄弟省市的对口帮扶下,攻坚克难,将黄沙弥漫的‘干沙滩’建设成为寸土寸金的‘金沙滩’的故事”可提炼为:兄弟省市的对口帮扶贫困地区:根据材料二“昔日贫困的农村、一心干实事的扶贫干部、走向田间地头的教授专家”可提炼为:教授走向田间地头指导生产;培养一心干实事的扶贫干部,深入一线,脱贫攻坚;根据材料二“没有偶像滤镜,没有玄幻情节,无须眼花缭乱的特效,饱含生活气息的细节和充沛的情感,已经足够引起观众的情感共鸣”可提炼为:拍摄制作精良的扶贫影视剧,对扶贫事业进行大力宣传。36.本题考查采访问题设计,设计的问题要切合目的“了解张桂梅创办云南女子高中的相关情况以及她的心路历程”,如:创办这样的学校有哪些困难?理由:切实的了解办学过程中的困难,有助于挖掘张桂梅办学背后的艰辛,获得针对性的帮扶。十二、写 作 1.阅读文字,按要求写作。学习新知时,人们要“领悟”;出现过失时,人们会“悔悟”;用心思考后,人们能“觉悟”……“悟”是明白,是理解,是觉醒……请以“悟”为题作文。要求:①文体不限;不少于600字(诗歌不少于30行);②文中不得出现真实的人名、学校名和地名。【答案】例文:悟人们常说:“做事要有悟性。”,那么,何为悟呢?我认为“悟”就是指长期思考积累后的想通与明白。前期的思考与准备是达成“悟”的必不可少的条件,如果没有这些,人们是不可能悟的。禅宗是佛教里的一个派别。佛教的大部分禅理都是禅宗努力的结果。禅宗又是做什么的呢?他们只需做四个字:参禅打坐。其实就是清心思索。许多禅宗的僧人都要经过长时间的思索才能有所悟。如果不参禅,那是绝对不会有悟的。就如慧能,在寺庙中做了多年杂役,经过了长时间的思索与准备,才会有后来令人惊艳的:“菩提本无树,明镜亦非台。本来无一物,何处惹尘埃。”,从而获得衣钵传承,成为禅宗的继承人。如果没有前期的准备与思索,他能够得此悟吗?曹雪芹经历了家族衰落,亲人离散,最后在“举家食粥”的困境下悟出了《红楼梦》这一著作。如果没有前期苦难的沉淀与他少年时的知识积累,是不可能诞生这部“怀金悼玉”,批判封建礼教的《红楼梦》的。这就告诉我们,在文学艺术上,不经过长期的积淀与准备,也是不可能悟的。司马迁撰《史记》积淀了多年,梅兰芳创梅腔也经历了几十年的苦功,这难道不是经过准备才得悟的吗?科学史上关于“悟”的故事更是多不胜数。在奥斯特发现电流磁效应之前,有几位科学家都做过同样的实验,但他们要么是准备不足,要么是思考不到位,都没有发现这一现象。直到奥斯特,他充分吸取了前辈的经验,进行了充足的准备,最终悟出了这一现象,为人类进入电气时代作出了杰出的贡献;牛顿在前人的基础上潜心研究,最终形成了三大定律和万有引力定律,形成了近代物理学体系。这些都告诉我们,只有在经历了思索,准备和积淀后,才能得悟。佛曰:“一花一世界,一佛一如来。”在生活的各个细节中我们都能得悟,得到各种各样的悟。但无论怎么悟,都要进行长足的准备与沉淀,就如同物理学上的量变,只有这样才能得悟,从量变到质变。所以,如果你想要有所悟的话,请先沉下心来,慢慢沉淀,你要相信水到渠成,瓜熟蒂落。只要你坚持不懈的攀登,终会登上顶峰!【解析】本题考查命题作文。第一,审题立意。“悟”是理解,明白,觉醒的意思,结合导语可知,文章的立意就是强调人应该多思考,理解,积累等去达到领悟的境界。可就这个中心具体立意为:要想领悟就要积累、沉淀;在生活中要多思考、多领悟;如何“悟”;什么才是真正的“悟”等。第二,选材构思。本题适合写成议论文,选取一个有关“悟”的中心论点,合理安排文章的论证思路,选取有说服力的论据如孔子的“吾日三省吾身”,牛顿在思考中发现物理定律,爱迪生凭借思考和探索成为发明家等,最后形成一篇论点明确,逻辑清晰,结构完整,论证方法恰当的议论文。也可以写成记叙文,记叙一件自身经历的“悟”的过程,表达有关“悟”的中心思想,事件中要有从没有“悟”到最终“悟”的过程,可以是醒悟,也可以是悔悟,还可以是觉悟等等。2.请以“春天,就这样来了”为题,写一篇600字左右的记叙文。要求:(1)不得透露个人相关信息。(2)不得抄袭。【答案】例文:春天,就这样来了一阵雨过后,空气也像被雨冲洗了一般,格外清新,如茵的草地上,点点露珠挂在草叶上,像一个个晶莹的水晶灯笼,太阳光照在上面,映出了缤纷的色彩。要考试了,我的心中掠过一丝迷惘与担忧:几个月的奋斗即将体现在那三天,几个月的等待只为这一战,万一考不好,万一临场失常……带着忧虑和不安,我走出了家门,出去散散心吧!路边的花儿被风雨打折了,想要站起来重新开放,似乎不可能了——这是不祥的预兆吗?明天,后天,大后天,我会失败吗?泪水不知不觉地滑过我的脸颊。迷蒙中,我看到了一个小小的、白色的身影——是纸吗?不,现在没有风,它却在微微颤动。走近一看,哦,是一只白蝴蝶,只见它全身湿湿的,显然是被雨淋的,两只触角极细微地颤动着,身上芝麻大的黑点依然可见。它停在花瓣上,似乎观察着周围的动静,就算周围有什么敌害,小白蝶也无力反抗,它太虚弱了。此时,我仿佛看到它在雨中挣扎的情景:大雨哗哗地下着,显示着它的淫威,小白蝶想找个地方躲起来,却被一个大雨点给打在了地上,它抖着翅膀想要飞起来,可它那细弱的脚连站都站不稳。第一次,它没有站起来,第二次,它又倒下了,第三次,第四次,第五次……啊,它终于站起来了,爬出了水洼……我的心怦然一动,多勇敢的小白蝶啊!我轻轻捉住它的翅膀,小心翼翼地把它放在朝阳的地方,让它感受一下阳光的温暖,它的身体又抖起来了,是怕我吗?噢,不要怕,我的小勇士,我微笑着离去,刚走不远,我突然想到我怎么可以把这么弱小的生灵放在那里呢,我又跑了回去,令人震惊的是,小白蝶飞向了天空,飞去之前,它在我的身边转了一圈,似乎感谢我,然后又飞走了,飞向了天空——多顽强的小生命啊!就在这一刻,我分明看见:天空,很蓝,阳光洒在我身上,很温暖!我忽然意识到:生命中会有太多的遗憾,但人要学会坚强,学会长大,对于困苦和迷惘,我们应该微笑面对。那一刻,我的心里洒满阳光。春天,就这样来了。【解析】本题考查全命题作文。第一:审题立意。以“春天,就这样来了”为题,题目关键字“春天”,代表着温暖、生长。春季,阴阳之气开始转变,万物随阳气上升而萌芽生长。“春”既可以是季节上的春天,万物复苏;也可以是人生的春天,带给自己美好感受的事件。“就这样来了”意味着已经到了。“春来了”可实写春天来了,美丽的景色呈现在眼前,也可虚写人生的春天来了,学习的春天来了,友情的春天来了等,表现一种新的气象、新的面貌、新的开始等。立意上应着眼于春的美好感受,给人以积极的期盼渴望,表达对真善美的追求和积极向上的人生态度。第二:选材构思。文体明确要求写成记叙文。可以选择发生在春天里的故事,也可以是故事带给我们春天般的感受,也可写描绘春天的景物。写故事,要扣住春天的背景,扣住故事带给人的美好感受,给人以希望,比如,通过自己的刻苦努力,终于在学习上有所突破,得到了老师同学的认可,让我觉得迎来了学习的春天;比如在和父母的不断沟通中,我慢慢理解了父母,让我迎来了亲子关系的春天等,写作时注意写清楚时间、地点、人物,起因、经过和结果;要用心理描写出自己的感受,用议论抒情语句故事对自己的影响或作用。写景物,应抓住春天万象更新、生机勃勃的特点,用春风、春雨、春花春草等景物以及人在春天的活动和心情来展开具体描写,用议论抒情语句,表达自己的喜爱和赞美之情。3.请以“窗外”为题,写一篇不少于600字的文章。要求:选好角度,确定立意,除诗歌外,文体不限不要套写抄袭,字迹工整,书写清楚,卷面整洁。不得泄露个人信息(真实的人名、地名、校名等)。【答案】例文:窗外又是忙碌的一天,回到家后,我把自己“甩”到书桌前。伸一个懒腰,舒活一下筋骨,视线又落到那窗外已是光秃秃的玉兰树上。又想起她盛开时的容颜。每当我趴在书桌前疲惫不堪的时候,我总是会不经意间忘向窗外那棵开得极好的木兰上。不知何时,我已经习惯于望向窗外,不知何时,我望向木兰总会和我记忆里的她联系在一起。窗外的那棵木兰是白中透着些许红色的。在天气还夹杂着丝丝寒风的时候,木兰树上就挂上了许多花骨朵。那一个个花骨朵小小的,被红色染了外衣,挺立在那不温不寒的微风中。我每回看到她们就有几分暖意从脚下蔓延,直至心头。她们替我驱逐了身中的寒气。从窗户向外望去,那一个个红扑扑的花苞,就如一个个灯盏上的红烛,照亮了我窗外的景色。那一个个红烛陪我度过那时枯燥无味的时光,在不知不觉中,她们慢慢舒展了身姿,露出了洁白无暇的“内衬”。从窗户向外望去,最外层火红的花瓣托住了里层纯白的内芯,花瓣层层相叠,有着一种不可描述的美丽。说那红白相间的木兰妖艳是最合适不过了,但若说她们清纯无比亦是不足为奇。我双手托腮,坐在窗前,把窗户大开,呼吸那新鲜的空气。清甜的香气扑面而来,有泥土的芬芳,青草的清新,野花的芳香,还有……嗯,还有一种若有若无的、淡淡的清香。哦,原来,那就是木兰的芳香啊!我总想摘下一朵,轻轻拢在手中,将茶沏在那木兰的花朵中,把那木兰的清香和着那茶香一同咽下。或者,把木兰的香气存在瓶子里,疲倦了,就拿出来闻闻,让那清香把那倦意消灭得无影无踪。后来,阅读书籍时,看到有人用木兰来象征着纯洁的友谊。开始我并不这么认为。可后来,我望向窗外,却想起了好友那张白里透红的小脸,也想到我们那坚不可摧的友谊。回忆的片段浮现:初见时,好友那如木兰一般的身影。再后来,我同好友一起爱上了木兰。自己对木兰的喜爱,而我,也在她的熏陶下爱上了木兰。最后,回忆定格在那即将分别时,互相赠送“木兰”的情景。回想罢,当初爱上木兰也许是因为好友的熏陶。可现在,我对窗外那一株株木兰的爱怕是已超过了我对那段友谊的爱,而那一段友谊已融入我对木兰的爱之中,成为不可分割的一部分。【解析】本题考查全命题作文。第一、审题立意。要求以“窗外”为题,写一篇文章。“窗外”的范围比较广,选择性很多,可以选择窗外景物的一个局部进行细致描写,也可以选择窗外的全景进行整体勾勒。“窗外”的世界是丰富多彩的,可以是美丽的风光,如花木茂盛的园圃,金色的麦田;也可以是人文景象,如小伙伴们玩耍的场景、农民伯伯丰收的场面等。不管选择什么材料,都要围绕一个中心,把自己的发现与体会作为文章的主旨,以窗外的景物给人带来的感受,或是由窗外感悟到的人生哲理进行立意,如:窗外的忙碌、窗外的善良、美丽、乐观、自由等。第二、选材构思。既然以“窗外”为题,不妨就选择生活中窗外常见的景象,并结合自己的生活写出“窗外”的感悟,如:生病期间,看到窗外遭受风雨击打的树木显得更加青翠,更加枝繁叶茂,感悟到生命的意义;窗外的小鸟一次次衔来小树枝搭窝,它一次次来回飞,它的执着、勤奋与欢乐,让我感动等。窗外的景也可以是人文景象,如写下雨时,看到窗外大家帮忙抢收粮食的场面,感受到邻里之间的和谐,人性的光辉等。窗外的景象并不是一成不变的,也可以写出窗外景象在空间、时间上的变化,感受不同的风光。写景时要注意通过细节描写,运用视觉听觉等五官感受使描写的景物如在眼前,生动鲜活;写人叙事要注意通过描写人物的语言、动作、神态等来突出人物的形象。写作时要恰当运用议论抒情等表达方式,突出自己的感悟,升华文章主旨。4.同学们,成长过程中,我们每个人都曾留下过或大或小的遗憾,有遗憾,也就有希望和期盼。请以“留点遗憾也挺好”为题,写一篇文章。要求:(1)诗歌、戏剧除外,文体不限。(2)表达真情实感,不得套写、抄袭。(3)文章中不得出现真实的地名、校名、人名。(4)字数600字以上。【答案】例文:留点遗憾也挺好“留点遗憾也挺好。”这是您——我的语文老师兼班主任的口头禅,想不到竟能在此时此地重温,真好!第一次听您说这句话,是在那次班会上。秋天的暖阳透过窗纱,落在我的身上,我觉得特别惬意。您的每一句话,都在拨动着每一个同学的心弦,合奏出和谐美妙的旋律:在“庆祝教师节歌咏比赛”中,我们班的合唱获得一等奖,我们班的诗朗诵获得一等奖,我们班的舞蹈获得一等奖……每一条好消息都让同学们禁不住欢呼,鼓掌,教室成了欢乐谷。然而,我渐渐垂下了头,因为我的脱口秀演砸了,我们班没有获得语言类节目的奖项,是我给班级留下了一点遗憾。我的脑海里又浮现出当时的情景:我站在舞台上,看着台下一片陌生的面孔,忘词儿了,卡壳了,傻愣愣地杵在那里;好不容易想起来,怕忘了,赶快说,像按了快进键。结果可想而知。如果我的表演不留遗憾,我们班岂不大获全胜?唉,真是无地自容啊!越想,我的头埋得越深。忽然,我觉得自己的肩头有一点沉重,抬起头,我看到了您明亮而温暖的目光。你抬起抚在我肩头上的手,轻轻地拍了拍我的肩膀,像对我,又像对大家说:“谁都希望得到十全十美的结果,然而,只要尽心尽力,没有得到完美,留点遗憾也挺好。”您再次把目光投向我:“请大家记住我的话,他的文笔,他的口才,将来一定是出类拔萃的!这一次的遗憾,说不定就是给将来的美好做铺垫的!”一阵热烈的掌声,震动着我的耳膜,在我的周身掀起了势不可挡的浪潮。于是,我站起来,语无伦次地说:“谢谢老师,谢谢同学,挺好,真的挺好!”时光,飞快地流逝。我在您的教导下,刻苦学习语文知识,积极锻炼书面和口头表达能力,不但学到了您的真传,而且也选定了自己未来的路——我也要像您那样,成为一个善于点燃豪情壮志,又能保护其不被旋风吹灭的人。昨天,我在您做考前动员时,又听那句口头禅:“我希望大家在一模考试中有完美的表现,尽力而为。尽力了,即使不完美,留点遗憾也挺好。”老师啊,老师,此刻我彻底明白了您的心。因为爱,您盼望着我们完美;也是因为爱,您对我们说:“留点遗憾也挺好。”【解析】本题考查命题作文。一、审题立意。这是一道命题作文,通过审题,我们可以提取三个关键词“留点”“遗憾”“也挺好”。“留点”,除限定了“遗憾”的比重不可过大之外,还指的是一种客观情况,而对于这些留下的遗憾,可以是自身主动留下,也可以是事件发展超出我们的预料和能力范围被动的留下,但无论是主观上还是客观上,都强调的是一种顺应规律的状态,是一种自我内心能够去完全接纳的坦然的释怀的状态,无论我们将来是否去争取,留下的终究已经留下。“遗憾”一词指的是不满意、悔恨、不甘心的事情,是无法控制或无力补救的情况所引起的后悔的心态,其本质上应该是一种负面的情绪。然而,在这个题目当中,需要我们透过遗憾客观消极的表象看到里边主观积极的内核,一切过往,皆为序章,上一秒留下的遗憾,可能恰恰是下一次辉煌的开场。而这种遗憾也在客观上告诉我们自己的不足,指出了我们还需要努力的方向,这对我们的成长来说是有一定的好处的。因此。对于遗憾,我们要辩证看待。这道题目当中最难的点就在于也挺好,如何将一个貌似消极的遗憾事件和结果,从不同的角度发现其中的闪光点,发现也挺好的内容,乃是本题的关键。其实,还是可用欲扬先抑的手法,前后对比去突出在起初之时遗憾,就是遗憾,令人失望,令人不甘,然而需要添加一个事件的转折点,可能是亲人师长等人的别样的“开示”,也可能是自我回溯过经历感受到一路走来自己的收获,或是认识到人生本来就没有完美之事,完美才是美最大的敌人,再美丽的玫瑰也是有刺的,物极必反,月盈必亏,上升到道家的哲学思想高度也是可以的,但一定要注意中间事件的转折要稍微自然一些。二、选材构思。从个人成长兴趣爱好的挑战角度来说,面对我们探索的领域,我们可能会经历无数次的挑战,这既是和他人竞争的过程,也是自己和自己竞争认清自我的过程。在这些时候我们不可能每一次都取得胜利,而那些失败、那些遗憾的瞬间,无论是个人原因还是集体的原因都可以从侧面告诉我们前方的路还很长,我们还有很多需要努力的地方。如果从写人的角度去诠释的话还是有一些难度的,要注意无论是亲情之间还是师生同学友谊之间,这个遗憾不能是过度的,欲扬先抑前边“抑”的部分,如果太过火的话,后边就“扬”不起来了,因此,可能是一个瑕不掩瑜的小遗憾,而这种遗憾,让我们意识到人和人之间的相互信赖,人和人之间的美好真情值得我们去珍视珍惜,有点遗憾也挺好。如果是写传统文化的角度,可选择的方向还挺多的。在我们认识传统文化,无论是物质性的还是非物质的时候,它们在传承的过程当中都可能留下一些瑕疵,但这并不会损害到它们作为精神层面的美好,甚至如断臂的维纳斯一般,正因为残缺,所以才更加美好;正因为遗憾,所以才更加圆满。故而后世雕塑家们根据《米洛斯的维纳斯》开始有意无意地在自己的创作中适当地“留白”,故意把人物中的某些部位略去不做,留给人们以想象的空间,让人们自由地发挥自己的想象。在本题目之中,“遗憾”便可以诠释为“留白”,尤其是自己在体验接触传统文化相关事情的过程中,留下的遗憾,也并不是缺憾,它会让我们下一次更加勇敢,更加真切地意识到,有一种快,叫做慢;有一种美满,叫遗憾。这就起到“以小见大”的效果了,会很有味道。5. __________伴我成长要求:①把题目补充完整,然后作文。②结合个人生活经历,选取真实的生活片段,写一篇700字左右的记叙文;或根据自己的所思所感,写一篇600字以上的散文。③写记叙文要求文章叙事清楚,结构完整,内容充实;恰当运用描写、抒情等表达方式,写出真情实感。④写散文要求叙事或抒情线索明朗,感情真挚。【答案】例文:家规伴我成长少年都爱标榜自己的成长,而说起我的成长,曾经贴在我家墙上的白纸黑字的家规便又浮现在眼前。虽然它已经被撕去,但是对我而言,它不仅伴随我一天天长大,更影响了我对家的认知,见证了我对爱的理解。那条家规,很简单——谁最后一个吃完饭谁洗碗。它源于我小时候总是边玩边吃。事实证明,这条家规的效果相当显著。从那以后,饭一上桌,我便冲上去狼吞虎咽,吃着吃着,一杯水便摆在我的碗边,映出我滑稽的样子。抬起头,妈妈还没有动筷子,阳光轻轻洒在她的脸上,映着她微微上扬的嘴角,但我无暇顾及这些——因为那条家规。那时,家规对我而言,是吞风卷残云的饭菜和那杯白开水。在这条家规的陪伴下,时间好像紧握的沙,我握得再紧也挡不住它的流逝。不知不觉中,我的嘴边涌出了细密的胡茬儿,妈妈的鬓角却沾染了几根白发。一天,妈妈满脸疲惫地对刚放下筷子正准备离开的我说:“帮妈妈把碗洗了吧。”我咽下最后一粒米,漠然地指了指墙上那条家规。六七年了,它不仅始终贴在那里,更铭刻在我的意识里。我转身走进卧室,关门的那一刻,好像有什么东西刺进了我的心里,但这刺痛感稍纵即逝。我成长的脚步仍在继续,但再看这条家规,却渐渐变得麻木,甚至泛起了苦味。直到那一天,妈妈生病的消息传入我的耳中。那一瞬间,我的眼前突然暗了一下,接着心中一团火腾地燃烧起来,烧毁了意识里那白纸黑字的家规。国有国法,家有家规。我一直自以为是地遵守着那条家规,可是我真读懂它了吗?家规,一个家庭的第一条规则不就应该是尊敬父母吗?坐在饭桌前,眼前闪过往昔的一幕幕,我不禁自问:我真的长大了吗?漫长的一个月过去了,妈妈的病终于好了,我们一家人终于能坐在一起吃晚饭了。只是这一次,我不再狼吞虎咽,而是细嚼慢咽,米饭的香味在舌尖上回荡着,灯光映在碗里,在抬头低头的瞬间闪烁着。妈妈吃完了,摇晃着站起身收拾碗筷。我鼻尖一酸,连忙拉住她,指着我的碗:“妈,你看!”最后几粒米正闪烁着晶莹的泪光。我亲手撕去了那张纸,“妈,家规我记下了!”家规,不是为了让冰冷的规则包围生活,而是为了守护一个家庭最真挚、最温暖的亲情,守护这份传承。【解析】本题考查半命题作文。第一,审题立意:这是一道半命题作文。“_______伴我成长”中的“伴”字是相伴相依,是一种陪伴,是凝聚着情感体验的。成长:指走向成熟,摆脱稚嫩的过程。补题,就是要解决“伴”的主语,可以虚指,也可以实指。题目中的关键词一明一暗。明的是“伴我成长”四字;暗的,就是要在横线上填充的词语。其实,这“明”与“暗”是一组因果关系,“暗的”是因,“明的”是果。搞清楚这组关系,就可以在横线上填充适当的词语。在个人的成长道路上,哪些东西与自己相伴相依呢?仔细一想,内容很多,有人、有事、有物,有具体的也有抽象的,比如:亲情、友情、诗歌、音乐、理想、美德、书刊、老街、老宅等等。根据题干提示,横线要补充的内容可以是一个人、一件事、一本书、一件物、一篇文、一句话等等。如,母爱伴我成长;老师伴我成长;大自然伴我成长;诗文伴我成长等。作文中要体现怎样“伴”,“伴”的意义,要强调“伴”的主语对“我”的成长变化所起到的促进作用。而选哪一种素材去写,要和写作者的成长环境相结合,也要和作者的情感寄托相联系。通常情况下,文章写作时切入口要小。第二,取材构思。在人生的历程中,能让自己收获成长的东西很多,它们都可以成为自己“成长”的助力源。写作的过程中,可以选取情感类的素材,如父母的陪伴伴我成长;也可以选取友谊类的素材,如朋友的鼓励伴我成长;可以选取成功类的素材,如自信梦想伴我成长;还可以选取读书类素材,如书中的名言警钟、哲理启示伴我成长……总之,只要是对我们成长有帮助的人、事、物、理均可成为写作的素材。选择符合题意的素材,抒写自己的成长历程中的助力因素,表达积极健康的思想作文。本文适合写作记叙文,在写作方法上,注意在叙述过程中,可以使用细节描写,通过具体的动作、心理、对话等描写,表达“我”内心的感受。在材料安排上,注意详略得当。写作时,多采用一些生动的画面展现、传神的细节描写、鲜明的对比手法等,可以更好地凸显文章主旨。在写法上,学会使用以小见大的写法,会突出作文主题,给读者带来亲切的感受。6.挑战无处不在,但勇者无惧。自然界,疾风里、骤雨下、暴雪中,小草挺拔、翠竹坚劲、青松直立:生活中,长跑途中一道道沟坎、题海里一道道难题、成长路上一次次考验,勇者却把挑战看成了机遇,在挑战的洗礼中成长自己:社会上,环境危机、时局微妙,挡不住同心筑梦,大国崛起……请以“面对挑战,我(们)______”为题目,写一篇记叙文。要求:①表达意图明确,内容具体充实。②600字左右。③文中请回避与你有关的人名、校名、地名。【答案】例文:面对挑战,我不惧前行大多数花是在春暖才开,它却不一样:大多数花遇到困难,就会退缩,它却不一样;大多数花惧怕风雪,它却不一样,反而开得越精神,越艳丽……它,就是梅花!去年寒假,到处冰天雪地,瑟瑟寒风迎面吹来,吹得人面如刀割。我虽坐在室内,却仍感寒气逼人。这一次,我是来参加“书法大赛”的。我坐在椅子上,一遍又一遍地练习毛笔字。练了几遍,我忍不住好奇心,偷偷地看了一眼我们要临摹的作品。我看了之后,大吃一惊,这些字竟然都是老师没讲过、我没练过的作品。我开始有些惧怕:唉,要写的字这么难,这次一定写不好,干脆弃权吧,免得出丑。可这是历经千辛万苦才到决赛的,么能轻易放弃呢?我强迫自己静下心来,开始练字。因为心里惧怕,笔尖颤抖,写出来的字像蝌蚪一样。我一着急,一惧怕,便上了一次又一次厕所。一气之下,我就到外面走走,散散心。天冷异常,我来到外边,不禁紧了紧衣服。我不知所措地走来走去,见地上满是野花的花瓣,枝干上有的光秃秃的,有的还有几朵稀稀拉拉的,虽未凋零,但已黯然失色。我此时的心情就像这些摇摇欲坠的残花,时刻都会退缩、凋零。我在原地徘徊,忽然闻到一股暗香,接着眼前一亮,一个个小红点在不远处若隐若现。我加快脚步,走近一看,原来是一株梅花。我看着她,但见花开得正旺,精神饱满。这时天空下起了雪,我不得不想办法躲避。而梅花,虽然面临着一场极大的考验,却毫不退缩,仍在风雪中傲然挺立,毫不低头折节。雪花轻轻落在梅花粉红的花瓣上,显得更加娇艳。瞬间,我的心一颤,勇气和力量又缓缓流进我的心田。我对自己说了声“我一定能行”便满怀信心回到赛场。虽然这次比赛成绩不理想,但我却明白了一个深刻的道理:世上无难事,只怕有心人!遇到困难不要退缩,只要努力,就能克服它。此后,每当我遇到困难和挑战时,我就会想起那傲雪怒放的红梅……【解析】考查半命题作文。第一,审题。题目中的关键词是“挑战”,“挑战”在这里应理解为“激励自己主动跟困难等做斗争”。青春是一段美丽的时光,是一段充满活力和希望的时光,是应该赞颂的时光。写作时要抓住“挑战”这一主题,将挑战与青春结合。青春会面对很多挑战,只有积极进取,才有可能超越自我,拔节成长。第二,立意。“挑战”意味着敢于正视困难,战胜困难,正视矛盾,解决矛盾。挑战的对象是不限定的,可以从以下几个方面立意:向他人、向客观环境挑战,如向“权威”“陋习”“环境污染”“困难”等挑战,让自己的青春更有价值;向自己心理、生理上的弱点挑战,如向“胆怯”“虚荣”“骄傲”“伤痛”等挑战,让自己变得更加完美等。第三,选材。人生处处有挑战:学习中解决难题,发挥潜力,是-种挑战;赛场上挥酒汗水,超越自我,是一种挑战;漆黑夜晚,独自回家,战胜怯懦,是一种挑战……我们可以写自己的亲身经历,如写原来的自己一直生活在舒适圈里,和同学们有了很远的距离。后来凭着一股不服输的劲头,努力挑战自己,最后拿到了年纪第一的好成绩。 由这一件小事, 让“我”明白了一个道理:面对挑战,我要勇往直前。7.阅读下面的材料,根据要求写作。赏兰园,观得青山园林的故事;习方言,解锁传统文化的密码;访博物馆,洞见人类文明的星火……大千世界,文化无处不在,但它常常隐藏在物之后、事之中,需要我们用心去探寻,去发现。请以“文化藏在_____里”为题,写一篇文章。要求:(1)将题目补充完整并填写在答题卡指定的位置;(2)不得抄袭、套作;(3)不少于600字;(4)文中不得出现真实的人名、校名。【答案】例文文化藏在传统习俗里不受君王任用的屈原,悲戚投江,而有了端午节;为天下着想的嫦娥,泪奔广寒宫,而有了中秋节;互相相爱的牛郎织女,只能靠喜鹊搭桥在七夕见面,而有了情人节。其实传统习俗带给我们的不只是与亲朋好友相聚的时光,更多的是文化与故事。鞭炮声窜进了我的耳朵,而这一窜就是15年,大街小巷无一处不是张灯结彩,祝贺恭喜声随着鞭炮烟雾飘到人们的心坎里,在这我感受到了传统文化的美;在热闹的湖畔旁,有人吃着香气扑鼻的粽子,有人正等待着中午要立蛋的时间,而更多的人正为一旁奋力比赛划龙舟的选手加油打气,在这我感受到了传统文化的美;明月当空的夜晚,有人正围着火炉烤肉,有人正放着绚烂的烟火,而更多的人正分享着自家的月饼,边欣赏那雪白的玉盘,在这我感受到了传统文化的美。在热闹的餐桌上,围着好久不见的亲戚,有人正发着红包,有人喝着热腾腾的鸡汤,有人吃着象征步步高升的萝卜糕,而我正把这一幕烙在心坎上,在这我体验到了传统文化的美;在欢乐的外婆家,有人泡着茶讨论着刚刚的龙舟比赛,有人正雀跃地让大家看他立好的蛋,而我正把粽子放进蒸笼里,在这我体验到了传统文化的美;在温馨的气氛里,有人用关爱的眼神注视着小宝宝熟睡的脸庞,有人和新手妈妈讨论着育儿方法,有人正把恭贺声布满了整间房子,而我正吃着弥月油饭,在这我体验到了传统文化的美。这些传统文化让亲朋好友之间的感情更加紧密了,像是刚出生的婴儿与家人密不可分一样,它让大家放下了手机,尽情享受相聚的欢乐时刻,并期许我们能把传统习俗一代一代地传承下去。在这样充满美的传统习俗里,我看见了文化,看见了传承。【解析】本题考查半命题作文。第一,审题。首先领悟题目中“文化”二字的含义。人类在不断认识自我、改造自我的过程中,在不断认识自然、改造自然的过程中,所创造的并获得人们共同认可和使用的符号与声音的总和,就是文化。文化的含义很广泛,人类社会相对于经济、政治而言的精神活动及其产物,分为物质文化和非物质文化。具体内容非常丰富,泛指群族历史、风土人情、传统习俗、生活方式、宗教信仰、艺术、伦理道德,以及法律制度、价值观念、审美情趣、精神图腾等很多方面。根据题干“赏兰园,观得青山园林的故事;习方言,解锁传统文化的密码;访博物馆,洞见人类文明的星火……大千世界,文化无处不在,但它常常隐藏在物之后、事之中,需要我们用心去探寻,去发现”可知,这里明确了文化可以具体到生活、技艺、建筑、器具、文艺、道德素养等内容。写作时可从中选取一个方面的具体的人、事、物等来立意,表达对某一文化内容的喜爱、自豪、赞美等感受,进而表现对中华优秀传统文化的自信。横线里可以填具体的地点,也可以写抽象的意境。例如:文化藏在烟火人间里,文化藏在传统习俗里,文化藏在岁月的长河里、文化藏在茶道里……第二,立意。文化就在身边,感受传统文化的魅力,体验传统文化的内涵,增强文化自信,提升生活品味。第三,选材。写作的过程就是寻找、发现、表达的过程,平日里我们看到、听到、亲身经历过的事情,其实都融入了文化的内涵。如“道遵思邈心存济世,德昭仲景志在医人”,就彰显了医者高尚的职业道德;再如“竹露松风蕉雨,茶烟琴韵书声”,则渲染了品茶的优雅意境。旅游时见到的名胜古迹、碑刻牌坊,无不富有文化韵味,如“胜地据淮南,看云影当空,与水平分秋一色;扁舟过桥下,闻箫声何处,有人吹到月三更”,写出了扬州二十四桥的独特风景。还有各民族独特的服饰、饮食文化,甚至许多商品包装上的图案文字,都包含着传统文化的元素。中华优秀传统文化,闪烁在国粹京剧色彩斑斓的脸谱上,跳动在民族乐器二胡哀婉凄美的旋律中,绽放在传统佳节异彩纷呈的民俗活动中,凝聚在具有千年历史的古代建筑的红墙绿瓦上……可以说,我们生活的方方面面都有中华优秀传统文化的印记。这个作文命题比较适合写成记叙文,横式纵式结构可以根据所写内容来定。根据材料的提示,不妨将文化内涵化虚为实,定位在一个具体的点上,如书法、剪纸、陶瓷等。写作时还要注重运用细节描写,写出这一优秀传统文化元素的特点和自己的感悟。最后在文章的结尾,要运用卒章显志的方法点题,以抒情笔法使主旨升华。字里行间不仅要体现出对中华优秀传统文化的喜爱和赞美之情,更要表达出坚定文化自信和对美好未来的期盼和向往。8.阅读文字,按要求写作。冰天雪地里,斯科特探险队员坦然的精神面貌震撼人心;荒芜的高原上,植树的牧羊人坚毅的精神品质令人钦佩;暴风雨中,独自寻找鸭群的顾小康脱胎换骨……人生茫茫,阻碍重重,或成功,或失败,胜之坦然,败之释然,但淋过的雨、吹过的风一定会成为我们生命历程中的宝贵经历。请结合你的生活体验,先将题目“_______的经历”补充完整,然后作文。要求:①要有真情实感:②不少于600字:④不得抄袭、套作:⑤不得出现真实的人名、校名和地名。【答案】例文:等待的经历回想2020年,注定是不平凡的一年,我们面临着场新型冠状病毒的挑战,我们陷入漫长的等待中。过年了,我坐在电视机前看春节联欢晚会,手机突然就收到了一条信息:武汉封城了。这才意识到这件事情的严重性,手机上也刷爆了关于新型冠状病毒的事。妈妈和哥哥为了这件事还推迟了好多天才外出工作,本想着去奶奶家拜年的,也延迟了好几天,街上人很少,但都戴着口罩,没有了往昔欢乐景象。确诊的人数已经有好几万了,形势也变得越来越严峻了。临近开学却又突然收到了要延迟开学的通知,说实话,当时还是挺开心的,因为可以多玩几天,但没几天后渐渐感到无聊了,就这样一直等啊,等啊,等到的还是延迟开学通知。还好,中途去了一趟学校领书跟小伙伴见了面,然后开始上网课,老师网上辅导,学习没落下。在这两个月的时间里,我无时不刻不关注着疫情的消息,其中有一段时间特别严重,令人揪心,但现在,疫情得到了控制,各地逐渐复工复产,学校也将迎来复课,漫长的等待也将结束。在这些日子里,有一群默默守护着我们的医护人员,他们是我们的天使,是“最美的逆行者”,我们等待着他们凯旋归来。除了医生,还有无数的志愿者,他们不怕被感染,为人们送去关爱和温暖,还有我们的物流人员、快递小哥、安保人员、公安民警……这些是多么震撼啊,每个人都为这次疫情做了很多事,而我们最应该做的就是好好读书,长大后为国家作出贡献,虽然我们无法像医护人员、志愿者们做出那么大的贡献,但呆在家里,预防冠状病毒,就是我们做出的最大贡献。等待春天,等待希望,等待疫情清零的那一刻。现在,疫情结束了,而从这次疫情,我也明白了很多,要注意个人防护,注重个人健康和卫生,珍惜家人之间的感情,培养独立自主的生活习惯,更要学会自律,在以后的日子里,学会感恩生活,热爱生命,积极快乐的过好每一天。【解析】本题考查半命题作文写作。首先,审题立意。题目“_____的经历”中的关键词为“经历”,“经历”意思指自身或他人见过、做过或遭遇过的事。题目揭示出写作内容必须是与自己密切相关的且已经发生过的真实事件。立意方面,可将这一“经历”带给自己的感悟、磨砺、提升等作为写作立意点,抒发这一“经历”的独特作用和深刻意义,表达对真善美的追求和积极向上的人生态度。其次,构思选材。应先根据对题意的理解完成补题,如一次成长的经历,一次后悔的经历、幸福的经历等。然后根据自己熟悉的体裁确定文体。如写成记叙文,可结合材料“人生茫茫,阻碍重重,或成功,或失败,胜之坦然,败之释然,但淋过的雨、吹过的风一定会成为我们生命历程中的宝贵经历”,联系自己的生活实际,记录自己难忘的一次经历。写作中心要紧紧围绕这一“经历”对自己成长的影响和作用。如写成议论文,可以摆出论点,然后引用事例作为论据,进行论证。也可讲故事作为开头引出中心论点并作为论据论证中心论点,然后选取名人名言和典型事例,旁征博引,从道理和事实两个角度充分论证,做到论点鲜明、论据充分、论证有力。9.写作苏东坡说:“古之立大事者,不惟有超世之才,亦必有坚忍不拔之志。”坚忍不拔恰恰就是人的一种性格。思想决定行为,行为决定习惯,习惯决定性格,性格决定命运!一个人的性格必为其带来巨大影响是古今中外的先贤们达成的共识。请你写一篇文章,讲一讲你在“性格决定命运”方面的自身经历或者所见所闻所想,让同学们读了你的文章以后受到启发。写作要求:①题目自拟,文体不限,但不要写成诗歌或戏剧,也不要写成书信体;②不少于600字,不要抄袭,不要套作;③文中不得出现自己的真实姓名、校名等相关信息。【答案】例文:性格决定命运 我们无法改变天气,但我们可以转换自己的心情;我们无法左右别人,但我们可以做好自己;我们无法决定出身,但我们可以改变命运。 一种性格,一种命运。家喻户晓的爱迪生,小时候被人看作是笨孩子,在大多人眼里,他长大后肯定没多大出息;可是后来他发明了灯泡,给了整个世界的夜晚一片光明,他也因此在世界上闪闪发光。人尽皆知的物理学家牛顿,因为被树上掉下来的苹果砸到了头,从而为物理学术界力学的研究作出了巨大贡献。难道他们的成功都只是巧合的遇到了某种外物诸如灯泡、苹果而发生的幸运奇迹吗?不,不是的。那是因为在他们身上都有一种善于观察、积极思考的性格,才有了他们后来的巨大成就。 一种性格,一种命运。有这样一个故事:从前,有两个结伴穿过沙漠。走到途中,水被喝光了,其中一人也因中暑而不能行动。同伴把一枝枪递给中暑者,再三吩咐:“枪里有五颗子弹,我走后,每隔两小时你就对空中鸣放一枪,枪声会指引我前来与你会合。”说完,便满怀信心找水去了。躺在沙漠里的中暑者却满腹狐疑:同伴能找到水吗?能听到枪声吗?他会不会丢下自己这个“包袱”独自离去?暮色降临的时候,枪里只剩下一颗子弹,而同伴还没有回来。中暑者确信同伴早已离去,自己只能等待死亡。想象中,沙漠里的秃鹰飞来,狠狠地啄瞎了他的眼睛,啄食他的身体……终于,中暑者彻底崩溃了,把最后一颗子弹送进了自己的太阳穴。枪声响过不久,同伴提着满壶清水,领着一队骆驼商旅赶来,但看到的却是中暑者温热的尸体……有时候就是这样,只要坚持一下,再坚持一下,也许命运就截然不同了。一种性格,一种命运。世人最崇敬的德国音乐家,被誉为“乐圣”的贝多芬,小时候家境清寒贫穷,全家人相处的气氛也不和谐。在他28岁时,双耳失聪,但他身残志坚,失聪后完成了九部《命运交响曲》。与此同时贝多芬留给后世的名言“我要扼住命运的咽喉,决不能倒下”成了不少身残志坚的人挑战不幸命运的人生信条。哲学家培根在《习惯论》说:“思想决定行为,行为决定习惯,习惯决定性格,性格决定命运。”拥有了优秀的性格和品质,一定能收获成功。【解析】本题考查材料作文写作。审题立意:提示材料列举了苏东坡说过的名言,紧接着引出“性格决定命运”的话题,要求围绕“性格决定命运”结合自身经历或者所见所闻所想写一篇文章启发同学。话题包含三个核心词,分别为“性格”、“决定”和“命运”,我们可据此进行立意。立意方面,可以从某种性格带给自己的积极作用或影响作为立意点,阐明性格对我们成长的重要作用。选材构思:围绕“性格决定命运”进行写作,可结合提示材料“思想决定行为,行为决定习惯,习惯决定性格,性格决定命运”回忆自己成长过程中的经历,选取自己印象深刻的某件事,这件事必须要与“性格”和“命运”相关。比如:记叙自己由自卑到自信的变化历程,可运用详略安排的手法记叙事件,运用多种人物描写方法塑造人物性格,写作过程中通过对比突出自己由“自卑”到“自信”蜕变后的变化,这样的文章才能立意深刻,打动人心。当然,也可以议论说理,以“性格决定命运”为中心论点,列举实例,引有名言,论证性格对于命运的决定性作用,做到有理有据,以理服人。10.青春路上,我们书声琅琅,歌声飞扬;青春路上,我们朝气蓬勃,欢呼争强;青春路上,我们意气风发,昂扬向上。青春路上,我们的脚步不会因困难而停止前进;青春路上,我们的旋律不会因风雨而中断奏响。青春路上,我们用微笑面对所有的迷茫;青春路上,我们用希望照亮前行的路。请从下面两个题目中任选一个,写一篇不少于600字的文章。题目一:以“希望照亮前行的路”为题,写一篇记叙文。题目二:围绕“青春”这个话题,写一篇演讲稿,在五四青年节活动上演讲。写作提示:①如果选择题目二,请自拟标题;②避开真实的人名、地名、校名;③书写规范,卷面整洁。【答案】例文:题目一:希望照亮前行的路梦想,是人生中最华美的篇章。梦想,究竟是什么呢?有的人可能会说:梦想是儿时天真的梦,是纯真美好的愿望。在我的眼中,梦想是人生奋斗的目标,是未来前进的方向!在我很小的时候,曾经有过很多梦想:看到花店就想去当花店老板;看见蛋糕店就想做蛋糕师;每次生病了,去看医生,就想做一名“白衣天使”。但是,随着时光的流逝,我希望我能当一名老师,这个梦想,在我的心中扎下了根。假如我当了老师,我会和同学们一起看书学习,让同学们拥有更多的知识,会让同学们做很多的游戏,开发他们的智力;我会登上那三尺的讲台,向孩子们传授知识,带领他们在知识的海洋里遨游;我会把我的学生当作我的孩子,像照顾自己的孩子一样照顾他们,教育他们。让他们从一个个真善美的小故事里领悟到人生的真谛;让他们从一首首如歌如画的唐诗宋词中,领略到“万籁此都寂,但余钟磬音”的寂静;让他们从一个个英语单词中,感受到语言的神奇;让他们从一个个活蹦乱跳的数字中感受到数学王国的神奇。我还会让孩子们从学习中找到乐趣,在乐趣中学习,爱上学习。当他们捧起奖杯时,我会告诉他们“不要骄傲,一定要再接再厉,在以后的道路上继续努力,才能有所成功”。当他们在成长的道路上遇到困难而气馁时,我会告诉他们“不要气馁,阳光总在风雨后,坚持就是胜利”。我会像当时老师教育我们一样,把他们培养成一个有爱心,有责任心,有正义感的好孩子,长大后成为一个对社会有用的人才。希望照亮了我前行的路,我一直在为这个梦想和希望而不懈地努力,扎扎实实学好每一门功课,一步一个脚印朝这个目标攀登。梦想就如同一双翅膀,带着我在宽广的天空中飞翔,让我的人生更有意义。题目二:青春宛如一首歌亲爱的同学们:大家好,今天我演讲的题目是“青春宛如一首歌”。青春宛如一首歌,歌词的细腻华美,是我们对未来梦想的追逐摸索;旋律的起伏跌宕,是我们面对现实的拼搏。我们就是词曲的作者,在人生的乐谱上写下青春的单纯快乐,写下青春的奋斗执着。青春的单纯快乐就如同歌曲的前奏。听,那前奏已在耳边响起。那次联欢会上,同学们个个神采飞扬。大家围坐在一起玩着猜拳打手的经典游戏。“剪刀石头布,哈哈……你输了,快乖乖地把手放下吧!”瞧!不知是哪一个同学正幸灾乐祸着呢!记得当时一向内向的我,也抛开了淑女的形象,在同学们青春的渲染下变得风趣了许多。“哈哈。”我放声大笑,“你们要注意手啦!我的‘魔爪’要伸向你们啦!”只见同学们个个提高警惕,屏住呼吸,教室里顿时变得鸦雀无声。突然,“啪”的一声,许多只手以迅雷不及掩耳之势缩了回去,沉默在此时顿时被打破,只见有的同学长舒一口气,好像逃过了什么大劫;有的同学则是欣喜若狂,庆幸自己没有中了我的“魔爪”。独有小可皱着八字眉,不停地抱怨:“太倒霉了,我小可英明一世,谁知竟落在你的‘魔爪’之中。”话音刚落,只见大家笑得前俯后仰,笑声在教室中回荡。如今想起那青春的快乐单纯仍温暖着我稚嫩的心房。歌曲的前奏如此美好,但高潮就蜿蜒曲折了。没错,青春的艰苦奋斗就是高潮。如今已是初三了,紧张的气氛、快节奏的生活压在我们的肩上。每晚,每当星星月亮挂在静谧的夜空,人们享受这祥和的夜晚之时,我们却还在奋笔疾书,在苦海中迷茫。我们顾不得是否已进入深夜,我们顾不得是否月光下只有我们的点点明亮,我们顾不得疲惫的身躯朦胧的睡眼。我们只是在艰难中奋斗,只是在黑暗中摸索,只是在被一道道难题所困。我们不知这样的夜晚度过多少,但这些夜晚决定着青春的希望,谱写着乐曲的高潮。乐曲的前奏是否让你倾心向往?乐曲的高潮是否让你的心情起伏跌宕?这就是我的青春,它宛如一首歌在我生命的岁月里被我奏响。看那词,听那曲,都是我青春的荡漾。我的演讲结束,谢谢大家。【解析】题目一:本题考查命题作文。第一,审题立意。以“希望照亮前行的路”为题,写一篇记叙文。“希望”在字典中的解释有两种:一,美好的愿望或理想。二,愿望或理想做寄托的对象。鲁迅曾经说过,“希望是附丽于存在的。有存在,便有希望。有希望,便是光明。”人活着不能没有希望,否则会像失去控制的小船,随波浮沉。若有了希望,便有了前进的动力,有了战胜困难的勇气,有了奋勇拼搏的力量。对于同学来说,“希望”可以是考试中取得理想的成绩,考上梦想的高中;对于父母来说,“希望”可以是家人的平安健康,工作的一帆风顺;对于医护工作者来说,“希望”可以是病患痊愈,疫情退散;对于国家来说,“希望”可以是国泰民安,繁荣富强。结构方面,一件事结构与多件事结构,均可。当同学们明确我们要写的“希望”是什么之后,我们就可以开始结合自己的学习生活,来展开叙述。成长感悟,人间真情和文化社会这几类都可以作为我们的立意角度。第二,选材构思。本题要求写成记叙文,可以写自己的希望是什么,也可以讲讲自己的希望是如何确立的,还可以讲一讲自己是如何为了自己的希望奋斗的。希望要具体,例如说希望自己期中考试成绩能比上一次进步,希望体育课跑800米的速度比上节课更快些,希望自己能够克服自卑心理变得乐观开朗起来等等。在写希望的过程中要加入具体的情节和细节,如写清楚自己为了体育测试都具体做了什么,自己又是如何克服自卑心理的。结尾可发出呼吁,呼吁大家认识到希望的重要性,从而用希望去创造更加美好辉煌的人生。题目二:本题考查话题作文。第一,审题立意。围绕“青春”这个话题,写一篇演讲稿,在五四青年节活动上演讲。审题可知,“青春”,寓意“善良”“勇敢”“无私”“无畏”“拼搏”“努力”“奉献”“责任”“奋斗”“梦想”,青春的时期,带有独特的气质,心里有火,眼里有光;心中有梦,执着追求;敢于拼搏、勇于承担。朝气蓬勃,充满希望,在努力中奋斗、拼搏,不断成长,不断进步,收获幸福。这是一段美丽的时光,是一段充满活力和希望的时光,是应该赞颂的时光。审读题干,针对材料之后的思考和感悟,叙写自己的经历,抒发自己的感情,发表自己的议论。也就是可以写青春时期的自己的故事,写青春的感悟,写对青春的看法。不管是从哪一个角度去写,主题必须积极、健康,必须展现青春的成长和幸福。第二,选材构思。可以写青春的美好,青春是人生最美好的时光,梦想是生命中最耀眼的阳光,为了梦想而奋斗的青春是最美好的风景。正是那千万颗在青春中奋斗的心,以及那千万双在天空中高飞的振翅,装扮着最美好最美丽最珍贵的人生季节,将青春变成人生最美好生活的出发点;再讲述当代中学生应如何度过自己的青春年华;最后号召同学们:青春正我在我们手中,梦想就在我们心中,命运就在我们手上,我们不能容忍青春在我们手中白白流逝,我们不能在叹息声中颓废了青春,我们不能在叹息声中了结生命,那么就让我们在青春时节奋发吧!选择好适合的材料后,行文时还要符合“写一篇演讲稿”的要求,注意演讲稿由开头、主体、结尾三部分构成。首先提出观点,然后分析,最后深化主题,注意多用断句,能够调动听众的情绪,给听众深刻印象。格式要准确,内容和语言上也要有激励性,将记叙、议论、抒情巧妙融合,进行呼吁和号召,以达到演讲的目的。11.阅读下面材料,根据要求完成写作。1988年出生的王娇,研究生毕业后进入敦煌研究院,7年来一直从事敦煌石窟考古报告的编写工作;1987年出生的杨金礼,19岁来到莫高窟,成为一名壁画修复师;90后刘小同2014年加入,专注为莫高窟“画像”……如今的敦煌研究院拥有一支200余人、产学研一体的保护队伍,其中不乏80后、90后、95后。他们在青春年华来到敦煌、深深扎根,为“把莫高窟保护好,把敦煌文化传承好”而坚守、奉献。读了这段文字,你有怎样的思考?请你结合参加上述各项主题活动(“青年,如何学做一个纯粹的人”主题活动)的收获,围绕“青年,如何学做一个纯粹的人”这一话题,或写作记叙性的文章,去参加校刊征稿比赛,或写作议论性文稿,在班级中和同学分享你的观点。要求:题目自拟,立意自定,不少于600字。不得抄袭、套作,不能泄露个人信息。【答案】例文:我们要做一个纯粹的人人生,要为祖国奉献,做一个纯粹的人!人生,不应虚度。他们的人生,在于奉献,在于人民,在于祖国。他们的梦想不在于“留取丹心照汗青”,而是为祖国的腾飞贡献出自己的一份力量。他们,是大国的工匠,是科学的巨人。是他们铸就着通往美好未来的桥。人生,若不为祖国读书,为祖国奉献,为人民的未来奉献,那人生的意义在于哪呢?他,一辈子没有虚度,一生属于核潜艇,属于祖国。1970年12月26日,当我国第一艘核潜艇下水,奔向大海之际,在场的人无不热血沸腾,他更是喜极而泣。尽管青丝变白发,依旧铁马踏冰河。如今,我国第一艘核潜艇已经退役,但年逾九旬的他仍在“服役”,他就是黄旭华。2021年了,如今已隔51载,是什么让这位老人依然对核潜艇如此痴迷?我想,是为祖国奉献的那份心,那团火在黄旭华老人心中熊熊燃烧着,“烈士暮年,壮心不已”。与像黄老这样的老前辈们比起来,越来越多的新兴科技强人正在逐渐崛起。梁建英,这位女70后,将CRH380AL高速动车组创造了时速为486.1公里的世界铁路运营试验第一速。CRH380AL被誉为中国高端装备自主创新的典范产品,是中国高铁的“金名片”。看着眼前的一切,梁建英再也掩饰不了内心的喜悦,22年的沐风栉雨,砥砺前行,让中国高铁不但填补了国内产品和技术空白,更丰富了中国高铁的产品谱系。为国家奉献,在于人生的勇气,在于勇于探索的精神。在浩瀚的伶仃洋,一条雄伟壮阔的“跨海长虹”已经交付使用。这就是连接香港、珠海与澳门的超级工程一著名的港珠澳大桥,而这个被世界公认的“当今世界上最具挑战性的工程”被总工程师林鸣建设成功。为祖国做贡献力量需要中国梦,中国智慧。世界正在改变,中国也应该更加强大。这就需要新一批人才站出来,为这努力与奋斗!【解析】本题考查话题作文。第一,审题立意。材料阐述了杨金礼、刘小同等一群80后、90后、95后献身敦煌壁画保护、修复事业的故事。启发我们要表现“凡人匠心”,持之以恒,为国献身的奉献精神,一丝不苟的工匠精神。正是这无数的“凡人匠心”使得敦煌文化得以延续。本题要求围绕“青年,如何学做一个纯粹的人”这一话题来作文。“纯粹的人”指心无杂念,有理想有追求的人。材料中的杨金礼、刘小同等正是这样的人。我们应在作文中体现对工匠精神的敬意和对献身国家事业的礼赞。第二,构思选材。本文可以写记叙文也可以写议论文。如写议论文,可以以“人生,要为祖国奉献,做一个纯粹的人!”为中心论点,我们可以寻到许多与此有关的事例,袁隆平、邓稼先、钟南山、张定宇、陈薇等人事迹皆可作为事实论据论证观点。要做到观点明确,论据充分,论证合理,条理清晰。写记叙文,叙述要有条理,内容要具体,能体现人物的精神品质。另外要注意的是,我们在写作时,要注意详略得当,不能写成了流水账,要写出必要的故事情节,人物的情态、语言等,明确这件事情带给了我们什么样的感受,它让我们从中学到了什么。12.学校的阳光广播站开展专题播音,围绕“胜与败”这一话题给出以下三个题目,请你任选一个,写一篇作文,向广播站采编部门投稿。(1)请以“__________无胜败”为题,写一篇记叙文,分享你面对胜败时的往事和随感。(2)请你为收听广播的初三同学写一封信,勉励他们正确看待胜败,努力应对中考冲刺。(3)请你阅读下面材料,写一篇议论文,向收听演播的听众们谈谈你的思考和感悟。胜者有计划,败者有托词。胜者常说虽有困难,还是办得到;败者常说,虽然办得到,但太困难。要求:①如果选择题目(1),请先在横线上填写合适的内容,把题目补充完整,然后作文;如果选择题目(2)(3),请自拟标题作文。②作文应符合所选文体的基本特征。③不少于600字。④文中请回避与你有关的人名、校名、地名。【答案】例文(1):比赛无胜败今天是运动节的第二天,原以为我是最早到达邻水中学运动场的,但莫道人行早,更有早行人。此时,绿色的运动场上早已是座无虚席,人声鼎沸了。比赛马上就要开始了,比赛场上运动员们个个摩拳擦掌,跃跃欲试,看着他们精神抖擞的样子,谁能断言,下一个刘翔不从他们中诞生呢?各班的啦啦队员们也做好了准备,他们有的挥舞着自制的标语;有的喊着鼓舞的口号;有的喝着水,润着嗓子为接下来的鼓舞士气做着准备。男子400米比赛开始了,我班首先上场的是“小飞侠”张展翼。你别看他瘦巴巴的样子,脸上却挂着自信的微笑,一副志在必得的样子。我班的拉拉队员们高喊着:“张展翼,加油!张展翼,必胜!”。他向我们自信地挥挥手,然后站在起跑线后。“各就各位——”老师一声令下,六名运动员屈膝弯腰,双眼坚定地注视着前方,一声枪响,他们如离弦的箭一样直射而出,开始还是齐头并进、不分上下,他们你追我赶,互不相让。但是转眼间拉开了微弱的距离,渐渐地,张展翼更是以迅雷不及掩耳之势,把其他队员甩在了后面。此时,操场上加油声、尖叫声更是彼此起伏,震耳欲聋。近了,近了,他以绝对的优势获得了小组第一名。我和同学们围着我们心中的小英雄,脸上挂满了胜利的喜悦。这时,我看着那个落在最后的运动员正低着头,有气无力地走着,一副垂头丧气的样子,不禁想:比赛为什么这么残酷,必须要分出胜败呢?其实比赛有输赢,快乐无胜败,在比赛中我们学会了坚强,学会了勇敢,也学会了团队合作,更得到了无限的快乐,不是吗?例文(2):永不言弃,终能成功亲爱的同学们:你们好,临近中考,我有很多话想要对你们说。蝶在沉默了一冬之后,积蓄了全身的力量,终于把飞的梦想变成现实;依米花在沉默了五年之后,爆发了自己毕生的心血,终于把花的芬芳吐露给大家。坚定信念,放飞梦想,永不言弃,方可成功。“天将降大任于是人也,必先苦其心志,劳其筋骨,饿其体肤,空乏其身……”孙膑在忍受了挖去膝骨的酷刑之后,忍耐着、坚持着,执著地用心完成一部流传千古的兵书《孙膑兵法》。孙膑,有多少人为他的学富五车所折服,有多少人被他的智谋所吸引。他在沉默中靠着坚定的信念,把自己的梦想放飞到蓝天中。“天行健,君子以自强不息。”这句话曾鼓舞着多少仁人志士,又曾让多少失路之人重获新生。盲人作曲家阿炳,他在“几家欢乐几家愁”的奔波流浪中,感到灵魂被深深地震撼了,于是一曲《二泉映月》便在他的二胡中诞生了。那柔和的乐音,那真挚的情感,使每一个人为之感动。当小征泽尔指挥完之后,他激动地跪了下来,“此曲只应天上有,人间能有几回闻?”他的眼睛虽然失明了,但有一盏信念之灯在你的心中为他指航。在心灯的指引下,他把梦想放飞到音乐殿堂,开创出民族音乐的新天地。 怎么能忘记,诸葛亮在南阳躬耕的身影。他的才华使刘备三顾茅庐而不悔,他的潇洒使群儒百口莫辩,而他游刃有余。这是他勤奋的结晶。“酒香不怕巷子深”,“桃李不言,下自成蹊”,他明白他的苦心终究会得到回报,他只是在等待,等待他的梦想成为现实的那一天。三气周瑜,他潇洒自如;七擒孟获,他镇定沉着。在信念的支撑下,他创造了经天纬地的大业。树立信念,放飞梦想,你将能随它一起到达美丽的地方。树立信念,放飞梦想,你将会拥有一个更加灿烂的明天。让梦想飞翔,让信念开花。不怕失败,永不言弃,调整好心态,相信你们一定能在中考中取得好成绩!例文(3):面对失败失败对于我来说意味着两点:它像晴天里突如其来的霹雳,让我灰心气馁;又像贝壳中的棱石。已那刻薄尖锐的角,通过不断磨练让我们成为晶莹剔透的珍珠。但面对失败,最重要的是永不放弃。面对失败,大多数人的情绪都会跌多低谷而失落,但意志力不强的人往往会选择放弃,我认为这种做法是大错特错的,是你不敢面对失败而逃离现实的懦弱表现,而你在失败后要做的,就是用持之以恒的信念以循序渐进的方式找到成功的突破口,这样令你失败的盲点就会逐一突破。“伟大的尝试,即使失败了,也是壮美的。”从文森特·隆巴迪这一句话中我也总结出:每一次失败,要找找这次失败的原因,有了以往的经验后,再次尝试的几率就会变小,下一次就会更小,直到你登上成功的舞台。面对失败,不要皱起眉头,更不要因为失败而出现挫败感,这是你对于问题置之不理的源头,要把失败看作是一件司空见惯的事,甚至为每一次的失败而欢呼雀跃并保持乐观的心态,俗话说的好:“失败是成功之母”。而每一次的失败就是为你通往成功搭上一层又一层的阶梯。对于失败,最好的代表人物就是“发明大王”爱迪生。爱迪生一生为人类科技发展作出了无人能比的伟大贡献,有着至高无上的荣耀,可他为他的辉煌事迹付出的代价,就是那如星星般繁多的失败。他曾在感慨万千中说了这样一句话:“失败也是我需要的,它和成功对我一样有价值。”这句话也使我受益匪浅,是啊,我们的生活何尝不需要失败呢?没有失败又何来成功呢?可以说失败更加丰富了我们的世界,在我们的人生也留着一道叫做“失败”的印记。可失败又是什么?是你快要到达终点时让你回到原点的坏消息,坚强的人凭着自己最初的理念,一次又一次的对自己说:“重来!”,最后他欣赏到的将是汇聚努力和毅力的彩虹;而脆弱的人,则会选择半途而废,这将使他之前的付出的汗水全部化为乌有。总而言之,失败是我们成长中必须的重大打击,是一株幼苗必经的倾盆大雨,是一艘小船在海上行驶必遇的大风大浪,是启发我们“吃一堑,长一智”的老师!所以我们要学会接受失败,更要勇敢的面对失败!【解析】(1)考查半命题作文。第一,审题立意。要求写一篇记叙文,分享你面对胜败时的往事和随感。首先要求补全题目。“无胜败”的意思就是没有胜利和失败,因为本文要求写成记叙文,所以可以理解为通过某件事,让自己明白了,胜利和失败不是最重要的,从事件中明白了……的道理,这才是最重要的,指让我们不要太注重于结果。据此横线上可填写:比赛、人生、学习、青春等词等等。第二,选材构思。例如可以填写“友谊”,“友谊无胜败”,可以叙述自己和同学朋友相处的点滴日常,有快乐的,也有冲突的,最后得出:友谊像金,但是友谊比金子还宝贵。友谊是无价的,毫无胜败可言,有的只是一片赤诚之心;还可以填写“学习”,“学习无胜败”,记录自己考试失败的一次经历,没有因为一次的失败就放弃努力,而是从中总结错题,吸取教训,后来终于又取得了下一次考试的成功,最后得出:考试无胜败,每一次考试都是对自己学习阶段性成果的验收,取得好成绩不应该骄傲自满,考得差也不应该沮丧放弃,而是要继续努力,获得下一次的“成功”。(2)考查应用文的写作。第一,审题立意。要求为收听广播的初三同学写一封信,勉励他们正确看待胜败,努力应对中考冲刺。在中考来临之际,很多初三同学可能因为成绩总是没有提高而感到灰心丧气,心灰意冷,想要放弃。所以可以以“永不言弃,终能成功”为主题来写作,意思是要想获得成功必须付出坚持不懈的努力;有不服输的意志、坚定的信念,理性看待成败,调整好心态才能取得最后的成功……第二,选材构思。我们可以从中选择一个主题进行选材构思。可以从生活中选材,以生活中的人物或者历史上的名人轶事来论述主题。开篇可以指出现在的初三学生们存在的问题:过于看重胜败,对中考没有信心等。接下来通过运用道理论据、事实论据来论证自己的观点,如:富兰克林的风筝实验,“天电”被引下来,总结经验后后他又证明了“天电”和地电一样能被金属传导,能融化金属,能点燃酒精。还有郭守敬善于总结前人的经验和教训,勇于探索创新和突破困难,为元朝治水,建立水运站,还参与制作《历法》的工作,为当时元朝做出了杰出的贡献等等。在列举事例时可列举古今中外的名人事例,增强文章的论证力度。证明之前的三年学习都是经验的积累,失败后总结经验和教训才能更好的取得成功。最后祝福初三学子们调整心态,在中考中取得好成绩,注意书信的格式。(3)考查材料作文。第一,审题立意。材料“胜者有计划,败者有托词。胜者常说虽有困难,还是办得到;败者常说,虽然办得到,但太困难”,说明, 成功的人找方法,失败的人找借口;要成功就没有借口,要借口就不可能会成功。面对失败时,不要为自己找借口,而是应该勇敢面对自己的失败,接受自己的失败。面对失败的态度决定了你的结果。有些人不会被失败打倒,而是继续努力,再度挑战,这是一种“败不馁”的顽强与坚韧的精神品格;但也会有人认为尝试过就不算失败,对“失败”的理解有个人的定义,只要付出了就是成功,只要挑战了就有收获,人生没有绝对的失败,这是一种乐观豁达的人生态度。两种看法都体现出积极的人生观,无须厚此薄彼。第二,选材构思。本次作文在于引导学生思考,人应当以什么样的态度面对失败挫折,以什么样的人生态度来生活。这样的作文其实很容易写成“心灵鸡汤”式的文章,喊喊口号。其实谁都知道应当以乐观的态度生活,但是生活其实很复杂,有时候并不是说到就能做到的,并且也不是所有的情况都适合“乐观”。考生应当深入思考,人在面对不同情况时该以什么样的态度来面对。这样才能把文章写得全面而深刻。写作须注意,不要陷入“失败是成功之母”的陈词滥调,作文不妨以新时期社会主义核心价值观作为思想指引,体现出新时代青年积极向上、朝气蓬勃、不畏艰难、笑对挫折的精神面貌。13.阅读下面的材料,根据要求写作。因为对光的信仰,勇毅前行的心中,充满暖阳;因为有风的鼓舞,磨砺与坚韧,撑起翱翔的翅膀;因为有前路的召唤,踔厉奋发的里程光辉闪亮。请你根据阅读后的联想、感悟或思考,自拟题目,写一篇不少于600字的文章。要求:①立意需在材料包含范围内;②除诗歌外,文体不限,文体特征鲜明;③不得抄袭或套作;④文中不得出现真实的校名、人名等信息。【答案】例文:追光追吧,追逐那光,所及之处,春色满园。——题记在我最烦躁,最迷茫的那段时间,我得到了一把种子,是一位老人给我的。我在一个阴雨天中将它们随意地种在我家楼下,抬头看着阴暗的天。它都不会冒芽吧,这天气,连光都没有。种下了就是种下了,我认为,最近的天气都十分不好,我也只是随便把种子按在了土里,不会长出什么东西来,于是也没注意过他们会长成什么样子。直到有一天,我照常出门上学,下楼后余光好像看到了点点星光摇曳——是我之前随便种下的那些花!我怀着几分惊讶走到那几朵花面前,蹲下。细细看着,是我叫不上名字的花,长得也没有其他花那样惊艳世人,可它黄色的裙摆,粉红的脸颊,朴素而平凡,我却从中道出了这花的不同。前几天阴天,雨天,黑夜轮流转,人们几乎两个星期没有见过早晨的阳光了,生活迷茫,又闷又躁,朋友家家养的花早就不知道蔫了多少盆了。可这些花却依旧从不见天日的土壤中破土而出,追着心目中的阳光,努力绽放。在别人都厌倦这种日复一日的生活时,它却依然矗立在这,心存希望,挺起胸膛,仿佛要拨开云层,伸出枝叶迎接光。静静地看着这朵依然摇曳的花,好像心中的迷雾散去了些,虽然天气依旧不好,但我似乎感觉有光,照进心田,滋润着万物。默默地想着,我们这种平凡的普通人,即使生活枯燥而又迷茫,可谁的生活是一帆风顺,阳光充盈的呢?我们的生活并不因为没有了阳光而失去了任何希望啊。我们总得自己去追逐属于自己个人的那束光吧。就像现在比较流行的话一样“等风来,不如追风去”。不要担心前路迷茫,看不见光就停下脚步,原地叹气。我们其实都只是一个在自己朴素的世界中,默默追光的人罢了。追吧,追逐那光,所及之处,总会春色满园。我与这花同行,愿你我都能向阳而生。【解析】本题考查材料作文。第一,审题立意。“因为对光的信仰,勇毅前行的心中,充满暖阳”寓意心中有梦想,有追求;“因为有风的鼓舞,磨砺与坚韧,撑起翱翔的翅膀”寓意遇到挫折和磨难,我们需要坚韧,才能成就未来;“因为有前路的召唤,绰厉奋发的里程光辉闪亮”寓意前方有路,有理想,与目标。结合三句话,我们可以围绕“梦想”“理想”以及“如何实现梦想”等话题进行写作。立意上,要展现出青少年学生积极奋发的精神状态和对梦想执着不懈的追求。第二,构思选材。如拟题为“追光”,可以写追逐个人梦想,可以谈对追梦的感悟,写“追光”的意义,如分析是否要“追光”,“追光”与“不追光”有什么不同;也可以写要做什么样的追光少年,追什么样的梦;还可以写如何追逐“光”。这束“光”可以是一本好书,写一本书指引“我”走出迷悯;这束光可以是父母的言传身教,给“我”心灵的熏陶;这束光可以是同学的鼓励,给“我”失落时的安慰;可以是逆行者无私奉献的精神,给群众以希望……本文可以以写人记事为主,要有具体的事例,选择自己印象最深刻的事,例如面对考试失利,没有一蹶不振,而是更加努力奋斗,朝着自己的目标永不放弃前进的脚步;又如在疫情到来之时,各行各业的人迎难而上,积极配合防疫工作等等。重点表现某人某物某场面对“我”产生的积极影响。此外,选取典型事件,能够表现出自己“追光”过程所展现的精神或感悟,记叙描写和抒情等表达方式可综合运用,使文章从不同角度、不同侧面突出文章主旨。14.根据要求作文。汪曾祺老先生曾说:“生活,是很好玩的。”“我想把生活中真实的东西、美好的东西、人的美、人的诗意告诉人们,使人们的心灵得到滋润,增强对生活的信心,信念。”根据以上内容,请你自拟题目,写一篇不少于500字的文章,要求内容具体充实,文体不限。(不能透露与考生有关的信息)【答案】例文:我的青春回忆 回忆总是向那调皮的色彩斑斓的蝴蝶,在空中舞一场翩飞的梦,又任性的消逝在时间的转轴里,再在不经意间闯进你世界。——题记光阴似箭,岁月如梭。那个曾令我挥洒热泪的六月已悄然离去,直到现在仍是不能相信我们一起奋斗流汗流泪的日子就这样在六月分离的季节结束了,那是怎样的岁月啊,那是怎样的青春啊!那些天还没亮就在被窝打着小电筒背书,那些捧着英语课本大步流星的到那刚睡醒的太阳的小花坛儿肆无忌惮的大声朗读,那些为了补上缺腿的科目一下课就缠着老师给解题,那些每晚都要老师赶着我们才抱着书冲回宿舍……现在这样平静的笑笑过去的自己。然而那些心情却是刻苦铭心的在生命中记下了厚厚的痕迹,光是泪水与汗水飘飞的时光,就足以令人一辈子铭记。那些日夜我们都在做着怎么样美丽的梦呢?相信大家初中大部分人都经历过,因为害怕,以至于一次次失败而又不得不站起来;因为害怕,以至于一下课就缠着老师解题;因为害怕,以至于不敢比别人早一丁点回宿舍;因为害怕,以至于一个人躲在房间里偷偷地擦泪;因为害怕,以至于成绩出来了也不敢去问;都是只有一个目的,中考!当这场仗打完了,心里却是一阵惆怅一阵欢喜。就在几个月前我仍为了中考而胆颤心惊,今天我却坐在高中的教室里看云淡风轻,望着老师在讲台上手中的粉笔在黑板上刷刷的笔画着,是的,我踏进了高中的门槛,这是我人生的第一站。我不能说我是一个骄傲的胜利者。或许没有进入自己心目中神圣的高中是我人生的一大遗憾。但今天起得每一天都把握在我的手里,我还有机会尽自己最大的力量为下一个目标而奋斗。高中这不是终点,是又一个起点,更大的考验在后面等着我,美丽的大学梦在今天走进了我的心里,我看到大学在向我招手。回忆三年,这样过的也值,回想时不会有太多的愁,红颜挥空,青春易逝。莫等闲,白了少年头,空悲切,切勿浪费了这美好的青春,举起旗帜,朝着前线进军。等到胜利的那天就尽情的任回忆翻飞,任泪花四溅。【解析】本题考查材料作文。第一,审题立意:分析材料可知,“生活,是很好玩的”“我想把生活中真实的东西、美好的东西、人的美、人的诗意告诉人们,使人们的心灵得到滋润,增强对生活的信心,信念”,汪曾祺的话强调了写作的目的,就是要把生活中那此真实美好的东西告诉读者,从而让读者心灵得到滋润,进而增强对生活的信心。写作内容应该是关于生活中美好的事物,可以是一处景物,可以是某个品质高尚的人物,也可以是一幅温暖的画面……总之,要突出美好的一面。例如,可以写美好的景物,也可以写景物背后的故事,这个景物带给你的记忆,是否有景物的美、有滋味的美、有人情的美,写出自己对这个景物的记忆,自己与这个景物之间的故事,也可以通过具体事件,运用各种描写方法突出人物的某种美好的品质;还可以刻画生活中感动我们的某个画面或某个暖间,让我们感受到美好等等。第二,选材构思:可以创作记叙文,从自己真实生活中选取事件与人物,立足于某一种感情,表达自己的真情实感。亲情的温暖,友情的抚慰,师恩的无私,陌生人的善良都可以作为写作的出发点。以具体的事件来刻画人物形象,以生动的描写来抒发自己的真情。可以创作议论文,以自己的某一种看法或观点作为中心论点,立足于中学生生活与学习实际,引经据典,运用富有逻辑的论证来证明这一观点的正确,做到有理有据,以理服人。15.挑战无处不在,挑战无时不在。我们该如何正确看待它的存在?如何做出正确的反应?请自选角度,自拟题目,写一篇简单的议论文,表达你的思考。要求:①观点明确,有理有据。②600字左右。③文中请回避与你有关的人名、校名、地名。【答案】例文:直面挑战漫漫人生路上,如果把生活比作是一汪大海,那么挑战就是浅滩的恼人沙石。如果把生活比作一座高山,那么挑战就是阻挠山路的荆棘,但当我们正面它们时,即便伤痕累累,道路坎坷,大胆向前迈进,我们就已成功。在刚刚结束的CBA半决赛北京与山西的对决中,来自NBA的“独狼”马布里用出色的发挥将赛季前无人看好的北京金隅不仅首次带入了总决赛,并最终捧起第一座属于北京的冠军奖杯。赛后,马布里在更衣室彻底爆发,放声痛哭。面对一个个挑战,甚至是恐吓,马布里没有逃避,更没有畏惧,在赛场上用上佳的表现将此前的恩怨踩在脚下,回应了挑战,不仅让对手敬服,也让自己心中的怒气得到发泄,直面挑战,体现的是勇气、气概,更是一种霸气,而躲避挑战,怯于勇敢面对现实,则只会让人生暗淡,就此沉沦。同为篮球运动员,丹特琼斯大学时代就被无数荣誉、头衔所包围,眼看众人艳羡的状元已经到手,他却在一次训练赛中被一名名不见经传的高中生彻底打爆,输掉了比赛,本来一件小事,却被好事媒体大肆宣扬渲染,而琼斯没有了勇气去直面媒体的质疑,没有用行动去证明自己,整日酗酒度日,沉沦于酒精之中,迷失自我,断送了大好前程,而成就的高中生就是率湖人队五夺总冠军的科比。一味逃避,只能躲一时风声,掩耳盗铃,更只能是自欺欺人,给人留下笑柄。纵观社会各界,其实还有许多直面困难的典范,李小龙在被外国人一致视为东亚病夫后,在失地奥兰多勇夺全美搏击大赛冠军,并进军好莱坞,创造了辉煌,直面挫折,勇敢面对,打出的是中国人的骨气,击碎的是东亚病夫的形象!直面挑战,无论何时何地,用勇气面对,以实力回应,扎痛过后我们就会享受到海的柔和,荆棘拨开,我们就会领略山的美景,纵览风景之时,直面挑战的磨练的痛苦已不再那么刻骨,只有勇气扎根于心,指引我们到达成功的彼岸,去有所发现,有所收获。【解析】考查材料作文写作。第一,审题。阅读材料可知,生活中的挑战无处不在。挑战对于我们的人生有着重要的意义。题目要求写一篇议论文,我们就要围绕“挑战的内涵与意义”来展开。如:挑战是对已经存在的某种状况的不满,是对某种理想境界的追求,是一步步向完美靠近。中考也是人生征途中重要的挑战,可以写中考对我们未来的重要影响。第二,立意。我们可以从小的方面立意:挑战自己,我们才能不断成长;也可以从大的方面来立意:迎接挑战,人类才能不断进步。如:挑战是人类不断进取的动力之一。迎接挑战,中华民族创造了遨游太空的美丽神话,迎接挑战,全国上下团结一心实现了战胜疫情的伟大壮举,这两个事例可以作为有力的道理论据。第三,选材。人生路上充满了挑战,挑战首先是挑战自己。我们可以先列举学习、生活中,挑战自己的种种表现,如改变不恰当的行为习惯,学会承担责任等;然后阐述挑战自己的意义,如:挑战,是对我们学习的考验;挑战,是我们锻炼意志的机会;挑战,是我们人生勇敢面对的宣言;最后进行总结:当挑战来临时,要勇敢面对,勇于接受,这样才能成就优秀的自己。16.阅读下面的材料写作。当老骆驼穿越塔里木沙漠,到达罗布泊旁的草原上休息的时候,一直在草原上转的黄羊们都围拢过来。黄羊们说:“老骆驼, 你跑得并不比我们快,蹦的也不比我们高,塔里木沙漠望不到边,你是怎么走过来的呢?”“那很简单,”老骆驼说,“只要认定一个目标,一步一步地走就成了。关键在于你不要停顿。”请根据上述材料,自选角度,自主立意,自拟题目,写一篇文章。要求:文体不限(诗歌除外);文中不得出现真实的人名、校名、地名;不少于600字。【答案】例文:成功贵在坚持梅花若不经过彻骨的寒冷,又何来满园扑鼻香?钢铁若不经过烈火的锻造,又何来那无比尖锐的宝剑?河蚌若不经过沙粒的磨练,又何来那绚丽的珍珠,这一切无一不告诉我们成功贵在坚持。“冰冻三尺,非一日之寒”,无论做什么事都不能随随便便成功,当我们遇到挫折时,唯有像梅花一样巍然矗立在寒冬中,坚持到底,才能战胜挫折,赢得胜利。爱迪生因为发明电灯而闻名于世,而他发明电灯的过程是十分艰辛的。爱迪生为了寻找耐用的灯丝做了上千次试验,但都失败了。有人劝他放弃,但他坚持说“我失败了一千次,但说不定第一千零一次就成功了。”他的确做到了,大约试用了6000多种纤维材料,在经历了六千次失败后,终于找到了一种耐用的灯丝。爱迪生如果没有这种坚持不懈的精神,恐怕我们还在用煤油灯照明呢!一个人尽管天资一般。只要有“坚持”的优秀品质,则必出成果。相反,一个人天资再高,如果不坚持努力,绝干不成大事业。方仲永富有天赋,有着超人的智慧,但却不坚持学习,其行为扼杀了他原有的闪光的天赋,最终导致“泯然众人矣”。如果没有坚持,哪能有水滴石穿,如果没有坚持,愚钝的乌龟怎么能跑过活蹦乱跳的兔子,如果没有坚持,哪来的千年磨一剑。但是坚持也不是盲目的,要学会分是非,不要像一个莽夫,错了还冥顽不灵,不撞南墙不回头,所以,我们应学会判断,明辨是非。坚持是成功的基础,是事业的奠基石,只有坚持不懈才能取得成功。【解析】本题考查写作材料作文。第一,审题立意。细读题目中给出的材料,可根据老骆驼说的话“只要认定一个目标,一步一步地走就成了。关键在于你不要停顿”从中提炼出本次作文应围绕“坚持”这一话题来展开,坚持是一种态度,一种对于自己理想的坚持和对命运的不妥协。在生活中,没有人会一帆风顺,总是会遭遇一些挫折,这时候,我们就需要坚持,坚持心中的理想和目标不动摇。根据老骆驼说的话“那很简单”,可以提炼出“简单造就不平凡”的主题。故可选立意为:认准目标,坚持就能成功;坚持精神很重要;简单孕育不平凡。第二,选材构思。可以写成记叙文,记叙自己练习长跑,四季不停,寒暑不怯,终于获得区五千米长跑第一名,启示自己坚持就能实现目标。也可以写成议论文。突出“坚持的重要性”,先提出要有坚持的精神,用名言“泰山不让土壤,故能成其大;河海不择细流,故能就其深”论证坚持的重要。再列举马克思呕心沥血四十年写成《资本论》,李时珍用了大约27年的时间才编写完成《本草纲目》,经过了三次改写,于万历六年才最终完成的事例,论证坚持才能成功的道理。绳锯木断,滴水石穿,都印证了坚持就能成功的道理。最后得出结论:目标专一,坚持不懈,终能成功。17.根据下面的语境,展开联想或想象,按要求将语段扩写成一篇文章。……16岁的我,茫然地走在小路上…………一个人总要走陌生的路,看陌生的风景,听陌生的歌。心持勇气,怀揣梦想,我将继续前行!要求:①自拟题目;②字迹工整,书写规范,不少于600字;③文中不得出现真实的人名、地名、校名。【答案】例文:怀揣梦想,笃志前行这里刚刚刮过一场台风,田野里倒伏的甘蔗就像一块块碧绿的地毯。香蕉树被台风齐刷刷地拦腰截断,那些还没成熟的香蕉无人问津。路两旁的电线杆像醉汉似的东倒西歪。路上一片狼藉,横七竖八地落满残枝败叶。16岁的我,茫然地走在小路上,匆匆赶往学校。昨天发生的一幕又浮现在我眼前。“发数学试卷啦!”班里一阵喧闹,大家都迫不及待地想知道自己的考试成绩。可我心里惴惴不安,真希望自己的试卷不翼而飞。最担心的事情发生了:我的数学成绩没有及格。当数学科代表把试卷交给我时,他瞟了一眼,只见他瞪圆了双目,失望而又鄙夷地走开了。那眼神真让人不寒而栗。接着,没想到的是,那张令我难堪的试卷竟如长了翅膀般“飞遍”了全班!大家都知道了我的考试成绩,他们七嘴八舌地嘀咕着,明目张胆地谈论着……一支支利箭从四面八方刺穿了我的心,我心如刀绞。放学回家后,妈妈和蔼地问我:“最近学习怎么样啊?”我听了,大声地哭了起来,眼里闪着委屈与酸楚的泪花,告诉了妈妈我的数学考试成绩。妈妈听后,沉默了一会儿,然后语重心长地对我说:“孩子,你也长大了。要知道,考试就像打仗一样,不会场场都赢的。这么一点点小挫折就把你打倒了,那以后可怎么办呢?妈妈总不能一直陪着你。面对挫折,你更应该努力地微笑啊!”妈妈的话语深深地烙在了我的心中。是啊!没有哪一个人的人生道路是一帆风顺的,只有怀揣梦想,笃志前行,幸运之神才会垂青于你。这时,几声鸟鸣划过天空,把我从昨天的回忆中拉回来。我看到台风过处满目疮痍的大地上,柔弱的小草仍然顽强地挺立着,而且,经过暴风雨的洗礼,它们显得更加青翠、更加精神。我不禁感叹道:“连小草都有这样顽强的品质,更何况我们人呢?”想到这里,我不再犹豫,加快脚步向学校走去。一个人总要走陌生的路,看陌生的风景,听陌生的歌。心持勇气,怀揣梦想,我将继续前行!【解析】本题考查情景作文。第一,审题。这是一篇形式特殊的作文,给出中间和结尾的两段文字,然后按要求将语段扩写成一篇文章,写作难度较大,很多同学会一下没有了思路,陷入思维的困境。仔细阅读所给段落可知,其表现的主题应包括人生规划、成长感悟、关注自我等方面。第二,立意。而扩写点主要有两处:一、由“16岁的我,茫然地走在小路上”可知,要写出“我”“茫然”的原因或背景;二、由“一个人总要走陌生的路,看陌生的风景,听陌生的歌。心持勇气,怀揣梦想,我将继续前行”可知,要写出“我”在“小路上”的见闻和感受(包括人、景、事、物)。第三,选材。写作时要展开联想或想象,选择自己体验深刻的立意点进行构思。可以通过叙述人和事引发对成长的思考;也可以由景和物展开联想,获得人生启迪……总之,前面可以补充环境、背景、原因,中间可以叙述见闻、抒发情感、展开情节。18.阅读下面材料,按要求作文。邓清明,目前唯一一位没有执行过飞天任务,又仍在现役的首批宇航员,二十多年来,他一丝不苟地做着“备份”,虽然没能飞入太空。但坚持完成与“主份”相同难度的训练。“无论主份还是备份,都是航天员的本分”,一句话道出了邓清明的职业操守、责任担当。当今中国在各方面的成就举世瞩目,每一项成就都离不开无数个“邓清明”的默默付出。正所谓“功成不必在我,功成必定有我”,谁说英雄一定要身披金甲、光芒万丈?如邓清明这样悄悄发出光和热的人,都是值得崇敬的英雄。阅读以上材料,你有哪些认识和感悟?筑梦终会圆梦,邓清明终于踏上了“飞天”的征途,去探索神秘的太空。请发挥你的想象,写一篇记叙文,抒写他的见闻和感受。要求:书写工整,卷面整洁,字数不少于600字:不得涉及个人相关信息,凡涉及与考生相关的真实地名、校名、人名等,请用xxx代替:不得抄袭试卷中的材料及他人作品。【答案】例文:我的太空之行我是邓清明,这一次,我终于坐着神舟第N号飞船踏上了“飞天”的征途,去探索神秘的太空。这一天,工作人员为我们戴好防护面罩,穿上宇航服,检查好所有设备后,我们就登上我国最新研制的“光速号”宇宙飞船。登上宇宙飞船后,我发现飞船十分宽敞、舒适。我找好自己的位置,躺了下来。没过多久,指挥部开始倒计时:“十、九、八、七、六、五、四、三、二、一!点火!起飞!”随后,我便感觉到有一股强大的推力让我离开地面,我向外面望去,所有的事物都迅速地向后掠过。大山不见了,白云不见了,“光速号”以极快的速度推离地面,直指苍穹。也就过了几分钟时间,蓝天从我眼前消失了,取而代之的是一阵刺眼的光芒,它将周围黑色的天都给照成了金色。渐渐地,“光速号”的速度变得缓慢了。我走出舱门,结果一不小心,在太空中连翻几个跟头。令我惊奇的是,这深不见底的黑暗,竟没有让我掉下去,而是稳稳地飘在空中。在这里,我体验到飞翔的畅快感,别提有多激动!我回头望去,有一颗蔚蓝的星球出现在我的眼前,这就是地球。在这颗巨大的“蓝宝石”中间,镶嵌着一块块绿岛。她是多么的美丽和伟大!我真不希望她的美丽被我们毁掉,最终变成一个荒芜贫瘠的星球。临返回之际,我拿出照相机,给地球、“光速号”和自己照了张合影,当作是我太空之行的纪念。飞船开动了,我望着这片黑暗,心想:这宇宙中到底还藏着多少未知的秘密呢?我一定要揭开它神秘的面纱!近了,近了,离地球越来越近了,白云出现了,大山出现了,地上的人们也出现了,我回来了,地球!【解析】本题考查材料作文。第一,审题立意。材料的主要内容是曾经的“备份航天员”邓清明的故事。自1998年就加入的首批现役航天员中唯一个没有执行过飞天任务的人——邓清明,在备份岗位上坚持和奋斗了25年之后,终于奔赴穹苍,迎来了属于自己的“主份”。在邓清明的这段人生经历中,我们可以看到主分与备份的奇妙转变:正是源于对祖国的热爱,对航天事业的忠诚,对自己事业的信念,才使得邓清明不计较自己的“备份”身份,仍然努力训练,以“主份”的标准要求自己。正是因为高度的信念感、使命感与责任感,以及默默奉献、不求名利的优秀品质,他才能坚持训练,无怨无悔,最终成为“主份”。从某种程度上说,“备份”甚至比走到了聚光灯下的“主份”更令人敬仰。也只有默默做好“备份”,才能够在祖国和事业需要你的时候,像邓清明那样“一飞冲天”“一鸣惊人”,成为“主份”。同时,在邓清明从备份走到主份的路上,我们应该认识到,正是一路以来的坚守和奋斗,才让他坚持到最后,实现了突破。第二,思路点拨。写作时,尽量打开写作思路。如果是航天员邓清明,终于踏上了“飞天”的征途,去探索神秘的太空,会有一场怎样的奇妙经历呢?请发挥联想和想象,抒写他的见闻和感受。我们以“我的太空之旅”为题,写一篇记叙文。注意要结合“筑梦终会圆梦”写作,我们可以想象如果有一天邓清明终于乘坐载人飞船驶入太空,开启一段探索未知之旅,会发生什么故事呢?发挥合理地想象,想象他是怎样上去太空的,在太空看到了什么,有什么有趣的发现,他们都干了些什么事情,最后结果如何?充分写出这次探索过程中的体验和收获。19.题目:传说,遇见五叶草就能默许愿望实现心愿。假如你得到了一片五叶草,可以实现自己的一个愿望,你会在什么情况下使用?使用后会发生什么事情?请以“太高兴了,我拥有了一片五叶草……”为开头,展开丰富的想象,自拟题目作文。要求:①文体不限,诗歌除外;②不得抄袭、套作;③书写工整,卷面整洁;④不少于600字;⑤不得泄露个人相关信息,如有必要,请用××代替。【答案】例文:神奇的宇宙探索之旅太高兴了,我拥有了一片五叶草……据说得到一片五叶草,可以实现自己的一个愿望,我就许下了乘坐太空飞船探索宇宙的心愿。睁开眼睛,我开着飞船,已经穿过了大气层,看到了茫茫的宇宙。我第一个要探索的是水星。我来到虫洞旁边,穿过了虫洞就看到了水星。大家谁都知道水星上面没有水源,不过据我观察,当我们人类还没有出生的时候水星上面应该有水源,后来不知道什么原因把水给变没了,我采了一些沙石和水星上面的东西,就准备去下一颗星球了。我要探索的第二个行星是金星,我找了半天,都没有找到虫洞,只能用虫洞制造机来制虫洞了。我穿过了虫洞,看到了金星,我开着飞船进去了金星里面,外面都是沙子,没有氧气,而且指南针也不受控制。我穿上了宇航服就出去采一些沙子,这些沙子很特殊,也不知道是不是沙子,随便叫一个名字就行了。我坐上了飞船,准备去下一个行星。在这里你们应该有一个小疑问吧?为什么不是从金星再到水星?这是因为我利用虫洞来穿梭就可以按九大行星的顺序来排列,当然,地球就不用去啦!我正在准备去火星的时候,看到了一个奇怪的东西从我面前飞过去,我急忙去追,可他的速度太快了,我追不上,只好返回火星了。我登陆在火星上了,这里面有好多的洞洞,我找了其中一个跳进去看,我发现了一根骨头。难道火星有生物吗?不可能呀!我把这块骨头收起来,想回去做进一步的研究,可到了飞船上这块骨头就不翼而飞,我也不知道怎么回事。我启动飞船,准备向下一颗行星出发。我要去探索木星了,木星是太阳系最大的行星,我不知道里面有什么东西,我穿过了虫洞,“不”刚好到了黑洞旁边,“救命”SOS,我忘记这是太空,没有人。我把飞船动力调到最大,我开了所有可以让飞船前进的开关,把所有动力调到最大,终于脱离了黑洞的引力,可是没油了,我出去加油,站在飞船的上面,我把油加满了以后,回到了飞船的里面,向木星出发。我到了木星,我刚要进去,就被里面的一股风给吹了回来,我怎么样都进不去,只好不开心的离开了木星,去下一颗行星。我飞到了土星,被土星的光环所迷住,真是太漂亮了,这是用小岩石和大岩石组成的,实在是太美丽了,我从来没有看过这么美丽的地方。我进去里面采了一些沙石就离开了土星,因为里面气温好低,要冷死我了,我做准备去下一颗行星了。我来到了天王星,不过有一些巨大的石头来攻击我,我闪、闪、闪,因为太冷了,飞船都快结冰了,我马上离开了这里,在我走的时候,飞船上有一点残冰,我把他刮下来,做成了天王星的标本,我真是太聪明了。由于海王星地表很冷,我的飞船下不去,所以,我在里面安了一个标本采集器,它会把标本采来给我,还有一些照片。因为我的飞船没油了,灯开不了,又太黑了,所以我就不去探索最后一个行星了。我这次的太阳系之旅到此结束了,我下次要来一个外星人大寻找,解开外星人之迷,我要回地球了,很快我还会再来的——宇宙!【解析】本题考查情景作文。第一,审题立意。以“太高兴了,我拥有了一片五叶草……”为开头,结合“假如你得到了一片五叶草,可以实现自己的一个愿望,你会在什么情况下使用?使用后会发生什么事情?”可知,我们可以以五叶草实现我们了某个心愿进行写作。在这篇文章里所有的一切都是因为想象而存在,想得越多,这个世界就越丰富多彩,反之,如果你的想象力贫乏的话,这个世界将会黯淡无光。当然,所有的想象从某个角度看,都是现实生活的折射,都离不开现实生活。只有一定现实依据的想象才是能够被大多数人所接受的。所以我们要在自己积累的科学方面的知识的基础上进行大胆合理的假设和创新。第二,构思选材。如写自己对着五叶草许愿——可以进行宇宙探索之旅。在到达目的地后,本来想象中的一片荒芜,没想到却出乎意料,你见到了一种神奇的生物,这种生物长相奇特,并且带你了解了这里的环境,之后你依依不舍地与它告别。再如,在未来的航天生活中,你经历了一场劫难,特殊恶劣的宇宙环境给了你一个巨大的考验,但是依靠自己过硬的本领和知识,成功的脱险,并发誓今后一定会再次造访,并征服它等等。本文是想象类作文,是记叙文。要通过具体的事件来表达自己的感情或情感。可以全文叙述一件事,一线串珠,也可以采用片段组合式,通过几个片段来共同展现某一个主题。五叶草是一个引子,引出一些思考和感受,更多的笔墨还是要集中在对具体生活的描摹上。20.请以“回身,看向那扇窗,我不会忘记……”为开头,写一篇文章。要求:①题目自拟,除诗歌外,文体不限;②可以记叙经历、抒发感情、发表见解等;③字迹工整,书写规范,字数600~800字;④文中请不要出现真实的姓名、校名。【答案】例文打开那扇窗回身,看向那扇窗,我不会忘记渴望自由的那只鸟…… 那是前年的立秋,天地略显苍茫,秋叶在枯黄且尽丧生机的空气中不停地打转。夜色渐临,我僵硬的右手松开了笔杆,抬起昏昏欲睡的头颅,窗外皎洁的月光映在桌上,流连在我的脸庞上,交织着凄凉,孤寂。 夜色的笼罩下,一切悄无声息,忽然一道灰影从上方掠过,未着痕迹。但转眼间就落到窗外的空调外箱上,它身下的那一片皎洁把它的身躯映衬得格外清晰。我着实被一惊,仔细一看,这是一只灰黑斑纹的鸟儿,一边的翅膀挣扎着,另一边呈张开状而且一动不动。见状,我连忙打开窗户,犀利的寒风刮在脸上,却也顾不得这么多了。我把它小心翼翼地捧在两手合并的手心中。它左翼内侧已渗透出了血丝,一对眸子充满着惶恐与不安。一些简单的包扎之后,我轻轻地把它放进久置多年的鸟笼中,就在我关上鸟窗的那一刻,发现它在颤抖,颤抖这扇窗的阻碍,我却只是以为天气寒冷。在放好水和食物后,揣摩着它的一切,抱着好奇的玩味走了。 随即,灯关上了,只留下黑暗中望着那扇小窗颤抖的身影躲在鸟笼的角落里…… 当我醒来时已是太阳高照,伸着懒腰走到书房,眼前一幕却令我惊呆至极,鸟笼还在窗旁挂着,竟已生机全无,它静静地卧在角落里,头颅向着前面的小窗。暖暖的阳光照在身上,却只剩下彻骨的寒意。一段失神之后,我的嘴唇轻微动了一下,想说什么却噎在喉咙口。我轻轻地打开了小窗,把它如往捧在手心,独自走向楼下的草坪。迈着沉重的步伐走回到了书房,把栓住小窗的铁丝用力一拉,轻轻把它扔向窗外,它在空中划过一道弧度……那扇小窗,不会再关上了。【解析】本题考查情景作文。(1)审题立意。根据“请以‘回身,看向那扇窗,我不会忘记……’为开头”的提示,可知本题只是提供了一个情境,据此应写与“那扇窗”有关的令自己难忘的故事。如在那扇窗下(或通过那扇窗)我看到了美丽的风景,通过那扇窗我看到了社区志愿者无私奉献的身影,通过那扇窗我看到了小区里邻里间和谐相处的美好生活场景,那扇窗下有妈妈勤劳的脚印,那扇窗下有我读书的快乐,那扇窗下有老师为我辅导的温馨等。立意上,应表达对真善美的追求和积极向上的人生态度。易写成记叙文。(2)选材构思。应先根据对题意的理解,回顾与某个地方的窗有关的故事,确定要写作的内容。如家中书房窗下,有我刻苦学习的身影;厨房窗下妈妈忙碌的故事;办公室窗内老师耐心谈心的画面等。然后写清楚具体的内容。要写清楚时间、地点、人物,起因、经过和结果;要用心理描写出自己的感受,用议论抒情语句点明“难忘”之处以及对自己的影响或作用。注意行文时以“回身,看向那扇窗,我不会忘记……”为开头。21.按要求作文。针对部分中学生在学习生活中害怕困难,逃避困难,不会正确克服困难等情况,学校开展了以“直面困难,勇毅前行”为主题的教育活动。请你从下面的写作任务中任选其一,完成一篇文章。要求:①题目自拟;②符合写作任务的文体要求;③有真情实感,不得套写抄袭;④不少于600字;⑤文章中不出现真实的地名、校名和人名。【答案】例文1:面对困难,勇往直前在我们的生活中,不免会有许多困难。其实,困难就是一个弹簧,你弱它就强,困难还是一只可怕的拦路虎,阻止我们前行……困难是躲不开的,它因为你的害怕而变大。就像我,遇到困难后,总能勇敢的去面对尝试克服困难。记得有一次,爸爸妈妈去外边和他们的同事共进午餐。学完英语的我回到家时已经是“肚皮贴肚皮了”。后来,我走进厨房一看还有一些新鲜的土豆!这可真是天意!于是,我围好围裙,学着妈妈平时做饭的模样开始自己做饭。“啪,啪,啪。”我刮完土豆皮后就开始切土豆丝,然后一个劲的炒了起来。我先放了一汤勺的食用油,然后把切好的土豆丝放进锅里。大概过了几分钟的,我根据自己喜爱的口味,依然放了一汤勺的盐……土豆炒熟后,我使用锅铲把它从铁锅里盛到菜盘里,打理好一切后,我就迫不及待的拿出筷子,尝尝我的艺。我夹起几根土豆丝放到我的嘴里,还没等我嚼烂,就从口中吐了出来。这盘土豆盐放得太多了,差点把我咸死了。于是,我只好倒掉了这盘我人生中第一次自己做的土豆丝,我既伤心又难过。心想:这可怎么办呢?我已经白白浪费两个土豆了!我再也不敢尝试第二次了!可后来,我又转念一想:不行!我一定要把这盘土豆做成!于是,我又重拾信心,开始了炒土豆的第二次。这次虽然盐没有放多,可是,那些顽皮的食用油们就像热锅上的蚂蚁一样蹦来蹦去,全部溅在了我拿锅铲的右手上,令我疼痛不已。后来,我灵机一动,从冰箱里拿出两层保鲜袋包在我的手上。哈哈,这下我可再也用不着害怕那些淘气的食用油了!经过了十几分钟,香气扑鼻的土豆丝终于上桌了!我掩饰不住内心的喜悦,连忙拿起筷子,把土豆丝夹起来尝了一口。哇,这可真是一道美味佳肴呀!我一边品尝自己克服困难后,经过辛苦的努力做出来的土豆丝,一边看着我右手上被食用油炸起的水泡,心想:只是在我的右手上炸出几个水泡而已,就能换到一盘自己亲手做的美味佳肴,即使多受点伤,也值了!今天,我心里喜滋滋的,比吃了蜂蜜还要甜。因为,我成为了一个战胜困难的勇士,不再害怕任何困难!朋友们,困难本身并不可怕,可怕的是你没有战胜它的勇气。勇敢面对它,拿出你智慧,你就能够像我一样攻无不克,战无不胜,成为一名打败困难的“常胜将军”!例文2:勇敢面对困难敬爱的老师,亲爱的同学们:你们好!今天很荣幸站在这里为大家演讲,我演讲的题目是《勇敢面对困难》。人生在世,说长,悠悠数万日,遥遥无期;说短,忽忽几时秋,弹指一挥间。人,只只有一次宝贵的生命,而我们又有几次成功,几次失败呢?人生道路是不平坦的,难免有磕磕绊绊,这就是挫折。面对挫折,我选择张开双臂,勇敢面对,乐观地看待挫折。只有这样,你才能永远立于不败之地。古人云:“不经一番寒彻骨,怎得梅花扑鼻香”。困难与挫折对于人来说,一把打向坯料的锥,打掉的应是脆弱的铁屑,铸成的将是锋利的钢刀。人最出色的工作,往往是处于逆境的情况下,思想上的励,甚至是肉体上的痛苦,都可能成为精神上的兴奋剂。很多杰出的伟人都曾遭受心理的打击及形形色的困难,若非如此,也许他们就不会付出超群出众所必需的那种劳动。正如法国的伏尔泰说:“人生布满了荆棘,我们唯一办法是从那些荆棘上迅速跨过。”翻开历史的浩浩长卷,古今中外所有的有成就的人士,哪一个没有经过挫折。众所周知,越王勾践兵败于吴国,只能向吴求和,还和妻子做了吴国人质,卧薪尝胆后才死里逃生。回到越国,发奋图强,实现了洗辱复国的志愿。德国音乐家贝多芬在失聪后,靠把一根木条的上端插入发声器,另一端插入口中感受声音,才谱写了很多名曲,铸就了音乐的神灵。张海迪在高位截瘫下,自学成医,完成了很多医学著作的翻译,被誉为“当代的保尔”。我国著名体操运动员桑兰,比赛中脊椎严重受伤,造成瘫痪,但她坚强乐观的生活态度,却使她成为2000年北京申奥形象大使。试想,这些名人,面对挫折如果自暴自弃,能铸就辉煌的篇章?挫折像弹簧,你强它就弱,你弱它就强。挫折面前,一蹶不振,必将走向死亡。有一个故事说:两人结伴横穿沙漠,一人中暑不能前行,当朋友出去找水,把手枪给了他用来鸣枪确定方位。当朋友离开时,他却把手枪绝望地对准了自己的太阳穴,提着满壶清水的朋友循声而至时,只看到了一具尸体。这正是面对挫折不同的态度导致的严重后果啊!面对挫折,我们不能一撅不振,要勇敢奋斗,向它挑战。笑对挫折,盈我们的心房;笑对挫折,让我们勇敢前进!每一次挫折都是对自我意志、决心的考验。我们要从挫折中总结经验教训,不要因为惧怕再一次的受伤而放弃了近在咫尺的成功。敢于面对挫折的人是最坚强的!朋友,挫折是磨练意志、增加能力的好机会。让我们勇敢地面对挫折,优雅地迎接成功吧!我的演讲完毕,谢谢大家!【解析】写作任务一:本题考查话题作文。一、审题立意。根据学校开展了以“直面困难,勇毅前行”为主题的教育活动和“在班会上,向老师和同学讲述自己在学习生活中克服困难的经历和感悟”的写作情境可知,作文围绕“直面困难,勇毅前行”来写,“直面”就是对所遇之事会回避、不退缩、不放弃。然后,要写出自己在学习、生活中克服困难的经历和感受。二、选材。人生会遇到很多的困难、挫折,选取你觉得有意义、印象深刻的事。比如:可以写考试考差了,心情非常低落悲伤,然后通过阅读,了解了司马迁的事迹后获得启发,学会了勇敢坚强面对困难;也可以写在生活中,做错了事,不小心打碎了别人的窗户,在妈妈的引导下,向别人勇敢承认错误,直面困难,勇敢前行;也可以写,在一次爬山过程中,不小心擦伤了腿,很难继续往上爬,但在朋友的帮助下,勇敢坚持走完了全程;也可以写,在一次演讲比赛时,因为紧张、害怕而想放弃比赛,在自己心理暗示下,勇敢走上讲台,最终赢得了比赛等。三、文体。根据“在班会上,向老师和同学讲述自己在学习生活中克服困难的经历和感悟”可知,本文适合写记叙文。写作任务二:本题考查演讲稿。一、审题立意。根据“在‘直面困难,勇毅前行’主题升旗仪式上做一次国旗下讲话”可知,本题考查以“直面困难,勇毅前行”为主题的演讲稿。演讲稿,也叫演讲词。它是在较为隆重的仪式上和某些公众场合发表的讲话文稿。演讲稿是进行演讲的依据,是对演讲内容和形式的规范和提示,它体现着演讲的目的和手段。演讲稿是人们在工作和社会生活中经常使用的一种文体。它可以用来交流思想、感情,表达主张、见解;演讲稿具有宣传、鼓动、教育和欣赏等作用,它可以把演讲者的观点、主张与思想感情传达给听众以及读者,使他们信服并在思想感情上产生共鸣。所以,演讲稿,主题要突出,观点要鲜明,要有宣传性、鼓动性,能让听众产生共鸣。二、选材。可以选取自己克服困难的事情,也可以选取古今中外的名人克服困难的事或者名言警句的等,如:大作曲家贝多芬由于贫穷没能上大学,十七岁是患了伤寒和天花病,二十六岁,不幸失去了听觉,在爱情上也屡受挫折。在这种情况下,贝多芬发誓“要扼住生命的咽喉”。在与命运的顽强搏斗中,在乐曲创作事业上,他的生命之火燃烧得越来越旺盛了。逆境不但没有吓倒他,反而成了他获得强大生命力的磁场;再如,孙康家贫,冬夜映雪光读书;车胤家贫,夏夜练囊盛萤,借萤火虫的微光读书;再如,数学家华罗庚,因家境贫困,从小就替父亲担起全家的生活重任。但一有空,就借几本数学书来看,他用5年时间自学了高中三年和大学初年级的全部数学课程。他一心在数学王国的海洋里劈波斩浪,将身躯的疼痛、生活的艰辛和世道的不公统统抛在脑后……三、写作思路。写作时,注意中心要突出,语言要流畅,思想要积极向上。可以阐述“困难是什么”,然后写“我们应该怎么面对困难”,还可以写“勇敢面对困难的意义”等,最后慷慨激昂,号召同学们一起勇敢直面困难!【考前技巧篇】☞字、词、句篇答题技巧一:字音1.辨别多音字读音的方法和技巧。对于多音字,我们可以采用以下方法来确定其正确的读音。(1)据义定音。即利用音义练习,区别一个字的不同读音。因为多音往往是和多义紧密联系的。如“难”,取“做起事来费事”义时,读“nán”;取“不幸的遭遇”义时,读“nàn”。(2)根据词性定音。有的字由于词性不同,读音也往往不同。如“数”,当作数词、名词时,一般读“shù”,如“数学”;当作动词时,一般读“shǔ”,如“数落”;当作副词时,应读“shuò”,如“数见不鲜”。(3)根据语体定音。语体有口头语和书面语之分,许多多音字的不同读音是由口头语和书面语的差别造成的,应注意这一点,这有助于我们读准多音字。如“翘”,书面语读“qiá”,如“翘首、翘楚”;口头语读“qià”,如“翘尾巴、翘辫子”。(4)根据用法定音。有些字在不同的词里,字义并没有显著的不同,字音却不同,这主要有两种情况:一种是这个字单用(单音节词)时和用这个字构成双音节词或多音节词时读法不同,如“剥花生”的“剥”读“bā”,“剥削”的“剥”读“bō”;另一种情况是个别词语中的读音与一般读音不同,如“松柏、柏树”的“柏”读“bǎi”,而德国首都“柏林”的“柏”读“Bó”。2.形近字的误读。形近字是指那些形旁或声旁相近的字。造成误读的情况主要有以下两种:(1)笔画增减形成的形近字。这类形近字笔画只有细微的差别,但读音迥异。如“大抵”的“抵”读“dǐ”,“扺掌”的“扺”读“zhǐ”。(2)偏旁不同形成的形近字。这类形近字往往读音相同或相近,是中考考查的重点。如“峻”与“竣”都读“jùn”,而“慨”和“概”分别读“kǎi”“ɡài”。对于形声字、多音字、形近字等易读错的字,我们一要加强读音的积累和训练,二要时刻防范字形误认、偏旁误读等错误倾向。答题技巧二:字形1.音辨法。即根据字的发音辨别字形。通过朗读该词,发现其中的书写错误。如“随声附和”,从字音的角度考虑,我们经常读“fù hè”,而“合”没有“hè”的读音,所以应该是“和”。再如将“辍学”的“辍”(音chuò)错读成“缀”(音zhuì),将“作祟”的“祟”(音suì)错读成“崇”(音chónɡ),将“渎职”的“渎”(音dú)错读成“赎”(音shú),将“气概”的“概”(音gài)错读成“慨”(音kǎi)等都可以用此法较为迅速地将其分辨。2.形辨法。通过分析形声字的形旁来推导这个字的含义,如“插科打诨”中的“打诨”指开玩笑,跟言语有关,“诨”应为“讠”旁,不能错写成“浑”。3.义辨法。即通过掌握、分析词语的意义来辨析字的书写是否正确。俗话说“字不离词,词不离句,句不离篇”。在理解词语含义的基础上去掌握字形,就不易出错。如“再接再厉”说的是公鸡相斗,每次交锋以前先在石头上磨一下嘴,“厉”在古代写作“砺”,是“磨刀石”的意思,因此在这一词语中不能写成“励”。4.结构辨析法。分析字的结构,主要是指那些并列结构的词语(特别是成语),可以由此一字推及另一字。汉语构词有一个规律,一个词(特别是成语)相应位置上的语素意义相同、相关或相反。对这类词,可根据前后位置的关系,推知相对应的字词的词义,从而帮助我们辨析字形。如:“名门望族”中的“名”“望”相对应,指有名望的家族,因此不能写成“名门旺族”。5.来源结合法。大部分成语都有一定的来源,所以在把握这类词语的形态时,就要结合其最初的来源和典故,这样有利于准确、牢固地记忆其形态。如在“子虚乌有”“班门弄斧”“墨守成规”这几个成语中,“乌有”指一个人,即成语故事中的“乌有先生”,所以“乌”就不能随意地写作“无”;“班”指的是“鲁班”,也不能写作“搬”;“墨”指的是“墨子”,就不能写成“默”。当然,要做到正确书写汉字,仅靠上述方法不能彻底解决问题,关键还在于平时的方法落实,尽量弄懂词语意思,日积月累,灵活地运用这些方法,这样才不会对中考的字形考查题“望题兴叹”。答题技巧三:标点符号1.句号、问号、叹号。表陈述语气或舒缓的祈使语气时用句号,表疑问或一般反问语气时用问号,表感叹语气、强烈的祈使语气或强烈的反问语气时用叹号。2.顿号、逗号、分号。顿号表示词语间的停顿,逗号表示句子间的停顿,分号表示分句间的停顿。三者都表示并列关系。3.冒号。其作用概括起来有两点:①提示下文,②总结上文。4.引号。作用有五点:①表示直接引用,②表示突出强调,③表示否定,④表示特定称谓,⑤用来引用俗语、谚语、歇后语、古语等。5.括号。表示文中注释的部分。只对句中的部分词语作注释或补充说明的叫句内括号,句内括号和被注释的词语之间不能用标点,标点用在括号后;对全句内容作注释或补充说明的叫句外括号,括号后面不需要加标点。6.破折号。作用有六点:①表解释说明,②表意思的递进,③表意思的转换、转折或跳跃,④表停顿或声音的延长,⑤表总结上文,⑥表分项列举。7.省略号。表引文的省略,表列举的省略,或表说话断断续续。8.书名号。表示书籍、篇章、报刊、剧作、歌曲、文件名、图画名的名称。书名号内还有书名时,外面用双书名号,里边用单书名号。9.间隔号、专名号、着重号。间隔号表示间隔或分界,专名号标示人名、地名、朝代名等,着重号表示强调。答题技巧四:病句辨析辨析病句可采用以下方法:1.语感法。辨析句子正误,首先得认真阅读,仔细考虑,从整体把握,看看句意是否明确,内容是否合理,句意间关系与关联词语是否一致等。凭借语感,往往可以发现一些句子的毛病。2.紧缩法。找出句子的主干,检查主谓宾(中心语)是否残缺,搭配是否得当。3.分解法。理清枝叶(附在主、谓、宾上的附加成分),检查同枝叶相应的主干是否搭配得当。4.聚焦法。对于句中出现的修饰性词语、关联词语应格外留意,尤其是成对出现的词语,先检查有无滥用、错用或搭配不当的毛病,再看分句次序是否合理。5.结构分析法。结构分析法,顾名思义就是对句子结构进行分析,找准句子成分,从而确定句子是否在语法上出现问题。结构分析法主要从主干和修饰语两个方面对病句进行辨识,是判断病句的根本,也是最重要的方法。6.词语标识法。在一个句子中往往有一些标识性的词语,这些词语通常是语病的症结所在。例如:答题技巧五:修改病句修改病句以改正确、改通顺为原则,不做语言修饰,不改变句子原意。正确把握句子的结构, 了解常见病句类型及修改方法,是基本的应对策略。主要采用“增”“删”“调”“换”四种方法进行针对性的修改。要特别注意成分残缺、搭配不当、语序不当、结构混乱四个方面的语病。修改病句的方法一般有:1.看句子成分是否完整。修改病句首先要找一找句子的主干,看一看句子的成分是否残缺或赘余。若是成分残缺,则补上;若是赘余则删去。2.看搭配是否得当。对于成分完整的句子,我们要看成分之间的搭配是否合理。如:主语和谓语是否搭配,谓语和宾语是否搭配,修饰语和中心语是否搭配。3.看语序是否合理。句子中的修饰语应按照一定的顺序排列,否则就会违反逻辑或事理。因此,我们还应看句子的语序是否合理。4.看结构是否清晰。有些句子往往将不同的句式杂糅在一起,造成结构混乱,我们必须对其进行调整。5.看用词是否恰当。有些词语的使用从成分、搭配上来说是没有问题的,但是从词语的感情色彩、使用对象、使用范围来说是不合理、不恰当的。还有关联词语的使用不恰当等,这些在修改时都应注意。☞语言表达与应用篇答题技巧一:句子的衔接句子的衔接所遵循的方法与原则如下:1.保持话题一致。话题一致,是指选择的句子和语段要紧紧围绕一个中心,集中表现一个事实、场景或思想观点。要做到“话题一致”,首先需要弄清语意,抓住句子的特点;其次,要明确所选句子与上下文之间的关系或联系。另外,要注意陈述对象的一致性,以及观点与材料的一致性。2.保持句式结构一致。“句式结构一致”就是强调衔接句与其前后语句结构、短语结构要一致,字数大体相同,句子形式整齐,音韵和谐。3.保持情调、氛围一致。所给语段往往有自己的风格和特色,对于景物描写的语段,要分析语境因素:景物、情调、写法的特点。景物分远近、动静的不同;色彩有亮丽、暗淡之别;气氛有热情、凄凉之分;视角有俯、仰之异;感情或悲或喜;态度有褒有贬。所有这些在语段中都应保持和谐一致。4.保持内在逻辑一致。选择的句子和所给语段要围绕明确的话题,符合事物的内在逻辑关系。判断时要看是否符合事实,要注意分句间的关系、衔接以及整体的和谐。答题技巧二:句子的排序1.找排列规律。句子的排列次序是句子根据语意的需要进行的组合,具有不可随意更改的必然性,句序的排列有一定的规律,如按时间顺序、空间顺序或逻辑顺序(由现象到本质,由个别到一般,由浅到深,由感性到理性,由原因到结果,由轻到重等)。2.抓语言标志。即要抓住关键字眼、总领句、总结句、相同句式等。有些句子上句句尾与下句句首有相同词语相互承接,有些句子上下句有关联词语、指示代词、序数词以及“又、再、也”等语言标志。3.理写作思路。①寻线索:记叙文段一般有明确的线索。线索可从时间变化、地点转换、事件发展、人物变换、情感变化等方面去寻找;②明结构:总分总式、并列式、层进式。4.明排列方法。先确定首句和尾句,然后再排列中间句。也可以从局部入手,然后再向整体扩展。句子排序过程中,我们要注意这些细节:代词的指代内容、连词(关联词)的连接对象、前后词句的对应等。答题技巧三:语言表达得体解题方法1.看对象。无论是说话还是写文章,都应首先考虑对象特征,如性别、年龄、职业、身份、文化水平、性格、爱好甚至禁忌等。这个对象不仅指信息接受者,也指信息表达者。同一个意思,对不同的人应有不同的说法。另外,还应注意谦称、敬称和习惯用语的使用。谦称只能用于称自己,敬称只能用于称对方。2.看场合。(1)在庄重场合,要求用语庄重、规范,用典范的书面语,而不能用拉家常式的口吻。(2)在公共场合,用语要准确、扼要,话题要集中,使用自然、亲切、灵活的语言,并尽量用口语。(3)在与人促膝谈心时,不宜用外交辞令,故弄玄虚。因为面对面说话,发话人和答话人都有一定的语境,许多话都可以十分简短甚至省略,不必追求完整。(4)在娱乐场合,用语要有趣、生动,有时还需要一点幽默。 3.看语体。一般来说,不同的文体应有与之相适应的语体,而不同的语体又常常表现出不同的语体色彩和语体特征。如文艺语体要具有形象性,科技语体要具有精确性和严密性,政论语体要具有逻辑性和鼓动性,公文语体要准确、简洁和程式化。而各种文体,具体来说,对用语又有不同的要求。就实用文来说,新闻要求简明扼要、概括性强,广播稿、演讲稿要求用语通俗、口语化,合同要求措辞严密、表述清晰,贺词要求热情、庄重,讣告要求严肃、郑重而沉郁。☞文学文化常识与名著阅读篇答题技巧一:文学文化常识1.纵横结合记忆法。古今作家生活时代不同,从而形成纵的联系;同一时期不同作品又有各自的特点,从而又形成了横的联系。将这些纵横联系的知识点组合起来,在脑中形成几条线或几个面,是一种快速而牢固的记忆方法。如果能据此自己动手制作几张图表,印象就更加深刻了。如我国古代戏剧史有三个高峰,一是元杂剧四大家加上王实甫,二是汤显祖的临川四梦,三是清代的南洪北孔。这样纵横结合加以记忆,中国古代戏剧史又何愁记不住呢?2.举一反三记忆法。如由《战国策》的国别体联想到《史记》的纪传体和《资治通鉴》的编年体;记忆屈原时,由屈原想到他的作品《离骚》;又因《离骚》是中国浪漫主义文学的源头,想到西方浪漫主义三大家:雪莱、雨果、拜伦。这样触一而发十,就能较系统地记住许多文学常识了。3.分门别类记忆法。即在分类的基础上把某些有相同点的知识按一定顺序集中在一起强化记忆。可以以考点为分类标准,如:(1)时代国别,可以借鉴古代史书体例中的编年体和国别体的方法,按不同时代和不同国家来记忆。(2)风格流派,如山水田园诗派、边塞诗派、婉约派、豪放派等。(3)地位评价,如四大名著、世界三大短篇小说巨匠等。(4)文章体裁,如小说、诗歌、散文、戏剧等。(5)题材人物,如《范进中举》《孔乙己》等都塑造了受封建科举制度迫害愚弄的旧知识分子形象。(6)作家作品,如苏轼的诗、词及文等。(7)字号称呼,如柳河东、杜拾遗、太史公等。4.点面结合记忆法。复习既要注意全方位复习,又要注意突出重点。有的文学常识关键就在某一要点,抓住了要点,其他问题也就迎刃而解了。答题技巧二:名著阅读针对文学名著阅读题的命题特点,在复习备考的过程中,需要注意以下几点:1.熟悉课本推荐的名著,了解全书的情节梗概和结构特点,理解主要人物形象的性格特点,把握全书的主题思想和语言风格;精读其中部分有代表性章节,做到点面结合。2.认真审题,根据题目的设问指向和要求组织答案,切忌答非所问。力求答案语言简明、连贯,书写字迹工整,标点规范。不同的题型有不同的答题规范:1.情节概述 解答此类题,同学们一定要熟悉名著中的故事情节,特别是关系到人物命运和生死转折的情节。同时也应注意情节勾连类的问题答案一般不会出现在选段内,命题者的目的是考查对整部名著重要情节内容的了解,而不仅仅是某选段的内容理解。2.艺术特色(语言特色) 这类题目往往需要学生对作品的写作手法、语言风格、价值观念有整体的认识和把握,可以从自己熟悉的片段或者给出的语段推导。3.人物形象 回答这一类题目,首先要对名著中主要人物的性格特点有一个总体的把握;其次要结合选段的具体内容,结合人物的语言、动作、肖像、心理、细节等描写,认真揣摩人物的内心世界,把握他们的性格特点。一定要根据问题的要求去组织答案,注意要结合选段的“具体内容”来分析,否则解答是不完整的。4.主题探究答题时要根据文段情节并结合作品内容,探究作品的主题思想,准确分析作者的思想感情。不要拘泥与所给的选段,要联系整部作品进行解读。5.阅读启示 解答体验看法类题目时,要在了解作品主要内容的基础上,围绕作品主题,联系自己的生活体验,简明地回答。具体方法有如下几种:①借助情节和内容的分析理解,梳理出作者的写作思路及目的,从而概括出主题思想。②联系写作背景、作家的思想观点及创作意图挖掘出主题。③通过分析人物的形象及人物之间的关系来把握主题。 ☞名句名篇默写篇考生该项答题失误主要表现在以下几个方面:1.字词写错。这类现象是默写失分最主要的原因。这些考生不是背不出,而是一默写在纸上就把字词写错。比如“人情练达即文章”中的“练”,有的考生错写成“炼”。这往往都是在背诵时不细心,或者是长期养成了错误书写的习惯造成的。2.张冠李戴。有的考生背诵的诗文名句多了,在大脑中就出现了程序错乱,经常会出现这首诗串到另外一首诗上去的情况,或者同一首诗,前句串到后句、后句串到前句,在默写时造成张冠李戴。之所以会出现这种情况,是因为考生在背诵时囫囵吞枣,不注重对诗文内容的理解,更不注重这首诗和另外一首诗的区别。3.滥竽充数。这部分考生是为了将空填满,不管正确不正确,就将自己所知道的名句胡乱填上去,滥竽充数。4.杜撰句子。这部分考生实在想不起来该是什么名句,就自己杜撰一个填上去。☞诗词曲鉴赏篇答题技巧一:鉴赏诗歌形象解题答题技巧1.鉴赏人物形象的技巧。一般从以下方面回答:①分析角度,即语言、动作、神态、心理等;②结合人物所处的时代背景及生活环境;③结合人物个性;④结合全诗表达的思想感情。答题步骤:①要在文段的开头点出诗人的形象特点,即用一句话“盖帽”(这一步也可以放在最后作为总括);②结合全诗中的文句进行分析,注意要围绕“诗人形象”这个中心来写,如果诗歌运用了一些表现手法,也可以概括出来;③概括出人物形象的社会意义或蕴含的作者情感。2.鉴赏景物形象的技巧。一般从以下方面回答:①明确全诗表达的情感;②抓住景物与情感的融合点;③紧扣景物的特点;④分析景物描写的作用。答题步骤:①描写诗中展现的图景画面;②概括景物所营造的氛围特点;③联系诗人的生平,分析景物所包含的作者的思想情感。3.鉴赏事物形象的技巧。一般从以下方面回答:①明确全诗表达的情感;②抓住事物与情感的关联;③紧扣事物的特点;④分析表现手法:象征、拟人、比喻、借代、移情于物、托物言志、借物抒情等。答题步骤:①概括总结出诗中所描绘歌咏的主要物象;②结合诗句分析物象的外在特征(形态、色泽等)和环境特点,着重挖掘物象的内在品格,抓住物与志的“契合点”;③概括塑造此形象的意义。答题技巧二:鉴赏诗歌语言解题答题技巧1.抓动词。动词的使用常常能起到画龙点睛的效果,可使诗歌中的意象化静为动,更加形象生动,具有动态美。2.抓形容词。考生要注意形容词的重叠运用和活用。形容词可绘景摹状,化抽象为具体,变无形为有形,使人如闻其声,如见其人,如触其物,如临其境。如“梦魂欲渡苍茫去,怕梦轻、还被愁遮”中的“轻”,描状梦魂,化无形为有形,且运用了“通感”的修辞手法,使本不可触的梦有了重量,堪称妙绝。3.抓数量词。诗(词)人精心提炼的数量词,往往可以产生丰富隽永的诗情。如唐代诗人齐己《早梅》中的“前村深雪里,昨夜一枝开”,“一枝”先于众梅,悄然“早”开,显出此梅不同寻常。4.抓表颜色的词。如“两个黄鹂鸣翠柳,一行白鹭上青天”,“黄”“翠”“白”“青”四种颜色,点缀得错落有致,而且由点到线,向着无限的空间延伸,画面静中有动,富有鲜明的立体感。答题技巧三:鉴赏诗歌表达技巧解题答题技巧1.表达方式:描写、抒情、叙述、议论四种表达方式。其中描写、抒情是考查的重点。2.描写技巧:动静、虚实、点面、正面描写和侧面描写,烘托、渲染、细节描写、白描等。可使用各种修辞手法。3.抒情技巧:主要有直接抒情和间接抒情两种。间接抒情包括借景抒情(情景交融)、托物言志、借事抒情、借古讽今等。可使用各种修辞手法。4.修辞手法:比喻、夸张、拟人、借代、对偶、反复、双关、设问、反问、通感、对比、衬托等。5.篇章结构:开门见山、欲扬先抑、伏笔铺垫、承上启下、以景结情、卒章显志、结构对比、首尾照应等。答题技巧四:主旨情感类题目解题答题技巧答题技巧五:描摹意境画面类答题技巧1.找形象。从诗句中找出描写的具体形象和画面。2.析特点。抓住诗句关键词与细节,并结合常见形象的特定含义,分析其特点。3.明情感。理解诗词形象直接或间接抒发了什么思想感情。4.绘画面。发挥想象,紧扣诗词中的形象和画面,用自己的语言简练概括或生动描绘,突出营造的氛围特点。不可单纯翻诗句。☞文言文阅读篇答题技巧一:文言文实词答题技巧实词的考查主要集中在古今异义、一词多义、词类活用、通假字几个方面,理解时必须结合具体语境。1.字形分析法。汉字中的形声字占80%以上,而形声字的形旁有表意功能。因此,通过对字形结构的分析可以帮助我们探求字的意义。2.语境推断法。实词绝大多数是具有多义性的,因此在解释词义时,要紧密联系上下文,结合具体语境理解。3.结构分析法。对实词所在的句子进行成分分析,对实词所在的词组进行结构分析,同样可以帮助我们确认该词的意义。如:“急湍甚箭,猛浪若奔”,“奔”与“箭”所处位置相同,且都作宾语,那么就可断定“奔”也是名词,意思是“飞奔的马”。4.成语助解法。在成语中,保留了大量的文言词义,因此借用成语中的实词词义有时也能让我们“绝处逢生”。5.联想推断法。我们要善于根据课内学过的知识举一反三,相互比照,辨其异同,以解释试题中的实词词义。如“辍耕之垄上”中对“之”字的理解,联系“送杜少府之任蜀州”中的“之”当“去、往”讲,再结合语境,可以推断此处的“之”也是“去、往”的意思。6.代入检验法。对于选择、判断类的词语解释题,最简单的方法是将所给的义项放入各自的具体语境中去贯通文意,解释准确而无滞碍之处的即为正确答案。答题技巧二:文言文虚词答题技巧1.词性界定法。有些文言虚词兼有空词和虚词的双重性质,根据上下文的语境,只要能推断出其词性不一样,那么,其用法也肯定不同。2.语境推断法。一般而言,需要判定的虚词上下文语境完整,语句的整体意义实际上限制了某个虚词的意义和用法,因此可考虑根据具体的上下文来进行断断。3.句位分析法。虚词在句中的位置不同,搭配关系不同,它的意义和语法功能也往往不同。4.标志识别法。有些文言虚词是构成特殊文言句式的标志词,如能记住教材中出现的一些有代表性的特殊文言句式,可帮助你快速答题。5.结构互训法。文言文中多结构一致、前后对称的整句,往往处在相同位置的虚词其词性意义或功能用法也相同,所以知道了其中一个的用法,也可大致判断另外一个的用法。6.关系分析法。判断虚词的用法更多的是看这个虚词在句中起的语法作用,它跟前后文构成什么样的意义关系,尤其是“而”“以”作连词用时,其意义关系往往比较复杂,有并列、承接、递进、转折、因果、假设……,一定要瞻前顾后,仔细揣摩,加以确定。7.成语印证法。很多成语源自古诗文,凝固了很多文言现象,包括虚词用法的一些规律特点,通过一些熟知的成语来推知印证文言材料中的虚词用法也不失为一个好方法。答题技巧三:文言文断句答题技巧如何把握文言断句1.通过句意断句。阅读文句,了解大意,注意文言文单音节词占多数的特点,抓住几个关键字词翻译以理解句意,然后根据逻辑关系断句。2.根据语法断句。(1)主语、谓语之间要停顿。(2)谓语动词与宾语、补语之间要停顿。(3)成分省略处要停顿。3.借助词语断句。(1)句首发语词、关联词后要停顿。(2)时间状语在句首要停顿。(3)句中表转折关系、并列关系、顺接关系的连词前要停顿。(4)连在一起的两个单音节词中间要停顿。(5)句中语气助词“也”后要停顿。答题技巧四:文言文翻译答题技巧1.文言翻译的原则:信、达、雅。信,要忠实于原文意思;达,要合乎现代汉语的语法习惯;雅,要尽量做到言辞优美。2.文言翻译的要求:直译为主,意译为辅。所谓直译,就是要忠实于原文的内容和句子的含意,用现代汉语字字句句落实到译文之中,甚至在表达方式上也要和原文保持一致。在难以直译或直译以后表达不了原文意蕴的时候,可酌情采用意译作为辅助手段。3.文言翻译的方法:对、换、留、删、补、调。(1)“对”就是对译,逐字逐句落实。(2)“换”就是将单音节词换成现代汉语双音节词。将词类活用词换成活用后的词,将通假字换成本字……凡该换的,一律换之。(3)“留”就是保留文言文中的一些基本词汇和专有名词。如人名、地名、国名、朝代名、官职名、年号、政区名、典章制度及度量衡名称以及古今意义不变的词语等。(4)“删”就是删去文言文中的某些虚词。有些文言虚词在句中只起语法作用,只要不影响语气就可以删去。一些句首的发语词、在句中表顺接的连词、起补充音节或停顿作用的结构助词等均可略过不译。(5)“补”就是补出文言文中的省略成分。如对“微斯人,吾谁与归”一句的翻译,结合语境,应加上表示假设的关联词“如果”。(6)“调”就是调整语序。如宾语前置、定语后置、主谓倒装等在翻译时就要把这些倒置的成分调整过来,使之符合现代汉语语法。如“吾谁与归”是“吾与谁归”的倒装,应翻译为:我同谁一道呢?答题技巧五:文言文归纳内容要点,概括中心意思答题技巧(一)传记类文章要点概括1.关注传主的主要生平事迹。严防事件发生的先后顺序颠倒或错位,还要严防内容上的断章取义,或者范围、作用、程度等的不明确。2.关注传文中的重要人物及其关系。对传记中的人物要有全面的把握,严防人物与事件之间张冠李戴、事件杂糅或无中生有。 3.关注人物的对话。严防内容上的主次不分、因果关系不明或文意理解上的错误。4.关注文言现象。对文中出现的重要文言实词、虚词、句式等,一定要能做出合理的解释,严防题干信息的错解或误译。5.关注题干。明确题干考查的信息重点,严防与题干要求无关的信息进入答案或不恰当的推断混入。考生理清文章的脉络,将文章中对传主的介绍梳理清楚,即可轻松答题。(二)非传记类文章要点概括1.分析具体内容,概括文章要点。对文章内容的归纳和概括要注重对每一自然段内容的分析和概括。2.分析选项内容,与原文一一对应。考生解题时要注意将选项内容与原文一一对应。尤其要关注选项中的关于写作手法的表述是否在原文中有具体的体现。答题技巧六:文言文艺术特色、表现手法答题技巧1.看结构。常见的结构形式有:并列式、递进式、总分总式等。2.看表现手法。常用的表现手法有:借景抒情、托物言志、抑扬结合、首尾照应、画龙点睛、叙议结合、烘托、伏笔、照应、悬念和释念等。3.看描写手法。常用的描写手法有:动静结合、虚实相生、白描、正侧相映等。4.看抒情方式。常见的抒情方式有:直接抒情(直抒胸臆)、借景抒情、情景交融、借古讽今等。5.看语言特点。品味语言的方法有:从修辞的角度品析、从表达方式(记叙、描写、议论、抒情、说明)的角度品析、抓住语言的形式(句式、音韵、节奏等)进行分析。☞小说阅读篇答题技巧一:如何分析人物形象优秀的作家总是通过他笔下的人物形象,来描绘他所处的时代,寄予他所领悟的生活哲理,读者要通过人物形象来把握作品所反映的生活本质。分析小说中的人物形象需要我们做到:(1)要抓住人物形象描绘的特点分析。①要注意分析人物的肖像描写,即外貌变化和那些富有特征的细节描写,从而揭示人物的内心世界。②要注意分析人物的动作描写,即人物在激烈的矛盾冲突中的表现。因为人物的性格主要是通过人物的行动来表现。③要注意分析人物语言描写,特别是要分析那些最能表现人物个性的语言,以及作者的议论或者作者借作品中其他人物对人物评价的语句,进而确定作者的感情倾向是褒还是贬,是颂扬还是讽刺。④要分析人物的心理描写,分析心理描写对揭示人物思想和表达主题的作用。(2)要联系人物所处的环境来分析人物形象。一般来说,人物都是一定的社会环境的产物,什么样的环境造就什么样的人物;同时,人物也能对环境起一定的作用,二者相互统一。因此,分析人物形象一定要结合社会环境。(3)要从故事情节和矛盾冲突入手分析人物形象。只有从故事情节和矛盾冲突入手,把握住人物在诸多矛盾冲突中的言行、内心动机,才能更好地理解人物的性格。(4)要以发展的观点多方面、多角度分析和评价人物形象。因为社会关系的复杂性决定社会中的人物性格往往是复杂的并且是发展变化的。需特别注意的是分析人物形象时,应从实际出发,不过分拔高人物的思想品质;立足原文,概括出原文中所能体现出来的东西,不无中生有;全面分析评价人物,不以偏概全。答题技巧二:如何分析环境描写及其作用环境描写分为自然环境描写、社会环境描写及场面描写。社会环境主要包括人物活动的历史背景、社会情态、阶级关系等因素的总和,它往往在文章中起交代作用。自然环境描写主要包括人物活动的时间、地点、季节、气候以及景物等。自然环境描写的作用是:表现人物身份、地位、行动,表达人物心情,烘托人物形象,渲染气氛,推动情节发展。场面,指由人物在一定时间、场合相互发生关系而构成的生活情景。环境描写并非可有可无,人物总是在具体的环境中活动着。在记叙类文章中它有着极其重要的作用。它的作用我们可以从以下几个方面考虑:1.交待故事发生的时代背景。2.为人物出场作铺垫。3.渲染( )气氛。4.衬托(烘托)人物( )心情(形象)。5.为下文的( )埋下伏笔,推动故事情节的发展。6.抒发情感,表达情怀。在解答环境作用时要结合具体的句子表示的内容和具体的语言环境即联系上下文来斟酌语言,回答问题。答题技巧三:体会重要语句的丰富含意对于含蓄句的理解:这类句子往往有一定的哲理,考生须要仔细品读其中内涵丰富的词语,立足句子的表面意义,结合前后语境,从“为什么”或“这样说的目的是什么”的角度挖掘其言外之意。如上题。对于技巧句的理解:首先要从比喻、拟人、夸张、反语、排比等常用修辞的角度考虑,分析所用修辞手法的表达效果与所体现的作者的情感态度;然后从语言、动作、心理、神态、细节等常用描写手法的角度考虑,分析所描写对象的特点与所体现的作者的情感态度;或者从直接抒情和间接抒情两种抒情手法的角度考虑,直接揭示作者的情感态度。答题技巧四:分析作品体裁的基本特征和主要表现手法1.依据不同的角度,辨别不同的表现手法。表现事物的手法有铺垫、衬托、象征、渲染、化动为静、以小见大等;描写人物的手法有语言描写、动作描写、肖像描写、细节描写等;构筑情节的手法有悬念、照应、巧合、抑扬、陡转等;修饰语言的手法有比喻、夸张、拟人、反复、对偶等。2.结合文本内容,分析表现手法的具体作用。 (1)象征,使抽象的思想、概念形象化,使感情的表达含蓄、耐人寻味。(2)渲染,对事物进行细致描写,突出主要表现对象。为行文做好铺垫,营造氛围。(3)抑扬,包括先抑后扬、先扬后抑,可使文章曲折有波澜,突出喜欢、赞美或批评、讽刺的感情。(4)对比,突出事物特点,使形象鲜明。和衬托相比,对比没有主次之分。(5)衬托,突出所要表现的事物的特点,强化思想感情,使要表现的形象更加鲜明。(6)动静结合,包括以动衬静、以静衬动,起烘托作用,相得益彰。(7)虚实结合,突出事物的本质特征,鲜明地刻画人物的性格,凸显事物、景物特点,能更集中地揭示主旨。(8)铺垫,引出议论,为下文张本,推动情节的发展等。(9)悬念,在情节发展中设置某种疑端或矛盾冲突,吸引读者关心情节发展或人物命运,引人入胜。3.安排好答题思路,一般是先明确手法名称,如“对比手法、象征手法”等,再摘引原文典型例句,进行深入分析,指出其表达效果等。[来☞散文阅读篇答题技巧一:分析重点词语的意思 “重要词语(含短语)”一般指:①体现作者立场观点的词语。②表现文章主题思想的词语。③反映深层次含义的词语。④对文章结构起照应连接作用的词语(代词)。⑤比喻、借代、反语等特殊的词语。⑥根据语境随情而作别种义项的词语。锤炼字词解题三步走:第一步:关注词语的含义。这里的含义包括词语本身的意思和语境义。从字词含义的角度体会妙处。第二步:关注词语的手法。字词的手法包括比喻、拟人、拟物、双关、通感、夸张等修辞。第三步:关注词语的作用。说作用时要从内容(突出描述对象的……特点)、情感(表达……情感)、结构(照应……、承上启下、线索等)三方面谈,尽量全面。表达效果主要是生动形象、含蓄蕴藉等。答题技巧二:体会重要语句的丰富含意词语在备用状态时的意义叫“基本意义”,即词语本身的字面义,而当词语进入语境后,它的意义就是动态的、变化的了,往往含有某种特殊的意义,即语境义。与一词多义不同,语境义是不固定的,离开了特定的语言环境,其意义就不存在。这种含义与词的基本义有一定的联系,但又与基本义不同。中考中考查本考点侧重考查语境义。对词语语境义的理解可以从以下几个方面入手:1.修辞义:主要指比喻义、借代义、移觉义、讳饰义、比拟义等,要从修辞本身的特点出发来理解,如比喻的相似性,借代的相关性,比拟的形象性,反语的讽刺性等。2.隐含义:隐含义即词语的深层含义,就是人们常说的“弦外之音”“言外之意”,它是相对于字面义而言的。理解隐含义应注意象征义。3.临时义:临时义是相对于词语的固定意思而言的,它是指词语在特定的语境中派生出来的意思,离开了这个语境,这个词的临时义也就消失了。4.附加义:作者从情味、感情色彩等方面给词语附加的含义。容易被考查的重点词语主要有以下几类:1.与文章内容、主旨密切相关的词语。2.体现作者感情、立场的词语。3.理解上容易发生偏差的词语。4.有着丰富内涵或强烈感情色彩的词语。5.包含修辞手法,能增强表达效果的词语。另外,一些重要的代词,对文章的结构起连接、照应作用的词语,在语境中作别种义项的词语,等等。都是文章中的重要词语,应特别重视。答题技巧三:分析作品蕴含的感情散文是以抒发情思为主的、以情动人的文学体裁,贵在有“我”,因此对于它的阅读,关键之一在于对作者“思想情感”的把握。散文中作者或触景生情,或托物言志,或写人表意。通过直抒胸臆或寓情于他人他物他景的方式来传递自身的思想情感。对于散文“思想情感”的考查,中考考点主要围绕以下几个方面来进行:概括作者思想情感的发展变化;通过品味词语来体会作者情感;通过分析景物描写来品味作者情感;通过分析人物描写来体会作者感情。我们在分析作者的思想感情时,应主要从这些方面入手:1.从分析散文类型入手。不同散文表达中心的方法不同,借此可以帮助我们确立归纳的着眼点和归结点。叙事散文应看写了什么人,记了什么事,表现了什么精神人格;写景状物散文应分析写了什么景物,抒发了什么感情;议论性散文主要分析阐发了什么观点或哲理;咏物散文最鲜明的就是托物言志,主要分析所托之物被赋予了哪些象征意义。2.从文章关键点入手。对文章中心而言,文章的标题、开头、结尾等都是特别要注意的地方。抓住这些关键点不失为概括中心意思的有效途径。3.从分析结构层次入手。有些文章主旨比较含蓄,难以把握,只能在理解和分析全文的基础上,划分文章层次,归纳各层层意,进而提炼出中心意旨。4.从分析文章抒情议论性的句子入手。文章抒情议论性的句子,往往能直接表达作者对所写对象的看法,同时又揭示事物的本质,表现文章的中心。5.从分析文章的写作背景入手。适当借助注解,调动自己的知识储备,尽可能多地了解事物、人物、活动的时代背景和写作意图。答题技巧四:根据文本适当拓展根据文本适当拓展的题型灵活多变,解答此类考题的关键是①能准确读懂原文的含义和作者的思想情感。②要有自己明确的观点或态度,不能模梭两可。③答题时,语言要流畅。不能出病句,语言要有层次性,可以引用诗文或名言回答此题。☞说明文阅读篇答题技巧一:把握说明对象及其特征类答题技巧1.如何分清说明对象。(1)看题目。有的标题直接表明说明对象,如课文《中国石拱桥》《苏州园林》,这些都是事物说明文,标题就直接表明了文中要说明的事物,即说明对象。(2)观首段。题目没有表明说明对象时,甚至有的文章没有题目时,那么我们就要看文章的第一段,在第一段的结尾处往往会点出文章的说明对象。(3)找结尾段。有时文章标题与首段均找不出说明对象,作者会在文章的结尾处揭示文章的说明对象,所以要十分关注结尾段落。(4)抓关键句。有些说明文还往往利用一些段首的总括句或段尾的总结句来点明文章的说明对象。要在通读全文的基础上,找到这些句子,进而把隐含在文中的说明对象挖掘出来。(5)归纳总结。如果以上方法都不可行,那么这时候,只能研读全文,逐段分析、归纳、总结,最后确定说明对象。2.如何把握说明对象的特征。(1)抓标题法。有的标题在指出说明对象的同时,也指出了其特征,如《奇妙的克隆》;有的标题用设问句的形式,引导读者有针对性地把握说明对象的特征,如《食物从何处来》;有的标题用比喻、拟人的手法说明事物的特征,如《大自然的语言》。(2)抓关键句法。许多说明文在说明事物时会通过关键语句明确交代对象的特征,以便于读者理解和接受,这类句子包括:总领句、过渡句、总结句等。因此,我们要学会抓住这些关键性语句,把握说明对象的特征,如《苏州园林》一文,抓住了“务必使游览者无论站在哪个点上,眼前总是一幅完美的图画”这个关键语句,就把握了说明对象的特征。(3)材料分析法。在整体感知全文的基础上,分析文章的组材和选材,段落层次的意义和联系,从而综合归纳出说明对象及其特征,如《中国石拱桥》一文列举了中国石拱桥中两个有代表性的例子——赵州桥和卢沟桥。我们在分析这两个例子时,不难发现这两座桥虽有不同之处,但却都具有“形式优美,结构坚固,历史悠久”的特点,因此可以综合概括出中国石拱桥的特征。答题技巧二:品析语言类答题技巧答题技巧三:辨析说明方法及其作用类答题技巧答题技巧四:概括说明内容,帅选重要信息类答题技巧1.概括说明内容。(1)如果有中心句,可以直接摘录中心句。(2)如果文段是几层并列的内容,就可以把这几层意思连接,归纳出一个主要意思。(3)如果文段中有结构类似的词语、短语或句子,那么这些往往是所要找的几个要点。(4)归纳部分内容要点时还可以找出段落中表示句子终结的标点符号,如句号、问号等,在这几层意思中进行合并、筛选,然后归纳出该部分的要点。2.筛选重要信息。筛选并把握说明文中的重要信息,是说明文阅读中能力层级较高的综合考查点。它包含两个内容:一是辨别并筛选文中重要的信息;二是把握作者在文中的观点及态度。(1)把握筛选信息的标准。(2)确定筛选信息的区间。在阅读中,选择材料信息必须按照一定的需求去选取,才有效率,才能准确。需求规定范围,即信息的筛选区间,要在确定的区间内进行取舍,也就是我们通常所说的“词不离句,句不离段,段不离篇”原则。(3)通过抓关键词筛选主要信息。阅读时要抓住诸如“首先”“其次”“另外”“还有”“也”“除此之外”等提示语。(4)通过抓关键句筛选主要信息。所谓关键句,就是指过渡句、段首句、文末点题句、问题前后句等,它们往往能给我们提供一些集中有效的信息。(5)把选项信息与原文语句进行比较分析。需要说明的是,以上有关筛选信息的手段,不论是直接的还是间接的,很少有单独使用的时候,一般都是综合使用的多。各种阅读方法综合运用得越熟练,筛选信息就越快。☞议论文阅读篇答题技巧一:归纳内容要点,概括中心意思类答题技巧1.提取法。如果题目要求归纳的内容,在文段中并无明显的主干句,那么考生需要通过阅读文章自己去体会、把握和提炼答案。2.合取法。题目要求归纳的内容往往不是一个方面,依据要求,所表达的意思要全面,这时最好的方法是将有关的两个或两个以上的表达意思相同的语段分别抽取出来并组合在一起。3.摘取法。题目要求归纳的内容往往是段落中的重要词语或句子,其中重要词语往往嵌在主要语句中,重要句子常常出现在文章或段落的首、尾或中间,考生在归纳时只要把这些词语、句子摘录出来就可以了。如上题,文章首段首句即揭示了音乐艺术的本质属性,可摘取出来。答题技巧二:理解文中重要句子的含意类答题技巧1.理解句子含意。所谓理解句子含意,指的是既要说出句子的表层意思,又要说出句子的深层意思。2.问题的主要形式有:上文中画线句子在文中的含意是什么;揣摩下列各句,说说你对句子的理解;揣摩文中画线句子,回答问题。3.谈谈自己的体会。这类句子多是文章的论点、观点、议论句、抒情句、主旨句等,多以主观题的形式出现。4.指出句子在文中的表达作用。这类句子多是在文章结构中起着关键作用,比如过渡衔接、前后照应、揭示主旨、升华主题等。5.指出句子运用的修辞手法及作用。答题技巧三:筛选并整合文中信息类答题技巧第一步:整体认知,微观把握。“整体认知”的内容主要包括:作者的观点和态度、文章的基本结构层次、说明对象及其特征或文章的内容要点等。“微观把握”是指在阅读的过程中,一方面要留心一些关键的句子,包括表明文章或文段主要意思的中心句、要点句,表明结构层次关系的句子以及表示疑问语气的句子等。二是随时勾画圈点出重要词语(指示代词、关联词、副词等),如“其”、“这”、“那”、“因为……所以……”、“之所以……是因为……”、“如果……那么……”、“目前”、“将会”、“已经”、“最……”、“除……之外”、“大概”、“也许”、“可能”等。这些关键词句往往是阅读理解的突破口,因此考生千万不能掉以轻心。第二步:审清题干,据题溯源。审题,就是要仔细分析题干,把握题干的指向性,即把握题干中包含的与答案相关的各种信息。题干一般由两个部分组成:一是文章的作者的话,一是命题者的话。命题者设置题干的目的,主要是限定答题内容;同时,命题者为了使考生不至于茫然无措,往往又会在题干中提示答案要点在文中的位置,甚至限定了在哪一自然段或哪个句子中。这样考生就可以根据题干的提示,找到每一道题的出题点,从而锁定答题区间。考生要注意切忌凭自己对文章的感性认识来代替作者的思路,给出武断的结论。第三步:对比印证,筛选整合。解答筛选信息类客观选择题,考生要对比印证,即把选项中的信息与原文相关词句进行对照,分析它们在表意的时候是否存在差异。在把选项中的信息与原文信息进行对照时,要逐字逐句对照,看哪个词被换了、删了,哪些词语、分句的顺序变了,从而对选项信息的正确与否作出准确的判断。考生在对照时要特别注意范围大小、程度深浅、数量多少等,从而准确判断选项的表述是否符合原文意思。答题技巧四:常见论证方法及作用类答题技巧1.举例论证。所举的能证明论点的具体事例、概括事实、各种现象、统计数据,及作者对此所做的分析,合为举例论证。2.道理论证。通过讲道理的方式证明论点,使论证更概括更深入。3.对比论证。对比论证的作用就是突出强调。4.比喻论证。通过比喻进行证明,使论证生动形象、浅显易懂。5.引用论证。增强说服力和权威性,增强论证的趣味性,吸引读者往下读。☞非连续性文本阅读篇答题技巧一:主题材料内容的理解与判断类答题技巧1.要准确审题,明确答题方向。解答问题时,首先要看题干的表述是对哪些概念或句子的理解,题目的要求是选正确的还是不正确的项。2.要逐段逐句阅读分析,掌握关键信息。此类试题一般设置四个选项,答题区域涉及文本的各个角落,因此需要仔细阅读文本和选项,准确把握选项中的部分关键词语为下面的分析判断,做好准备。3.分析选项,确定关键句段和关键点。根据选项中的关键词语,再次快速浏览文本,找出与之匹配的关键词句,从而确定选项的答题区间。4.比较核对,确定选项。确定答题区间之后,可以把选项和相关区域的原文进行比较,再具体比较时可以思考以下三个问题:一是选项有没有篡改的内容?二是有没有混淆逻辑?三是有没有主观臆断?也就是说,选项是否篡改了原文中的表范围、程度、时间等的内容,是否混淆了原文中几个语句之间的逻辑关系,看原文中是否有此类信息等。答题技巧二:对主题材料信息的筛选、提取与归纳类答题技巧1.速读文本,把握材料大意。通读各则材料,了解各材料所说的主要内容,以及共同的话题,只有这样才能从总体上把握所有材料的主要内容,才不会在做题时生断章取义、以偏概全的错误。2.仔细审题,找准答题的角度。仔细审读题干,分析题干的要求,明确答题的内容,抓住题干中的限定角度范围、时间、空间等方面的信息,明确答题的角度。3.确定答题区间,取舍提炼材料。首先要略读所有材料,明确每则材料的主要内容,然后细读材料并及时带着问题,重新通读材料,确定大致的答题区域,再逐个材料寻找并提取答题的关键信息。这时,一要注意明辨相同或相似的信息。二要善于发现隐蔽的信息,对于这些比较隐蔽的信息,在阅读时一定要加以关注。三要注意挖掘潜在的信息,潜在的信息常常隐藏在文字之中,同学们要善于挖掘这些信息的深层意义。4.提炼整合,规范作答。把筛选出来的相关信息分门别类的进行整合归纳和概括,根据题干要求及文本层次分点整合,精心组织答案。答题技巧三:对文体知识的把握类答题技巧1.要速读文本,从整体上把握文体特征。首先要根据所给材料当中的不同段落内容,从整体上明确把握其文体归属,根据自己平时的积累,准确区分所给材料到底属于哪种文体,这是回答这类问题的前提。2.要仔细审题,明确题干的要求。首先要仔细审读题干,找到命题点,看题干涉及的到底是哪类文体的知识,在切合题干要求的前提下,提出和制定相应的应对策略。3.根据相关文体知识,具体分析作答。在确定了材料的文体归属之后,就要联系自己储备、掌握的相关文体知识,并结合材料的具体内容进行分析归纳作答。4.规范作答,力求简明。这类试题的答案应该既包含文体知识,又与材料内容紧密相关,因此,作答时一定要紧扣材料分析,力求答案层次清楚,语言流畅,力求简明扼要。答题技巧四:联系实际拓展、评价与探究类答题技巧1.要通读全文,把握材料要领。对材料的评价探究要建立在整体把握材料内容和作者观点、态度的基础上,通读是前提,面对不同材料,要善于抓住关键语句理清全文的脉络,明白材料陈述的对象和作者的观点,对全文有一个整体的认识和把握。2.选准角度,亮明观点。要归纳整理作者在材料中表达了什么态度,观点有哪些要点,明确评价探究的对象,在切合题干要求的前提下,选准切入点,提出的问题要具体;提出的方法,要有操作性;提出的建议,要有针对性。3.紧扣材料,理由充分。答题时要紧扣材料内容,根据具体探究、客观评价的原则进行作答,这是组织答案的核心部分。理由要充分合理。陈述理由,应要紧扣材料,多方分析;答案要层次清楚,语言流畅,力求从多角度回答。4.解答评价、探究类试题应注意以下几点:(1)抓住关键,全面分析把握文体的观点倾向,可以采用抓共同话题和关键句、梳理材料层次等方法。其中关键句包括总结句、主旨句、过渡句以及作者的评论句等,评论句又包括材料对人物、事物、事理等的评价内容。(2)尊重事实,有理有据。评价探究时,虽然允许发表不同的见解,但不能脱离事实,或只抓住细小的一点无限放大。对问题的探究评价,一定要以尊重事实为前提,对文本进行全面、客观、恰当的分析。(3)深入拓展,深度分析。评价探究时,要从具体事件,具体问题入手,由表及里,深入拓展,从而把握文体的主要观点和基本倾向。分析时不能仅停留在表象上,要用学过的理论知识,用历史的、发展的、辩证的眼光来分析,这样才能使最终的评价探究具有一定的精度和深度。☞写作篇一、立意构思考生在作文时怎样审准文题,使其立意高远、富有思想,进而提升境界呢?这里有两种方法为你支着儿。一法:多向思维,纵横比较,独辟蹊径。考生在审题立意时,可突破常规,独辟蹊径,围绕题材进行多角度思考,对所给材料进行多层次分析;根据生活实际与自己的人生体验,对同一件事情进行横向、纵向、逆向的多维审视,得出有创意的结论。二法:逆向思维,反弹琵琶,求变出新。考生应摆脱思维定式的约束和影响,打通思路,展开联想;善于逆向思维,同中求异,异中求同。新颖独到的见解需以切题和思想深刻为要,力避为追求新颖独到而偏离题意、弄巧成拙。二、结构规划思路、层次、开头、结尾、过渡、照应等都是文章结构的重要形式,而思路不清、层次不明、顺序颠倒、内容残缺等都是结构混乱的表现。那么怎样避免“结构混乱”,做到“结构严谨”呢?考生应对以下几个方面加以重视:1.思路清晰,层次合理。考场作文中议论文居多,记叙文次之,记叙有记叙的思路,议论有议论的套路,考生在动笔之前,要有总体的思路,并做到思路清晰;在思路清晰的前提下,进一步规划文章结构,做到层次清晰。审题完毕,立意确定,问题选定之后,要在草稿纸上列出所写文章的结构提纲。若写议论文,就把文章的结构列出,并把分论点、主要论据一同列出,以免忘记;若写记叙文,要依据选定的叙述顺序,理出线索,设计好情节。2.段落完整,思路严密。段落完整是思路严密的具体体现。写记叙文,一段文字要写出人物性格的一个侧面或事情发展的某个阶段。写议论文,一个文段应论证一个论点,或列举一个事例等,且段落安排应疏密相间,前后有序。3.衔接巧妙,过渡自然。在联系紧密的段落之间,利用相关词语、相应句子巧妙衔接;在联系不太紧密的段落之间,巧妙过渡,使文章前后连贯,脉络分明。当然,衔接过渡,有的需要意合,不需要具体词语、句子来指明;有的需要明接,要用语言材料来点明。无论采用哪一种衔接方法,都要顾及“起承转合”“引议联结”。三、内容充实1.要做到材料丰富,就必须做好两件事:有选择地占有,有对应地运用。因为材料是否丰富,不是以数量多寡论,而是以材料是否“有效”(是否围绕中心思想)来说的。当然,还要考虑材料的摄取面是否广泛。2.有选择地占有,是材料丰富的关键一环。这就需要我们“思接千载,精骛八极”,占有足够多的资源,否则一则素材,如何去选?3.有对应地运用,是对材料运用的提升。4.丰富性还体现在摄取面的广泛上。名言警句、神话传说、历史典故、名人传记、古典诗词……无一不可入作文,无一不可进行有效的剪裁,我们所要做的就是找到适当的角度切入。四、中心突出考场作文应立足表述准确,主题一贯,进而实现中心突出、思想深刻的目标。如何实现这一目标?考生可识记以下三种技法:1.务求表现主题的语句鲜明显豁。考场作文的主题最忌“犹抱琵琶半遮面”,因此考生作文时应将一些能体现中心的语句放在最突出、最显豁之处,用准确的叙议语句把文旨清晰地凸显出来。2.务求承载主题的文体对应得宜。考场作文的主题在不同的文体里有不同的表现形式,如记叙类文章的主题,多借用人物形象、典型故事来抒写某种思想感受,或通过议论抒情的方式来点睛表达;议论类文章的主旨,多运用哲理分析、逻辑判断形式来概括表达。由此,承载文章中心的文体与其表达形式应当恰当地选择。3.务求体现中心的文脉单一集中。考场作文时,无论是侧重一人一事,还是构思一叙一议,都只能有一个主题。主题一旦确立,全文都应着力围绕这一主题叙议引申,处处紧扣主题,做到文脉一气贯通。五、感情真挚作文要以“真”为基础,那么,如何才能写出“真”呢?1.要有对生活的真切热爱与思考。没有热爱,就没有对生活的观察,有观察才能把生活诉诸笔端;没有思考,就不能写出思想的厚度,有思考才能把自己的独特个性展现在读者的面前。2.要有对观察所得的加工与提炼。把最能体现真情实感的东西表现在细节之中:细微之处见真情。3.要有对记叙的恰当议论与抒情。只有源于生活而又高于生活的文章才能打动人。而这“高于生活”就包括议论与抒情的运用。比如附情于理。六、有文采1.要防止语言与观点脱节。这种类型的问题多半是考生平时背诵了不少精彩的语段而不加鉴别生搬硬套导致的。他们往往不对议论的对象进行深入分析就加以套用,结果写出的语段看似漂亮,却驴唇不对马嘴,最后适得其反。2.要防止论述偏少,重生动而缺逻辑。议论文的文采必须建立在符合逻辑推理的基础上。【考前心理篇】中考获胜第一步:培养健康心理在影响考试成功的20因素中,最重要的因素是“考试中间的心态”,其次是考试前的心理状态,第三位是学习方法,第四位是学习基础。中考考的是知识,是能力,这一点应该没有谁不知道,没有谁不重视;但是,中考也是对考生心理的考查,知道并且重视的恐怕就不多了。在这里,有必要提醒广大考生与他们的老师和家长:注意培养考生的健康心理,过好“心理关”,对成功参加中考,有着十分重大的意义。我们常常看到,在中考前的复习过程中,不少考生会产生一种焦虑、浮躁的情绪,心中莫名的烦躁,既害怕中考时间一天天的临近,又巴不得明天就考了算了;他们听课时注意力难以集中,记忆力下降,好像什么都复习了,又好像什么都没记住;失眠、多梦、常有体力不支的感觉;思维迟钝,有时甚至连面对简单题目,头脑都好像出现“断路”现象,不会做了。这一切带来的后果是:信心下降,效率下降,导致各种测试的成绩下降,这又使信心下降,产生严重的恶性循环,对考试十分不利。为什么会产生这种现象?笔者认为,原因主要有四:一是当下社会普遍存在浮躁心理给学校、学生和家长都产生不小影响,导致所有相关人员在考前都产生一种十分严重的焦躁心理,大家都在自觉不自觉地制造考前紧迫情绪,这种情况越临近考试越明显。二是家长和学生对考试的期望值过高,脱离了考生的实际水平。三是目前我国的考试制度和严峻的就业形势对考生的压力太大,超过了考生的承受能力。四是随着复习过程的一步步进展,要求考生尽快形成厚积薄发的态势,加快把知识转化为能力的步伐,部分考生一时不能适应这种要求,因而产生焦虑和急躁的情绪。怎样改变这种状况?关键在于增强信心,培养健康心理。办法主要有两点:一、学会善于从纵向和横向两方面客观积极地、正确真实地认识和评价自己所谓“纵向”,就是自己和自己比,自己的现在和自己的以前比,在比较中发现自己的进步,在比较中增强自己的信心。特别是从考前两个来月开始,各科教师在复习过程中,一定要尽力用考试或作业、辅导等各种方式向学生多提供一些有利于增强信心的信息,经常表扬学生,让学生心情愉快,受到鼓舞。所谓“横向”,指的是学生对各科知识的掌握情况。教师在复习方法上,要更加注意科学性和有效性,要设法让学生把学科中的薄弱环节转换成努力的目标,转换成一步步进步的台阶。即使是发现了学生知识的缺陷、漏洞或欠缺,都既要给学生补正,更要注意方式方法。这样,学生的焦虑和急躁就可以逐步转化为愉悦和信心。在这里,“形式”甚至比“知识内容”更为重要。二、明白知识从积累到释放的演变道理从复习到考试的过程,是知识网络的形成过程,是将知识转化为能力的过程,也可以说是能量积累到能量释放的过程。在这个过程中,考生的心态便是一剂很重要的催化剂。心态越好,知识与能力的转化就越完全,越充分,因而也就越完美。良好的心态来自于充足的信心。因此,相信自己、长自己的志气便显得十分重要。一些考生在考场上一碰上难题就左顾右盼,看见别人总在写,就总以为别人比自己行,这在心态上就先输了一分,是考场大忌。须知,你不会做的别人也很可能不会做,而你会做的别人却不一定会做。这种考试心态一定要建立。另外,老师和家长也要鼓励学生注意以下规律:“取法其上,得乎其中;取法其中,得乎其下。”在考前,如果只是鼓励学生“别害怕,别怯场”,这只是一种下策;鼓励学生“正常发挥”,这也只能算是中策;只有鼓励学生“超常发挥,尽情释放”,才是最好的上策!此外必须指出的是,克服考前焦虑,不单单是考生一方面的事,需要教师和家长(甚至社会各方面)的共同努力和关注。在这里,过分的关心和过多的指责,都是绝对不行的,考生需要的是科学的态度和方法。我们不能只注重了知识而忽视了心理,一定要把对心理的重视提高到应有的地步。送给考生的中考小贴士童鞋们,我的亲,马上就要中考啦。你的身体、心理、“装备”都准备好了嘛?!快来跟着我一起做考前最后的准备和检查吧!!NO1.你需要准备的东西之“装备篇”有了装备才能“打妖怪”,童鞋们,要提前准备好哟!别等到了学校,手忙脚乱痛哭流涕哦!哈哈!一、准考证和身份证。考生可以找一条喜欢的丝带,把准考证和身份证挂在脖子前,以防丢失、遗忘。二、文具2B铅笔至少两支、黑色签字笔至少两支、直尺、圆规等(不要带涂改液、胶带、修正带)。特别提醒:涂答题卡使用的铅笔削得太细,会延长涂卡时间,建议把铅笔削成扁扁的“鸭嘴”形,也有卖现成的楔形头的考试专用笔,请确保正规商场购买正品。进入考场的有效证件,必须妥善保存。一些有经验的老师建议。三、手表。考试时最好带一块手表合理安排时间。提前调好时间。四、必要的生活用品,如清凉油、水、纸巾。五、雨具。考试前两天考生或家长应注意天气预报,了解中考当天的天气情况,如果有雨,提前准备好雨具。六、着装。如果中考当天温度较高,应准备舒适、宽松、透气性好的衣服,如棉、麻质地,避免考场中暑。七、眼镜。戴眼镜的同学最好准备副备用眼镜,要提前试戴其是否舒服。戴隐形眼镜的同学要准备好一副框架眼镜和明目眼药水。八、出行。如果骑自行车去考场,提前检查好"座驾",车胎气是否足等;准备乘公共汽车或出租车的考生,准备好零钱;家人驾车送,请提前看好线路。NO2.你需要准备的东西之“心理篇”马上中考,很多孩子出现焦躁不安,甚至是头痛、失眠、脾气暴躁等问题,你要知道现在再紧张再压力再使劲熬夜补习,已木有用了,放松心态,深呼吸~~~吸气~~~吐气~~~吸气~~~吐气~~~告诉自己,心态决定一切,遇事有个好心态才能事事顺利,不就是“伸头一刀么”,中考来啊,我不怕你!!PS:考生的心理状态与考试成绩的关系一、给自己好的心理暗示,相信自己。“我一定能考好!一定能实现目标;之前学过的都记得都会用得着;题难难所有人,我不会不一定别人的就会;我今天真精神一切都会顺利……”二、要充满斗志,昂首挺胸进考场。天上写着五个字:这都不是事。见到监考老师,记得微笑以待哦!三、为自己准备平时最喜欢穿的最舒服的衣服和鞋子,让自己保有安全感和舒适感。四、考前不要喝太多水。免得进了考场突然想上厕所,造成心理上的慌张。NO3.你需要准备的东西之“身体篇”俗话说身体是革命的本钱,身体棒精神好,做什么都好。考生也一样,有一个健康的身体迎接考试,绝对是重中之重。一、饮食上平时吃啥 考试也吃啥。考前大换食谱可是饮食的大忌,不要食用平时考生没有吃过的新食物。因为新食物平日没吃过,可能存在过敏、不耐受、胃肠不适等问题。也要注意忌口,少吃荔枝、芒果等热性水果,防上火,冷饮也要少吃噢。不要临时大补、迷信保健品,如果不吸收、不适应的话,反而会导致腹泻、过敏、感冒上火等病症,适得其反。二、适当运动。早晚,天气不热的时候可以外出散散步,既可以缓解紧张情绪,平复内心紧张,也可以保持精神的状态。三、洗热水澡或热水泡脚泡。热水澡是最古老的镇静剂,要放松自己,最好浸泡在比自己的体温高一些些的热水里,时间不要超过15分钟。热水泡脚同样能舒缓压力,放松身心的疲惫。四、睡眠。保证充足的睡眠,不要临时改变自己的生物钟。考前不要晚上临时熬夜抱佛脚,这只会让你第二天考试更疲惫。终极考问:遇上不会做的题怎么办?中考不难,但是也一定会遇到不会的试题,这个时候,怎么办呢?讲究方法,就能将失分点降到最低。一问:要不要把全卷看一遍?拿到卷子以后看一下,是看考卷一共几页,多少道题一定要先知道,千万不能落题和落页。关于是否要把全卷的题目全看一遍,同学们按自己的习惯来做,没有对错之分。模拟考你们怎么做的,中考还是怎么做,不要改变你的习惯做法。对于语文试卷,作文题是要先看一看的,因为这是一道得分最高的题目。看了作文,做到心里有数,等到真正开始作文的时候再认真考虑,不会出现无暇细想的情况。二问:如何提高一卷的得分率?调查显示:一般试卷前几题的错误率比较高,因为一开始考生一般心情比较紧张,所以提醒大家,在心情恢复正常时要着重检查一下前几题。三问:遇上不会做的题怎么办?中考是选拔考试,碰到难题是非常正常的。碰到不会做的题不要紧张,要想到,我不会做,那好多人也未必会做。我只要把能拿到的分拿到就行了,一定要稳定心态。四问:有的题可以上手,但做半截又不会了,怎么办?碰到这样的题不要慌,仔细审题,能做一步做一步,能做两步做两步。试卷中做对一步便有一步的分,答对一点有一点的分。所以心态一定要放松,能做几步做几步,能得几分得分。不要想着一道题会做,就一定能做到底。中考考题看重的是区分度。五问:要不要最后检查一下全卷?相当一部分同学在规定时间内答不完题,但一定要留下15分钟左右时间检查全卷。往往检查一遍,能检查出一个错误,从而多得几分,这也是中考成功的一个重要方法。【考场注意篇】面对中考,很多考生第一次面临如此重要的考试,不免会因紧张而出现一些疏漏。稍有不慎就会影响考生情绪而影响临场发挥。可以从以下几点提醒孩子:做好技术准备,决胜中考考场1.早6:30左右起床,择其要点再温习一下。早饭要吃好(不要吃太多难消化、油腻的食物),记得带水。2.到考场时间:一定有家长送考。时间不宜过早或过晚,一般在考前30分钟至40分钟为佳。太早了,与各种干扰因素接触过多,容易破坏良好的心理感觉;太晚了,开考后才进考场,往往出现慌张的感觉。3.进入考场前上一次厕所。根据考点要求持准考证进考场,准考证放在考桌左上角备查,保持安静,认真听取监考老师的讲解,不明白的地方要先举手报告,获准后方可发问。4.进考场后让自己平静下来。一般开考前30分钟,老师会提前让我们进入考场。刚进考场的时候,考生往往会感到紧张,在气氛严肃的考场里稍微有些紧张很正常,我们要做的是别让这种紧张情绪扩散下去,蔓延开来。把注意力从考试上引开,让自己平静下来。避免失分,我有对策1.开始浏览试卷,先看看试卷有没有缺页或者印刷不清的地方,这些情况当然比较少见,但如果对试卷有疑问一定要立即向老师提问,请监考老师帮忙解决。确认无误后,在姓名栏写好自己的名字、准考号。答题卡贴条形码前核对自己的姓名、考号信息。做完这一系列工作以后,可以开始看卷子:主要是看卷子大概有多少页,一共有几种题型,最后有多少道大题。对试卷的整体构成心中有数,可以避免做题的时候没有控制好节奏,考试终场时才发现没时间做完试题的情况。2.稳定情绪、排除干扰、集中注意力。在考试全过程中这一点都是至关重要的。一切准备就绪,现在我们可以开始动笔做题了。3.答卷时要根据先易后难的原则。不会的暂时空着(可用铅笔画个问号,全卷答完后再回过头来逐个攻克,做完后,把问号擦掉)。4.要特别重视前40分钟的高效、合理使用:考生的思维状态和能力、头脑的灵敏度和计算的准确率,在前40分钟都是最高的。建议前40分钟不可在自己一时感到困难的试题上冥思苦想,一定要暂时放下,以尽可能的高速度,把有把握、较熟悉、解法思路明确的试题,一口气写完。这段时间保证顺利解题,失误必然最少,得分最多。5.接下来30分钟,从头开始去做自以为困难的试题。不要在原思路下一味苦想,而应重新读题,认真分析已知条件,由于此时心中已有底了,还未被自以为的难题打击信心,未造成慌乱紧张,心理正常,在心情稳定的情况下,冷静思考,为灵感的出现创造条件,此时很可能发现难题不难了。这段考试时间内,仍可把暂时无法求解的题目,再次放下,选难中之易者,从读题开始,改变思考角度、转化已知条件,使用不同方法,深入探索这一命题,这使一部分难题得以攻克。 6.第3个时间段,30分钟。对仍未解出的题目做死马当活马医式的最后一搏,以决定应否放弃。如是选择题,可在排除法无法从两个可能答案中找出正确选项时,任选其一,做对的概率也高达50%。7.最后10~20分钟是留做复查的:应把试卷看作是他人的,想方设法判断答案是否正确,尽量避免陷入个人解题的原思路和过程中去,那样将很难发现失误;修正答案要理由充分,似是而非,两个答案难于取舍时以不改为宜,切忌将正确答案改错;要检查过程是否严谨,所答与所问是否一致;一题两问的有无漏做,有无整页未答的情况;最后一项要检查姓名、号码等是否按规定填好了。8.每科出考场后要做到:不回忆考题,不讨论答案,不估算成绩,别人问就说感觉很好,要信心百倍地全身心投入未考科目上。9.考完后,休息一会儿后再吃午饭。午饭后不要马上睡觉,可以散散步,听听音乐。午睡时间安排30分钟到40分钟即可。家长一定要注意提醒考生,以免考试迟到。【考后心理篇】7种方法舒缓考生考试后心理压力结合工作经验,给广大家长和中考考生提供了7种舒缓心理的方法,大家不妨一试:方法一:转移思路法离开令人不快的情境,转移到高兴的情景或事情上,做自己喜欢做的事,如打球、听音乐、和朋友聊天等,忘却烦恼。方法二:亲近动物和植物法走入大自然,与动植物亲近,让植物的生机盎然、小动物的顽皮可爱来感染自己,排遣心中的烦恼,改变自己的阴郁心情。方法三:谈心法把心里的郁闷或牢骚倾诉于自己最亲密和信赖的人。这种谈心不仅可以用面对面的方式,也可用写信的方式来交流。有时候,自己的一些想法可能是错误的,朋友的一番话可能会使人有“柳暗花明又一村”的感觉。方法四:自我暗示法不妨经常对自己说“我能行”。只要有自信,任何人都不能使你自卑。方法五:日记法把内心的愤恨和痛苦尽情地宣泄在日记中,别人也不会知道。慢慢地自己的心情就会平静下来。方法六:补偿升华法以其他方面的成功(例如学得某种专长)补偿中考失败造成的痛苦;把不良情绪导向积极方面,通过做出突出成绩化解心理矛盾,获得心理平衡。方法七:宣泄放松法把积压在内心的消极情绪释放出来,或大声叫喊,或痛哭一场,或“潇洒”玩一回(当然要有益),通过宣泄让心情慢慢平静下来。也许这才是中考的真正内涵:它虽然不能定义你的未来,却浓缩了你最好的青春时光。考后提醒:中考结束适当娱乐可以放松不可放纵黑色的六月过去之后,你就迎来了五彩缤纷的假期,接下来即将打开人生中的另一个重要阶段——高中的大门。接下来的时间,你要怎么过?许多考生已经有了自己的打算,不过,无论怎么安排这个暑假,都应该好好珍惜,因为,接下来,你还有另外三年的一场硬仗要打。当然,还有考生没想过怎么安排,那就参考下吧。N1.锻炼身体,保持身材如果考生有条件,可以选择自己喜欢的一项运动,加强锻炼。或者,学习一种新的运动,瑜伽、游泳等皆可。N2.外出旅行,长长见识出门旅行是一个度过假期的好方式。读万卷书,行万里路。所以,趁着年轻出去看看吧,外面的世界不是一般的精彩!N3.睡觉聚会上网,也不能少考完要先睡上三天三夜,这是众多考生考完后的第一想法。的确,睡觉、聚会、上网这些方式是放松的好方法。虽然不提倡考生以此打发漫漫长假,但睡觉是储备体力等重要方式,聚会是同学间联络感情的重要渠道,而上网是了解这个世界的重要窗口,所以,只要不是长时间沉迷,家长没必要阻止考生做这些事情。温馨提示:可以放松,但不可放纵友情提示各位考生,可以放松,但不可放纵。最好的方法是,给自己的假期划分几个阶段,每个阶段都充分利用。要对自己的即将到来的高中生活进行规划,同时可以外出游玩,长长见识。假期的每一天,都要合理安排时间,并注意饮食和作息规律。名词①菡萏②帐蓬③虬(qiú)枝动词④镌(juān)刻⑤诘责⑥拜谒(è)形容词⑦亘(gèng)古⑧妥贴(tiē)⑨拙(zhuō)劣成语⑩殚精竭虑⑪如座针毡⑫深恶(è)痛绝小贴士:洪泽湖大堤,亦称“高家堰”,始建于东汉。大堤北起淮阴区码头镇,南迄洪泽区蒋坝镇。洪泽湖大堤结构规格统一、筑工精细,展示了中国古代水利建设的高超技艺。洪泽湖大堤既是先民综合治理黄河、淮河、运河水系的工程,又是一处可作为文化研究、旅游发展的历史文物和治水丰碑。中文名称中国文化遗产标志英文名称Chinese cultural heritage确定时间2005年08月17日图案成都金沙“四鸟绕日”实施方案环节主要任务辩手分工时长立论正面陈述己方观点正方一辩 反方一辩各约3分钟攻辩一质疑对方观点,回答对方提问正方二辩 反方二辩各约3分钟攻辩二质疑对方观点,回答对方提问正方三辩 反方三辩各约3分钟自由辩论①正方 反方各约4分钟总结陈词②正方四辩 反方四辩各约4分钟古典小说“闹”的情节“闹”的原因人物形象《西游记》①___嫌弼马温官小,又怒王母娘娘未请他参加蟠桃会。桀骜不驯敢于反抗《水浒传》鲁智深大闹野猪林林冲被公差押解至野猪林时,公差要杀害林冲。②_____家园是什么古诗文名句故乡风物的萦绕露从今夜白,(1)___________。(杜甫《月夜忆舍弟》)(2)_________,万里送行舟。(李白《渡荆门送别》)因思杜陵梦,(3)___________。(温庭筠《商山早行》)家中亲人的牵挂何当共剪西窗烛,(4)_________。(李商隐《夜雨寄北》)马上相逢无纸笔,(5)_________。(岑参《逢入京使》)闲适宁静的隐居生活(6)__________,悠然见南山。(陶渊明《饮酒》)勇士报国的梦中疆场(7)________,_________。(陆游《十一月四日风雨大作》)文人志士家国天下的情怀(8)________,__________。“我”的表现儿子眼中“我”的主要问题(1)絮絮叨叨“我”将书画礼盒背在身后添人麻烦(2)(3)“我”询问酒店为啥缺少顾客(4)方法提示形象分析关注对猫婆居住环境描写(1)_________________。关注对猫婆言行的描写示例:从猫婆说自己收养的猫都是叫人打残、摔坏的猫,还抢救了一只差点被顽童烧死的小黄猫,可见猫婆仁慈善良。关注对猫婆身世的介绍(2)___________________。关注对小黄猫的描写(3)____________________。时间刚退休头四年① _______休息日行动②______开始修墙停止修墙③ ______感情充满深情④________力不从心念念不忘情节结构内容概括开端她要离开村里,可一夜的雨冲坏了村里通往外面的桥。发展(1)高潮铺桥的木板不够,柱子奶奶坚持用自己的寿材板铺桥。结局(2)探究商代青釉器的“身份”提出问题:(1)_______探究问题(参照下面问题解决的角度)(2)_______商代青釉器的烧成温度是多少?(3)_______解决问题器物种类原料(氧化硅、氧化铝含量占比)烧成温度施釉情况一般陶器80%左右较低,700-1100℃大多数无釉商代青釉器(4)_______1200℃左右(5)_______一般瓷器含量很高较高,>1200℃表面有高温釉得出结论青釉器是(6)________(填写“陶器”或者“瓷器”)时间、地点2021年6月28日中央电视台采访对象张桂梅采访方式深度访问,录像采访目的了解张桂梅创办云南女子高中的相关情况以及她的心路历程。采访器材纸、笔、录像机采访问题(1)您为什么想筹建这样的学校?(2)为什么不男女生一起招收?(3)写作任务写作情境及内容写作任务一在班会上,向老师和同学讲述自己在学习生活中克服困难的经历和感悟。写作任务二在“直面困难,勇毅前行”主题升旗仪式上做一次国旗下讲话。两面词能否、是否、好坏、得失、成败、高低、有没有等并列词和、与、同、及、以及、跟等否定词不、不再、没有、否认、避免、忌、防止等代词这、那、他、此、彼等介词通过、经过、由于、对、对于、在等技巧内容示例一看题目诗词题目往往提示内容、点明情思《送杜少府之任蜀州》题目就暗示了诗人对朋友的深情厚谊二看词句诗词中往往有直接表明诗人情感的关键词句表达情感的词,通常有“别、惜、独、愁、思、沾襟”等,结尾句一般直接表达诗人情感三看意象诗人的情感常常蕴含在具体的景物中,由此形成了一些特有的意象杨柳、浮云等往往用来表达离愁别绪,而明月、鸿雁、杜鹃等则往往表达思乡怀人之情四看题材每一题材的古诗常常表达相对固定的情感田园诗多抒发对自然美景的热爱之情,送别诗多表达与友人离别时的惜别之情题型答题模式加点字词有何作用?准确科学、生动形象地说明了事物……的特征(事理)。符合实际情况,具有科学性,激发了读者的阅读兴趣。“××”一词能否替换为另一个词语?并说明理由。不可以。“××”一词的意思是……,所换词语的意思是……,替换后意思有所改变,不符合实际(或:没原文形象生动)。限制性词语能否删去?不能删去。如果删去“××”一词,句子的意思就变成了……,显得太绝对;用了“××”一词,准确地说明了……,符合实际情况,具有科学性。(“比较”“几乎”“相当”等词表程度,“大约”“可能”“左右”等词表估计,“多”“有余”等词表数量)从文章中找出一个能体现说明文语言准确、形象特点的词语并简要分析。“××”句中的“××”一词用得准确、形象,它表示的意思是……“这些条件”“这种现象”“同样道理”等含有指代意义的词语在文中具体指代什么?一般指的就是代词前面的那句话,找最近的一句话。有时要注意可能不是整句话,而是其中的一部分。引用俗语、诗歌、神话传说有什么作用?从内容上说,生动形象、通俗易懂地说明了……,增强了文章的形象性和文学性或增添了文章的神奇色彩(或增强文章的趣味性和可读性)。从结构上说,起到了承上启下的作用或者是引出说明对象。说明方法常用标志词作用(答题模板)举例子例如、比如、据说、譬如等举……例子,具体说明……特点,从而使说明更具体、更有说服力。列数字数词、确数、约数、小数、倍数、百分数用数据准确地说明该事物的……特点,使说明更准确、更有说服力。作比较也、相对于、比、和……不同把……和……作比较,强调了被说明对象的……特点(地位、影响等),使人印象鲜明。打比方像、如、仿佛把……比作……,形象地说明该事物的……特点,增强文章的趣味性。分类别一类……一类……条理清楚地说明了事物的……特点,对事物的特征、事理分门别类加以说明,使说明更有条理性。下定义这就是…… ……是……运用下定义的说明方法,用简明科学的语言对说明的对象、科学事理加以揭示,从而更科学、更本质、更概括地揭示事物的特征、事理。作诠释因为……、由于……、这是由……造成的运用作诠释的说明方法,对事物……的特征、事理具体解释说明,使说明更通俗易懂。列图表各种类型的图片、表格等运用列图表的说明方法,用列图表的方式对事物……的特征、事理加以说明,使说明更简明更直观。引用引用名言、诗文、新闻报道、轶闻趣事等各类资料运用引用的说明方法,引用……进行说明,既增强了说服力,又增强了趣味性。

相关试卷

2022届青岛中考语文考前热身卷(三): 这是一份2022届青岛中考语文考前热身卷(三),共11页。试卷主要包含了本试题共三道大题,含23道小题, 下列句子没有语病的一项是, 默写, 阅读下面材料,按要求作答等内容,欢迎下载使用。

2022届北京中考语文考前热身卷(一): 这是一份2022届北京中考语文考前热身卷(一),共17页。试卷主要包含了请将答案正确填写在答题卡上,根据语境作答,阅读材料,回答问题,在下列横线上填写出相应的句子,名句恒久远,千古永流传等内容,欢迎下载使用。

2022届江西中考语文考前热身卷(一): 这是一份2022届江西中考语文考前热身卷(一),共13页。试卷主要包含了语言知识及其运用,古诗文阅读与积累,现代文阅读,综合性学习和写作等内容,欢迎下载使用。



【本文地址】

公司简介

联系我们

今日新闻

    推荐新闻

    专题文章
      CopyRight 2018-2019 实验室设备网 版权所有